You are on page 1of 561

LISTENING PRACTICE FOR GIFTED STUDENTS

CHAPTER 1: IELTS LISTENING

A- SECTION 1

Exercise 1:

You will hear a student called Janet talking on the phone to the manager of a sports centre
about a job and complete the notes below. Write NO MORE THAN THREE WORDS
AND/OR A NUMBER for each answer.

White Water Sports Centre

Manager's name: 1. ______________

Job available: 2. ______________ (part-time)

Job responsibilities:

– supervising swimmers

– care of 3.______________ for beginners' classes

– carrying out 4.______________ regularly

Days: Mondays and Wednesdays

Working hours: 5.______________ to ______________

Maximum pay: 6.______________ per hour

Interview: Friday at 2 p.m.

Address of Sports Centre: 23–27 7.______________, East Gate

Steve's direct line: 8.______________,

Remember to bring: 9.______________

10.______________
1. Steve Thompson

2. pool attendant

3. (the) equipment

4. water quality tests

5. 6 (p.m.), 10 (p.m.)

6. $19

7. Farndon Avenue

8. 053210

9-10. (in either order) application form, certificates

V1: Hello, White Water Sports Centre.

Janet: Hello, er, I wanted to enquire about a job at the centre.

V1: Right, I’ll just put you through to the manager.

Steve: Hello, Steve Thompson speaking.

Janet: Hello, er, my name’s Janet Willis, Er, I’m looking for a part-time job and I saw an ad
saying that you have some vacancies. I was wondering what sort of people you were looking for.

Steve: Well at present we’re looking for a part-time pool attendant.

Janet: oh, yes, I’ve spent the last three summers working for a children’s summer camp, so I did
a lot of pool supervision, and I’m actually a sports student – water sports is my special area.

Steve: Ok, well no need to ask if you can swim then …

Janet: No,  I’m  certainly not afraid  of the water. So what does the job at the pool involve?

Steve: you’d mainly be responsible for supervising the swimmers – we have to watch them all
the time, obviously, in case of accidents, so you’d have regular shifts there.

Janet: Ok.

Steve: Then as well as that, you’d have to look after the equipment that’s used by the beginners’
classes

Janet: Right – and would I be involved in teaching them at all? I’d be quite interested in that.
Steve: Well, they have their own instructor, so that’s not really part of the job. The attendant’s
job does involve taking regular water quality tests … but you wouldn’t be involved in cleaning
the pool or anything like that.

Janet: Ok. And the ad said you wanted someone just twice a week.

Steve: yes, that’s right.

Janet: Can I choose which days?

Steve: Erm, well if you’d rung up earlier you could have done but i’m afraid it’s got to be
Mondays and Wednesday – we’ve got someone for Tuesday and Thursdays, and the weekends
are already fully staffed. Is that going to be a problem for you?

Janet: No, that should be alright. And the ad said it was evening work, right?

Steve: yes, you start at 6, and the pool closes at 9:30, but you wouldn’t get away until 10 by
the time you’ve checked the lockers and changing rooms.

Janet: fine.

Before you hear the rest of the conversation, you have some time to look at questions 6 – 10

Janet: And how muck do you pay?

Steve: The basic hourly rate is $15, but we’d go up to $19 for someone with the right
qualifications.

Janet: Well I’ve got life-saving  certificates and first aid qualifications.

Steve: Oh, with that and your experience you’d probably get the maximum rate then. Obviously,
you’d probably get the maximum rate then. Obviously, you’d have to come along for an
interview, if you’re interested?

Janet: Oh it sounds just the job I’m looking for. Shall we fix a time for the interview now?

Steve: Ok … er, it’s Janet, isn’t it?

Janet: Yeah, Janet Willis.

Steve: How about Friday morning, Janet? Around 11.00

Janet: Oh sorry – I have lectures, but I could make the afternoon.

Steve: 2p.m?

Janet: Fine. And can I just check on where you are … is it Findon Avenue?
Steve: No, it 23- 27 Farndon Avenue, that’s FARNDON, it’s off East Gate.

Janet: East Gate … Fine. I’ll looking forward to meeting you, then.

Steve: ok, so if you need to phone me before then, you can get through to me directly on 053210

Janet: Is there anything I need to bring along to the interview?

Steve: Well, you do need to fill in an application form- I’ll put one in the post for you, so can
you fill that in and bring it along.

Janet: you don’t want me to post it back to you?

Steve: No, just remember to bring it along with you.

Janet: What about references? Should I bring any?

Steve: No, but do have your certificates with you when you come, we need to see those.

Jane: Great. Thanks very much then., I’ll see you on Friday. Bye.

Steve: Bye.
Exercise 2: Complete the notes below.

Write NO MORE THAN THREE WORDS AND/OR A NUMBER for each answer.

Temporary Patient Record Form

Name: Example

Peter Smith

Street address: 1. ______________

Suburb: 2. ______________

Phone number: 3. ______________

Details of injury

Sport: Tennis

Type of injury: Sprained 4. ______________

Date of injury: 5. ______________

Previous treatment and current problems

The patient’s private doctor suggested treatment with 6.


___________.

The patient is unable to 7. ______________ and he is experiencing


some pain in his 8. ______________ at night.

Advice given

Stop using the 9. ______________

Do regular 10. ______________ at home


1. 95 Cross Street

2. Walkley

3. 4689 5324

4. knees

5. June 18th

6. ice pack

7. go upstairs

8. back

9. stick

10. exercises

Girl: Good morning, what can I do for you'?


Peter: Uh, yes, I am currently visiting this area, but I injured myself when I was doing sports a
while ago, and I still feel painful, so I wondered whether 1 could go for a doctor here.
Girl: Sure sir, we can take you on as a temporary patient. I'll just take down some personal
details of you. May I have your name, please?
Peter: Yes, it's Peter Smith.
Girl: All right, Peter. And where are you currently staying here?
Peter: At 95 Cross Street.
Girl: And the county?
Peter: Walkley. That's W-a-l-k-l-e-y.
Girl: OK, and can I have a contact number?
Peter: Uh, it's 4689 5324.
Girl: OK, thanks. May you just wait down there! The doctor will see you in a minute.
Doctor: Now, how can I help you? It's Peter Smith, isn't it?
Peter: Yes, I had a sporting accident, and a doctor at home treated me, but I'm still getting some
pain.
Doctor: Hum, right. Uh, first of all, I want to ask you a few questions.
Peter: Okay.
Doctor: Well, what sport were you doing when you got injured?
Peter: Playing tennis with my friends.
Doctor: Now, I see. Did you hurt your elbow or wrist?
Peter: Oh, no. I had my knees sprained, which was the original problem.
Doctor: Right, and when did this happen?
Peter: Uh, that was three weeks ago now, so it was about June 18th.
Doctor: Hum. And, you said you had medical treatment at home?
Peter: Uh, yeah. The doctor said I didn't need an X-ray or things alike, and he just told me to use
an ice pack.
Doctor: Fine, anything else?
Peter: Yes, and I've been using a walking stick to help me get around.
Doctor: Right, now what problems are you having during walking?
Peter: Well, actually I can walk, yet I still can't go upstairs, so I've been sleeping downstairs.
Doctor: Hum, now you said your knee still gets hurt'?
Peter: Well, no. Actually, it's getting better. It's my back that's hurting me now. It really aches
at night, and I cannot sleep well.
Doctor: Hum, I have several suggestions for that.
Peter: Great.
Doctor: First you should put the stick away as that's probably the source of the problem.
Peter: Oh, really? I wish I'd know.
Doctor: After that, I can prescribe you something to relax the muscles in your back.
Peter: Oh, sorry to be difficult, but I've had something like that in the past, and there were many
side effects, and I don't want to take it. Would you recommend anything else?
Doctor: Well, yes. We do have a leaflet showing some exercises you can do yourself at home. If
you do them every day, they'll soon be effective.
Peter: Great. I'll do that.
Exercise 3:
Questions 1-5
Complete the sentences. Write NO MORE THAN TWO WORDS for each answer.
Peter and Mary’s diet will be both sensible and 1………………
Every two months, they can eat 2………………
On Saturdays, they will go 3………………
This coming Saturday, they will go to 4………………
In every meal, there will be 5………………
Questions 6-10
Choose the correct letter, C, I, or B.
C Chocolate

I Ice cream

B Biscuits

Peter’s opinion Mary’s opinion

Tuesdays B 6………………

Thursdays 7……………… 8………………

Sundays 9……………… 10………………


1. practical

2. pizza(s)

3. (light) walking

4. Pine Park

5. fruit juice

6. B

7. I

8. C

9. B

10. I

Peter: Okay Mary, I know I’m putting on weight with all my bad eating habits.
Mary: That’s right. Chocolate bars, pizzas, and all washed down with litres of soft drink.
And with all the sugar in soft drink, I think we can say you’re definitely overindulging,
don’t you think?
Peter: I know, I know. I intend to definitely stop drinking that from now on.
Mary: And that’s part of the sensible eating plan that we have agreed we need to
formulate—one that we can both follow.
Peter: That’s right, sensible, and practical — allowing us to indulge in some sweet
foods on occasions.
Mary: On rare occasions, and we’re going to cut out all the pizzas and hamburgers.
Peter: Well, not totally. I thought you said pizzas could be eaten once a week. You know
how I like pizzas.
Mary: No. I said once every two months. Once a week was ice cream, not pizza.
Peter: Yeah, that’s because you like ice cream. I prefer pizza.
Mary: Peter, if you want to lose weight, you need to be disciplined, in both what
you eat, and what you do, which brings us to the issue of exercise.
Peter: That’s right. So, jogging twice a week, and hiking on Saturdays, right?
Mary: No, hiking is Sundays, allowing Saturdays for the light walking exercise. I’ve
investigated, and discovered there are a lot of walking trails around this suburb, with
many beautiful places if you just look around.
Peter: So what’s happening this Saturday? A walk around the city?
Mary: That’s later in the month. This Saturday we’ll visit Pine Park
Peter: Hill Park is nicer, with great views, too,
Mary: Yes, but we have to travel too far to reach Hill Park, so I ruled that out, and I love
the smell of pines.
Peter: And what about the weekday meals? Salads every night, I suppose?
Mary: No, it’s important to vary the meals so that the food remains interesting and fun.
The only common factor is the fruit juice, but otherwise we’ll have a variety of
healthy fare, from salads, to fish, to lean meat.
Peter: I'd rather have meat every night, but you’re the boss.
Mary: If you want to lose weight, you’ll have to accept this program, okay?
Peter:Okay.
Peter: Now Mary, you said we could have a little treat in our diet plan three times a week:
specifically, Tuesdays, Thursdays, and Sundays.
Mary: Yes. A very small treat, but something to reward us for all the discipline shown.
Such rewards will motivate us to continue the diet.
Peter: So, on Tuesdays, what can I choose?
Mary: You have a choice between a small serving of chocolate, ice cream, or biscuits.
Peter: Okay, well, I’ll go for the biscuits. I love those crunchy little things with my cup of
tea. So do you, right?
Mary: I do indeed, so I’ll have them, too, saving the ice cream for later.
Peter: But I know how much you like ice cream, so why don’t you save it for the end of
the week? That will give you the motivation to keep going, and you can have chocolate
midweek?
Mary: That’s a good idea, but I think the most important thing is for you and I to eat different
things on those other two days.
Peter: Well, if you think that way, I’ll eat the ice cream on Thursdays, not the chocolate.
Okay?
Mary: Well, if you’re having that, then I can’t have ice cream. In order to be different, I’ll have
the chocolate.
Peter: So, we can compare our different snacks, and maybe even share a little with each
other.
Mary: Exactly! And the variety and fun involved will take our minds of wanting more.
Peter: Eating for fun! A good idea!
Mary: And then you can have the chocolate on Sunday, right?
Peter: But I like biscuits? Actually, I want biscuits again—to have with my cup of tea.
Mary: But you have the biscuits on Monday, remember, and we can’t repeat.
Peter: Ah come on! This isn’t fixed in concrete. I can have them again instead of chocolate. You
might like variety, but I don’t. I’m a creature of habit.
Mary: Well, the whole point of this was to taste different snacks, but if you want the
same thing, okay. I, for one, am sticking to the original plan, with ice cream to finish the week,
and not another dose of chocolate.
Exercise 4:
Questions 1-7
Complete the notes below.
Write ONE WORD AND/OR A NUMBER for each answer.

Enquiry about booking hotel room for event


Rooms
Adelphi Room
   number of people who can sit down to eat: 1………………….
   has a gallery suitable for musicians
   can go out and see the 2………………… in pots on the terrace
   terrace has a view of a group of 3……………………
Carlton Room
    number of people who can sit down to eat: 110
    has a 4……………………
    view of the lake
Options
Master of Ceremonies:
    can give a 5………………… while people are eating
    will provide 6………………… if there are any problems
Accommodation:
    in the hotel rooms or 7…………………
Questions 8-10
What is said about using each of the following hotel facilities?
Choose THREE answers from the box and write the correct letter, A, B or C, next to
Questions 8-10.
Availability
A     included in cost of hiring room
B     available at extra charge
C     not available
Hotel facilities
8   outdoor swimming pool ……………….
9   gym                                ……………….
10  tennis courts                  ……………….
1. 85

2. roses

3. trees

4. stage

5. speech

6. support

7. cabins

8. C

9. A

10. B

ANDREW:        Good morning, Clare House Hotel. Andrew speaking. I’m the Events Manager.

SAM:               Good morning, Andrew. My name’s Samantha. I’m arranging a party for my
parents’ fiftieth wedding anniversary, and I’m ringing to ask about hiring a room some time next
September. Also my parents and several of the guests will need accommodation.          

ANDREW:        OK, I’m sure we can help you with that. Will you be having a sit-down meal or
a buffet?

SAM:               Probably a sit-down.

ANDREW:        And do you know how many people there’ll be?

SAM:               Around eighty, I think.

ANDREW:        Well we have two rooms that can hold that number. One is the Adelphi Room.
That can seat (Q1) eighty-five, or hold over a hundred if people are standing for a buffet.

SAM:               Right.

ANDREW:        If you have live music, there’s room for four or five musicians in the gallery
overlooking the room. Our guests usually appreciate the fact that the music can be loud enough
for dancing, but not too loud for conversation.

SAM:               Yes, I really don’t like it when you can’t talk.


ANDREW:        Exactly. Now the Adelphi Room is at the back of the hotel, and there are French
windows leading out onto the terrace. This has a beautiful display of pots of (Q2) roses at that
time of the year.

SAM:               Which direction does it face?

ANDREW:        Southwest, so that side of the hotel gets the sun in the afternoon and early
evening.

SAM:               Very nice.

ANDREW:        From the terrace you can see the area of (Q3) trees within the grounds of the
hotel, or you can stroll through there to the river – that’s on the far side, so it isn’t visible from
the hotel.

SAM:               OK.

ANDREW:        Then another option is the Carlton Room. This is a bit bigger – it can hold up to
a hundred and ten people – and it has the advantage of a (Q4) stage, which is useful if you have
any entertainment, or indeed a small band can fit onto it.

SAM:               And can you go outside from the room?

ANDREW:        No, the Carlton Room is on the first floor, but on one side the windows look out
onto the lake.

SAM:               Lovely. I think either of those rooms would be suitable.

ANDREW:        Can I tell you about some of the options we offer in addition?

SAM:               Please do.

ANDREW:        As well as a meal, you can have an MC, a Master of Ceremonies, who’ll be with
you throughout the party.

SAM:               What exactly is the MC’s function? I suppose they make a (Q5) speech during
the meal if we need one, do they?

ANDREW:        That’s right. All our MCs are trained as public speakers, so they can easily get
people’s attention – many guests are glad to have someone who can make themselves heard
above the chatter! And they’re also your (Q6) support – if anything goes wrong, the MC will
deal with it, so you can relax.

SAM:               Great! I’ll need to ask you about food, but something else that’s important is
accommodation. You obviously have rooms in the hotel, but do you also have any other
accommodation, like (Q7) cabins, for example?
ANDREW:        Yes, there are five in the grounds, all self-contained. They each sleep two to
four people and have their own living room, bathroom and small kitchen.

SAM:               That sounds perfect for what we’ll need.

SAM:               Now you have various facilities, don’t you? Are they all included in the price of
hiring the room? The pool, for instance.

ANDREW:        Normally you’d be able to use it, but (Q8) it’ll be closed throughout
September for refurbishment, I’m afraid. (Q9) The gym will be available, though, at no
extra charge. That’s open all day, from six in the morning until midnight.

SAM:               Right.

ANDREW:        And the tennis courts, but (Q10) there is a small additional payment for
those. We have four courts, and it’s worth booking in advance if you possibly can, as there can
be quite a long waiting list for them!

SAM:               Right. Now could we discuss the food? This would be dinner, around seven
o’clock …
Exercise 5:

Questions 1-4: Choose the correct letter, A, B or C.

1. A TFN is a number used

A.exclusively for tax administration.

B.exclusively by individual tax payers.

C.for managing several government services.

2. Without a TFN, the applicant would be

A.taxed at a higher rate.

B.unable to work.

C.liable for Medicare contributions.

3. What kind of visa does the applicant have?

A.a visa granting indefinite stay

B.a visa with work rights

C.a student visa

4. A TFN is issued

A.if you change your name.

B.only once in a lifetime.

C.when you claim a government benefit.

Questions 5-10: Complete the form below. Write NO MORE THAN TWO WORDS
AND/OR A NUMBER for each answer.

Passport #: 5. ______________

Nationality: Spanish

Postal address: 1339 6.______________

Hollywell 1517

Contact details: 09-5577 5076


Preferred contact person: Martha 7. ______________

Title: 8. ______________

Family name: Farina

First given name: Maria

Other given name(s): Rosa Ana

Any other names: Mary = (9. ______________)

Date of birth:   10. ______________ (mm/dd/yy)

Gender: Female

Name of spouse: N/A

1C
2A
3A
4B
5 JGW 1100536
6 Harbour Drive
7 Pearce
8 Miss
9 first name
10 15/11/(19)83

A: Good morning, Tax Office. How can I help you?


B: I'd like to apply for a tax file number.
A: Are you a citizen?
B: No, but I'm told I still need a number.
A: So, what is your residency status? Are you a permanent migrant or perhaps a temporary
visitor?
B: Oh...I'm...I'm a permanent migrant.
A: And you need a TFN?
B: A what?
A: A TFN—it stands for Tax File Number.
B: Yes, what is that exactly?
A: It's a unique number we issue to individuals and organisations to help administer tax
collection and other government systems.
B: Why do I need one?
A: In actual fact, you don't.
B: What do you mean?
A: It's not compulsory, you know.
B: But...I should get one?
A: Well, it's a good idea, otherwise you'll have more tax withheld from your wages or salary
—in fact, you won't be eligible to participate in the PAYE (that's pay-as-you-earn') system and
you couldn't apply for income support or other benefits. You wouldn't have to pay the Medicare
levy but then you wouldn't be entitled to claim Medicare benefits either. I'd better have one then.
Okay. So, you're currently living in this country, right?
B: Yes.
A: What kind of visa do you have: a working visa?
B: Well, I did have one because after my student visa expired I went back home and worked for
a year before applying for a job here. The job turned out to be permanent and full time and my
employers wanted me to settle here...so I now have a permanent migrant visa.
A: Now, once you have a file number you never need to reapply, even if your circumstances
change, for example, if you get married or decide to take an English name. They'll even use the
same one when you retire and apply for a government pension. I see. Have you ever had a TFN
before?
B: No.
A: Right...let's get on with the application process now.
A: What is your passport or travel document number?
B: Oh, just a moment, I have it here: JGW-double one-double zero-536.
A: Where do you come from?
B: I came from Greece.
A: Is that your country of origin?
B: Yes, I came from Greece.
A: Are you Greek? Do you have a Greek passport?
B: No, I was born in Spain. I have a Spanish passport but I've been living in Greece.
A: And where do you live now? Where will the department send your TFN?
B: That's 1339 Harbour Drive, Hollywell. Postcode? 1517.
A: Thank you. Now, what's the best way to get in touch with you?
B: By phone—you can ring my landline number 09 5577 5076. I'm sorry I haven't got a cell
phone at the moment.
A: I see...can you give me the details of someone else we could contact if we can't get you during
office hours?
B: That would be my landlady.
A: What's her name?
B: Martha Pearce.
A: Is that Pierce P-I-E-R-C-E?
B: No, it's P-E-A-R-C-E.
A: And her number is?
B: The same as the one I gave you.
A: Yes, I've got that. Now, for some more personal details...What title do you use? Excuse me?
Are you Mrs, Miss or Ms?
B: I'm not married. Put me down as Miss.
A: All right, what's your surname?
B: Farina.
A: What's your first name?
B: Maria.
A: Do you have a second or middle name?
B: Well, two actually.
A: What are they?
B: Rosa Ana.
A: I'm guessing Farina is your maiden name, since you haven't been married, but, are you known
by any other names?
B: Farina is my only surname...but people call me Mary.
A: As a first name?
B: Yes
A: When were you born, Mary? Can you give it to me in the following order: day, month, and
year?
B. The 15th of November 1983.
A: You're obviously female...so that brings me to the last question which I don't really need to
ask.
B: What's that?
A: Your husband's name!
B: I don't have one.
A: I know—so just write down N/A for not applicable'.
B: Thank you.

Exercise 6:

Questions 1 – 2
Circle the correct answer A – C.
1. The festival lasts for
A. 2 days.
B. 3 days.
C. 4 days.
2. It will take place on
A. 14th July.
B. 4th July.
C. 4th August.
Questions 3 – 4
Circle TWO answers A – E.
You can buy tickets from
A. the Town Hall.
B. the festival office.
C. libraries.
D. tourist advice centres.
E. post offices.
Questions 5-7
Write NO MORE THAN THREE WORDS in each space.
5. The weather forecast for the festival is…………………………..
6. Food will be available from……………………………………………
7. There will be a bar selling soft drinks,…………………………………
Questions 8-10
Write ONE WORD in each space.
Name of band Where they come from What kind of music

Petie’s Dozen 8_______________ Jazz

Strings Poland Poland 9_______________

The Fiddlers 10______________ Folk

1. B
2. C
3-4. B, E (in any order)
5. hot and sunny
6. stands
7. beer and wine
8. New Orleans
9. Classical Music/Favourites
10. Ireland

Now, as I'm sure you know, the Red River Festival - that is, the Red River International Music
and Arts Festival, to give it its full name - is on while you are here, and I'm sure I speak for all
the inhabitants of Red River in inviting you most cordially to attend. The festival takes place
over a long weekend - that is, it starts on the Friday afternoon and runs until Sunday
evening. Normally the festival would take place on 4th July, the same day as American
Independence Day. but this year we have rescheduled it for the fourth of August.
Now, you can buy tickets for this festival either by the day or for the whole festival. The second
option is cheaper, although of course not everyone can attend for the whole time. A day's ticket
is $10, and it's $25 for the whole festival. That's very good value. If you want tickets, you are
advised to get them early, because there are always more visitors than tickets. Space is limited,
so buy early! You can get them direct from the festival organisers' headquarters, the
festival office, and I'll give you that address later, or you can get them from any of our
three post offices or one of the many bookshops in the town. Last year we issued them from
tourist advice centres and the Town Hall, but this year it was decided to limit the number of
outlets to cut down on administrative costs. The weather is looking good. The forecasters expect
it to be one of the hottest and sunniest weekends of the year, so it's perfect for the festival,
although I would remind you to cover up and be aware of the dangers of too much sunshine. If it
gets too hot for you, you could always stay inside for some of the indoor events. And of course
you'll be able to get food - from sandwiches and snacks to barbecues, as well as ethnic fast food
— from several stands. There will be a bar this year, but after last year we will only be selling
soft drinks, beer and wine. We have decided not to bother with a spirits licence, there won't be
any whisky on sale. So. what's on? Well, I can only give you a flavour of the many attractions
we have coming this year, but if I can name one of my personal favourites, you must see Petie's
Dozen, a traditional New Orleans jazz band. They were here last year, and were so popular that
we've invited them back. If you like classical music, we've got a string quartet from Poland,
appropriately called Strings, playing classical favourites. We've also got rock bands, a blues
band from the UK, a group of traditional Bavarian beer hail singers, and another of my
favourites, The Fiddlers, who come from Ireland. Their special brand of folk music is popular
all over the world. Moving on, then, to other attractions in the Red River area. For children,
there's lots to do and see, from museums to theme parks.
Exercise 7:
Questions 1-4: Choose the correct letter, A, B or C.
1 The workshop takes place on
A the 5th.
B the 6th.
C the 16th.
2 The maximum number of participants is
A five.
B twelve.
C fifteen.
3 The workshop lasts for
A half an hour.
B one hour.
C over an hour.
4 Each workshop participant must contribute
A £2.
B £5.
C £10.

Questions 5-6: Choose TWO letters A-F.


Which TWO items must participants bring to the workshop?
A a bag
B a container
C floral supplies
D paper
E a penknife
F scissors

Questions 7-10: Complete the booking form below. Write ONE WORD AND/OR A
NUMBER for each answer.

JAPANESE FLORAL ART WORKSHOP BOOKING FORM

PARTICIPANT DETAILS
First name: 7 ...................
Surname: 8 ...................
Contact telephone number: 9 ...................
Email address: 10 ...................@yahoo.com

1. Answer: B
Note You hear: 'No, it's on the 6th - Saturday the 6th'. C is mentio'ned by the information
assistant but Lubna corrects her so this choice would be incorrect.
2 Answer: C
Note The words 'maximum number' paraphrases 'limited to' in the recording. You hear:
'... this workshop is limited to fifteen participants'. B is also mentioned but this refers to
the number of people who have booked already so is not correct.
3 Answer: B
Note You hear: 'I'm really glad it lasts for a full hour – I don't think I would be able to
come up with any kind of floral arrangement in less time than that - and certainly not in
30 minutes!'.
4 Answer: B
Note You hear: 'The workshop itself is free but we're asking participants to pay £5 each -
just to cover the cost of floral supplies'.
5 and 6 Answers: B and F
Note You hear B mentioned: '...please remember to bring scissors or cutters to the
workshop ...' and you hear F mentioned: 'and you'll also need to bring your own
container'. E (a penknife) is mentioned but only as an item that is not appropriate for use
at the workshop. C is also mentioned but is an incorrect answer because floral supplies
are supplied to participants as part of the workshop fee. A is mentioned but is unsuitable.
7 Answer: Lubna
Note The name is spelt out: 'L-U-B-N-A'
8 Answer: Awan
Note The receptionist mistakes the last letter for an 'm'.
9 Answer: 0759 830 5321
Note The phone number is given.
10 Answer: lawan25
Note You have to listen for the first part of the email address so you need to be prepared
to write it quickly. Note that we say dot when giving email addresses.

Receptionist: Good morning, you're through to the Tree house at the Botanic Garden.
How can I help?
Lubna: Oh, Hello, I'd like to book a place on the ... er ... Japanese floral art workshop.
R: Ah, yes, do you mean our workshop on the 16th?
Lubna: No, it's on the 6th - Saturday the 6th.
R: Ah, yes - got it up here on screen now! Japanese Floral Art workshop ... That's great -
you're just in time - we're nearly full! Twelve people have already booked a place and
this workshop is
limited to fifteen participants. It's one of our most popular workshops - in fact it's the fifth
one we've run! And this is the last one this term. There won't be another workshop until
next year now.
Lubna: Oh great, thanks a lot. And can I just check the start time? It says on the leaflet
I've got here that it begins at 12.30.
R: That's right - it finishes at 1.30. Most of our other workshops are only half an hour
long but this one is longer. In fact we're thinking of running some longer ones in the
future.
Lubna: Oh, I see. Well I'm really glad it lasts for a full hour - I don't think I would be
able to come up with any kind of floral arrangement in less time than that - and certainly
not in 30 minutes!
R: Me neither! Now, one more thing - the workshop itself is free but we're asking
participants t.o....p_a_y £5 each - just to cover the cost of the floral supplies.
Lubna: That's fine.
R: Now before I book you a place I'll just give you some information about what'II
happen on the day.
R: Basically, it's a real hands-on workshop so you'll be making your own floral
arrangement that you can take home afterwards.
Lubna: That's great - I'm really looking forward to that.
R: Now, a couple of important things - please remember to .b.ring scissors or cutters to
the workshop - unfortunately we only have a limited number of pairs to lend people who
forget them on the day. Last year, some participants ended up using penknives which are
not at all suitable for floral arranging!
Lubna: Right - I'll make a note of that.
R: And you'll also need to bring your own container.
Lubna: Do you mean a bag - that sort of thing?
R: No, you need a shallow container - basically it has to be shallow with a wide base so
that you can use it to work on your arrangement. It can be pottery, wood, plastic,
whatever you like.
Lubna: Oh, of course. I see. I'm sure I can find something suitable.
R: And when you arrive for the workshop - ask for Elizabeth McMillan. She's leading the
event - she's a really experienced workshop leader. She's been running the event for us
for many years so you'll certainly be in good hands!
Lubna: That's good to know.
R: Now I'll just take your details for the booking form. Can I have your first name?
Lubna: Yes, it's Lubna.
R: Lubna - can you spell that for me?
Lubna: L-u-b-n-a.
R: And your surname?
Lubna: Awan.
R: Is that A-w-a-m?
Lubna: No, it's an 'n' at the end.
R: Thank you. And a contact number - just in case we need to get in touch with you
before the workshop?
Lubna: Yes, it's 0759 830 5321.
R: Thanks, that's all the information we need so that's you booked in. Oh, actually, just
one more thing ... would you like to be on our mailing list? This means that we can send
you information about future events and workshops. All we need is your email address.
Lubna: Yes, that's great. My email address is Lawan25@yahoo.com
R: OK, I'll just read that back - L-a-w-a-n 25 at yahoo.co.com
Lubna: Yes, that's correct.
R: OK, I'll pop all your details on the system and we'll see you at the workshop next
weekend.
Lubna: Thanks a lot for your help. Bye!

Exercise 8:

1. How many members does the cycling club have currently?

2. How much does Youth Membership cost?


3. From whom must you get a signature when applying to join?

4. How long does it take to process a membership application?

5. How often do family rides take place?

6. How long are the Saturday rides usually?

7. What must you get for your bike?

8. When is the next camping tour?

9. What is happening on May 5th?

10. How much discount do members get at Wheels Bike Shop?

1. 76

2. £30 per year

3. teacher or parent
4. 3 weeks

5. every month

6. 60km

7. (a) safety certificate

8. (on) July 14

9. (a) picnic

10. 15%

Man: Cuxford Cycle Club.


Woman: Oh, yes, hello. l'm calling to enquire about joining the club.
Man: Fine. What would you like to know?
Woman: Well, I wanted to get a picture of what the club is like. For example, how big it is.
Man: Now that changes each year, of course. Cycling is growing in popularity ...
Woman: I'm sure
Man: So, last year we had nearly 70 members, which was a record in itself, but this year there are
76 people on the list and I'd say at least 60 of them come to events regularly. I should think
something like 85 is a likely figure by next year.
Woman: That's bigger than I expected.
Man: Yes, there are plenty of opportunities to meet people.
Woman: And how much does it cost to join?
Man: It depends. £40 is for standard members, and there are reductions for certain categories.
For example, Veteran and Youth members pay £10 less, £30, and family membership works out
at £25 per head. All those charges are per year.
Woman: And Youth means?
Man: Under 18.
Woman: Oh, that covers me - at least at the moment!
Man: Then, for safety reasons, your application will need to be endorsed, so your teacher or
parent needs to sign your form.
Woman: No problem. So, what happens after rye sent the form in to you?
Man: We deal with it and get a confirmation of acceptance with a membership card out to you in
3 weeks and then you're ready to ride. It lasts a year and we send you a renewal one month
before it's due to expire.
Woman: OK. And can you tell me something about the activities you do?
Man: Yes, there's a range of things, to reflect the varied membership. There are the family rides,
which are pretty popular, held every month ... and that might get increased to every two weeks.
Woman: I don't know if that's really for me.
Man: Mm, you might prefer the Saturday rides, which are more popular with the Youth
members. We don't go huge distances, 100km or anything like that; 60 km is about average ...
But the pace is fairly brisk.
Woman: Let's hope I'd be able to keep up!
Man: Oh, actually, there's something I should have mentioned before. We've got to be sure
everyone's bike is roadworthy, so you'll need to have your bike checked and obtain a safety
certificate for it, Most bike shops'll do that for you.
Woman: Fine. Do you do any longer tours, like holidays?
Man: Yes, there's a camping tour at least twice a year, There's one on July 14, though it'll get
booked up very soon. If you miss that, then there's another on August 17th
Woman: Oh, good.
Man: But obviously there's plenty going on before then. You might want to come along on May
5th, Your membership should be through by then, and that's when we have a picnic. Everyone
brings some food to share, and we go out to the hills and eat there.
Woman: That sounds fun. I'm going to fill in my form as soon as I get off the phone.
Man: And a further benefit of membership is the discount with Wheels.
Woman; The shop on Mill Road?
Man: Yes, the manager's a member of the club, and he'll give you a 15% reduction it means
membership can pay for itself.
Woman: Great. Well, you'll be getting my form soon. Man: Good, I look forward to meeting
you...

Exercise 9.

INCIDENT REPORT
Questions 1 – 10

Complete the notes below.

Write ONE WORD AND/OR A NUMBER for each answer.

Name: Anna Lumley

Telephone:                        1…………….

Date of arrival:                 2…………….

Address:                            235 3…………… Road, East sea

The total value of insurance: $ 4…………….

Missing items:

– lamps and chairs (not expensive)

– furniture and 5…………….

– a rocking horse, some 6…………. and fruit bowls

Items ordered: – a clock

               – a 7……………

Damaged items:    – the 8………….. needs to be replaced

                                  – a 9…………… of one of the dining chairs is split

                  – four 10……….. were broken

1 077876345

2 27(th) February/Feb.
3 Akendale

4 3,450

5 books

6 paintings

7 mirror

8 desk

9 leg

10 plates

Hello, this is the complaints centre of the service department at Clifton Antiques. Before we
start, I'll just need to take more details from you, OK?
Sure, no problem.
Well, could you please tell me your full name, madam?
It is Anna Lumley, that's Anna, L-U-M-L-E-Y.
All right, L-U-M-O-E-Y.
Not exactly. The fourth letter is L, not 0.
Oh, sorry. Let me rewrite it... and may I have your contact number that we can use to reach you
during the week?
Sure, my mobile phone number is 0-double 7-8-7-6-3-4-5.
Great. Now, what can I do for you today, Anna?
I produced an order of a large quantity of items from you last week on the 20th February, and
has been expecting them to arrive at the office in a week. However, only half of the shipment
has been delivered. 1 just would like to ensure that they haven't been missing in transit.
All right, madam. It's a pity that we brought the inconvenience to you. I'll look for the parcel
track record and see if I can give you a date when it is okay to receive the rest of your orderings.
What was your delivery address listed?
I made the arrangements for the parcel to be conveyed from your warehouse on Ardale Road to
my office address at 235 Akendale Road. Sony, could you say it again, the address? 235
Akendale Road. A-K-E-N-D-A-L-E.
In east sea?
Yes, sir. Well, OK. Our system has tracked your parcel, which shows that your shipment has
been received; however, there's no record about the lost items. I would suggest that you wait for
two days, and if the other items don't arrive then you might necessarily consider to claim
insurance coverage for the value of the rest.
How much are them worth?
Yet it is necessary to make a list of your lost items so we can recheck it against our records, is
that ok?
Of course, okay.
It is unfortunate that a few missing items arc the sole one of a kind and thus, irreplaceable. Also,
there were many small items that aren't very valuable, like lamps and chairs. However, there was
a large item of antique furniture and a bag filled with first edition books, which were some of the
first ever to be printed on a press.
Right, ok. Anything else?
Yeah, there were also some important items, a Victorian rocking horse for my daughter, some
large oil paintings originating from the Edwardian period and some few decorative fruit bowls.
Right, ok. I've taken down the list of these missing items, so I'll call the warehouse to see if any
of them are still there.
Ah, I almost forgot! There are several other pieces that I've spotted on your website and wanted
to order. May I do that now?
It is definitely okay. Would you describe me a little about the items that you are looking to buy?
There is a gold clock and a golden framed vintage mirror. Ok, perfect. The charge will be taken
from payment card that you used before, you can expect them to be delivered within the next
week.
Is there anything else that I can do for you today?
Yes, I received two damaged items in the shipment, so I need to claim for a partial refund. Oh,
sorry. I need to know more about the details of the actual damage over the phoneline before you
put in a full report.
What kinds of pieces are damaged?
A drawer is missing from the antique mahogany desk, and I also spotted a dent on one of the
corners, so it's basically unusable.
I see. Do you know how much will it cost to repair it?
No. Well, I don't think it's repairable. I will have to buy a new one.
Sorry. take a note of that and see what we can do for compensation. Anything else?
I also purchased a set of dining chairs with navy leather padding. However, the colour is faded
and one of the legs has completely split down the middle.
Ok, are there any other damaged pieces?
Yes, there is a set of Chinese crockery to furnish my dining room table. But when. I opened the
case I found that a cup was lost and that some plates had smashed- four, actually. Receptionist:
And is that all of the items?
Yes. Think that's all.
Right, I'll estimate the value of the damaged and a refund will be issued.
Ok, cheers for your great help. Receptionist: No problem, my pleasure. Goodbye!
Exercise 10.
Question 1-7 choose letter A, B, C
1 The librarian says that training always includes
A computer skills.
B basic medical skills.
C interpersonal skills.
2 All library service volunteers have to
A record their arrival and departure.
B stay within ‘staff only’ sections.
C wear a uniform.
3 The woman would be entitled to a contribution towards the cost of
A transport by minibus.
B parking at the library.
C public transport.
4 One recent library project involved
A labeling historical objects.
B protecting historical photographs.
C cataloguing historical documents.
5 At present. the library is looking for people to
A record books onto CD.
B tell stories to children.
C read books to the blind.
6 The woman says she is interested in a project involving
A taking library books to people in hospital.
B delivering library books to people at home.
C driving the disabled to the library.
7 The woman agrees to work
A two hours per week.
B four hours per week.
C six hour’s week.
Questions 8-10. Choose THREE letters A-G. Which THREE of the following must be
provided by all volunteers?
A             civil conviction check
B             signed copy of commitment
C             certificates to indicate qualifications
D             emergency contact information
E              date of birth
F              signature of parent or guardian
G             referees

Woman:    Hmm .. I'm interested in doing some work for the library - are you the person
to speak to?
Librarian:    Yes Right, well, erm, what sort of work are you interested in?

Woman:    I've just come to live here in Australia I don't want a full-time job until my
children have settled down, but t really need to get out of the house a bit, and l heard
you need voluntary workers for various projects...  

Librarian:    Right.

Woman:      but I don't know if I have the right skills.

Librarian:    Well, we do provide training

Woman:    Oh.

Librarian:    We always include an orientation to the library, together with emergency
procedures, that's fire regulations, emergency exits, first aid. So you can cope with
accidents or sudden illness, things like that which are necessary for anyone who’s
working with the public. Then we give specialist training for particular projects - like
using our database system.

Woman:    I do have quite good computer skills, in fact.

Librarian:    Umm Great!

Woman:    Is there any sort of dress requirement?

Librarian:    Well, all staff have to wear a name badge so they can be identified if they
go outside the ’staff only' areas. But apart from that there aren't many regulations - we
ask you to sign in and sign out for insurance purposes, but that's all. How about
transport do you live locally?

Woman:    Not too far away I'm at Porpoise Beach. My husband needs the car during
the day but it's only about twenty minutes on the bus.

Librarian:    In fact, we can reimburse part of your travel expenses in that case.

Woman:    Oh Would that be the same if I came by car?

Librarian:    No, because parking is such a problem here. One thing we are looking for
though is someone who can drive a minibus.

Woman:    No problem So. do the projects involve going outside the library?
Librarian:    Some, yes. But not all. We’ve just finished one which involved working with
photographs taken of the area 50 or 100 years ago it basically involved what we call
encapsulation ..

Woman:    Putting them in some sort of covers to keep them safe?

Librarian:    Exactly, it’s time-consuming work, and we were very grateful to have help
with it. Then, sometime next year we're hoping to begin working on an initiative involving
the sorting and labelling of objects relating to local history. We'll be needing help with
the cataloguing.

Woman:    I'd definitely be interested. How about at present?

Librarian:    Well, we have a small team who work to support those who are unable to
read.

Woman:    Working with the blind.

Librarian:    Yes, or other groups who have reading difficulties. We provide volunteers
with equipment so that they can take books home with them and read them aloud onto
CDs. We're gradually building up a collection that can be lent to those who need them

Woman:    Mmm. I can see it would be useful, but I'd really like to do some sort of work
where I can get the chance to meet people. How about reading stories to children?

Librarian:    Mmm. That's done by our regular staff. But we do have another project -
it's a very long established scheme which involves helping those who are unable to
have direct access to the library.

Woman:    Oh. I noticed someone with a trolley of books when i was at the hospital last
week. That sort of thing?

Librarian:    That would have been one of ours, yes. It’s one of our most popular
services - lots of people who wouldn't dream of going to the library normally, when
they're at home, borrow a book when the trolley comes round the ward.

Woman:    I can imagine. Yes, I'd definitely be interested in that. Right, so how do I
enroll?

Librarian:    Well, we do ask all volunteers to commit themselves to a regular period


each week.

Woman:    I could probably do five or six hours.


Librarian:    Oh ... be careful not to take on too much - but we do need someone for a
couple of afternoons from 2 to 4 ... so four hours altogether.

Woman:    That sounds fine.

Librarian:    Right, so here's the application form . .. it asks the usual questions, name
and address and telephone number. You also need to fill in details of who we should get
in touch with in case of any accident or problem like that, we do need to have that filled
in, and there's a space for date of birth, but that’s only if you're over 75 so. we won't
worry about that.

Woman:    No. Oh. it asks for qualifications do I need to provide certificates?

Librarian:    They're not necessary. We'll need the names of two referees not relatives
or family members, obviously. What else . . signature of parent or guardian - that won't
be necessary as i assume you're over 18?

Woman:    Yes. What's this? it says 'civil conviction check'

Librarian:    That's a document we have to provide by law for those working on projects
involving children, so we’ won’t need it in your case. But you will need to sign this
separate document that’s a copy of commitment, it's basically an agreement to work
according to the library guidelines. So if you'd like to fill this all in - you can do it here, or
take it home, whichever you prefer.

Woman:    I'll take it home if that’s OK. Right, well thank you for your time ...
B.- SECTION 2
Exercise 11.
Questions 11 and 12: Choose TWO letters, A-E. Which TWO things are included in the
price of the tour?
A garden gloves
B ladybugs
C bush timbering lessons
D food
E hummingbirds
Questions 13 and 14: Choose TWO letters, A-E. Which TWO facilities of Pine Garden are
open today?
A plant care centre
B cafe
C gift shop
D model town
E tourist office
Questions 15-20 : Choose SIX answers from the box and write the correct letter, A –H.
A varieties of desert
B edible plants
C lawns and lawn alternatives
D native plants
E storing water
F plants attracting wildlife
G unified design
H soil nutrients
15 Mary …………….
16 Berson …………….
17 Smith …………….
18 Nunee …………….
19 Acanlan …………….
20 Mandelson …………….

11-12 B/C
13-14 A/D
15 D
16 A
17 B
18 C
19 H
20 G

Hello, welcome you all to Pine Garden. My name is Manuel. Before you wander off and begin
your exploration of the garden, I am going to keep all of you informed of several things about
this building. I know you are eager to start your wandering and exploring, so I will try to keep
this as short as possible.
At the very first, I think I should explain to you something that you can do with your ticket. If
you would like to be much more into nature, the optimal section is our planting area, where all
the visitors can plant small flowers and bulbs on their own. These plants will gradually grow and
then become part of our garden. The activities of planting are totally free. However, if your
hands are sensitive, we strongly recommend you to buy a pair of garden gloves in order to
protect your skin.
Also, here at Pine Garden, we use wooden materials from the trees that have been felled in our
very own pine forest to make carved goods! If you are interested and want to get involved and
try by yourself, you can join one of our bush timbering lessons for free, where you will have
the opportunity to make your own key ring with the help of a skilled craftsman.
Our aviary is the most popular attraction, where you can see a whole range of bird species. More
surprisingly, it is free to enter this section, yet you should pay a small amount of supplement
for the entry to the hummingbird section. Also, the insect section that is not far from the
aviary might arouse your interest. There you will find a number of interesting insects, such as
butterflies, pocket ladybugs, dragonflies and so on, and no extra fee for it will be paid.
Unfortunately, some areas are now temporarily limited to visitors today. For example, the gift
shop that has been closed earlier this year will remain out of boundary for another month
or so. As I have said before, the restaurant still offers free food and snacks for you, and if you do
feel like purchasing a gift, why not buy that special potted bush or orchid from our plant
care centre? What's more, our new treetop cafe is now in the process of construction; it will
be very compelling when it's finished. Actually, our model town has already opened in
advance, and that is such great interest to the public. Also, our tourist office is ordinarily
available to give tourists many aids. But the officer is sick at home. Please do not be
disappointed by this, since our opening visiting areas also provide quite an experience!
I'd also like to introduce our plant experts responsible for the wonderful plant exhibitions here at
Pine Garden, Mrs. Mary is one of our specialists, who is personally in charge of our awesome
displays that can all be found in the local wild nature. Mr. Berson is responsible for looking
after some varieties of plants that grow in much drier and hotter climates than ours with
difficult tasks, which means there is no need to conserve much water for it on site. If you go into
the glass house, there are a large number of plants that he has managed to grow without any need
for raining or irrigation. Mr. Smith is in charge of keeping all the visitors fed at our
restaurant, which contains some of the varieties that grow under the ground and those that
on trees and bushes. Now, Mr. Nunee here is our specialist on the most universally growing
plant in the world: grass! You may have noticed how beautifully green and lush our grounds
are thanks to his specialist knowledge. Mr. Acanlan guarantees our soil is compiled with
nutrients; all the specialists' habitat is hence supported and encouraged. He succeeded in doing
this by fertilizing the earth with his special formula that originally constructed by himself. At
last, I'd like to invite you all to meet Dr. Mandelson, the manager of our landscaping team,
who works closely with all other experts to make sure everybody works together to create a
landscape that is pretty as well as sustainable.
Well, that just about rounds it up. Now, if anyone has any questions...

Exercise 12:
Questions 11 and 12
Which TWO activities that volunteers do are mentioned?
A     decorating
B     cleaning
C     delivering meals
D     shopping
E     childcare
Questions 13 and 14
Choose TWO letters, A-E.
Which TWO ways that volunteers can benefit from volunteering are mentioned?
A     learning how to be part of a team
B     having a sense of purpose
C     realising how lucky they are
D     improved ability at time management
E     boosting their employment prospects
Questions 15-20
What has each of the following volunteers helped someone to do?
Choose SIX answers from the box and write the correct letter, A-G, next to Questions 15-20
What volunteers have helped people to do
A     overcome physical difficulties
B     rediscover skills not used for a long time
C     improve their communication skills
D     solve problems independently
E     escape isolation
F     remember past times
G     start a new hobby
Volunteers
15   Habib                   ………………
16   Consuela              ………………
17   Minh                    ………………
18   Tanya                   ………………
19   Alexei                   ………………
20   Juba                      ………………

11&12. A, E
13&14. B, E
15. F
16. A
17. E
18. G
19. D
20. C

SECTION 2
Good morning. My name’s Lucy Crittenden, and I’m the Director of Operations for an
organisation that arranges volunteering in this part of the country. I’m hoping I can persuade one
or two of you to become volunteers yourselves. Let me start by briefly explaining what we mean
by volunteering.
Volunteers are teenagers and adults who choose to spend some time, unpaid, helping other
people in some way. Most volunteers devote two or three hours to this every week, while a few
do much more. The people they help may have physical or behavioural difficulties, for example.
Volunteers can do all sorts of things, depending on their own abilities and interests. If they’re
supporting a family that’s struggling, for example, they may be able to give them tips on
cooking, or recommend how to plan their budget or how to shop sensibly on their income. They
might even do (Q11) some painting or wallpapering, perhaps alongside any members of the
family who are able to do it. Or even do (Q12) some babysitting so that parents can go out for a
while.
The benefit from volunteering isn’t only for the people being helped. Volunteers also gain from
it: they’re using their skills to cope with somebody’s mental or physical ill health, and
(Q13) volunteering may be a valuable element of their CV when they’re applying for jobs:
employers usually look favourably on someone who’s given up time to help others.
Significantly, most volunteers (Q14) feel that what they’re doing gives them a purpose in
their lives. And in my opinion, they’re lucky in that respect, as many people don’t have that
feeling.
——————–
Now I’d like to tell you what some of our volunteers have said about what they do, to give you
an idea of the range of ways in which they can help people.
Habib supports an elderly lady who’s beginning to show signs of dementia. Once a week they,
along with other elderly people, go to the local community centre, where a group of people come
in and sing. The songs (Q15) take the listeners back to their youth, and for a little while they
can forget the difficulties that they face now.
Our volunteer Consuela is an amazing woman. (Q16) She has difficulty walking herself, but
she doesn’t let that stop her. She helps a couple of people with similar difficulties, who had
almost stopped walking altogether. By using herself as an example, Consuela encourages
them to walk more and more.
Minh visits a young man who lives alone and can’t leave his home on his own, so he hardly
ever saw anyone. But together (Q17) they go out to the cinema, or to see friends the young
man hadn’t been able to visit for a long time.
Tanya visits an elderly woman once a week. When the woman found out that Tanya is a
professional dressmaker, she got interested. Tanya showed her some soft toys she’d made, and
(Q18) the woman decided to try it herself. And now she really enjoys it, and spends hours
making toys. They’re not perhaps up to Tanya’s standard yet, but she gains a lot of pleasure from
doing it.
Alexei is a volunteer with a family that faces a number of difficulties. By calmly talking over
possible solutions with family members, he’s helping them to realise that they aren’t helpless,
and that (Q19) they can do something themselves to improve their situation. This has been
great for their self-esteem.
And the last volunteer I’ll mention, though there are plenty more, is Juba. She volunteers with a
teenage girl with learning difficulties, who wasn’t very good at talking to other people. Juba’s
worked very patiently with her, (Q20) and now the girl is far better at expressing herself, and
at understanding other people.
OK, I hope that’s given you an idea of what volunteering is all about. Now I’d like …

Exercise 13.
Induction talk for new apprentices
Questions 11 and 12
Choose TWO letters, A-E.
Which TWO pieces of advice for the first week of an apprenticeship does the manager give?
A     get to know colleagues
B     learn from any mistakes
C     ask lots of questions
D     react positively to feedback
E     enjoy new challenges
Questions 13 and 14
Choose TWO letters, A-E.
Which TWO things does the manager say mentors can help with?
A     confidence-building
B     making career plans
C     completing difficult tasks
D     making a weekly timetable
E     reviewing progress
Questions 15-20
What does the manager say about each of the following aspects of the company policy for
apprentices?
Write the correct letter, A, B or C, next to Questions 15-20.
A     It is encouraged.
B     There are some restrictions.
C     It is against the rules.
Company policy for apprentices
15   Using the internet          …………….
16   Flexible working              …………….
17   Booking holidays            …………….
18   Working overtime           …………….
19   Wearing trainers             …………….
20   Bringing food to work    …………….

11&12. A, C (IN ANY ORDER)


13&14. B, E (IN ANY ORDER)
15. B
16. B
17. C
18. A
19. A
20. C

Good morning everyone. My name’s Janet Parker and I’m the human resources manager. We’re
very happy to welcome you to your new apprenticeship. I hope that the next six months will be a
positive and enjoyable experience for you.
I’d like to start with some general advice about being an apprentice. Most of you have very little
or no experience of working for a big organisation and the first week or so may be quite
challenging. There will be a lot of new information to take in but don’t worry too much about
trying to remember everything. The important thing is to (Q11) check with someone if you’re
not sure what to do – you’ll find your supervisor is very approachable and won’t mind
explaining things or helping you out. You’re here to learn so make the most of that opportunity.
You’ll be spending time in different departments during your first week so make an effort to
(Q12) talk to as many people as possible about their work – you’ll make some new friends and
find out lots of useful information.
As well as having a supervisor, you’ll each be assigned a mentor. This person will be someone
who’s recently completed an apprenticeship and you’ll meet with them on a weekly basis. Their
role is to provide help and support throughout your apprenticeship. Of course, this doesn’t mean
they’ll actually do any of your work for you – instead they’ll be asking you about (Q13) what
goals you’ve achieved so far, as well as helping you to identify any areas for improvement.
You can also (Q14) discuss your more long-term ambitions with them as well.
————————–
Now I just want to run through a few company policies for our apprenticeship scheme with
you… Most importantly, the internet. As part of your job you’ll be doing some research online
so obviously you’ll have unlimited access for that but please (Q15) don’t use it for personal
use – you’ll have your own phones for that.
Some of you have already asked me about flexible working. After your probationary three-month
period – some of you will be eligible for this – but (Q16) it will depend on which department
you’re in and what your personal circumstances are. So please don’t assume you’ll
automatically be permitted to do this.
I want to make sure there’s no confusion about our holiday policy. Apart from any statutory
public holidays (Q17) we ask that you don’t book any holidays until after your six-month
apprenticeship has finished. Time off should only be taken if you are unwell. Please speak to
your supervisor if this is going to be a problem.
You’ll be expected to work a 40-hour week but there may be opportunities to do overtime during
busy periods. Although you’re not required to do this, (Q18) it can be a valuable experience –
so we advise you to take it up if possible. Obviously, we understand that people do have
commitments outside work, so don’t worry if there are times when you are unavailable.
As you know, we don’t have a formal dress code here – you may wear casual clothes as long as
they’re practical – and the only restriction for shoes we have is on high heels for health and
safety reasons. (Q19) Comfortable shoes like trainers are preferable.
There’s a heavily subsidised canteen on site where you can get hot meals or salads cheaply.
Snacks and drinks are also provided – so (Q20) we’ve decided to introduce a no packed lunch
policy. This is partly to encourage healthy eating at work and partly to stop people from eating at
their workstation, which is unhygienic.
OK moving on to …

Exercise 14.

Questions 11-16
What advantage does the speaker mention for each of the following physical activities?
Choose SIX answers from the box and write the correct letter, A-G, next to Questions 11-
16.

Advantages
A not dependent on season
B enjoyable
C low risk of injury
D fitness level unimportant
E sociable
F fast results
G motivating

11. using a gym


12. running
13. swimming
14. cycling
15. doing yoga
16. training with a personal trainer

Questions 17 and 18
Choose TWO letters, A-E
For which TWO reasons does the speaker say people give up doing the gym
A. lack of time
B. loss of confidence
C. too much effort required
D. high costs
E. feeling less successful than others

Questions 19 and 20
Choose TWO letters, A-E
Which TWO pieces of advice does the speaker give for setting goals?
A. write goals down
B. have achievable aims
C. set a time limit
D. give yourself rewards
E. challenge yourself
11. F
The first question to ask yourself is whether you would enjoy training in a gym. Many people are
put off by the idea of having to fit a visit to the gym into their busy day - you often have to go
very early or late as some gyms can get very crowded. But with regular training you’ll see a big
difference in a relatively short space of time.
12. D
Running has become incredibly popular in recent years. That’s probably got a lot to do with the
fact that it’s a very accessible form of exercise - anyone can run - even if you can only run a
few metres to begin with.
13. A
Swimming is another really good way to build fitness. What attracts many people is that you can
swim in an indoor pool at any time of year.
14. B
Cycling has become almost as popular as running in recent years. That’s probably because as
well as improving their fitness, many people say being out in the fresh air in a park or in the
countryside can be fun, provided the conditions are right, of course - only fanatics go out in the
wind and rain!
15. C
Yoga is a good choice for those of you looking for exercise which focuses on developing both a
healthy mind and boby. It’s good way of building strength and with the right instructor, there’s
less chance of hurting yourself than with other more active sports.
16. G
Getting a personal trainer is a good way to start your fitness programme. Obviously there can be
significant costs involved. But if you’ve got someone there to encourage you and help you
achieve your goals, you’re less likely to give up.
17-18. B, C
I don't think people suddenly stop caring about improving their fitness, or decide they have more
important things to do. I think people lose interest when they don't think they're making
enough progress. That’s when they give up hope and stop believing they’ll ever achieve their
goals. Also, what people sometimes don’t realise when they start is that it takes a lot of
determination and hard work to keep training week after week and lots of people don’t have
that kind of commitment.
19 -20. B, D
One thing you can do to help yourself is to set manageable goals - be realistic and don’t push
yourself too far. Some people advise writing goals down, but I think it’s better to have a flexible
approach. Give yourself a really nice treat every time you reach one of your goals. And don’t
get too upset if you experience setbacks - it’s a journey - there are bound to be difficulties along
the way.

Exercise 15.
Questions 11-17
Complete the sentences. Write ONE WORD ONLY for each answer
It is important for everyone to know simple first aid 11…………..
After an accident, one must firstly be aware of 12…………..
After that, the first-aider must 13…………..
Clearing of airways may not happen when patients are 14…………..
The mouth must be checked for 15…………..
CPR must be done if breathing is absent or 16…………..
CPR must be done up to the arrival of 17…………..
Questions 18-20
Choose THREE answers from the list and write the correct letter, A—G, next to the
questions. Which THREE pieces of advice does the first-aid officer say are most
important?
A. Have proper equipment
B. Give regular first-aid training
C. Have a safety officer
D. Instil safe behaviour
E. Put posters on walls
F. Have safety meetings
G. Have first-aid boxes

11. steps
12. danger
13. respond
14. unconscious
15. blockages
16. irregular
17. medics
18-20. C,D,F

Hello, everyone. Now, you know why I'm here. You all work in a factory, and in this
environment, there always remains a significant possibility of accidents happening, in which
case, first aid will be necessary. What is first aid? It is the provision of emergency on-site care
when an injury occurs, and it is essential for everyone to know, if only simply, the steps which
must be followed.

There are, of course, minor injuries which may happen, not needing further medical care beyond
the intervention of the first-aider, but you can never be sure, thus the following steps must
always he followed. These can be abbreviated to the words. 'Dr ABC"— in other words. D — R
— A —B—C . The 'D' stands for "danger", and that's the first issue to keep in mind. When an
accident happens, immediately ascertain that the environment is safe — that, for example,
nothing else will fall or break or cause accidents. If you, the first-aider, are also injured, the
problem is even worse. The 'R' stands for "respond'. You must then ascertain the best response.
Once all the danger has been eliminated, and the distress calls sent out, the appropriate action is,
obviously, to help the injured party.

Alright, that leads to 'A', which stands for "airway". In order to stay alive, all people need to have
an open airway to allow breathing. A conscious person will automatically clear their own airway,
but if unconscious, this may not happen. The brain is stopped or hindered from properly
directing the body and, obviously, in the worst case, death can result, for this reason, the injured
person is normally put into the 'recovery' position — placed on the side, tilling the head back,
and ensuring that there are no blockages in the mouth. To free any such blockages, the back can
be slapped, or the chest compressed, allowing anything to be spat out. Now that the airway is
free, cheek for 'B' or 'breathing'. If breathing is not happening, or is irregular, the fust-aider may
have to assist with what is technically known as cardiopulmonary resuscitation, or more
commonly. C P R. This involves breathing for the patient, through mouth-to-mouth contact,
while periodically massaging the heart through compressions to the chest. This combination
allows blood, and oxygen, to flow around the body, keeping the patient alive, hopefully
until medics, such as doctors, advanced first-aiders, or ambulance staff, arrive.

Now, it’s certainly good to have know ledge of emergency first aid. but. obviously, the best
situation is simply not to have accidents occur in the first place, for that, you need to be aware of
safety issues, but just saying "be aware' does not usually achieve much. It is more important to
have an appointed person whose job is to ensure awareness and work-safety. Safely inspections
would obviously be part of their job, whereby'they can make sure, for example, that the first-aid
boxes are fully equipped. Another idea is to put posters on the walls, but. interestingly, research
has proven that these lend to he ignored, becoming just part of the wallpaper—seen, but not put
into practice. It is much better if everyone is just instilled to not be reckless, that is,to not rush
into situations without thinking about the possibility of accidents, and instilling this mindset is
part of the job of the safety officer. Some other suggestions are ongoing first-aid training, and
‘no accident’ reward or star systems. These have had some success, but nothing beats a regular
meeting, say, once a month, in which the subject of safety is brought to the attention of everyone,
and any outstanding issues related to this are thoroughly discussed.

Exercise 16.
Questions 1-4
Look at the following reasons why different groups use social networks
Match each group with the correct reason, A-E
Write the correct letter, A-E.
1 The elderly
2 Teenagers
3 Mothers
4 Single people

List of reasons
A Loneliness
B Popularity
C Reconnecting
D Entertainment
E Reassurance

Questions 5-10
Complete the table below
Write NO MORE THAN THREE WORDS for each answer.
Social networking Pros Cons Future
site
SoPals 5.______________ Harvests data on users Moving into
Popular in different 6.______________
countries postings.
EverywhereUs A network for 8.______________ is Continue to grow
7.______________ poor
Sweet Targeted at Childish design Expand to other
9.______________ No app 10.______________
Most media supported

1D
2B
3E
4C
5 Easy to use
6 temporary
7 close friends
8 web version
9 teens
10 demographic
I: And that's the latest news today. Next we're going to be speaking to our technology editor,
Simon Smith. He'll be telling us all about the latest report in social networking that has been
collated from over a million participants around the world. Good afternoon, Simon.
S: Good afternoon, Carol.
I: So, this report
S: Yes, it's the biggest report like this I think we've ever seen, and it certainly raises some
questions for social media sites. And indeed for us, the people that use them.
I: I hear that its tapped into the motivations we have for using social media. What can you tell us
about this?
S: It has. There were over 50 different groups surveyed and there were some interesting
discoveries. For example, senior citizens, who you would think use social networks like
Facebook to make friends and avoid being lonely, actually used them to do things like look
at articles and watch videos. This group of people actually spends the most time on the Internet
too, which is in stark contrast to the situation ten years ago when a lot of older people weren't
even online. The research also found that young people tend to use social networks for
reasons relating to social acceptance - simply, they want to stay 'in' with their friends. This
group tends to switch sites a lot, so Facebook might be popular for a short amount of time. but
they'll move on to other sites like Instagram or Twitter pretty swiftly. Loyalty isn't a big feature
of this group. If we look at mothers however, we can see a completely different story. Their
loyalty is actually pretty high, and they tend to form close bonds with people in their online
community. People that they haven't actually met. This is not simply to allay the isolation of
staying at home with a small child, which is what we might suppose. Actually, it's to find out
whether they are doing the right thing when it comes to caring for their young children or indeed
their teens. These sites can be extremely supportive tools. Lastly, let's look at another group
which showed surprising motivations for using social networks; people who are not in
relationships. This group of people, again, you may think are quite lonely, but statistics
show they post more and go out more than any other group. In fact, often their motivation
is to get back in touch with people they've met. Perhaps from school or just from their busy
social lives.
I: That's very interesting and not at all what I would have thought. How do you think this
information will affect social networking sites at the moment? Do you think we'll see some
changes?
S: Yes, I think so. I've been looking at some of the most popular sites at the moment, SoPals,
EvermhereLls and Sweet. They all have their advantages and disadvantages but I can see all
three of them going in different directions in the future, Lets talk about SoPals firstly, as it's
probably one of the most well-known social networks around at the moment. And why is it so
popular? Well, I think it's pretty easy to use and also it's seen a good uptake in lots of different
countries. I think nearly every country in the world has access to it and there are millions of
international users. However, the one thing many people are not at all happy about is the way in
which SoPals quite brutally collects user data. There has been some talk in the press about who
can access this data, and although the wider community might not be too bothered, there are
some rights groups who are outraged, On the other hand, they also have quite a good strategy for
going forwards. They are going to develop posts which are more temporary. So, say, they'll stay
on for an hour and are then archived. This is something which is increasingly popular for
younger users, so it sounds like a good move to me. Now, moving onto EverywhereUs ... This is
more of a context-specific social networking website_ One of its best points is that it's geared
towards close friends. This means that people are more likely to share more things. The app has
been a great success. However, we can't say the same for the web version, which has had some
dreadful feedback, They need to work on this in my opinion, but I don't think it will stop this site
from getting bigger and biker in the future. One final site that I do want to mention is Sweet
Now, this isn't really a site for the likes of me and you Carol. It's actually a social network for
teens. Now, the reason why this site is becoming so popular for this age group is that it supports
so many different forms of media. So, you can post audios, pictures, videos, little voice
messages, games. It's really booming with this age range. It's not all perfect though_ I've got to
say that the design is quite simplistic and child-like. even for the age range. And also, there's no
app yet. They'll want to get that developed if they want to be a big player in this market, And „,
they obviously do want to be a big player as they're planning next year to branch out to other
demographics, such as young parents and older people. I can see this being a really successful
move if they get the design spot-on. One thing I will say though is that social networking doesn't
appear to be going anywhere. It looks to just grow and grow.

Exercise 17.
Questions 11–14
Which counsellor should you see?
Write the correct letter, A, B or C, next to questions 11–14.
A Louise Bagshaw
B Tony Denby
C Naomi Flynn
11) if it is your first time seeing a counsellor
12) if you are unable to see a counsellor during normal office hours
13) if you do not arrange an appointment
14) if your concerns are related to anxiety
 
Questions 15–20
Complete the table below.
Write NO MORE THAN TWO WORDS for each answer.
Workshop Content Target group

what you need to succeed


Adjusting academically 15 ………………… students

use time effectively, find 16


………………… between study and
Getting Organised leisure all students

talking with staff, communicating all students, especially


Communicating across cultures 17………………… students

ways to relax, breathing techniques,


Anxiety 18…………………, etc. students about to sit exams

19……………… 20 ………………… students


… staying on track for long periods only

11. C
12. C
13. A
14. B
15. first/1st year
16. (optimal) balance
17. international/foreign
18. meditation
19. motivation
20. research

Good morning, boys and girls. I'm Steven, working as the counselling administrator at College's
counselling administration. Today I would like to talk with you about the counselling team of the
school and the services you can be offered.

There are now three professional counsellors in our team here at St. Court. They are Louise
Bagshaw, Tony Denby and Naomi Flynn. Each of them holds regular one-on-one sessions with
students, but you cannot start counselling with them until you should make an appointment with
Naomi Flynn first. Naomi is an expert in meeting freshmen and delivers a preliminary session in
which she will tell ou what you can ex ect from counselling; also she would ask you a few
simple questions related to what you would like to discuss. For those who are feeling a bit of
worried about the counselling steps, this can be indeed helpful. Naomi is also the best choice for
students who can only communicate with a counsellor beyond office hours. She's not in the
office on Mondays, but she will start working on Wednesday mornings and works late on
Thursday evenings, so before your first class or after your last class on those days, you can see
her. Louise is in our drop-in centre office a whole day. If you want to ask some counsellors for
help without a prior appointment,shvill be the optimal one. But do notice that if you choose this
service, Louise will either see you herself, or send you to the next available counsellor. If you
want to see certain counsellor each time when you visit, an appointment in advance is strongly
recommended. Online or at reception during office hours are booking forms now available.

Tony is our latest-joined member of the counselling team. He is the sole male counsellor and has
solid foundation and expertise in stress management and relaxation techniques. Anyone who is
trying to handle anxiety is encouraged to see him. A variety of techniques, like body awareness,
time management and positive reinforcement will be introduced to you by Tony to help you
address this problem.

Each term, there will be some small team workshops operated by the counselling team, which
last for two hours and are all free to the enrolled students.

The first workshop we offer is Adjusting. For some people, college education is found to be a big
shock. It tends to be simple to get lost after the structured learning surroundings of school.
Therefore, what is essential for academic success in this workshop will be shared. As anticipated,
this offering is targeting first-year students.

Getting organised is the service that the second workshop offers, where we are active to motivate
you to break off the habit of putting things off. We will try to help you get the most out of your
time and find out the optimal balance between academic and recreational activities. In this
workshop, we're catering to a broader crowd, ranging from undergraduates to postgraduates.
The next one is a Communication workshop. If you've come from overseas, the way people
interact here may be quite different to what you're used to. This workshop will recommend some
ways to foreign students about handling many situations. For example, they might find
struggling on how to talk with teachers and other staff. It will also cover all aspects of
multicultural communication. International students will learn a lot from this class, so we
particularly encourage you to come along, but I have to say that occasionally local students can
find it helpful as well. Everyone is welcome!

The workshop called Anxiety will be available later on in the year and it will target something
you might be familiar with, that is the nerves and the anxiety brought by the coming exams. Lots
of students experience their entire academic careers like this, but surely there is a way to solve
problems. Welcome to Anxiety workshop and you could learn several ways of relaxation and the
proper way to breathe, as well as meditation and other methods to keep calm. This workshop is
designed for everyone who is going to take exams.

The last workshop we have is the Motivation workshop. The theme for this workshop is how to
stay on target and be motivated during the long-run projects. This workshop is only available for
research students. Less-advanced students already have some workshops dealing with their
needs. Well, that's it, thanks for listening. If you would like to learn more information about our
services, do visit us at Counselling Service.

Exercise 18.
Questions 11 and 12
Choose TWO letters, A-E. Which TWO things happened during the presenter's absence?
A Surprise guests were on the show.
B Tony replaced him for good.
C Radio Western got a new manager.
D Radio Western invited some audience to come.
E Listeners wrote to the radio.
Questions 13 and 14
Choose TWO letters, A-E. Which TWO of the following descriptions are about the presenter's
holiday?
A friendly local people
B good food
C tedious flight to vacation
D gorgeous weather
E disappointed children
Questions 15-20
Who is concerned about each of the following shows or musical genres? Write the correct letter,
A, B or C, next to Questions 15-20.
A Clive
B Sally
C John
Shows and musical genres
15 music in traditional and serious style
16 country music
17 music with very strong rhythms
18 interactive live shows through online platforms
19 recorded shows
20 music for specific groups

11-12 C, E (IN ANY ORDER)


13-14 A, B ( IN ANY ORDER)
15 B
16 B
17 B
18 C
19 C
20 A
Hi you’re listening to Macrc Ambrose and I will be with you through to midday on Talk Back,
the show where you tell us what you think about Radio Western. I am sure you have all heard
we have a new boss at Radio Western and she’s very keen to get your feedback. Well,
you've certainly taken her invitation seriously. We’ll be dipping into the postbag for your
feedback in just a moment and speaking to a surprise guest or two later in the show.

I must start by saying a big 'thank you’ to Tony Marsons — judging by your letters. he did a
great job of covering the show last week whilst I was away. Thanks Tony, if you're listening.
And thanks for all the emails asking about my holiday. I had a long, rather boring flight home
late last night but I must say I had a wonderful time. The food was absolutely delicious and the
locals we spoke with were really friendly. Shame about the weather, which was an absolute
washout, but you can't have everything I suppose. And the kids loved it, so everyone was happy.

Anyway, on to the first of your letters ... Sally from Liverpool is very concerned about the
consequences of the cutbacks we've been experiencing here at Radio Western, in particular
whether some of our late-night music shows could be facing the axe.'We're force-fed a great deal
of pop music throughout the day: writes Sally, 'and some of your listeners look forward to the
more niche musical genres you cover in the evening. Are these in danger when you rearrange the
schedule?' Not at all, Sally. In fact look out for one or two exciting new shows over the coming
weeks. We've got a brand new classical music show coming soon and the return of the ever-
popular Chris Greene with his show on international folk music. And of course there's Carol
Whittaker's History of Jazz every Friday night. Hopefully this will put your mind at rest, Sally.
John from Leicester writes in to point out that many of our guests on Talk Back and other shows
seem to run out of time before they have the chance to finish the interview. 'It happens again and
again,' writes John. 'As the programmes draw to a close guests get rushed and many questions go
unanswered, Why don't you offer some kind of after-show online channel where the guest can
continue answering Iisteners'questions?' I think that's a great idea, John. And as you go on to
say, if it were recorded, people who don't get the chance to hear the live show could catch
up later. Well certainly pass this one up to management. I'm sure a service like this would go
down really well on our website. Now Clive has a question that will be of interest to all us over-
50s fans of Radio Western. Clive wants to know why we don't feature more issues related to
this age group and cater more to this group's taste in music, As Clive explains, As a regular
listener, I'm concerned about your age profiling. Presenters seem intent on covering topics that
appeal very much to the 30-somethings, which is great for them, but what about people of other
ages?' Clive would like to see music shows aimed at the older generation and more on problems
facing the over-50s in our consumer affairs shows. Well Clive, wed certainly hate to think you're
feeling excluded from our schedule. What about other listeners? Do you think were getting it
about right or is there room for improvement?
Exercise 19.
Questions 11-16
Choose the correct letter, A, B or C.
11 The self-access centre
A is always available.
B is generally quiet.
C gets busy during exams.
12 The Internet PCs
A should be used for no more than half an hour.
B must not be used to access Facebook.
C are generally used to study English.
13 Usernames and passwords
A are created by the teachers.
B are issued during induction.
C are created by students.
14 The help desk
A is staffed by technicians.
B offers language support.
C. is only open when two members of staff are available.
15 Photocopying of reference books
A is not allowed.
B is done by teachers.
C has a small cost.
16 The high-spec PCs
A can be used to browse the Internet.
B are available for word processing.
C may need a CD-ROM.
Questions 17-20
Complete the sentences below.
Write ONE WORD AND/OR A NUMBER for each answer
17 Students can purchase a pair of ………………… from the help desk.
18 The graded readers can be borrowed for a maximum of ………………….
19 The centre opens at 8.30 and closes at …………………, Monday to Friday.
20 On occasion the room is ………………… by a teacher and might not be available.

11. C

12 A

13 C

14 B

15 B

16 C
17 headphones

18 14 days

19 5: 30

20 booked

OK, so here we are on the first floor. The self-access centre is just along here on the left. This
room's very popular with students and can get quite busy. When it’s quiet, you can come here as
often as you want, but if there's a lot of demand, such as coming up to exam time, we have to
limit sessions to make sure everyone gets a chance to use the resources.

If you'd like to follow me in. So, here we are. As you can see, it’s a lovely bright room with lots
of resources to help you with your English studies. Over there against the wall we have row of
Internet-connected computers. As you can imagine these get taken very quickly with students
wanting to check their email, Facebook, that kind of thing. Because of the demand we ask
students to try and to stick to about 30 minutes maximum.

You'll need to log in with your username and password. You should have created these
already during your induction. Please do not share your details with anyone else, and please
make sure you read our policy on using the Internet.

I mentioned the help desk earlier, and that's it over there just past the computers in the corner.
There are usually two members of staff available to help you, and these will often be teachers, so
if you have any language questions that's where you can go. Now, over there you can see the
reference section, You'll find dictionaries, exam practice tests, vocabulary and grammar books. I
should point out that these books are for reference only, and we don't offer a loan service. We are
allowed to make photocopies of one or two pages, so if there's an exercise you need a copy of
ask the staff to help. Now these computers on the oval table here, they're our high-spec PCs.
You'll find programmes to help you with your English but also opportunities to practise other
languages such as Spanish, Chinese, German; several languages in fact. Some of them require a
CD-ROM ....You'can collect them from the help desk. By the way, there's no Internet
connection on these computers nor any office software for the time being. If you want to do any
word processing, you'll need to use one of the laptops we keep for this purpose. Again, see the
staff if you need one of these,

Some of you were asking earlier about extra listening practice, and these small rooms here are
dedicated language labs. If you want to use this resource, see a member of staff to buy a set of
headphones. And finally here on the left we have two stands with our large collection of readers.
These are simplified novels by well-known British authors like Charlotte Bronte, Charles
Dickens, and Shakespeare. The books are graded, and you'll find lots that will be appropriate to
your level. Unlike the reference books you can take these readers home and keep them for up to
14 days. I think that's everything, The centre is open from 8.30 till 5.30 during the week so, as
your classes start at 9.00, those of you who get here early can use the resource before your
lesson. Although it's open all day long, it's booked by a tutor for their class now and again, so
you might not be able to gain access if you have any free time during the day. It's best to ask the
tutor concerned if it's okay to pop in Oh, and it's open an Saturday as well, just for a few hours,
from 10.00 till 1.00.

Exercise 20.

Questions 11-13: Choose the correct letter, A, B or C.

11. The main aim of the arts festival is to

A support the creative industries.

B increase the number of spectators.

C get more people into the creative arts.


12 After attending a drawing workshop, Ben thought

A it was surprising that he was talented.

B it could remind him of his childhood.

C. it could give joy to other people.

13 According to Ben, research indicates that

A being creative brings more happiness.

B being creative can lead to positive mindset.

C the more creative you become, the more curious you get.

Questions 14-17 Which group of people can do each of the following activities? Write the
correct letter, A, B, C or D, next to Questions 14-17.

A older people

B any interested people

C the unemployed

D creative people

Activities

14 walk for creativity

15 work with children

16 learn the art of story-telling

17 create your own artwork

Questions 18-20

Complete the sentences below

Write ONE WORD AND/OR A NUMBER for each answer.

18. Sessions must be ………………… in advance.

19 All workshops are free of charge, and ………………… will be provided.

20 The office is open from 9,00-5.00, Monday-Friday, and until ………………… on Saturdays.
Presenter: Hi again. I'm joined today by Ben Knightly from the Media and Arts Centre. He's here
to tell us about the launch of the city's arts festival. Hi Ben. This year has a particular focus,
doesn't it?

Ben: It does, yes. This year we want to encourage more people who would not normally
describe themselves as being creative to get involved with some of our many events and
workshops. Not simply turning up as spectators but to get involved themselves, to get their
hands dirty as it were. There's such a wide offering this year that I'm sure we'll have something to
suit all tastes.
Presenter: You were telling me earlier how beneficial being creative can be for us.

Ben: Absolutely. I recently attended a drawing workshop, and even if I do say so myself, came
away with a very good sketch I'd done. But what was particularly surprising for me was my
feeling of pride and joy when I looked at the sketch again and showed it to the family. It really
took me back to the feelings I had as a youngster when I'd made something. I realised that
even as an adult we can get just as much pleasure and happiness from creative activity. Actually,
research has shown that the more we allow ourselves to be creative, the happier we feel ;
and the more positive our frame of mind, the more creative and the more curious we become
about the world we live in.

Presenter: Well, you've certainly persuaded me. So, what kind of events can we look forward to?
Ben: We want to try and include as wide a range of people as possible this year, from people
already involved in the creative arts through to elderly people who haven't been creative in years.
So for example, we’re inviting people in the creative industries who occasionally suffer from
writer's block to join us on one of our creative walks. Walking has been proved to aid creative
thinking, and we're running a series of walks during the spring and summer around some of the
many beauty spots in and around the city. Then there's our knitting programme. We're working
with schools in the area to invite grandparents in to teach kids how to knit. It’s a great
opportunity to bridge the generation gap and rekindle that interest in knitting you may have
forgotten about. We also aim to inspire and support people without jobs through a series of
free courses starting with creative writing workshops. These courses will give them an
insight into the basic ingredients of a good short story and help participants get their ideas
into shape. And for anyone out there who is looking for the chance to explore their creative
side, come along to our printmaking workshops, You'll have the chance to study some
fantastic prints by local artists, explore different print processes, and take home a print of your
own to hang on a wall.

Presenter: Excellent. So how do we go about getting involved?

Ben: If anyone is interested in joining one of these sessions, its important that you contact us first
as places need to be booked beforehand. We ran similar sessions last year and demand was high.
As I said previously, there's no charge for any of the workshops, and materials where
appropriate will be provided on the day. You can get further tion on our website, and if you don't
have access to the Internet, call us on 514 2261. The booking office open Monday to Friday from
nine to five, but closes early on a Saturday at 12.30.

Presenter: Many thanks, Ben.


C – SECTION 3
Exercise 21.
Questions 21-24 Choose the correct letter, A, B or C.
21 What does Howard say about the experience of writing his dissertation?
A It was difficult in unexpected ways.
B It was more enjoyable than he’d anticipated.
C It helped him understand previous course work.
22 What is Joanne most worried about?
A Finding enough material.
B Missing deadlines.
C Writing too much.
23 What does Howard say was his main worry a year previously?
A Forgetting what he’d read about.
B Not understanding what he’d read.
C Taking such a long time to read each book
24 What motivated Howard to start writing his dissertation?
A Talking to his tutor about his problems.
B Seeing an inspirational TV show.
C Reading a controversial journal article.
Questions 25-26
Choose TWO letters, A-E. What TWO things does Howard advise Joanne to do in the first
month of tutorials?
A See her tutor every week.
B Review all the module booklists.
C Buy all the key books.
D Write full references for everything she reads.
E Write a draft of the first chapter.
Questions 27-28
Choose TWO letters, A-E. What TWO things does Howard say about library provision?
A Staff are particularly helpful to undergraduates.
B Inter-library loans are very reliable.
C Students can borrow extra books when writing a dissertation.
D Staff recommend relevant old dissertations.
E It’s difficult to access electronic resources.
Questions 29-30
Choose TWO letters, A-E. What TWO things does Joanne agree to discuss with her tutor?
A The best ways to collaborate with other students.
B Who to get help from during college vacations.
C The best way to present the research.
D Whether she can use web sources.
E How to manage her study time.

21. C 26. B OR E IN EITHER ORDER

22. C 27. C, D (in either order)

23. A 28. C, D (in either order)


24. B 29. B OR D IN EITHER ORDER

25. B OR E IN EITHER ORDER 30. B OR D IN EITHER ORDER

Hi Howard ... I haven’t seen you for a while.


Hi Joanne. Yeah, they’re keeping us really busy on the postgraduate programme. But how are
you? You’ll be starting your dissertation soon, won’t you?
Yeah ... tutorials start next week ... I’ve got Dr Peterson. You’ll remember it all from last year, of
course!
It’s not something you forget easily. But seriously, although I didn’t expect to enjoy writing my
dissertation ... and in fact I didn’t really find it much fun, I wouldn’t have missed the
experience ... I found it really improved my understanding of the whole degree programme, you
know, from the first year on ...
Right.
So what are you doing yours on?
Glaciated landscapes ... although I haven’t decided exactly what aspect yet.
I did mine on climate systems, so I can’t help you much I’m afraid. But you’ll be fine once you
start your tutorials ... Dr Peterson’ll help you focus.
I know, and he’ll set me deadlines for the different stages ... which is what I need.  My concern is
that I’ve got tons of material on the topic, and I won’t be able to stick to the word limit, you
know.
Hmm. I remember I had different concerns when I was doing my dissertation.
Last year?
Yeah ... before my first tutorial I did a lot of fairly general reading ‘cause I hadn’t fixed on my
topic at that stage. I actually enjoyed that quite a lot and really improved my reading speed, you
know, so I was getting through a lot of material. I was frightened I wouldn’t remember it
all though so I got into the habit of making very detailed notes.
So did you find your tutor helpful .in getting you started?
Yeah we certainly had some interesting discussions but it’s funny ... I saw a brilliant programme
about climate change and it was that that really fired me up. It was talking about some recent
research which seemed to contradict some of the articles I’d been reading. Hmmm.
So you say your tutorials start next week?
Yeah.
Well, the first month’s crucial. You’ve got to meet your tutor and decide on your focus but don’t
become too dependent on him...you know, don’t see him every week...only when you want to
check something.
Right.
Once you’ve got the focus you’ve got to get reading - it’s helpful to look through the
bibliographies for all the course modules relating to your topic. And get hold of any books you
think you’ll need.
I haven’t got much money ...
I mean get the books from the library, far better.
And I suppose I should prepare a detailed outline of the chapters?
Yeah absolutely, but don’t feel you have to follow it slavishly, it’s meant to be flexible.
Now, I’m someone who likes to get writing quickly ... I can’t just sit and read for a month.
Not like me then ... but if that’s what suits you, you know, your natural approach, then you really
ought to start immediately and write the first chapter ...
Right.
Now Joanne, about the library ... it’s worthwhile getting on good terms with the staff ... they
aren’t always helpful with undergraduates .
I suppose they focus on post-grads more.
Maybe but show them you’re serious about wanting to do good work.
And what if I can’t find what I need?
Well, there’s inter-library loans ...
Borrowing books from other libraries ... but I’ve heard it isn’t all that reliable.
You’re right.. .but you probably won’t need it anyway ... be positive, the library is likely to have
most things you need ... and during the dissertation writing period, you can take out fifteen
instead of the usual ten books.
Should I look at previous years’ dissertations?
You can do .
But I won’t know which are the good ones.
The library only keeps the best and the staff can advise you.
Are they willing to do that?
Oh yeah .
And I’m worried about getting journal articles ... from the electronic library.
Well, have you tried to find any yet?
No.
Well you should, it’s really straightforward.
That’s obviously something I’ll have to look into.
Dr Peterson will help.
Yeah, I know I can go to him if I have any worries.
Except he will be away in the second month - it’s the holidays. You should ask him what to do
while he’s away.
Gosh, yeah ... but I suppose I can get a lot of support from coursemates ... I know a couple of
people who are thinking of doing the same topic as me.
Take care, collaboration can become dependency. I think you’d better see how that works out
what the people are like.
You’re probably right ... About other reading, I suppose Dr Peterson’ll recommend plenty of
good articles to get me started.
One thing I’d find out is what his attitude is to internet sources.
Surely not in this day and age? I’d better get that sorted out right at the beginning.
I would if I were you.
And I’ve also got some questions about the research sections - how much time I should spend
explaining the process.
Well, I think that’s up to you ... you can see how it develops as you’re writing.
OK.
It’s the same with things like time management . that’s something a tutor can’t really help you
with .
I agree!
So is there anything else you need me to go ove
Exercise 22.

Questions 21–30

Complete the notes below. Write no more than three words for each answer.

Novel: 21 …………………

Protagonists: Mary Lennox; Colin Craven

Time period: Early in 22 …………………


Plot: Mary → UK – meets Colin who thinks he’ll never be capable of 23 ………………… .
They become friends.

Point of view: “Omniscient” – narrator knows all about characters’ feelings, opinions and 24
…………………

Audience: Good for children – story simple to follow

Symbols (physical items that represent 25 …..…):

• the robin redbreast

• 26 …………………

• the portrait of Mistress Craven

Motifs (patterns in the story):

• the Garden of Eden

• secrecy – metaphorical and literal transition from 27 …………………

Themes: Connections between

• 28 ………………… and outlook

• 29 ………………… and well-being

• individuals and the need for 30 …………………

21) The Secret Garden

22) (the) 20th/twentieth century

23) walking

24) motivations/motivation

25) abstract ideas

26) roses
27) dark(ness) to light(ness)

28) health

29) environment

30) human companionship

Professor: Good morning, Lorna and Ian! I'm glad that you both chose to make it. You're the
only two who take the names down for this literature test. So let's get started, shall we? I would
like to go through some aspects of the novel, The Secret Garden, with you before the test next
week. Do take some notes and feel free to interrupt me if you have questions.

Ian: Hey Lorna, have you got a spare pen?

Lorna: Yeah, here you are.

Professor: All right, so, the story follows two key characters. You should refer to them as
protagonists who go by the names of Mary Lennox and Colin Craven. The story is set shortly
after the turn of the twentieth century, and the narrative tracks the development of the
protagonists as they learn to overcome their own personal troubles together.

Lorna: That's quite a common storyline, isn't it?

Professor: Yes, you're right, Lorna. So could you share something you've already known about
the character of Mary?

Lorna: Well, in the beginning, she is an angry and rude child who is orphaned after a cholera
outbreak and forces to leave India for the United Kingdom to her uncle's house in Yorkshire.
Exactly, and there she comes across Colin who spends his days in an isolated room, believing
himself to be permanently crippled with no hope of being ever possible of walking. The two
strike up a friendship and gradually learn by encouraging each other that both of them can have a
healthy, happy and fulfilled life.

Ian: Is there any need for us to remember these details for the exam? Just the fundamental
structure. Examiners don't want to read a plot summary. They know what the book is about.
Focus on narrative techniques instead, such as point of view.

Lorna: What does that mean?

Professor: It's all about how we see the story. For example, it's written based on what is called an
'omniscient narrator', which means all-knowing. So, readers can feel the same as how all the
characters do about things, including what they like and don't like, and what their motivations
are in the story.

Ian: Won't it be that difficult to perform a technical analysis? After all, it's a kid's book.

Professor: Well, it was initially pitched at adults, you know, but over the years it has shifted to a
more youth-orientated work. In this case, your understanding is correct in some way. The simple
lexical items and absence of foreshadowing make the story relatively easy to follow and
supposedly suited for children. But that doesn't mean there isn't much to analyse. Look at the
symbolism, for instance.

Lorna: Symbols are things, right? Material things like objects that stand for abstract ideas.
Professor: Absolutely right. The author also uses many of them. There's the robin redbreast, for
example, which symbolises the wise and gentle nature that Mary will soon adopt. Note that the
robin is regarded as 'not at all like the birds in India'. Roses are treated as well as a personal
symbol for Mistress Craven. You'll see they're always mentioned alongside her name. And
Mistress Craven's portrait can also be interpreted as a symbol of her spirit.

Ian: Are symbols just another name for motifs?

Professor: No, motifs are a bit different. They don't have a direct connection with something the
way a symbol does. Motifs are simply recurring elements of the story that support the mood.
Lorna: Are there any in this novel?

Professor: Yes, two very key ones. The Garden of Eden is a motif, which comes up a few times
in association with the garden of the story. And then you've got the role that secrets play in the
story. At the very beginning, everything is steeped in secrecy, and slowly the characters share
their secrets and in the process move from darkness to lightness, metaphorically, but also in the
case of Colin, quite literally. His room used to have the curtains drawn, but in the end, he appears
in the brightness of the garden.

Ian: Anything else needed to know about?

Professor: Yes. Nearly all novels explore universal concepts that everyone has witnessed, things
like love, family, loneliness, friendship. These are called themes. The Secret Garden has a few
themes that all concentrate on the idea of connections. The novel explores, for example, the way
that health can determine and be determined by our outlook on life. As Colin's health conditions
get improved, so do his perceptions of his strength and possibility. The author also examines the
relationship between our surroundings and our physical and spiritual prosperity. The dark,
cramped rooms of the manor house stifle the development of our protagonists; the garden and
natural environments allow them to blossom, just as the flowers do. Finally, this book looks at
the connections between individuals, namely Mary and Colin. This necessity of human
companionship is the novel's most important theme because none of their development as
individuals would have appeared without their knowing each other. Well, that about sums it up, I
think.

Lorna: That's a great help, thanks.

Ian: Yes, thanks very much.

Exercise 23.

Questions 21-26
Choose the correct letter, A, B or C.
21 The tutor suggests Fergus attend the fair on
A Monday through to Friday.
B Tuesday and Wednesday.
C Monday or the end of the week.
22 At the fair Fergus can
A impress potential companies he is interested in.
B decide his career path.
C do nothing since he is only in his first ear.
23 Fergus needs to do something in advance such as
A researching about the programme.
B visiting some company websites
C making a good conversation with the people
24 Fergus says that
A there is one company he is particularly interested in.
B he has done some research already
C he knows the boss at one of the companies.
25 The tutor thinks Fergus should
A ask some questions that concern the employers.
B identify the skills required for jobs before the event.
C talk about what the salaries are during job interviews.
26 Fergus plans
A to wear a suit and tie.
B to wear smart but casual clothes.
C to buy an outfit for the event.

Questions 27 and 28 .
Chose TWO letters, A-E.
Which TWO groups should Fergus target his questions at?
A more than one representative
B those who give away free gifts
C alurnni who work for those companies
D those responsible for interview
E ex-students who have attended the fair

Questions 29 and 30
Choose TWO letters, A-E.
Which TWO benefits can Fergus get from attending the job fair?
A get feedbacks from former students
B apply socialising skills
C secure a job on the spot
D introduce himself to new people
E find potential employees
21 B

22 A

23 B

24 B

25 A

26 B
27-28 C,D (IN ANY ORDER)

29-30 B,D (IN ANY ORDER)

Tutor: OK, Fergus, so we've looked at your assignment, which was OK. Now, before you go,
you know about. the jobs fair that's coming up, don't you?

Fergus: Yes, it’s the week after next, isn't it? The whole weer is that right?

Tutor: That's right. Monday through to Friday. I'd suggest making sure you get along there on
Tuesday and Wednesday. Engineering companies tend to be more prominent then rather than
on Monday or the end of the week.

Fergus: Um, yes, I've got the programme for this year. And it looks like those days will be best
for me. I'm only in my first year, so I'm not expecting too much from the day. But I’ve heard you
can pick up some valuable ideas for career paths.

Tutor: Well, you've still got a few years here, I know, but it's never too soon to make a good
impression on potential employers. You've got the programme, so do some research. Have a
look at company websites so you've got the basis for a good conversation with the people on the
stands.

Fergus: Yes, I was looking at one the other day. The boss was being interviewed about their
staff development programme, and there are one or two other firms I'm also interested in.

Tutor: Mm, that's good. You've made a start already. Remember to think about what you're
going to ask people before you turn up. Not how much you're likely to earn, of course! You
only discuss salaries at job interviews. No questions about the skills you need for the job,
the kind of personal qualities employers are looking for, that kind of thing.

Fergus: Yes, I see what you mean. It's best to go prepared and make the most of the
opportunities. And I'm sure you don't need telling that it’s a good idea to dress correctly for the
event. You need to give off a professional air.

Fergus: Well, I won't be buying anything special for the occasion, that's for sure. I've got a suit
and tie at my parents’, but I don't have time to collect it. I ‘ll make an effort, though. A nice pair
of trousers and a jacket, nothing too formal.

Tutor: I'm sure you'll look the part.

Tutor: By the way, you'll often find companies have more than one representative, maybe
someone from marketing handing out free gifts, someone who’ll explain the interview process,
an ex-student who now works for them, that kind of thing. Try and direct your questions
towards the best person.

Fergus: Yes, that's a good idea. I’ll certainly be keen to talk to any ex-students that are around.

Tutor: I'm sure you'll find the whole thing really useful. It’s important to go to these events, and
we always get great feedback from students who have attended. As long as you go with the right
expectations. It's unlikely you'll come away with the promise of a job, of course. It’s more about
discovering what companies are looking for in potential employees.

Fergus: Yes, plus they are a great opportunity to practise things like networking, meeting
new people, talking about yourself and what you do, d’you know what I mean?

Tutor: Definitely, yes. There’ll he several high-profile companies in the engineering sector, and
you’ll have the chance to get to know some useful people. If they give you their card or contact
information, make sure you keep it safe. It’s a sign they like you and want you to keep in touch.

Exercise 24.

Questions 1-4  

You will hear two undergraduates doing a research methods course - a girl called Leela and a
boy called Jake - having a seminar with their tutor.

Choose the correct letter, A, B or C.

1. Leela and Jake chose this article because


A it was on a topic familiar to most students.
B it covered both IT and education issues.
C it dealt with a very straightforward concept.
2. How did Leela and Jake persuade students to take part in their research?
A They convinced them they would enjoy the experience.
B They said it would help them do a particular test.
C They offered to help them with their own research later on.
3. Leela and Jake changed the design of the original questionnaire because
A it was too short for their purposes.
B it asked misleading questions.
C it contained out-of-date points.
4. Leela was surprised by the fact that
A it is normal for questionnaire returns to be low.
B so many students sent back their questionnaires.
C the questionnaire responses were of such high quality.

Questions 5-6

Choose TWO letters, A-E.

What TWO things did respondents say they liked most about doing the crossword?

A It helped them spell complex technical terms.


B It was an enjoyable experience.
C It helped them concentrate effectively.
D It increased their general motivation to study.
E It showed what they still needed to study.

Questions 7-8

Choose TWO letters, A-E.

In which TWO areas did these research findings differ from those of the original study?

A Students’ interest in doing similar exercises.


B How much students liked doing the crossword.
C Time taken to do the crossword.
D Gender differences in appreciation.
E Opinions about using crosswords for formal assessment

Questions 9-10

Choose TWO letters, A-E.

What TWO skills did Leela and Jake agree they had learned from the project?
A How to manage their time effectively.
B How to process numerical data.
C How to design research tools.
D How to reference other people’s work.
E How to collaborate in research.

1. C 6. C, E (in either order)

2. B 7. D OR E IN EITHER ORDER

3. C 8. D OR E IN EITHER ORDER

4. A 9. C, E (in either order)

5. C, E (in either order) 10. C, E (in either order)


So, the task I gave you both was to choose an article about a small-scale research project.

Jake: Yes ...

You were then required to try to reproduce the research procedures in your own context ... i.e. try
it out for yourselves.

Yeah ... and that’s what we’ve done.

Great. So I’d like you to tell me a bit about the article and why you chose it.

Well, the article’s written by two university lecturers who had started using crosswords to help
their students revise terminology for exams .

And the crosswords were designed and set on computers.

And we selected the article because ... well it seemed an accessible topic, even though we
weren’t familiar with the technique ... you know ... using IT to design crosswords for higher
education.

That’s a good reason. So these lecturers wanted to see how well this innovation was received by
their students?

Jake: Yes.

So how did you go about reproducing the research?

Well, we drew up a list of terms from one of our own modules ... and designed a crossword for
revising these terms.

Then we asked our classmates to try out the crossword and give us feedback, you know, their
opinions, on how they felt about using the technique.

Was it easy to find participants?

It wasn’t easy at first. But then we convinced them that by taking part in the research they were
actually benefiting themselves by preparing for an exam which is coming up later this term.

Leela:              And it worked!

Tutor:              Good. So how did you find out what the students thought about doing the
crosswords?
Jake:                A questionnaire. The original article used a two-page long questionnaire. There
were lots of excellent questions on it but the whole section on difficulties using IT is now
obsolete ... old-fashioned even, even though it had only been written a couple of years ago.

Tutor:              So you designed a shorter version?

Leela:              Yeah. Then we sent it to the forty students by email and got twenty-eight replies. I
was taken aback by the fact that everybody we talked to thought this was a good return! I mean
the responses were well written, you know, people had taken a lot of care, but I was really
disappointed with the low numbers.

Tutor:              Yes, an important lesson to learn for an apprentice researcher .

Leela and Jake: Yeah.

Tutor:              So what results did you get?

Jake:                Well, basically the responses were extremely positive. The students said
that doing the crossword on a computer helped them really focus on the work in hand and not be
distracted, which is something that commonly happens with other ways of doing of revision.

Yeah ... that was really clear. But something that struck me was that ‘having fun’ hardly featured
in their responses ... nor did anything to do with spelling of hard words ... which I thought would
be an obvious benefit.

No? OK ...

Respondents also said that doing the crossword hadn’t really increased their general motivation
to study but that it had highlighted the gaps in their memory so they knew what further work was
necessary.

Right ... So how did your findings tally with those of the original researchers?

There were lots of similarities but ...

.there were probably two main differences. We found that more males than females liked the
technique, whereas the original study found the reverse.

Also our respondents said they wouldn’t mind doing a crossword as a final official exam ...
whereas in the original study students said they would hate doing it even if it meant having a
shorter test.

But of course both sets of respondents said they’d be interested in doing more crosswords for
informal purposes, revision and so forth.
Right, so let’s have a think about the whole project and what you’ve learned from doing it.
Well ... it was very time consuming!

Yeah! And I don’t think we managed that aspect very well.

It could have been worse ... I mean we didn’t have a lot of data so we didn’t have to spend ages
processing it. And of course, we’d already done a course on numerical data processing so there
wasn’t much new there.

Yeah, that’s true. Anyway, I think we designed our questions well so that they gave us
manageable data.

Yeah, it really helped having the original study to guide us, as it were ... and that helped us see
what a good research instrument is ....what a good questionnaire should be like.

Absolutely - we got a lot from that. But when we were writing up the project, I’m not sure
whether we’ll know how to acknowledge the work of the original study ... you know, our
referencing.

No ... that’s something we’ll both have to work on in the future.

Actually that part’s been great, finding ways to share and support another person.

That’s the real plus from the project ... learning ways to do that.

Well, it’s obviously been very successful ...

Exercise 25.

Qestions 21-23 What does Judy say about each of the following courses?

Write the correct letter, A, B or C, next to Questions 21-23.

A She transferred from this course.

B She transferred to this course.

C Her transfer request was turned down for this course.

Courses
21 History of Art

22 English

23 Fine Art

Questions 24-26

Choose THREE letters, A-G.

Which THREE reasons does Graham give for wanting to transfer?

A The German course is too difficult.

B He does not like the people he is studying with

C He has more interest in History

D He plans to do a combined degree

E It suits his career plans.

F He is not bothered that he will not spend a year abroad.

G His housemate has persuaded him to transfer.

Questions 27-30

Complete the notes below. Write NO MORE THAN THREE WORDS for each answer.

Transferring to another course at the university

Step 1: Identify your reasons for wanting to transfer.

Step 2: Check that you satisfy the 27………………… for the new course.

Step 3: Speak with the Careers Service.

Step 4: Find out if there are any 28………………… implications.

Step 5: Speak to the Admissions Tutor in the department you want to transfer to.

Step 6: Complete an 29 ………………… form.

Please note: The form must also be signed by the 30 …………………of your current course and
the one you are transferring to.
21 C

22 A

23 B

24-26 A, C, F (IN ANY ORDER)

27 entry requirements

28 funding

29 internal transfer
30 Head of Department

Judy: Hi Graham, How are you dong?


Graham: Hi Judy, I'm fine, thanks. And thanks for popping round. I was hoping I could pick your
brains about transferring to another course.

Judy: Yes, I remember you saying you wanted to do something else. Are you planning on staying
here, or are you looking to go to a different university?

Graham: No, I'm happy to stay here. I just feel I need to do something else. How did you go
about your transfer? You did Histoty originally, didn’t you?

Judy: That was the course I initially wanted to transfer to. I'd studied Art at college and
wanted to continue, but my parents persuaded me that English would be more useful so I
took their advice. But I really didn't enjoy it and tried to transfer to History of Art, but the
course was full. Anyway, l spoke with the course tutors, and they told me about the Fine
Art programme. They thought I had the talent to do it, so that was that.

Graham: I see. How did your parents take the news?

Judy: They were OK about it, really. They just want me to do what I enjoy, so everything's fine
there. So, you're hoping to transfer as well, then?

Graham: Yes, I think I've given the German course a good try, but I'm not really happy. Most of
the other students on the course seem to have at least one German parent, or they've spent a great
deal of time in Germany, so their German is much better than mine. We get huge novels to
read, and I'm still struggling with the first chapter, while they're already finished and writing
their assignments. I thought about doing a combined degree, German with another course, as the
workload would be less, but in the end I feel a complete change would be best. One of my other
subjects at school was History, and 1 realise now I'm actually more interested in that. One
of my housemates is doing History, and it sounds like a great degree, so I've decided to focus on
that.

Judy: Well, people do transfer, so it's not out of the ordinary. But won't you miss the
opportunity to study abroad for a year? You spend Year 3 in Germany, don't you?

Graham: I know. Not only Germany, actually. You have a choice of Germany, Switzerland, or
Austria. That would be exciting, I know, but that's really the only thing I like about the course,
and I can always do some travelling after I've finished my studies. I still don't know what I
want to do as far as a career is concerned, so I think it's best I study something I enjoy, first and
foremost.
Graham: Anyway, I thought I'd ask you about how you went about your transfer before taking it
any further.

Judy: Well, the first thing you have to do is make sure you're clear in your own mind why you
want to transfer. You'll need to persuade people that it's a good idea, so get your arguments clear.
Then ... I'm sure you've already done this, but check what the entry requirements are for
History and whether you've got the necessary qualifications. They might be more flexible now
you're actually here, but you need to show them you'd be able to keep up with your studies.

Graham: Well I've got History qualifications, so I'm hoping that won't be a problem. Who do I
have to speak to?

Judy: Well, they always tell you to speak with your course tutor first in case there's a way of
making the course you're on more appealing to you. But it sounds like you've definitely decided
to get out, so I'd go to the Careers Service next, just to make sure there aren't any restrictions on
transferring. It shouldn't be a problem, but it's best to check first, And then there's your funding.
I'm no expert but I would imagine this won't be a problem; German's a four-year course and
History is three, so you could just start from Year 1 next September. If the Careers Service think
it’s OK and there are no funding problems, go and speak to the Admissions Tutor in the History
Department, just to see if they're happy with your qualifications — and if there are places.

Graham: OK. Well, I'm hoping it'll be OK. I'm planning to transfer at the beginning of next year
rather than half-way through this year.

Judy: Yes, hopefully it'll be alright. If they accept you, it's pretty straightforward after that. It’s
not like you're going to another university. You just have to complete a form — I think it's called
an internal transfer form. Your current Head of Department and the one for the course you're
transferring to have to add their signature as well . And that’s it.

Graham: Well, thanks for that, Judy. I'd better start making my case for transferring, I suppose.

Exercise 26.

Questions 21-25

Choose the correct letter, A, B or C.

21 What did Alice do when she first arrived at the university Open Day?

A She had a coffee.

B She looked around the campus.

C She went to one of the events.

22 Which of the following is NOT on the campus?


A a supermarket

B a bank

C a bookshop

23 Why does Oliver think it is a good idea to register?

A The university will know you attended

B You get an information pack.

C You cannot attend events unless you register

24 What is the main factor for Alice in her choise of university?

A its facilities

B its links to industry

C the number of mature students

25 Which area of work is Oliver interested in?

A teaching

B finance

C software development

Questions 26-28

What does Alice say about each of the following events in the afternoon?

Write the correct letter, A, B or C, next to Questions 26-28.

A She will attend.

B She might attend.

C She will not attend.

Events

26 a talk by the Head of Department

27 a talk about the year abroad

28 the exhibition in the Physics Department


Questions 29 and 30

Choose the correct letter, A, B or C

29 Which employers offer only unpaid work?

A accountancy firms

B the university

C campus retail outlets

30 Oliver

A plans to live at home if he gets a place at the university.

B will live on campus if he gets a place at the university.

C does not yet know where he will live if he gets a place at the university.

21 B

22 A

23 A

24 C

25 C

26 A

27 C
28 B

29 A

30 C

Oliver: Excuse me, is this seat taken?

Alice: No, by all means, have a seat. Are you here for the Open Day?

Oliver: Yes, I think I've just about finished now. I got here first thing this morning. What about
you?

Alice: I got here a little while ago. I spent some time walking around the place first, just to
get a feel for what it's like. I'm doing the organised events this afternoon. I thought I'd have a
coffee before I get started. It's a lovely campus, isn't it?

Oliver: Yes, I love it. And the facilities are unbelievable. I've just been over to have a look at the
sports centre. There's an Olympic size swimming pool, a gym, squash courts, everything really.
All the high street banks are here, and the bookshop looks better than the one in town. There's
supposed to be a big supermarket a few minutes' walk from the main entrance, so there's
pretty much everything you need here.

Alice: Yes, I really like the look of it .... um, I wonder if you can help me. I think I need to
register to let them know I've arrived, don't I?

Oliver: I'm not sure you have to. You can just pick up an information pack from the desk over
there. And nobody asked my name or anything when I turned up for the events earlier. I just
walked in. But you never know; they might check after to see if people have bothered to
come to the Open Day, so I think it's best to register.

Alice: Thanks. just finish my coffee and then I'll get started.

Oliver: So, is this your first Open Day?

Alice: No, it's my fourth. I've been to Sussex, Coventry, and Birmingham so far. They've all got
their good points. But being a bit older, I'm particularly keen on somewhere that has a few
students my age on the course. Apart from that, they all seem to have great links to businesses,
and there isn't much to choose between them as far as their facilities are concerned. How about
you?

Oliver: I haven't been to any other Open Days yet, but I'm hoping I end up here. I've just been to
a presentation by the Head of Department. It sounds like a great place to do Maths — that's my
subject. He was telling us about all the avenues open to Maths graduates and the kind of work
you can end up doing. A lot of students go into finance, accountancy, banking, that kind of thing.
I can't say that's ever appealed to me, though. My Maths teacher at college was telling me
about the opportunities in the software industry, which I quite like the sound of.

Alice: Well, I hope you manage to get in. According to the letter they sent me, my department is
doing something similar. There's a talk later this afternoon by the head. I can't miss that. There's
also someone who'll be explaining about the year abroad. Apparently, you can spend your third
year at one of their partner universities in Spain or Germany. I'm going to have to give that a
miss, though, to catch my train. Oh, and there's also an exhibition area in the Physics
Department with some of the things people are doing here, try and catch that.

Oliver: There were a few second- and third-year students at the exhibition I went to. One of them
gave me some great tips on finding work as well. I already knew about a couple of
accountancy firms in the area that offer work experience. That's on a voluntary basis,
though. But apparently the students helping here on the Open Day get paid, and the university
advertises other jobs that come up now and again, so that's worth remembering. And a lot of the
shops here are always looking for staff.

Alice: Mm, that's useful to know, I overheard someone saying there's a tour of some of the halls
of residence in about half an hour, so I think I’ll register and try to fit that in before I go to the
talk. Are you thinking of living on campus?

Oliver: I've not made my mind up yet. I don’t live far away from here. My parents' place is just
the other side of town, I could easily get the bus to campus, plus it would be a lot cheaper if I
stayed at home. But it would be nice to get some independence as well, so I don’t know. I'll
have to see. But I didn't know about the tour. Would you mind if I get along with you?

Alice: No, not at all. Let me just finish my coffee and I’ll go and register.

Exercise 27.

Questions 21-30

Choose the correct letter, A, B or C.

21. The purpose of the meeting is to:

A. Report on problems with the program.

B. Help to make the program better for future students.

C. Discuss what was most enjoyable about the program.

22. Freya found it easy to adapt to studying in Germany because:


A. She had been there before.

B. She speaks perfect German.

C. Many things were familiar.

23. Tom felt that the university's pre-program support was not enough because:

A. The preparation booklet didn't have enough information about living in China.

B. It didn't provide enough practical skills for living independently abroad.

C. It is impossible to prepare everything in advance.

24. After three months in China Torn felt unhappy because:

A. He didn't have any friends.

B. He wasn't making much progress with learning Chinese.

C. He was frustrated about no longer being the best student.

25. Tom improved his experience by:

A. Becoming more socially active.

B. Having a lot of trips around China.

C. Spending more time on study.

26 According to Freya, the number of students attending classes was:

A. Less than in the U.K.

B. About the same as in the U.K.

C. More than in the U.K.

27. Freya found that when studying management case studies in Germany:

A. There were a lot of arguments between the students.

B. There was more focus on solving problems with theory.

C. Increasing profit was one of the most important factors.

28. Tom found it hard to adapt to his new course at first because:
A. He was not able to use his first language.

B. He found it difficult to get along with his classmates.

C. The course materials were not interesting.

29. Tom and Freya both found it useful to:

A. Ask other students for help

B. Go on trips with other students.

C. Study with other students.

30. Before starting the exchange program, Tom and Freya both suggest:

A. Making friends with local people in the new country.

B. Visiting the country you will be studying in.

C. Watching TV shows about the country you will be studying in.

21. B

22. C

23. C

24. B

25. A

26. C

27. B

28. A
29. C

30. B

Tutor: Hi Freya, Tom, thanks for coming today. You both spent the last academic year on
exchange programs at overseas universities. Now, at the start of this new semester I'd like to
get some feedback from you to help us to provide more support to our exchange students in
the future. Does that sound OK?

Tom: Sounds fine to me.

Freya: Me too. By the way, where were you Tom? I was in Germany.

Tom: Oh, are you a languages student too? I was in Beijing studying Chines

Freya: Wow, cool! Actually, I was studying management. All my lectures were in English, and
my father's German, so I felt very much at home.

Tutor: I actually wanted to get started by talking about day-to-day challenges. It seems like you
were able to adapt quite smoothly, Freya. How about you Tom, did you feel that the university
provided enough support?

Tom: Well, yes and no really. I mean, I did have a meeting with my course tutor which was quite
useful, and I also remember reading some travel guides and the university's own preparation
booklet. However, it was my first time to travel independently to another country, and I don't
really think there is any program that can completely prepare someone for having to deal
with living in a new country.

Tutor: That's true, and in our booklet, it does mention the stages of culture shock, especially
coping with the most difficult stage, stage 2, which sets in after two or three months.

Tom: Yes, the negotiation stage isn't it? Well. I'd been studying Chinese for a year before I went
over to China and was considered one of the best students in the class, but suddenly I was in
China and still using English all the time except in the classroom. It just made me feel really
frustrated that my Chinese skills were still pretty limited and that I was too lazy to improve.
Like you say, that period around three months was really hard. It just seemed like even the
simplest things like going to the shops were incredibly difficult.

Tutor: But you managed to get through it?

Tom: Well I didn't have much choice! Yes, it was a pretty steep learning curve. It was mostly
making friends with locals, forcing myself to use Chinese all the time and getting more
involved in campus activities... that did the trick. The second six months just flew by, and I
can't wait to go back at some point in the future.
Tutor: Great! So overall sounds like a pretty successful year, yeah?

Tom: Yes, it was, on the whole.

Tutor: OK, let's move on now to discuss in more detail some of the differences in the educational
environment. Freya, shall we start with you? What were some of your main impressions?

Freya: Well, the number of people attending lectures was much greater than I was used to,
and the seminars were also quite large. so it was quite difficult to get direct feedback from tutors.

Tutor: And what about the learning process, was it similar to what you were used to?

Freya: Pretty much, Like I said, all the classes were in English. I did notice something different
when analyzing case studies though. For example, in the U.K, it seems generally more a case of
trying to find the most practical solution, whereas in Germany there was much more focus on
finding a model or framework to deal with the problem more effectively. That was a real
eye-opener, and it's definitely something to keep in mind in the future.

Tutor: So you mean that applying theory to practice is more common in German management
training than in the U.K,? Interesting. Anything else? How about you, Tom? Thinking about the
study side of things, urn, was there anything in your experience of studying in China that
surprised you or took a bit of getting used to?

Tom: Well, I suppose the biggest challenge was that the teacher spoke in Chinese the whole
time, and all the study materials were in Chinese, so there was no use of English to make
things easier from time to time.

Tutor: Wow, sounds tough! And you were able to cope with that?

Tom: Yes, eventually. Getting to know new people helped a lot as well, and as time went by we
got together more often to do homework and practices together. That really helped a lot,
especially as I'm more of a group learner.

Freya: I can definitely relate to that, Tom, I learnt so much from other students, sometimes
even more than from the teachers, just because we were all in J the same boat and wanted to get
the most out of being there.

Tutor: Great. Before we finish, any other tips for potential exchange students?

Freya: Well, yes, I'd certainly recommend going to take a look at the place before you start
the course, just as a tourist. Just to get familiar with the people is and the living environment.

Tom: Yes, I'd go along with that, though not easy when you have to go as far as China!
Tutor: Well, Tom and Freya, thanks again for coming today, the feedback you've given me has
been really useful.

Exercise 28.

Questions 21 and 22

Choose TWO letters, A–E

Professor Jones thinks that Mark will ask for more time for his essay because:

A two other students have already asked her

B her students never plan their assignments

C most students have handed in their assignments

D it is the day before the assignment is due in


E Mark is always late with his assignments

Questions 23–25

Choose ONE letter, A, B or C

23 Mark needs an extension because:

A he was sick and went to the doctor

B he had a problem with his computer

C his brother had a personal problem

24 Mark didn’t come to see Professor Jones a week ago because:

A he thought his writing was very bad

B he planned to finish the essay on time

C he wanted to keep his grade point average

25 Which rule below is NOT correct? You should

A ask at least 3 days before the essay is due

B fill out a form and then email it

C talk to your teacher in person

Questions 26–30

Complete the notes below.

Write NO MORE THAN TWO WORDS AND/OR A NUMBER for each answer.

Professor Jones’s feedback:

• Mark needs to add 26 …………………… to his bibliography

• He should look in the online databases, e.g. Languageline

• His essay has no 27 ……………………

• Mark plans to explain the theories and show 28 …………… for each of them
• He should 29 …………………… the essay before he hands it in

• He can have one 30 …………………… for his assignment

21 – 22 A, D (IN ANY ORDER)

23 C

24 B

25 A

26 three / 3 references

27 introduction

28 evidence

29 edit

30 extra day
Jones: Hello Mark. What can I do for you?

Mark: Oh, hi Professor Jones. I’ve come to ask you a big favour.

Jones: Let me guess, Mark, you want more time for your essay assignment. You’re the third
student I’ve seen today, all with the same request! It beats me why a few people leave their
planning to the last minute and then think they can come at a day’s notice and get a
reprieve. It’s really not fair to all the students who are well planned and organised is it?

Mark: Yes, you’re right, and I’m really sorry I need to ask. I’ve never asked for an extension
before.

Jones: So, Mark, why should I grant you an extension when I said no to everyone else today?

Mark: Well, Professor Jones, I really do have a valid reason. It‘s not just the usual, like I have a
cold, and I have a certificate from my doctor, or my computer crashed and I lost all my data. I
wouldn’t bother you with those old excuses. You see, my twin brother was planning to get
married next weekend, but last week his girlfriend told him the wedding was off and he
landed on my doorstep. He was really upset. I couldn’t just tell him to go away because I was
busy. And of course I would have had three assignments due this week, and all my study plans
just went out the window.

Jones: I see. So, why didn’t you come to see me a week ago when your brother first turned up?

Mark: Because I was hoping I would still be able to get it done, but I just can’t manage it .
Well, actually I have written the required number of words, more or less, but frankly, I feel it’s
terrible. I don’t want to let myself down by handing it in as it is at the moment, and I really don’t
want to drop my grade point average by getting a low mark on this assignment.

Jones: Well, the course handbook states very clearly that at least 5 days’ notice is required
for any extension, except in emergencies. Those rules were designed to make it fair for
everyone you see. You’re supposed to submit a request on the proper form…. and you can send
it by email. And you also need to make an appointment and discuss your request in person, as
you have done now. Your problem does come very squarely under the ‘family issues’ category,
so you probably would have got some extra time if you’d done that. I’m not really inclined to
grant this request now, you know.

Mark: Yes, I know I’ve really messed up. I suppose I’ll just have to hand it in as it is,
and take a lower grade.

Jones: Well, at least you’ve made a start, which is more than can be said for your other
colleagues today. Have you got what you’ve done so far? Come on, we’ll look through it
together and I’ll see if I can make a few suggestions for a quick fix, how about that?

Mark: Oh, thank you, yes here it is on my laptop.


Jones: Well, let’s see. Let’s start with the bibliography … Yes, you’ve consulted most of the
sources I suggested, but you don’t have any references of your own, and that was part of the task,
wasn’t it? You need at least three references of your own, see?

Mark: Yes, I know. I did find one more, McDonald and Ferris 2014.

Jones: OK, that’s a good one, I suggest you look at their reference list, too, and chase up a couple
more from there. You should be able to find everything you need in the online library databases,
especially Languageline.

Mark: Yes, OK. So, what about the essay structure, and the argument? Are they OK, or am I
barking up the wrong tree?

Jones: Well, you just seem to jump straight in to your first point here. I’m not sure where your
essay is going. Where’s the introduction?

Mark: Oh right … I always leave the introduction till the end and write it after I’ve finished the
rest of the essay. That way, it ties in to what I’ve already written …Well, that’s what we were
taught in the study skills course, anyway.

Jones: Yes, and you’re quite right. So, tell me about your ideas. What’s your main argument?

Mark: Well, I’m not really sure. I think I understand the 3 opposing theories OK, but I don’t
really have a strong opinion about which one is correct. I was more thinking of explaining them
all as clearly as I can, and then giving the evidence for each one. I don’t think there is a clear-cut
right or wrong, well, that’s my opinion anyway. Do we need to say that one is better than the
other two?

Jones: No, you don’t, and what you are planning to do is often the best way to go about it. Well,
I’ve been skimming through what you’ve written while we’ve been talking and one thing you
definitely must do before you submit this assignment is to edit it for grammar and spelling
mistakes. There are a lot of errors here, just simple things that are easy to fix. But still, I can see
you’ve done the basic work, and I do sympathise with your brother, it must be very difficult for
him. So, I’ll give you one extra day on this essay.

Mark: Oh, thank you Professor Jones, thank you! I’ll hand it in before 5pm on Wednesday, then.

Jones: Yes, that’s the final deadline. Goodbye now!


Exercise 29.

Questions 21-25

Choose the correct letter, A, B or C.

21 Why does the student want to study Tourism?

A good future

B good pay

C parents’ will

22 What kind of skill will the student gain in the course?

A time-management

B financial planning
C note-taking

23 The student has the ability of

A independence.

B communication

C coping well with statistics.

24 The teacher believes that the industry of tourism is

A shrinking

B seeing a bright future.

C growing popular.

25 How does the student compare the university course with polytechnics?

A There are summer schools.

B The course is structured in modules.

C The price is reasonable.

Questions 26-30

What feature do the speakers identify for each of the following courses? Choose
FIVE answers from the box and write the correct letters, A-G, next to questions 26-
30.

Features

A limited value

B useful

C relevant to career

D flexible admission

E intensive
F improving leadership

G self-control and time management

Courses

26 Travel and Business

27 Japanese

28 Medical Care

29 Computer

30 Public Relations

21 A

22 B

23 A

24 C

25 C

26 E

27 A

28 B

29 D

30 C

CHARLOTTE: Excuse me, Dr Twain. May I speak with you for a minute?
DR TWAIN:    Of course, please come in.

CHARLOTTE: I’m Charlotte York. I’m considering taking your course in Tourism.

DR TWAIN:    Right. Well, Charlotte, how can I help you?

CHARLOTTE: I have been considering studying Tourism, however, it is such an important


decision that I would like to seek some advice about it first. Would you mind answering some of
my questions?

DR TWAIN:    Absolutely, fire away!

CHARLOTTE: Well I have been discussing courses with my parents and they are concerned that
I will not be able to get a well-paid job with a degree in Tourism. The reason that I want to
study the course is that I have a great interest in the subject and I think I would really
enjoy it. I believe the only way that I will enjoy my life is if I enjoy my career. Happiness is
far more important than money, don’t you think?

DR TWAIN:    Absolutely. I would much rather be happy and poor rather than rich and
miserable. Money cannot buy you happiness.

CHARLOTTE: I’m glad you agree.

DR TWAIN:    You needn’t worry about money, Charlotte. A large part of the Tourism course
is dedicated to teaching students how to manage finances, a skill that you can apply to your
everyday life as well. I would also recommend that you take a sideline course in time
management, as this can be incredibly useful in efficiently planning your workload. Efficiency is
the key to success!

CHARLOTTE: I’ll remember that.

DR TWAIN:    Now I have found that some students have natural talents that really help them to
succeed in the course. Communication skills, for example, can be very beneficial. Do you have
any strengths?

CHARLOTTE: Maths was always my favourite subject at school so I really enjoy solving
mathematical problems, however I find statistics quite difficult. I have always been very
capable and self-sufficient. I have a lot of confidence in my abilities and will take the initiative
in situations without needing to depend on anyone else for their help.

DR TWAIN:    That’s a really great quality to have and will be particularly useful if you choose
to study Tourism.

CHARLOTTE: That’s great.


DR TWAIN:    I would recommend that you spend some of your time researching the course. A
lot of people who are uneducated on the subject claim that Tourism is a shrinking industry
and that it will become irrelevant in the future. If you study the published research however, you
will see that the truth is quite the opposite. The industry has, in fact, grown significantly
as people have developed an ever-increasing interest in culture and travel. Have you
compared the university course with a polytechnic?

CHARLOTTE: Yes, I have. I was interested in studying the course in modules. However, the
university doesn’t offer that option. I don’t have enough funds to be able to attend an expensive
university, so I was relieved to see that the course is quite affordable. I also considered attending
a summer school instead of university to save money and so that I could work during the rest of
the year, but I really wanted the university experience.

DR TWAIN:    I think that university would suit you well.

DR TWAIN:    Now, what about the courses? Are you interested in any of the

other subjects on offer?

MELANI E:    I have looked at a few. I was interested in Travel and Business as it sounds similar
to Tourism.

DR TWAIN:    That is really worth learning, however, be aware that it is difficult and will
demand a lot of your time.

MELANIE:    OK, that’s good to know.

DR TWAIN:    You might find that Japanese is an interesting course and it will teach you
valuable skills in speaking the language. Personally, it’s not bad and could be of some help,
but not that much.

MELANIE:    OK, Japanese, got that.. .What about Medical Care?

DR TWAIN:    Well, if you have time, the course will teach you a lot about curing diseases and
illnesses, or dealing with injuries outside, although it’s not essential.

CHARLOTTE: So, OK, if it’s useful, I’ll take it.

DR TWAIN:    If you enjoy using technology and are worried about fulfilling the entry
requirements, computing is very relaxed about the skills that applicants must possess.

CHARLOTTE: I’m terrible with computers so I’m not sure that I would enjoy that course. How
about Public Relations?
DR TWAIN:    Yes, I would recommend that course. It would be related to entering the
Tourism industry as it will educate you on how to approach clients and develop associations
with them.

CHARLOTTE: That’s great! Thank you so much for your help!

Exercise 30.

Question 21-24

Choose the correct letter A, B or C.

Project on using natural dyes to colour fabrics

21. What first inspired Jim to choose this project?

A.textiles displayed in an exhibition

B.a book about a botanic garden

C.carpets he saw on holiday

22. Jim eventually decided to do a practical investigation which involved

A.using a range of dyes with different fibres.

B.applying different dyes to one type of fibre.


C.testing one dye and a range of fibres.

23. When doing his experiments, Jim was surprised by

A.how much natural material was needed to make the dye.

B.the fact that dyes were widely available on the internet

C.the time that he had to leave the fabric in the dye.

24. What problem did Jim have with using tartrazine as a fabric dye?

A.It caused a slight allergic reaction.

B.It was not a permanent dye on cotton.

C.It was ineffective when used on nylon.

Question 25-30

What problem is identified with each of the following natural dyes?

Problems

A. It is expensive.

B. The colour is too strong.

C. The colour is not long-lasting

D. It is very poisonous.

E. It can damage the fabric.

F. The colour may be unexpected.

G. It is unsuitable for some fabrics.

H. It is not generally available.


Natural dyes

25 turmeric

26 beetroot

27 Tyrian purple

28 logwood

29 cochineal

30 metal oxide

21 C

22 A

23 A

24 B

25 C

26 F

27 H

28 D

29 A

30 E

TUTOR: OK, Jim. You wanted to see me about your textile design project.
JIM: That's right. I've been looking at how a range of natural dyes can be used to colour fabrics
like cotton and wool.

TUTOR: Why did you choose that topic?

Jim: Well, I got a lot of useful ideas from the museum, you know, at that exhibition of textiles.
But I've always been interested in anything to do with colour. Years ago, I went to a carpet
shop with my parents when we were on holiday in Turkey, and I remember all the amazing
colours.

TUTOR: They might not all have been natural dyes.

JIM: Maybe not, but for the project I decided to follow it up. And I found a great book about a
botanic garden in California that specialises in plants used for dyes.

TUTOR: OK. So, in your project, you had to include a practical investigation.

JIM: Yeah. At first I couldn't decide on my variables. I was going to just look at one type of fibre
for example, like cotton

TUTOR: ... and see how different types of dyes affected it?

JIM: Yes. Then I decided to include others as well. so I looked at cotton and wool and nylon,
TUTOR. With just one type of dye?

JIM: Various types. including some that weren't natural. for comparison.

TUTOR: OK.

JIM: So. I did the experiments last week. I used some ready-made natural dyes, I found a website
which supplied them, they came in just a few days, but I also made some of my own.

TUTOR: That must have taken quite a bit of time.

JIM Yes, I'd thought it'd just be a matter of a teaspoon or so of dye, and actually that
wasn't the case at all. Like I was using one vegetable, beetroot. for a red dye, and I had to
chop up a whole pile of it. So it all took longer than I‘d expected.

TUTOR: One possibility is to use food colourings.

JIM: I did use one. That was a yellow dye. an artificial one.

TUTOR: Tartrazine?

JIM: Yeah. I used it on cotton first. It came out a great colour, but when I rinsed the
material, the colour just washed away. I'd been going to try it out on nylon, but I abandoned
that idea.
TUTOR: Were you worried about health issues?

JIM: I'd thought if it's a legal food colouring, it must be safe.

TUTOR: Well, it can occasionally cause allergic reactions, I believe.

TUTOR: So what natural dyes did you look at?

JIM: Well, one was turmeric. The colour's great, its a really strong yellow. It's generally used in
dishes like curry.

TUTOR It’s meant to be quite good for your health when eaten, but you might find it's not
permanent when it's used as a dye — a few washes, and it's gone.

JIM: Right. I used beetroot as a dye for wool. When I chop up beetroot to eat I always end up
with bright red hands, but the wool ended up just a sort of watery cream shade.
Disappointing.

TUTOR There's a natural dye called Tyrian purple. Have you heard of that?

JIM: Yes. It comes from a shellfish, and it was worn in ancient times but only by important
people as it was so rare. I didn't use it.

TUTOR: It fell out of use centuries ago, though one researcher managed to get hold of some
recently. But that shade of purple can be produced by chemical dyes nowadays. Did you use any
black dyes?

JIM: Logwood. That was quite complicated. I had to prepare the fabric so the dye would take.

TUTOR: I hope you were careful to wear gloves.

JIM: Yes. I know the danger with that dye.

TUTOR: Good. It can be extremely dangerous if it's ingested. Now, presumably you had a
look at an insect-based dye? Like cochineal, for example?

JIM: Yes. I didn't actually make that. I didn't have time to start crushing up insects to get the red
colour and anyway they're not available here, but I managed to get the dye quite easily from a
website. But it cost a fortune. I can see why it's generally just used in cooking, and in small
quantities.

TUTOR: Yes. it's very effective. but that's precisely why it's not used as a dye.

JIM: I also read about using metal oxide. Apparently you can allow iron to rust while it's in
contact with the fabric: and that colours it.
TUTOR: Yes. that works well for dying cotton. But you have to. be careful as the metal can
actually affect the fabric and so you can't expect to get a lot of wear out of fabrics treated
in this way. And the colours are quite subtle, not everyone likes them. Anyway. it looks as if
you've done a lot of work

Exercise 31.

For questions 16-25, listen to a talk about the history of the electric guitar and supply the
blanks with the missing information. Write NO MORE THAN TWO WORDS taken from
the recording for each answer in the space provided.

DATE MAKER NAME OF GUITAR FEATURES

1890s Orville Gibson no name similar in shape to a


16.________________
1925 Jogn Dopyera The National Guitar made of metal, good for playing
17.________________
1930s C.F. Martin The Dreadnought strings made of
Company 18.________________
1931 George Beauchamp The 19.________________ used two
20.________________shaped
like horseshoes to increase sound
1935 Adolph The Rickenbacker Electro made from
Rickenbacker Spanish 21.________________
1941 Les Paul The Log the first to be completely
22.________________
1950 Leo Fender The Fender Broadcaster its simplicity made it ideal for
23.________________
1951 Leo Fender The 24.________________ easy to carry around

1952 Ted McCarty The Gibson Les Paul 25.________________in colour

1954 Leo Fender The Fender Stratocaster double cutaway design

16. violin
17. blues
18. steel/metal
19. Frying Pan / frying pan
20. magnets
21. plastic
22. solid
23. mass production
24. Precision / precision
25. gold

Lecturer During today’s lecture in this series about the history of popular music. I’m going to
look at the different stages the electric guitar went through before we ended up with the
instrument we know so well today.
The driving force behind the invention of the electric guitar was simply the search for a louder
sound. In the late 1890s Orville Gibson, founder of the Gibson Mandolin-Guitar Manufacturing
Company, designed a guitar with an arched or curved lop, as is found on a violin. This made it
both stronger and louder than earlier designs but it was still hard to hear amongst other louder
instruments.

During the 1920s with the beginnings of big–band music, commercial radio and the rise of the
recording industry, the need to increase the volume of the guitar became even more important.
Around 1925 John Dopyera came up with a solution. He designed a guitar, known as The
National Guitar, with a metal body which had metal resonating cones built into the top it
produced a brash tone which became popular with guitarists who play blues, but was unsuitable
for many other types of music.

Another way of increasing the volume was thought of in the 1930s. The C. F. Martin Company
became known for its ‘Dreadnought‘, a large flat-top acoustic guitar that used steel strings
instead of the traditional gut ones. It was widely imitated by other makers.

These mechanical fixes helped, but only up to a point. So guitarists began to look at the
possibilities offered by the new field of electronic amplification. What guitar players needed was
a way to separate the guitar’s sound and boost it in isolation from the rest of a band or the
surroundings.

Guitar makers and players began experimenting with electrical pickups which are the main
means of amplification used today. The first successful one was invented in 1931 by George
Beauchamp. He introduced to the market a guitar known as The Frying Pan’ because the
playing area consisted of a small round disk. The guitar was hollow and was made of aluminium
and steel. He amplified the sound by using a pair of horseshoe-shaped magnets. It was the first
commercially successful electric guitar.

So by the mid-1930s, an entirely new kind of sound was born. Yet along with its benefits, the
new technology brought problems. The traditional hollow body of a guitar caused distortion and
feedback when combined with electromagnetic pickups. Musicians and manufacturers realised
that a new kind of guitar should be designed from scratch with amplification in mind.

In 1935 Adolph Rickenbacker produced a guitar which took his name – ‘The Rickenbacker
Electro Spanish’. It was the first guitar produced in plastic, which, because of its weight,
vibrated less readily than wood. It eliminated the problems of earlier versions which were
plagued by acoustic feedback. ‘The Electro Spanish’ had its own problems, however, because it
was very heavy, smaller than other guitars of the period, and was quite awkward to play.
Developments continued and in 1941 Les Paul made a guitar which he called ‘The Log’, and true
to its name, it was totally solid. All previous guitars had been hollow or partly hollow. It looked
slightly strange but the next step had been made towards the modem electric guitar.

The first guitar successfully produced in large numbers was made in 1950 by Leo Fender. His
Spanish-style electric guitar, known as a ‘Fender Broadcaster’, had a bolt-on neck, and was
initially criticised by competitors as being very simple and lacking in craftsmanship. Yet it was
immediately successful and was particularly suited to mass production, spurring other guitar
companies to follow Fender’s lead.

In 1951 Leo Fender revolutionised the music world yet again when he produced an electric bass
guitar. This was the first commercially successful bass model to be played like a guitar. It was
easier for players to hit an exact note: that’s why it was called “The Precision”. Although there
had already been electric standup basses, this was much more portable. It is now standard in the
line-up of any rock band and some historians suggest that entire genres of music, such as reggae
and funk, could not exist without it.

In 1952 the Gibson company became Fender’s first major competitor when Ted McCarty created
The Gibson Les Paul’ guitar. It was distinctive because it was coloured gold. The reason for this
was to disguise the fact that it was made from two different kinds of wood. In 1954 Leo Fender
responded to this successful instrument by introducing The Fender Stratocaster’. It is easily
identified by its double cutaway design and three pickups. This model may be the most
influential electric guitar ever produced. The modern guitar as we know it was here to stay.

Exercise 32.

For questions 16-25, listen to a lecture about dust storms and supply the blanks with
the missing information. Write NO MORE THAN THREE WORDS taken from the
recording for each answer in the space provided.

Main focus of lecture: the impact of 16_________ on the occurrence of dust storms.
• Two main types of impact:
A) break up ground surface, e.g. off-road vehicle use
B) remove protective plants, e.g. 17________ and__________

Name of area Details


USA ‘dust bowl Caused by mismanagement of farmland
Decade renamed the 18_____________
West Africa Steady rise in dust storms over 20-year period

Arizona Worst dust clouds arise from 19_______________


Dust deposits are hazardous to
20_______________
Sahara Increased wind erosion has occurred along with
long-term 21___________

Drying-up of Aral Sea


Intensive 22_____________ in Central Asian Republics
Drop in water in major tributaries
Total volume of water in lake reduced by 23________________
Increase in wind-blown material
Lake has become more 24_______________
Serious effects on 25______________nearby

16. human activity/ activities

17. farming and drainage

18. Dirty Thirties/ 30s

19. dry thunderstorms

20. machine operators

21. drought

22. irrigation

23. two-thirds
24. salty/ saline/ toxic

25. crops/ plants/ agriculture

Lecturer: In the last lecture, we looked at the adverse effects of desert dust on global climate.
Today we're going to examine more closely what causes dust storms and what other effects they
can have. As you know, dust storms have always been a feature of desert climates, but what we
want to focus on today is the extent to which human activity is causing them. And it is this trend
that I want to look at, because it has wide-ranging implications. So — what are these human
activities? Well, there are two main types that affect the wind erosion process, and thus the
frequency of dust storms. There are activities that break up naturally wind-resistant surfaces such
as off-road vehicle use and construction and there are those that remove protective vegetation
cover from soils, for example, mainly farming and drainage. In many cases the two effects
occur simultaneously which adds to the problem.

Let's look at some real examples and see what I'm talking about. Perhaps the best-known
example of agricultural impact on desert dust is the creation of the USA's 'dust bowl' in the
1930s. The dramatic rise in the number of dust storms during the latter part of that decade was
the result of farmers mismanaging their land. In fact, choking dust storms became so
commonplace that the decade became known as the 'Dirty Thirties'.

Researchers observed a similar, but more prolonged, increase in dustiness in West Africa
between the 1960s and the 1980s when the frequency of the storms rose to 80 a year and the dust
was so thick that visibility was reduced to 1,000 metres. This was a hazard to pilots and road
users. in places like Arizona, the most dangerous dust clouds are those generated by dry
thunderstorms. Here, this type of storm is so common that the problem inspired officials to
develop an alert system to warn people of oncoming thunderstorms. When this dust is deposited
it causes all sorts of problems for machine operators. It can penetrate the smallest nooks and
crannies and play havoc with the way things operate because most of the dust is made up of
quartz which is very hard.

Another example — the concentration of dust originating from the Sahara has risen steadily
since the mid-1960s. This increase in wind erosion has coincided with a prolonged drought,
which has gripped the Sahara's southern fringe. Drought is commonly associated with an
increase in dust-raising activity but it's actually caused by low rainfall which results in vegetation
dying off.

One of the foremost examples of modern human-induced environmental degradation is the


drying up of the Aral Sea in Central Asia. Its ecological demise dates from the 1950s when
intensive irrigation began in the then Central Asian republics of the USSR. This produced a
dramatic decline in the volume of water entering the sea from its two major tributaries. In 1960,
the Arai Sea was the fourth-largest lake in the world, but since that time it has lost two-thirds of
its volume, its surface area has halved and its water level has dropped by more than 216 metres.
A knock-on effect of this ecological disaster has been the release of significant new sources of
wind-blown material, as the water level has dropped.

And the problems don't stop there. The salinity of the lake has increased so that it is now
virtually the same as seawater. This means that the material that is blown from the dry bed of
the Aral Sea is highly saline. Scientists believe it is adversely affecting crops around the sea
because salts are toxic to plants.

This shows that dust storms have numerous consequences beyond their effects on climate, both
for the workings of environmental systems and for people living in drylands …

Exercise 33.

For questions 16-25, listen to a lecture about baby’s memory and supply the blanks with the
missing information. Write NO MORE THAN THREE WORDS AND/OR A NUMBER taken
from the recording for each answer in the space provided.
Question: Can babies remember any 16_________________?

Experiment with babies


Apparatus:
baby in cot

colourful mobile

some 17____________________
Re-introduce mobile between one and 18__________________ later.

Table showing memory test results

Baby’s age Maximum memory


span

2 months 2
days

3 months 19______________

21 months several weeks

2 years 20______________

Research questions:

Is memory linked to 21_________________ development?

Can babies 22_________________ their memories?

Experiment with older children

Stages in incident:

a) lecture taking place

b) object falls over

c) 23___________________

Table showing memory test results

Age % remembered % remembered after


next day 5 months
Adults 70% 24__________________

9—year-o1ds 70%, Less than 60%

6—year-olds Just under 70% 25_________________

16. (particular) events

17. string

18. 14 days

19. (a) fortnight/ 2 weeks/ two weeks

20. six months

21. language

22. retrieve/ recall/ recover

23. (an) argument

24. 70%
25. 40%

Lecturer: We're going to look today at some experiments that have been done on memory in
babies and young children.

Our memories, it's true to say, work very differently depending upon whether we are very old,
very young or somewhere in the middle. But when exactly do we start to remember things and
how much can we recall?

One of the first questions that we might ask is — do babies have any kind of episodic memory ...
can they remember particular events? Obviously, we can't ask them, so how do we find out?

Well, one experiment that's been used has produced some interesting results. It's quite simple and
involves a baby, in its cot, a colourful mobile and a piece of string. It works like this. If you
suspend the mobile above the cot and connect the baby's foot to it with the string the mobile will
move every time the baby kicks. Now you can allow time for the baby to learn what happens and
enjoy the activity. Then you remove the mobile for a time and re-introduce it some time from
one to fourteen days later.

If you look at this table of results ... at the top two rows ... you can see that what is observed
shows that two-month-old babies can remember the trick for up to two days and three-month-old
babies for up to a fortnight.

And although babies trained on one mobile will respond only if you use the familiar mobile, if
you train them on a variety of colors and designs, they will happily respond to each one in turn.

Now, looking at the third row on the table, you will see that when they learn to speak, babies as
young as 21 months demonstrate an ability to remember events which happened several weeks
earlier. And by the time they are two, some children's memories will stretch back over six
months, though their recall will be random, with little distinction between key events and trivial
ones and very few of these memories, if any, will survive into later life. So we can conclude
from this that even very tiny babies are capable of grasping and remembering a concept.

So how is it that young infants can suddenly remember for a considerably longer period of time?
Well, one theory accounting for all of this — and this relates to the next question we might ask
— is that memory develops with language. Very young children with limited vocabularies are
not good at organising their thoughts. Though they may be capable of storing memories, do they
have the ability to retrieve them? One expert has suggested an analogy with books on a library
shelf. With infants, he says, 'it is as if early books are hard to find because they were acquired
before the cataloguing system was developed'.

But even older children forget far more quickly than adults do. In another experiment, several
six-year-olds, nine-year-olds and adults were shown a staged incident. In other words, they all
watched what they thought was a natural sequence of events. The incident went like this ... a
lecture which they were listening to was suddenly interrupted by something accidentally
overturning, in this case it was a slide projector. To add a third stage and make the recall more
demanding, this 'accident' was then followed by an argument. In a memory test the following
day, the adults and the nine-year-olds scored an average 70% and the six-year-olds did only
slightly worse. In a retest five months later, the pattern was very different. The adults' memory
recall hadn't changed but the nine-year-olds' had slipped to less than 60% and the six-year-olds
could manage little better than 40% recall.

In similar experiments with numbers, digit span is shown to...

Exervise 34.

For questions 16-25, listen to a lecture about healthy cities and supply the blanks with the
missing information. Write NO MORE THAN TWO WORDS taken from the recording for
each answer in the space provided.
The World Health Organization says a healthy city must:

- have a 16 ______________ and _________________environment.


- meet the 17 ________________ of all its inhabitants.
- provide easily accessible health services.
- encourage ordinary people to take part in 18_______________

Place/ Project Aim Method Achievement


Sri Lanka To upgrade squatter The Better housing and
Community Contracts settlements 19_____________ infrastructure
System constructed Provided better
infrastructure, e.g. 20_____________
drains, paths opportunities
Mali To improve sanitation 21_______________ Greater environmental
Cooperative in city graduates organizing awareness
garbage collection Improved living
public education conditions
campaign via
22______________
and discussion groups
Egypt To support Women provided with Rise in the
(Mokattam) disadvantaged women the 25________________
23______________ 24________________ and quality of life of
and equipment for young women
sewing and weaving

16. clean, safe

17. basic needs

18. local government

19. residents

20. economic

21. secondary school

22. films

23. Women’s Centre


24. skills

25. status

Well, Adam's just been talking about some of the problems that have resulted from the rapid
growth of cities in the last hundred years - things like housing, sanitation, crime, and so on. For
my presentation, I'd like to look at some examples of what cities are doing to try to solve some of
these problems.

As part of its healthy city programme, the World Health Organization - the WHO has come up
with a set of criteria for a healthy city. The WHO says, that amongst other things, a healthy city
must provide a clean environment which is also safe - it mustn't be dirty, or dangerous for its
inhabitants. As well as that, the WHO says a healthy city has got to be able to satisfy its
inhabitants' basic needs - that's all its inhabitants, not just the rich ones or the ones with jobs.
Everyone who lives there. A third thing….. a third criterion, is that it's got to have health services
which can be used by all the inhabitants, and which they can access easily. The final point's to do
with local government - the WHO says this is something that the whole community should be
involved in, not just a few powerful politicians or businessmen. So, a healthy city's not just a
matter of avoiding illness, that sort of 'healthiness', it's the way that the whole city works together
for the benefit of its population.

OK. So what I'd like to do now is to look at some projects in different cities around the world
where cities have tried to meet these criteria to make their cities 'healthy' ones.

Right, the first project I'm going to discuss took place in Sri Lanka, and this project was called
the 'Community Contracts System'. Its aim was to improve the places where the poorest section
of the population lived - the squatter settlements. Basically, the problem was lack of
infrastructure things like drains, paths, wells for water and so on. So, a programme was set in
place to construct this infrastructure, but what was different about it was that the residents did
this - the people who actually lived there, not people from outside. And this meant that not only
did the people end up with improved housing and infrastructure, but also because they had
contracts with the community, it improved their chances from an economic point of view. So
that's a way the lives of people in one urban environment were improved.

The next project I'd like to discuss took place in the capital city of Mali, in West Africa. This
project involved setting up a cooperative to try to solve the problems of sanitation in the old
central quarters of the city. One of the main problems was a lack of a system for garbage
collection, which meant that there were a lot of insects, and this was causing disease. And again
it's interesting to look at who was involved in dealing with this problem - in this case, the
cooperative involved students who had graduated from secondary school in getting a system
going. As well as that, the cooperative set up a campaign to educate the public about the
importance of good sanitation, through showing films and setting up discussion groups among
the local people, especially women and adolescents. And the outcome was an increased
environmental awareness which led to changes in household behavior as well as improved living
conditions.

OK, the third project was in Egypt, just outside the capital, Cairo, which is a city that's grown
very rapidly in the last few decades. This project was based in a Women's Centre in a poor area
called Mokattam. The aim of the project was to support girls….. young women from the area
from poor families, so these were women who had no education they'd never been to school, so
they were totally illiterate, and they had no chance of getting jobs. At the Women's Centre, they
were shown how to sew and how to weave, and once they'd learned these skills they were given
the equipment - a sewing machine or a loom - so that they could make things to sell, and have a
chance of earning their own living. And this project has meant that these young women have
greater status in the community, but as well as that, they can enjoy a better quality of life.

So I don't think the problem is that cities are bad. This world and its cities have the resources to
provide for the population that lives there. What it takes is a stronger will and a better
distribution of resources.

Exercise 35.

For questions 16-25, listen to a recording about earthquakes and supply the blanks with the
missing information. Write NO MORE THAN THREE WORDS AND/OR A NUMBER taken
from the recording for each answer in the space provided.
16. When did Mt. Pinatubo erupt for the first time?

______________________________________________________________________________

17. When was the earthquake measuring 7.8 recorded?

______________________________________________________________________________

18. When did the experts begin to study Mt. Pinatubo?


______________________________________________________________________________

19. What fell on the local villages on 2nd April?

______________________________________________________________________________

20. What does a 'Level 5' alert mean?

______________________________________________________________________________

Several large earthquakes were triggered by the eruption of Pinatubo on 15th June 1991 because
of the 21______________ of the summit. The ‘caldera' thus created considerably reduced the
height of the mountain. At the same time a 22________________ was passing by and the rain
associated with it mixed with the cinders in the air to form a substance called tephra' which fell
on the 23_________________ of homes causing them to collapse, crushing hundreds of people.

During the eruption, large amounts of sulphur dioxide gas were emitted, which combined with
24__________________ to make sulphuric acid which was responsible for a great deal of ozone
depletion above Antarctica. The overall effect of the cloud from this great eruption was the
lowering of 25__________________.

16. 5,500 years ago

17. July 1990

18. mid-March 1991

19. ash

20. eruption in progress

21. collapse

22. tropical storm

23. roofs
24. water and oxygen

25. global temperatures

Good morning, everyone. In these environmental science lectures I guess you're all used to
hearing about global warming—well, I'm here today to talk to you about one particular volcano
and its effect of global cooling. I'll begin by going back a little bit in time.

Towards the middle of 1991, the second largest volcanic eruption of the last century occurred in
the Philippines, not far from the capital city, Manila, on the island of Luzon.

Mount Pinatubo belongs to a chain of volcanoes in the area and this was by no means its first
eruption. There is evidence of eruptions from approximately 500, 3,000 and 5,500 years ago.

The events of the 1991 Mount Pinatubo eruption began in July 1990, when a magnitude 7.8
earthquake occurred 100 kilometers northeast of the Pinatubo region. The sleeping giant was re-
awakened but few people had any idea of what was in store for them. In mid-March 1991, many
earthquakes were experienced around Mount Pinatubo and this is when volcano scientists (or
volcanologists as they are called) started their investigation of the mountain. Before the disaster,
thousands of people lived in very close proximity to the mountain and, on April 2nd, small
explosions from vents near the crater dusted their villages with ash. This resulted in the order for
evacuations of 5,000 people later that month.

Earthquakes and explosions continued to harass the residents and, on June 5th, a Level 3 alert
was issued for two weeks because of the possibility of a major eruption. However, the
appearance of a large amount of lava protruding from the mountain on June 7th led to the
announcement of a Level 5 alert on June 9th, indicating an eruption in progress. An evacuation
area within 20 kilometers of the volcano was established and this time 25,000 people were
evacuated.

On the following day, Clark Air Base was evacuated and the danger radius was extended to 30
kilometers from the volcano resulting in the total evacuation of 58,000 people.

On June 15th, just after midday, the eruption of Mount Pinatubo commenced and lasted for nine
hours causing numerous major earthquakes due to the collapse of the land at the top of the
mountain and the creation of a huge caldera. `What's a caldera?' I hear you say. Well, it's obvious
really—with a huge eruption such as this where enormous amounts of material have exploded
into the air, the summit falls into what is now an empty chamber and thus forms a large crater.

As luck would have it, as the eruption was taking place, a tropical storm was passing just to the
northeast of Mount Pinatubo, bringing a lot of rainfall to the area. The dust and cinders that had
been thrown up into the atmosphere combined with the water vapor from the storm to cause a
rainfall of tephra that fell across the whole island of Luzon. Most of the people who perished
during the eruption did so because of the weight of the ash collapsing roofs and killing the
occupants of the houses. If it hadn't been for that passing storm, the death toll would certainly
have been much lower.

But that's not all, besides the ash, Mount Pinatubo expelled between 15 and 30 million tons of
sulfur dioxide gas. Can you guess what happened next? Yes, the sulfur dioxide mixed with
water and oxygen in the atmosphere to become sulphuric acid, which is a major contributor to
ozone reduction.

The eruption plume from Mount Pinatubo reached high into the atmosphere, attaining an altitude
of 34 kilometers and the resulting aerosol cloud spread around the earth in two weeks and had
covered the planet within a year. During the years 1992 and 1993, the ozone hole situated over
Antarctica reached an unprecedented size.

The cooling effects of this cloud over the earth were remarkable. It reduced global
temperatures considerably. In the United States, for example, we experienced our third coldest
and third wettest summer in 77 years during 1992.

Exercise 36.
For questions 16-25, listen to a talk given by a lecturer on Agriculture and Environment and
supply the blanks with the missing information. Write NO MORE THAN THREE WORDS
taken from the recording for each answer in the space provided.
Agriculture and Environment

• 16_____________ production = biggest problem in today's world


• Agriculture is important for jobs, exports and foreign exchange
• 'Agriculture means:
• growing crops
• raising animals
• 17________________
• 18________________
• Agriculture must be sustainable: old methods, & new, chemical methods are all unsustainable
 19__________________ of biodiversity
• Biotechnology -> GM or GE —> bio-prospecting (bio-piracy) i.e. large companies steal
samples of native plants to use the 20_______________ for their own crop improvement
• 21__________________ is responsible for less food and higher prices
• Farmers need to be educated but governments also need to pay attention to 22______________
in order to protect the environment and re-nourish the soil
• Experts from around the world could come together to form a 23______________ to observe
farm systems aiming to prevent pollution and erosion and encourage safe procedures that are also
24_________________
• Creating the project's 25________________ would be very expensive and more money would
be needed for the monitoring system but it could solve the problem of food shortages.

16. food

17. fishing

18. forestry

19. (the) destruction

20. genetic material

21. climate change/ global warming

22. policy development/ environmental policies

23. global network


24. cost-effective

25. (online) infrastructure

Welcome to this lecture on Agriculture and the Environment. I hope it is enough to make some
of you decide on a career in the field of agricultural science. As you all know, food is a basic
human need and producing enough of it is the single greatest challenge facing the modern world.

Developing nations have rapidly expanding populations so agriculture should be central to any
development agenda for those countries. What's more, 75% of people in the developing world
are dependant, directly or indirectly, on agriculture for their livelihood. And, for many low-
income countries, it's the most important sector of the economy accounting for 50% of GDP and
sometimes it's the primary, if not only, source of foreign currency.

Now, of course, when I talk about `agriculture', I am using the term to encompass more than just
growing food crops. Of course, livestock farming, fishing, and forestry are included.

In order to combat wide-scale food shortages, agricultural research programs are underway in
many areas. Using science is one way to increase productivity; but, a word of warning:
agriculture must also be sustainable. Let's look at approaches that are not sustainable. Firstly,
overgrazing and intensive cropping are two ancient but destructive practices that lead to loss of
soil fertility. Secondly, the modern idea of liberal application of chemical pesticides and
herbicides has had disastrous consequences for the health of the land, ranging from the pollution
of water sources to the destruction of wildlife.

These practices have ignored the mechanisms that sustain ecological communities. Ignorance has
led to the destruction of the very biodiversity that is essential for sustainable food production.
However, introducing new agricultural techniques, especially things like genetic engineering,
can be difficult because many people remain suspicious of the fact that plants have had their
genetic material modified by scientists.

Biotechnology has also led to the dubious practice of bio-prospecting, or as some prefer to call it,
bio-piracy. Foreign multinational companies have been accused of illegally obtaining samples of
indigenous plants of other countries in order to get their hands on genetic material to improve the
quality or yield of their own crops.

We must put aside the controversy surrounding the field of agricultural biotechnology in order to
concentrate on the biggest threat to food production on this planet.....which is.....? Yes, climate
change. The effects of global warming so far have been to shrink the food supply thereby
pushing up prices and making, even the most basic necessities, unaffordable.

As I see it, the international community must address this, and other challenges to agricultural
production, with urgency. Concrete scientific and technological achievements need to be
presented for farmers to evaluate and learn to use but, apart from that, governments need to
address the complex issues of policy development if the world's hungry are to be fed.

Environmental policies need to be put in place to protect ecosystems and correct soil
degradation where possible. Countries cannot continue to exploit natural resources whilst
ignoring the consequences. In fact, I'd like to see teams of agriculture and environment experts
making up a global network which would monitor the world's farming systems. Different
farming systems should be studied not only with a view to analyzing the environmental effects,
but the social and economic effects as well, The studies would be carried out with a view to
stemming pollution and erosion and promoting safe, cost-effective practices that will guarantee a
secure food supply in the future.

Monitoring sites would need to be set up all across the world and data collected in a systematic
way. Of course, building the online infrastructure for such a project would cost millions of
dollars and there would be ongoing costs involved with the monitoring system but the
information gathered would go a long way towards solving the problem of feeding the masses
and ensuring millions of people don't face a hungry future.

Exercise 37.

For questions 16-25, listen to a talk given by a lecturer on Agriculture and Environment and
supply the blanks with the missing information. Write NO MORE THAN THREE WORDS
AND/OR A NUMBER taken from the recording for each answer in the space provided.
16. Frederick was the name of the duke made famous in a well-known________________.
17. Frederick was aged_____________ when he was sent to war.
18. His battles against the_______________ ended in total disaster.
19. In 1798, George III gave his son a______________ within the ranks of the military.
20. Frederick should not be held wholly accountable for failing: the__________________ of his
fighters made victory difficult to achieve.
21. The people Frederick commanded were mostly old men, new conscripts
or_______________.
The Grand Old Duke of York:
— wanted to enlist better types of 22__________________
— made the army better by bringing in chaplains, medics and 23_________________
— established a college for preparing army 24__________________
— established school in Chelsea for sons of soldiers
— gave up the position of Commander-in-Chief in 1807 because of a 25_______________
— re-appointed later (in 1811) by his brother (George IV)
— died in 1827

16. rhyme/song
17. 30/thirty
18. French
19. promotion
20. nature
21. (ex-)convicts
22. soldiers
23. vets/ veterinary surgeons
24. officers
25. scandal
Good evening. I'd like to thank the Continuing Education Department for hosting this series of
lectures on 'People behind the names you thought were fiction''. Welcome to this talk on The
Grand Old Duke of York".
I'm sure you're all familiar with the old nursery rhyme: The Grand Old Duke of York. He had
ten thousand men, He marched them up to the top of the hill. And he marched them down again,
and so on. But did you know that the Duke of York immortalized in this popular song was
actually Frederick Augustus, second son of King George the Third of England and Queen
Charlotte? He achieved fame in this way because of the humiliation he suffered at the hands of
the French during the Revolutionary Wars at the end of the 18th century.
Frederick was born on the 16th of August 1763 and from the age of seventeen, he had been
trained as a soldier. When war broke out between England and France in 1793, his father, the
king, insisted that he should command the British contingent that was being dispatched to
Flanders to co-operate with the Austrians and the Dutch. The Duke was a brave soldier, but,
remember, he was only thirty at the time; not only was he young but he was also inexperienced
in battle and was unable to cope with the enthusiastic French Revolutionary Army, He was let
down by his allies too, and in spite of the arrival of ten thousand fresh troops from England, his
campaigns were a disaster.
He was driven out of Dunkirk in September 1793, Handers in May 1794, and Belgium in July
1794, Finally, during the winter of 1794 to 1795, his army retreated to the border of Hanover.
And...with his unsuccessful campaigns over, the Duke returned to England. It was after this
military fiasco that the Duke of York came to be (rather unkindly) satirized in song.
Would you believe, despite all this, King George the Third arranged his son's promotion to the
position of Commander-in-Chief of the army in 1798? And, in the following year, he was
appointed to command an army sent to invade Holland, Again he was unsuccessful and this
confirmed the general opinion that he was not capable of commanding an army in the field.
Nevertheless, the rhyme is a bit cruel and harsh because it doesn't take into account the nature of
the soldiers who served with Frederick. All the blame for lack of success should not have been
attached to the Duke alone because the army he had under his command was made up from what
is commonly described as lithe scum of the earth"—this is a somewhat offensive term used to
refer to a group of people regarded as despicable and worthless. Who were they—these ordinary
soldiers? Well, they were mostly vicious, brutal ex-convicts or raw recruits and elderly men. The
officers who commanded them were all untrained as military men. In fact, they were anybody
who could afford to buy a commission.
Urn, but here's the really great thing that, unfortunately, the Duke of York is not remembered for:
he realized that this was a hopeless kind of army and he set about improving conditions in order
to recruit higher-quality soldiers. He introduced padres—are you familiar with the term? No?
Well, let me explain...you see, members of the British Armed Forces are generally Christians of
one denomination or another, and a padre is a Christian cleric or chaplain who ministers to the
soldiers and attends to their spiritual needs without belonging to any particular grouping within
the Christian faith.
Now, where was I? Yes, Frederick introduced padres, doctors and veterinary surgeons to the
battlefield. Why vets? To attend to the horses, of course! Remember we're talking about late 18th
century battlefields. He was also the founder of the Royal Military College for the training of
officers at Sandhurst - yes, the very same one where the princes and other members of the royal
family receive their military training today! Frederick also founded the Duke of York's School in
London for sons of soldiers killed in battle. His name is perhaps better commemorated by this
school in Chelsea than by the column that stands at the top of Waterloo Steps in St. James's Park.
In 1807, the Duke was involved in a scandal with a woman and as a result, resigned as
Commander-in-Chief but he was reinstated in 1811 by his elder brother, the Prince Regent, who
later became George the Fourth of England. He continued in this post until his death in 1827.

Exercise 38.

For questions 16-25, listen to a presentation of a student and supply the blanks with the
missing information. Write NO MORE THAN TWO WORDS taken from the recording for
each answer in the space provided.
Saving the juniper plant
Background

16. Juniper was one of the first plants to colonise Britain after the last____________________
17 Its smoke is virtually___________________, so juniper wood was used as fuel in illegal
activities.

18. Oils from the plant were used to prevent_________________ spreading.

19. Nowadays, its berries are widely used to_______________ food and drink.

Ecology

20. Juniper plants also support several species of insects and_______________

Problems

21. In current juniper populations, ratios of the__________________ are poor.

22. Many of the bushes in each group are of the same age so______________ of whole
populations is rapid.

Solutions

23. Plantlife is trialling novel techniques across__________________ areas of England.

24. One measure is to introduce_________________ for seedlings.

25. A further step is to plant________________________ from healthy bushes.

16. ice age

It’s has been discovered that it’s actually among the first species of plant to establish itself in
Britain in the period following the most recent ice age

17. invisible

…when burned, the smoke given off is all but invisible.

18. infection(s)
Particularly, during large epidemics, oils were extracted from the Juniper wood and sprayed in
the air to try to prevent spreading infections in….

19. flavor

These days, perhaps its most well-known use is in cuisine cooking where its berries are a much-
valued ingredient used to flavor a variety of…

20. fungus/ fungi

If Juniper bushes are wiped out, it would radically affect many different insects and also fungus
species…

21. sexes

In present-day populations, ratios between the sexes are unbalanced

22. extinction

….the plants are the same age so this means that bushes grow old and start to die at a similar
time leading to swift extinction of whole populations

23. lowland

….this time, focusing on lowland regions of England

24. shelter(s)

The first thing is to try to provide shelters for seedlings….

25. cuttings

A further measure is that in areas where colonies are all but die out, numbers are being replaced
by the planted cuttings.

For my presentation, I’m going to summarise what I’ve found out about efforts to save one plant
species ... the juniper bush. It once flourished in Britain and throughout the world’s temperate
zones, but over the last few decades has declined considerably. Before I go on to explain the
steps being taken to save it in England, let me start by looking at some background information
and why the juniper has been so important in cultural as well as ecological terms, historically and
in the present day.

Firstly, I want to emphasise the fact that juniper is a very ancient plant. It has been discovered
that it was actually amongst the first species of plants to establish itself in Britain in the period
following the most recent Ice Age. And, as I say, it has a much valued place in British culture. It
was used widely as a fuel during the Middle Ages because, when burnt, the smoke given off is
all but invisible and so any illicit activities involving fire could go on without being detected, for
example, cooking game hunted illegally. It also has valuable medicinal properties. Particularly
during large epidemics, oils were extracted from the juniper wood and sprayed in the air to try to
prevent the spread of infection in hospital wards. And these days, perhaps its most well-known
use is in cuisine...cooking, where its berries are a much-valued ingredient, used to flavour a
variety of meat dishes and also drinks.

Turning now to ecological issues, juniper bushes play an important role in supporting other
living things. If juniper bushes are wiped out, this would radically affect many different insect
and also fungus species. We simply cannot afford to let this species die out.

So, why is the juniper plant declining at such a rapid rate? Well a survey conducted in the north
and west of Britain in two thousand and four to five showed that a major problem is the fact that
in present-day populations, ratios between the sexes are unbalanced and without a proper mix of
male and female, bushes don’t get pollinated. Also, the survey found that in a lot of these
populations, the plants are the same age, so this means that bushes grow old and start to die at
similar times leading to swift extinction of whole populations.

Now, the charity Plantlife is trying to do something to halt the decline in juniper species. It’s
currently trying out two new major salvage techniques, this time focusing on lowland regions of
England. The first thing it’s trying is to provide shelters for the seedlings in areas where juniper
populations are fairly well established. These, of course, are designed to help protect the plants at
their most vulnerable stage. A further measure is that in areas where colonies have all but died
out, numbers are being bolstered by the planting of cuttings which have been taken from healthy
bushes elsewhere.

Now, I hope I’ve given a clear picture of the problems facing this culturally and ecologically
valuable plant and of the measures being taken by Plantlife to tackle them. If anyone has any
questions, I’d be happy to ...

Exercise 39.

For questions 16-25, listen to a talk about the birds in New Zealand and supply the
blanks with the missing information. Write NO MORE THAN TWO WORDS taken
from the recording for each answer in the space provided.
Birds in New Zealand
Facts
– The birds often build their nests along the coastline or next to the 16____________
– In 1984. researchers found that there were 17____________ of this kind of bird left.
– It is difficult to 18____________ the number of birds accurately.
Influencing factors
– Human activities, such as 19____________ and building homes threaten the
bird population.
– The birds are influenced by many species which can eat their 20____________
– Natural disasters like 21____________ can also reduce the population of the birds.
Ways of protection
– The zoo should hire a 22____________ to keep the birds from being poached
– The organizer could build 23____________ to prevent the public from getting closer.
– People should make more of an effort to protect 24____________ birds.
– Through the 25____________, people will learn more about bird protection.

16. river
17. 3 pairs
18. estimate
19. farming
20. eggs
21. storms
22. guard
23. fence
24. wild
25. media
Ladies and Gentlemen, good afternoon. In this session, we're going to talk about birds
indigenous to New Zealand, delving into the facts and analysing methods of protecting these
species from extinction.

It may be a surprise to you to know that there are millions of bird species that are endemic to
New Zealand; in other words, they are rarely found anywhere else throughout the world.
Because quite a large number of these birds lead a living on a diet of freshly-caught fish,
they often nest alongside the coastline or, follow the neighbouring river, if necessary to
track freshwater fish. Once they have mated, the female will collect twigs and pieces of debris
to build nests where she can hatch her eggs under safe circumstances, like out the reach of
predators.

When first immigrants landed on New Zealand roughly 7 centuries ago, rats were carried on the
ships with them. The rats flourished in the warm climate and gradually became a threat to the
native bird populations, particularly to the flightless species. In 1984, researchers discovered
that just 3 pairs of flightless bird species remained in existence, and that even these faced
severe pressure. Now, New Zealand is a global pioneer in facilitating the recovery of severely
endangered species from the brink of extinction.
Every year, researchers conduct certain surveys aimed at monitoring the fluctuation in the levels
of bird numbers and species living in New Zealand. Owing to migration patterns of a few bird
species, it is far from easy to accurately estimate the bird numbers since many may have
flown to other regions in search of mates and warmer climates, and thus not be included in
the investigation.

Probably, there is a sensitive link between bird numbers and environmental influences, especially
those closely linked to human activities. Routine activities, such as farming or building
houses, can have a massive impact on the local populations. Forests that play a role of a
habitat for thousands of birds can be entirely eradicated to produce fields for cropping or to
provide wood for construction, which is such a catastrophic impact.

It is not only the activities of humans that threaten the living conditions of bird species, but the
population of many predatory animal species has dramatically risen in New Zealand. One of
these predators is the Mantane snake, which was introduced from Australia and has decimated
the population of Killdeer birds. These birds nest on the ground, and often return to find
their eggs have been devoured by the egg-eating snakes.

One cannot dismiss; of course, it is the disadvantage of that nature itself that imposes on the
survival of many bird species. Natural disasters such as storms can be devastating, tearing
apart forests and leaving thousands of destroyed nests in their wake. Monsoons flood the
rivers and often drown many of the flightless bird species that are unable to escape.

Unfortunately, illegal hunting, which is the greatest threat to bird species in New Zealand, is
nearly impossible to prohibit. Several bird species, which are going to be extinct, now appear on
display at the national zoo, and there is such a crucial need that urgent policies are adopted to
protect them. The zoo recently employed an expert in bird protection, who strongly
suggested that a guard should be employed to protect their birds from poachers.
The expert also lays much stress on the fact that the birds can be haunted by quite a
stressed situation where the public can approach them too closely, suggesting that the
administration install a fence network to keep the public a safe distance away, from the
birds. In spite of this threat to birds in captivity, it is the freely roaming birds that are most at
risk. Many efforts are being made to educate the public in terms of how they can contribute
towards protecting birds that are living in the wild.

Finally, research has indicated that one of the most feasible measures of keeping the public
noticed about the significance of protecting the bird populations is through the media. I
advise you all to read the related articles in specialist journals, and also do some research on the
Internet. Everyone can make your own contribution if you put your mind upon it!

Exercise 40.

For questions 16-25, listen to a postgraduate psychology student talking to other


students about a job-satisfaction study he has investigated and supply the blanks with
the missing information. Write NO MORE THAN TWO WORDS taken from the
recording for each answer in the space provided.
Workers involved in the study were employed at a 16___________.
Despite some apparent differences between groups of workers, the survey results were
statistically 17___________.
The speaker analysed the study's 18___________ to identify any problems with it.
The various sub-groups were 19___________ in size.
Workers in the part-time group were mainly 20___________.
The 21___________ of workers who agreed to take part in the study was disappointing.
Researchers were unable to 22___________ the circumstances in which workers filled
out the questionnaire.
In future, the overall size of the 23___________ should be increased.
In future studies, workers should be prevented from having discussions with
24___________.
Workers should be reassured that their responses to questions are 25___________.

16. call centre 21. response


17. inconclusive 22. control
18. methodology/methods 23. sample/group
19. unequal 24. their/other colleagues
20. female/women 25. confidential

Good morning everyone. For my presentation today I’m going to report on an assignment that I
did recently. My brief was to analyse the methods used in a small study about job satisfaction,
and then to make recommendations for future studies of a similar kind.
The study that I looked at had investigated the relationship between differences in gender and
differences in working hours, and levels of job satisfaction amongst workers. For this purpose,
employees at a call centre had been asked to complete a questionnaire about their work.

I’ll summarise the findings of that study briefly now. First of all, female full-time workers
reported slightly higher levels of job satisfaction than male full-time workers. Secondly, female
part-time workers reported slightly higher levels of satisfaction than female fulltime ones did. On
the other hand, male part-time workers experienced slightly less job satisfaction than male full-
time workers. But although these results seemed interesting, and capable of being explained,
perhaps the most important thing to mention here is that in statistical terms they were
inconclusive.

Personally, I was surprised that the findings hadn’t been more definite, because I would have
expected to find that men and women as well as full and part-time workers would experience
different levels of satisfaction. So I then looked more carefully at the methodology employed by
the researchers, to see where there may have been problems. This is what I found.

First of all, the size of the sample was probably too small. The overall total of workers who took
part in the survey was two hundred twenty-three, which sounds quite a lot, but they had to be
divided up into sub-groups. Also the numbers in the different sub-groups were unequal. For
example, there were one hundred fifty-four workers in the full-time group, but only sixty-nine in
the part-time group. And amongst this part-time group, only ten were male, compared to fifty-
nine who were female.

Secondly, although quite a large number of people had been asked to take part in the survey, the
response was disappointingly low - a lot of them just ignored the invitation. And workers who
did respond may have differed in important respects from those who didn’t. Thirdly, as the
questionnaires had been posted to the call centre for distribution, the researchers had had very
limited control over the conditions in which participants completed them. For instance, their
responses to questions may have been influenced by the views of their colleagues. All these
problems may have biased the results.

In the last part of my assignment I made recommendations for a similar study, attempting to
remove the problems that I’ve just mentioned.

Firstly, a much larger sample should be targeted, and care should be taken to ensure that equal
numbers of both genders, and both full and part-time workers, are surveyed. Secondly, the
researchers should ensure that they are present to administer the questionnaires to the workers
themselves. And they should require the workers to complete the questionnaire under supervised
conditions, so that the possibility of influence from other colleagues is eliminated. Finally, as
workers may be unwilling to provide details of their job satisfaction when they are on work
premises, it’s important that the researchers reassure them that their responses will remain
confidential, and also that they have the right to withdraw from the study at any time if they
want to. By taking measures like these, the reliability of the responses to the questionnaires is
likely to be increased, and any comparisons that are made are likely to be more valid.
So that was a summary of my assignment. Does anyone have any questions?

Exercise 41.

Complete the form below. Write NO MORE THAN TWO WORDS AND/OR A NUMBER
for each answer

Accident Report
Time of accident: 1 ___________

Details of the accident:


The driver failed to stop at the junction of Monks Road and High Street.

There were 2___________ people in the car, all around 20 years old.

The car hit a woman on her 3___________.

The car was last seen heading towards the 4___________.

The 5___________ at the junction were out of order.

Make and model of the car:

red Ford Fiesta, with one door 6___________

Registration number: 7___________

Witness name: Rita 8___________

Address: 19, 9___________, Bradford

Telephone number: 0232 566788

Mobile number: 10 ___________

1 1.15/one fifteen
2 3/three
3 bicycle/bike
4 town centre/center
5 traffic lights
6 damaged
7 Y48 BYW
8 Stansfield
9 Althorpe Road
10 07834 889772
Police officier: Hello madam. I understand you witnessed the accident. Have you got a few
minutes to tell me what you saw?
Woman: Yes, no problem. I don't have to be back at work for a while, so I'm pleased to help.
Police officier: Did you actually see what happened?
Woman: Yes. I was standing over there, near the bus stop. I was on my way to get something for
lunch and just happened to be looking at a shop across the road. That's when I saw the red car
come out from the junction over there.
Police officier: You don't happen to know what time it occurred, do you?
Woman: Well, I left work for my lunch break at one, and its only about ten minutes' walk away
— the office, I mean — so it might have been about ten past one. Although I did pop into the
shop for something, so it was probably closer to one fifteen.
Police officier: So it pulled out of Monks Road — that's the road over there — straight onto High
Street?
Woman: That's right, yes.
Police officier: Did you get a view of who was in the car?
Woman: There were three of them. Two in the front, the driver, of course, someone in the
passenger seat, and there was someone in the back. They were quite young. I doubt if they were
much older than twenty. Anyway, they came speeding out of the side road over there and hit that
lady's bicycle. The driver didn't bother to stop to find out if she was OK. He just drove off along
the main road towards the town centre. Er, is the woman OK?
Police officier: She should be fine. She banged her head when she came off the bike, so we've
called for an ambulance — they always like to check you out in case you have concussion. But
no, she seems fine.
Woman: The bike doesn't look too good, though. I don't think she'll be using that again. I
suppose she was very lucky, really. If they'd hit her instead of the front wheel, she could have
been seriously injured. It looked like they were just in a hurry and didn't want to stop at the
junction. I know the traffic lights aren't working there, so perhaps they thought they could just
pull out.
Police officier: Could you give me a description of the car? Do you know the make and model?
Woman: Well, I'm not very good with cars, but I’m pretty sure it was the same model as my
husband’s car, a Ford Fiesta. It was really like I said and quite old, and the door on the driver’s
side was damaged. It looked like it had been in another accident some time ago.
Police officier: I don’t suppose you had a chance to take down the registration number, did you?
Woman: I did, actually. Let me see ... um, Y 4 8 B Y W. Will that help you trace them?
Police officier: That's really helpful. It depends. It might be a stolen car, but at least we’ll be able
to trace the owner. If it wasn’t stolen, then yes, we’ll be able to find out the name of the driver.
Now would you mind giving me your contact details, just in case we need to get in touch about
anything?
Woman: Of course
Police officer: What's your name?
Woman: Mrs Stansfield. Rita Stansfield. That's STANSFIELD.
Police officier: And your address. Mrs Stansfield?
Woman: 10 Althorpe Road Bradford. That's ALTHORPE.
Police officer: Have you got a telephone number we can get you on?
Woman: Yes, it’s 0232566788.
Police officer: And do you have a mobile number?
Woman: Yes ... 07834889772.
Police officer: That's great, Mrs Stansfield. As I said, we may get in touch if we need any further
information, but probably what you've told me is enough. Thanks for your time.
Woman: No problem. I'm glad to have been of help.

Exercise 42.

Harry’s Hire Company

Questions 1–6
Complete the notes below.
Write NO MORE THAN TWO WORDS AND / OR A NUMBER for each answer.
Hire for: birthday party
Equipment Hire:
Day and date of event: 1 …………………… November
Number attending event: 2 ……………………
Rental: 5 dozen dinner plates, bowls
5 dozen sets of 3 ………… and …………
40 plastic 4 ……………………
4 dozen each small / medium glasses
six 5 ……………………
Costings:
Weekend Package: 5 p.m. Fri – 10 a.m. Mon: $1600 + tax
6 …………………… Package: 5 p.m. Sat – 10 a.m. Mon: $1350 + tax
7 ……………………: $50 (within 10 km)
Breakage Insurance: $60
Replacement costs: plates, bowls – $3.55 per item
small glasses – 8 …………………… per item
medium glasses – $4.40 per item
Customer Details:
Full name: 9 ……………………
Address: 28B Sandstone Close, Martinsborough
Contact number: 10. 084 ……………………

1 Saturday 25th / 25
2 55 / fifty-five / fifty five
3 knives / forks
4 (garden) chairs
5 ice buckets
6 same-day / same Day
7 home delivery
8 $3.50 / three dollars fifty
9 Susan Millins
10 3987695
MAN: Hello, welcome to Harry’s Hire Company. How can I help you?
WOMAN: Oh, hi, yes, I’ve come in to find out about renting stuff for a 21st birthday
party.
MAN: Yes, of course … First of all, what date is the party?
WOMAN: It’s next Saturday. That was the closest we could get to the actual birthday,
which is the 22nd of November. Gosh, it’s hard to believe it was 21 years ago,
seems like yesterday …
MAN: So, the 18th of November?
WOMAN: No, sorry, I meant the following Saturday, the 25th
MAN: OK, we have just about everything here; tableware, marquees … you name it,
we rent it! What size of event are we talking about here?
WOMAN: Yes, that’s a good question. We were planning to have about 40 people, but
you know how these things grow and it went up to 60 at one stage … umm …
I think it’s back to 55 now. Yes, that’s right. It was all getting a bit out of hand.
MAN: Ok, and what kind of catering and entertainment are you having? We can
help with entertainment hire, too, you know, if you need microphones or a
sound system.
WOMAN: Oh, that’s good! We’ve booked a catering company, and they’re providing a
meal. It’s nothing elaborate, just finger food snacks and then a simple buffet
meal, so we’ll need all the usual dinner plates and bowls … I suppose 5
dozen of everything, oh and knives and forks too, 5 dozen sets. We won’t
need any cooking equipment, because the caterers will do that, and they’re
providing tea and coffee as well.
MAN: I see, and do you need any tables or chairs.
WOMAN: Well, not tables, because we wouldn’t have room for them, but I suppose
some extra chairs might come in handy. What type do you have?
MAN: Come over here and I’ll show you. We have a couple of different kinds. We do
have folding wooden ones, like these, but the most popular ones are just
those stackable plastic garden chairs, we rent a lot of those…
WOMAN: Yes, the plastic ones look great. Maybe 40 of those.
MAN: Ok … I’m making a list here as we speak. Was there anything else? Oh, do
you want small or medium glasses? People generally want both sizes.
WOMAN: Yep, better get both kinds. Four dozen of each. Umm … and what else? The
caterers are supplying a punch bowl, so that’s OK. Oh, I know, what about six
ice buckets, for keeping the drinks cold? We’re providing all the drinks,
because I have a friend who is helping us with that. Umm … I suppose this is
going to get very expensive.
MAN: Well, let’s talk about our packages and rental deals. Firstly… what day do you
want to collect the equipment?
WOMAN: Oh, I’m not sure. Does that make a difference to the price?
MAN: Well … the weekend package deal is to pick up after 5 p.m. on Friday, and
drop off before 10 a.m. on Monday. That will be sixteen hundred dollars plus
tax. If you want to save a bit of money, you can collect the equipment on the
day of the party before 5 p.m., and drop off on the Monday before 10 a.m.,
and that will be thirteen hundred and fifty dollars plus tax. That’s called the
same-day package. Your party numbers come between our small and
medium price packages, I’m afraid, so in fact you could rent a few extra of
everything for the same price.
WOMAN: I see. Well, we’re not inviting more guests! I think we have quite enough
already … umm … are there any other hidden charges with those packages?
MAN: No, not really. But if you want us to drop off and pick up at your house, there
is an extra home delivery charge of $50, provided you live within 10 km of
here. Oh, and if you want to take out breakage insurance, that’s a $60 flat fee.
Otherwise, you pay for every item you break, at the replacement cost.
WOMAN: Wow, so how much is that then? I bet that soon adds up.
MAN: Well, yes, it does a bit. Let’s see … tableware is $3.55 a piece, small glasses
are $3.50 and medium glasses are $4.40. Oh, and if you break a chair,
they’re expensive: $15 each, and you’d be surprised what happens when the
party gets going!
WOMAN: Yes, insurance sounds like a good idea, and I think I’ll take the weekend
package deal, thanks, it’s much more convenient, isn’t it? And not much more
expensive.
MAN: OK, so let’s take a few details then. Your name?
WOMAN: Oh, it’s Susan Millins.
WOMAN: No, it’s M-I-L-L-I-N-S
MAN: Right. And your address please?
WOMAN: 28B Sandstone Close, Martinsborough
MAN: And just to confirm the order; the medium-size party weekend package with
breakage insurance, and did you want to collect this yourself?
WOMAN: Yes, thank you. I do live within 10 km, but I don’t want to pay any extra
charges. I’ll get my son to help me.
MAN: OK, we’ll need an emergency contact number, just in case anything goes
wrong. Oh, and credit card details, of course.
WOMAN: Oh, yes, of course, the phone number is 084 – 398 – 7695
MAN: OK, thank you, and now the credit card…..
Exercise 43.
Questions 1–6
Complete the form below.
Write NO MORE THAN TWO WORDS AND/OR A NUMBER for each answer.

Application Form for use of Library Internet Service

Family name: Milton


First names: 1 ……………….. Jayne
Address: 2 ………………..
35 Maximilian Way
Whitfield
Post Code: 3 ………………..
Occupation: Nurse
(works the 4 ………………..)
Home phone: N/A
Mobile: 0412 214 418
Type of ID: 5 ………………..
ID number: AZ 1985331
Date of Birth: 25th 6 ………………..

Questions 7 and 8
Choose TWO letters, A–E.
What will the woman use the internet for?
A trade & exchange
B research
C email
D social networking
E job vacancies

Questions 9 and 10
Write NO MORE THAN TWO WORDS AND/OR A NUMBER for each answer.
9 How much does it cost to register as an internet user?
………………………………….
10 What is the maximum amount of time allowed per single daily internet session?
……………………………

1. Lynda

2. Unit 15 / Unit fifteen

3. 5577

4. night shift

5. swipe card

6. September 1975 / Sept 1975

7&8. A, C (in either order)


9. fifteen dollars / $15 / 15 dollars

10. 60 minutes/sixty minutes/one hour/1 hour

WOMAN: Good morning, I’d like to register to use the internet in the library.

MAN: Do you have a library card?

WOMAN: Yes, I’ve been a member for 6 months but I’ve never used the internet services
before.

MAN: No problem. Can I have your full name please?

WOMAN: Lynda Jayne Milton.

MAN: So, Milton is your surname?

WOMAN: Yes.

MAN: And Linda, L-i-n-d-a?

WOMAN: Well, no, actually it’s L-Y-N-D-A.

MAN: Lynda … Jane.

WOMAN: And, erm, Jane isn’t spelt J-a-n-e either. It’s J-A-Y-N-E.

MAN: Okay, got that …thank you. Now, where do you live, Lynda?

WOMAN: Unit 15, 35 Maximilian Way.

MAN: That’s in Whitfield, right? I have a cousin who lives in that area.

WOMAN: Yes, Whitfield.

MAN: And the postcode is double seven double five?

WOMAN: Not quite – you’ve got it round the wrong way. It’s double five double seven.

MAN: Whoops, okay, moving on now … Do you work or are you at home during the day?

WOMAN: Well, both, actually. I work as a nurse but I’m on permanent night shift.

MAN: Oh, I see. In that case, we won’t put down your home phone number because I’m sure
you don’t want to be disturbed when you’re trying to sleep.

WOMAN: Thanks, I appreciate that – you can always leave a message on my mobile if you have
to contact me. I have it turned off when I’m sleeping, but I regularly check my voicemail for
messages.
MAN: And that number is?

WOMAN: 0412 214 418

MAN: Good. Now I’ll need to see some form of photo ID – have you got something with you?

WOMAN: Yes, just a minute, here’s my swipe card for the hospital.

MAN: Thank you. I just need to make a note of the number … AZ 1985331

WOMAN: Is that all you need?

MAN: Just one more thing – your date of birth – but I can get that from the card. One moment …

WOMAN: Look – I’m afraid you haven’t copied it down correctly. I was born on the 25th
September, 1975.

MAN: What have I written? Oh yes, I see it now, I’ve got the 25th of the eighth month, but that
would make it August … Thanks for spotting the mistake.

MAN: Well, that’s the application form done – now, I wonder if you’d mind taking part in a
survey we’re doing?

WOMAN: That’s fine. What do you want to know?

MAN: Basically, we’re trying to find out why people access the internet. I mean, what would
you be using it for? Social networking, I suppose.

WOMAN: I don’t really think so – I haven’t got the time for something like that. But I do want
to keep in touch with friends and family both here and abroad, so I’d mostly be making use of
my email account.

MAN: I see. A lot of students come in here to do research – is that something that you might be
doing?

WOMAN: I think the internet is a great tool for research but it’s not something that interests me
at the moment.

MAN: What about checking out a new job?

WOMAN: A lot of my friends use the internet for job hunting and they say it’s the best way to
look for a new position – I’m quite settled where I am, though. You can get access to the other
classifieds, can’t you? Trade and exchange, that kind of thing …

MAN: Yes, and I’m told it’s a very popular way of buying and selling these days.

WOMAN: Well, I’ll definitely be using it for that.


MAN: Thanks a lot. Now, do you have any questions?

WOMAN: Is there a charge for the service?

MAN: It used to be free but we’ve decided to set a one-off payment of fifteen dollars for the
initial registration.

WOMAN: Oh, that’s not bad at all. One more thing … is there a time limit for each session, like
half an hour or something?

MAN: Oh, it’s better than that – one 60-minute session per day – quite generous really. But
we’re very strict about it.

WOMAN: I’d rarely spend more than 45 minutes on the internet at any one time so that’ll be
more than enough. Thank you.

Exercise 44:

Questions 1–10
Questions 1–5
Complete the notes below.
Write NO MORE THAN ONE WORD AND/OR A NUMBER for each answer.

Second-hand Bedroom Furniture for Sale

Example Answer

Number of items for sale: three


Bedside tables

Construction: wood

Colour: 1 ………………..

two (in each table)


Drawers:
handles made of 2 ………………..

Height: 3 ……………….. cm

Condition: 4 ………………..

Price: 5 ……………….. (for both)

Questions 6–10
Complete the notes below.
Write NO MORE THAN ONE WORD AND/OR A NUMBER for each answer

Dressing table

Drawers: five (two are 6 ………………..)

Width: 7 ………………..

three: one large, two small (all


Mirrors:
8 ………………..)

Condition: good

Price: 9 ………………..
Seller’s details

Name: Carolyn Kline

Address: 19 10 ……………….. Road

Answers
1. cream
2. brass
3. 65 / sixty-five
4. perfect
5. £30 / 30 pounds / thirty pounds
6. deep
7. 1.25 metres / 1.25 m
8. adjustable
9. £50 / 50 pounds / fifty pounds
10. Domain
Transcript
You will hear a telephone conversation between a woman who is selling some furniture and a
man who is making enquiries about it. First, you have some time to look at questions 1 to 5.

[20 seconds]

You will see that there is an example that has been done for you. On this occasion only the
conversation relating to this will be played first.

WOMAN: Hello, Carolyn speaking.


MAN: Hello. My name is Lincoln Farraday and I’m ringing to see if you still have the bedroom
furniture that you advertised for sale?
WOMAN: Yes, there are three items left – two bedside tables and a dressing table.

The woman said she has three items available so three has been written in the space. Now we
shall begin. You should answer the questions as you listen because you will not hear the
recording a second time. Listen carefully and answer questions 1 to 5.

WOMAN: Hello, Carolyn speaking.


MAN: Hello. My name is Lincoln Farraday and I’m ringing to see if you still have the bedroom
furniture that you advertised for sale?
WOMAN: Yes, there are three items left – two bedside tables and a dressing table.
MAN: Oh, good. They’re just the items I’m after. Tell me, what’s the construction of the bedside
tables – I mean, what are they made of?
WOMAN: Well, they’re a matching pair and they’re made of wood – but the wood has been
painted, it’s not brown anymore, it’s been painted cream.
MAN: I see.
WOMAN: Each table has a shelf and 2 drawers. Oh, and the drawers have square brass handles
– quite modern and quite nice really.
MAN: And what about the dimensions?
WOMAN: Well, each table is 50 centimetres wide …
MAN: That’s good, much bigger than that and they wouldn’t fit beside my bed. I live in an
apartment where the bedrooms are quite small. What I really need to know is how tall they are –
you see my bed’s quite high.
WOMAN: 65 centimetres high and 45 centimetres deep.
MAN: Thanks. Just a couple more questions about the bedside tables, what condition are they in
and how much are they?
WOMAN: They’re in perfect condition – there isn’t a mark on them, you know. I had them
painted professionally, you see, so the finish is much better than you’d normally expect. As for
how much … well, I guess … I think fifteen pounds each would be a fair price, but I’ve decided
that I’ll only sell them together, as a pair, so that means the price is thirty pounds all up.
……………………………………………………………………………………………
Before you hear the rest of the conversation, you have some time to look at questions 6 to 10.
[20 seconds]
Now listen and answer questions 6 to 10.

MAN: Now, can you tell me about the dressing table?


WOMAN: Yes, it matches the other tables in colour and style.
MAN: Good – how many drawers does it have?
WOMAN: Five altogether. Um … the bottom two drawers hold more as they’re deep.
MAN: Mmm … and the dimensions – how wide is it? That’s all I need to know – it wouldn’t be
more than a metre and a half, would it?
WOMAN: Well, just under actually … it’s … ah, 1.25 metres across.
MAN: Does it have a mirror?
WOMAN: Three.
MAN: Sorry?
WOMAN: It has three mirrors – you know … a central one and a narrower one on each side.
And they’re all adjustable.
MAN: I see – and the overall condition of the dressing table?
WOMAN: Well, it has a couple of scratches on the surface but it’s still in good condition so I’m
asking fifty pounds.
MAN: Could I call round and have a look later today?
WOMAN: What time were you thinking of?
MAN: In about half an hour …
WOMAN: Oh, yes, that’s fine. By the way, my name is Carolyn Kline – it’s on the gate at the
front of the house.
MAN: Kline – is that K-L-I-N-E?
WOMAN: That’s right. And I live at 19 Domain Road.
MAN: Did you say the main road?
WOMAN: No, Domain – D-O-M-A-I-N Road.
MAN: That’s just off Ash Grove, isn’t it?
WOMAN: Yes. See you soon then …
MAN: Yes, in about 30 minutes.

Exercise 45:

Questions 1-10

Complete the notes below. Write NO MORE THAN TWO WORDS AND/OR A NUMBER for
each answer.

West Bay Hotel – details of job

Example                           Answer
ŸNewspaper advert for temporary staff

Ÿ   Vacancies for 1…………………..

Ÿ   Two shifts

Ÿ    Can choose your 2………………….. . (must be the same each week)

Ÿ    Pay: £5.50 per hour,  including a 3………………….. .

Ÿ    A 4…………………..  is provided in the hotel

Ÿ   Total weekly pay: £231

Ÿ   Dress: a while shirt and 5…………………..  trousers (not supplied)


a 6…………………..  (supplied)

Ÿ    Starting date: 7………………….. .

Ÿ    Call Jane 8…………………..  (Service Manager) before 9…………………..  tomorrow


(Tel: 832009)

Ÿ    She’ll require a 10………………….. .

Answer:

1: waiter(s)
2: day off
3: break
4: (free) meal
5: dark (coloured/colored)
6: jacket
7: 28 June
8: Urwin
9: 12.00 (pm)/noon/mid-day
10: reference

WOMAN:                  Hello, West Bay Hotel. Can I help you?

MAN:                         Oh, good morning. I’m ringing about your advertisement in the Evening
Gazette.

WOMAN:                  Is that the one for temporary (Example) staff?

MAN:                         That’s right.

WOMAN:                  Yes. I’m afraid the person who’s dealing with that isn’t in today, but I can
give you the main details if you like.

MAN:                         Yes, please. Could you tell me what kinds of  staff you are looking for?

WOMAN:                  We’re looking for waiters (Q1) at the moment. There was one post for a
cook, but that’s already been taken.

MAN:                         Oh right. Erin, what are the hours of work?

WOMAN:                  There are two different shifts – there’s a day shift from 7 to 2 and a late
shift from 4 till 11.

MAN:                         And can people choose which one they want to do?

WOMAN:                  Not normally, because everyone would choose the day shift I suppose. You
alternate from one week to another.

MAN:                         Okay. I’m just writing all this down. What about time off?

WOMAN:                 You get one day off and I think you can negotiate which one you want,
it’s more or less up to you (Q2). But it has to be the same one every week.

MAN:                         Do you know what the rates of pay are?

WOMAN:                  Yes, I’ve got them here. You get £5.50 an hour, and that includes a break
(Q3).

MAN:                         Do I have to go home to eat or…

WOMAN:                  You don’t have to. You can get a meal in the hotel (Q4) if you want to,
and there’s no charge for it so you might as well.
MAN:                         Oh good. Yes, so let’s see. I’d get er, two hundred  and twenty one, no,
two hundred and thirty one pounds  a week?

WOMAN:                  You’d also get tips-our guests tend to be quite generous.

MAN:                         Ern, is there a uniform? What about clothes?

WOMAN:                  Yes, I forgot to mention that. You need to wear a white shirt, just a plain
one, and dark (Q5) trousers. You know, not  green or anything like that. And we don’t supply
those.

MAN:                         That’s okay, I’ve got trousers, I’d just have to buy a couple of shirts. What
about anything else? Do I need a waistcoat or anything?

WOMAN:                 You have to wear a jacket, but the hotel lends you that. (Q6)

MAN:                      I see. Er, one last thing – I don’t know what the starting date is.

WOMAN:                Just a minute, I think it’s sometime around the end of June. Yes, the 28th ,
(Q7) in time for the summer.

MAN:                     That’s great. I’m available from the 10th.

WOMAN:Oh good. Well, if you can call again you need to speak to the Service Manager. Her
name’s Jane Urwin, that’s U-R-W-I-N (Q8), and she’ll probably arrange to meet you.

MAN:                     Okay. And when’s the best time to ring?

WOMAN:                Could you call tomorrow? Um, she usually starts checking the rooms
at midday, so before then if you can (Q9), so she’ll  have more time to chat. I’ll just give you
her number because she’s got a direct line.

MAN:                     Thanks.

WOMAN:                It’s 832 double – 0 9.

MAN:                     823 double -0 9?

WOMAN:                832

MAN:                     Oh, okay. Yes, I’ll do that.

WOMAN:                And by the way, she will ask you for a reference (Q10), so you might like
to be thinking about that. You know, just someone who knows you and can vouch for you.
MAN:                     Yes, no problem. Well, thanks very much for your help.

WOMAN:                You’re welcome. Bye.

MAN:                     Bye.

Exercise 46:

Questions 1 and 2

Choose the correct letter A, B or C


1.    In the lobby of the library George saw

A. a group playing music


B. a display of instruments

C. a video about the festival

2.    George wants to sit at the back so they can


A. see well

B. hear clearly

C. pay less

Questions 3-10

Complete the form below. Write NO MORE THAN TWO WORDS AND / OR A NUMBER for
each answer.
SUMMER MUSIC FESTIVAL

BOOKING FORM

NAME:                            George O’Neill

ADDRESS:                     3 …………………. , West sea

POSTCODE:                  4 …………………..

TELEPHONE:               5 ………………….

Date
Event Price per ticket No. Of tickets

Instumental group
5 June
£7.50 2
– Guitarrini

17 June Singer (price £6 2


includes
6 ………………….

in the garden)

22 June
7
…………………….
£7.00 1
(Anna Ventura)

23 June

Spanish Dance & 8£ 9


Guitar Concert ………………… ……………………

NB Children / Student / Senior Citizens have 10 …………………discount on all tickets.


 

1. C : a video about the festiva


2. B. hear clearly
3. 48 North Avenue 
4. WS6 2YH 
5. 01674553242 
6. free drink 
7. the pianist
8. 10.50
9. 4 
10. 50%

NINA: Hi, George! Glad you’re back. Loads of people have phoned you.

GEORGE : Really?

NINA: I felt just like your secretary!

GEORGE : Sorry! I went into the library this afternoon to have a look at a newspaper and I
came across something really interesting.

NINA: What? A book?

GEORGE : No, a brochure from a summer festival – mainly Spanish music. Look, I’ve got it
here.

NINA: Spanish music? I really love the guitar. Let’s have a look. So what’s this group
‘Guitarrini’?

GEORGE : They’re really good. They had a video (Q1) with all the highlights of the festival
at a stand in the lobby to the library,

so I heard them. They play fantastic instruments – drums and flutes and old kinds of guitars. I’ve
never heard anything like it before.

NINA: Sounds great.

GEORGE : Okay. Shall we go then? Spoil ourselves?

NINA: Yes, let’s.

GEORGE : The only problem is there aren’t any cheap seats…it’s all one price.

NINA: Well, in that case we could sit right at the front – we’d have a really good view.

GEORGE : Yeah, though I think that if you sit at the back you can actually hear the whole
thing better. (Q2)

NINA: Yes, Anyway we can decided when we get there.

NINA: So will you fill in the form or shall I?


NINA: Just a minute – I’ve got it written down here. WS6 2YH (Q4). Do you need the phone
too?

GEORGE : I’ll do it. Name: George O’Neill. Address: 48 North Avenue (Q3), Westsea. Do
you remember our new postcode? Still can’ remember it.

GEORGE : Please. I’m really bad at numbers.

NINA: 01674553242 (Q5). So, let’s book two tickets for Guitarrini.

GEORGE : Okay. If you’re sure £7.50 each is right. How do you feel about the singer?

NINA: I haven’t quite decided. But I’ve noticed something on the booking form that might just
persuade me!

GEORGE : What’s that then?

NINA: Free refreshments!

GEORGE : Really?

NINA: Yes, look here. Sunday 17th of June. Singer, ticket £6.00 includes drinks (Q6) in the
garden.

GEORGE : Sounds like a bargain to me!

NINA: Yes, let’s book two tickets for that. So, what else? I’m feeling quite keen now! How
about the pianist (Q7) on the 22nd of June?

GEORGE : Anna Ventura? I’ve just remembered that’s my evening class right.

NINA: That’s okay. I’ll just have to do on my own – but we can go to the Spanish dance and
guitar concert together, can’t we?

GEORGE : Yes – I’m sure Tom and Kieran would enjoy that too. Good heavens – £10.50 (Q8)
a ticket! I can see we’re going to have to go without food for the rest of the week – we’ll need to
book four! (Q9)

NINA: Wish we were students – look! Children, Students and Senior Citizens get a 50% (Q10)
discount on everything.

GEORGE : If only!

 
Exercise 47:

Questions 1-3

Complete the form below. Write NO MORE THAN THREE WORDS AND/OR A NUMBER
for each answer.
TOTAL INSURANCE INCIDENT REPORT

Example              Answer

Name                    Michael Alexander


Address                24 Manly Street, (1) ………………….., Sydney

Shipping agent   2) …………………..

Place of origin    China

Date of arrival    3) …………………..

Reference number   601 ACK

Questions 4-10

Complete the table below. Write ONE WORD AND/OR A NUMBER for each answer.
Item Damage Cost to repair/replace

The 4  ………………….. is
Television Not known
damaged

The 5 ………………
The 6  …………………..of
…. cabinet 7 $ …………………..
the cabinet is damaged

A 8  …………………..is
Dining room table $200
split

Six 9  ………………….. we About 10 $  ………………….. 
Set of china
re broken in total
 

ANSWER
1. Milperra
2. First Class Movers
3. 28 November
4. screen
5. bathroom
6. door
7. 140
8. leg
9. plates
10. 60 
JUDY:                  Good morning. Total insurance. Judy speaking, how may I help you?

MICHAEL:           I recently shipper my belongings from overseas back here to Australian and  I
took out insurance with your company. Some items were damaged during the move so I need to
make a claim. What do I have do?

JUDY:                  Okay, well first I need to get a few details about this. Can you give me your
name please?

MICHAEL:          Yes, It’s Michael Alexander.  (Example) 

JUDY:                  Okay. And your address please?

MICHAEL:          My old address or mu current one?

JUDY:                  Your current one.

MICHAEL:          It’s 24 Manly Street, Milperra near Sydney.

JUDY:                  What was the suburb, sorry?

MICHAEL:          Milperra. M-I-L-P-E-R-R-A (Q1).

JUDY:                  Right. Now, who was the shipping adent Mr. Alexander?

MICHAEL:          You mean the company we used?

JUDY:                  Yes, the company who packed everything up at he point of  origin.

MICHAEL:          Oh, it was…er…First Class Movers (Q2) .

JUDY:                  Okay … where were the goods shipped from?

MICHAEL:          China,  but the ship came visa Singapore and was there for about a week.

JUDY:                  Don’t worry, all of  that information will be in the documentation. Now, the
dates. Do you know when the ship arrived?

MICHAEL:          It left on the 11th of October and got to Sydney on the 28th of November (Q3) .

JUDY:                  Okay, I need one more thing. There’s a referent number. It should be in the top
right-hand comer of the pink from they gave you.

MICHAEL:          Let me have a look. I have so many papers. Yes, here it is. It’s 601 ACK.
JUDY:                  Thanks.

MICHAEL:          Yes, well four things actually. I’ll start with the big things. My TV first of all.
It’s a large one…very expensive.JUDY:                  I need to take down a few details of the actual
damage over the phone before you put in a full report. Can you tell me how many items were
damaged and what the damage was?

JUDY:                  Our insurance doesn’t cover electrical problems.

MICHAEL:          It isn’t an electrical problem. The screen has a huge crack (Q4) in it so it’s
unusable.

JUDY:                  I see. Any idea of the price to repair it?

MICHAEL:          No, Well. I don’t think  it can be repaired. It will need a new one (Q4).

JUDY:                  Okay. I’ll make a note of that and we’ll see what we can do. Now, what was
the second item?

MICHAEL:          The cabinet from the bathroom (Q5) was damaged as well. It’s lovely
cabinet, we use it to keep our towels in.

JUDY:                  And what is the extent of the damage?

MICHAEL:          Well, the back and the sides seem okay but the door has a huge hole in it
(Q6). It can’t be repaired . I’m really not very happy about it.

JUDY:                  And how much do you think it will cost to replace it?

MICHAEL:          Well, when I bought it last year I paid $125 for it. But the one I’ve seen here in
Sydney is a bit more expensive, it’s $140 (Q7).

JUDY:                  Right, and what was the third item?


Exercise 48:

Question 1-10

Listen to the conversation and complete each gap with no more than THREE words.
BUSINESS NATIONWIDE
Courses available:
Name of Course: (0) Getting Started
Time: Two hours from (1) …….
Cost: Free
Course Content: Is starting a business right for me?
Writing a (2)........
Some legal issues
Nearest Location: Handbridge
Next Course Date: 20th March

Name of Course: (3)………….


Length of course: (4)............
Cost: (5) ………..
or £20 for recently unemployed
Course Content:
Day One: Legal Issues
Day Two: Marketing and Pricing
Day Three: Accounting and (6)………..
Nearest Location: Renton
Next Course Date: 5th March or (7)………..

CALLER’S DETAILS
Name: (8)…………..
Address: (9) ........., Eastleigh
email: (10)………….

Answer

1.6pm-8pm

2.Business Plan

3.Business Basics

4.three days

5.£80

6.bookkeeping

7.18th April

8.Lila Park

9.39 White Lane


10.lila.park@rainbow.com

Man: Hello, this is Business Nationwide, Daniel speaking, how can I help you?
Woman: Hi there, er, I’ve recently started up a small business, and I noticed on your website that
you run some courses for people who are starting up.
Man: That’s right, we do. We offer two courses which may be of interest to you. Our first course
is called ‘Getting Started’. It’s a two–hour evening course, and it runs from 6pm to 8pm. We
discuss things like ‘Is starting a business right for me?’, writing a business plan and some of the
legal issues. It runs at various locations in the area. Where are you based?
Woman: I live in Eastleigh.
Man: Eastleigh. So, the closest course to you would be in Handbridge, and the next one is on the
20th March.
Woman: Uh-huh, and how much is that.
Man: That one is free.
Woman: Okay, well it might be worth it.
Man: But did you say you’re trading already?
Woman: Yes, since about August.
Man: Well, you might be better off taking our three-day course – ‘Business Basics’. It’s not free
I’m afraid – it’s subsided and costs £80 for the three days, unless you’ve been unemployed in
the past six months, in which case it’s just £20.
Woman: No, that doesn’t apply to me.
Man: Well, it’s well worth the money. The three days cover the essential aspects of running a
business. The first day covers legal issues, such as tax, insurance, employment laws and health
and safety. The second day covers marketing and pricing, and the third covers accounting and
book-keeping.
Woman: It sounds useful. Does the ‘Business Basics’ course take place in Handbridge too?
Man: Er, let me see. No, it’s not available in Handbridge, I’m afraid. The nearest course to you
would be in Renton. There’s one on the 5th March, and another on the 18th April.
Woman: Yes, that might be useful.
Man: I’ll send out a pack to you if you like, with some details of the courses and also some
information about what you need to do to set up and who you need to register with.
Woman: Great.
Man: Can I take your name?
Woman: Yes, it’s Lila Park.
Man: Lila? Is that L-A-I-L-A?
Woman: NO, L-I-L-A.
Man: Lila Park. And your address please?
Woman: 39 White Lane, Eastleigh.
Man: And have you got an email address? If so, we can send you details of any courses that are
happening near you that you might be interested in.
Woman: Yes, it’s lila dot park at rainbow dot com
Man: Great, well I’ll have the information pack sent out to you today.
Woman: Thanks, that’d be great.
Man: My pleasure. Bye.
Exercise 49:

Complete the table below.

Write ONE WORD AND / OR A NUMBER for each answer.

Art classes at Bramley Community Centre

Class Things to bring Fees /Timetable Tutor


Example a set of 1 The cost is 2 $ ............. A local artist
‘is essential for two terms
called Steve 3 . .....
‘Movement and light : Monday evenings 6-8
painting in the style of
p.m., Room 15
French impressionists

‘Clay Basics’: using the an old 5 The cost is $180 per Theresa Clark –
pottery wheel to make would be a good term her work is
several 4 ......................... displayed in the
idea Every 6 community centre
6:30-8:30 p.m.,
‘Sketching Architecture’: people usually take a Room
The cost3 is $160 per Annie Li
drawing old buildings, fold-up chair and a term Annie’s cell
starting with the phone number:
8 Fridays 11-1 p.m. meet
7 at the corner of 10
Victoria Street and
9.................. Road

Answer:

1. brushes

2. 285

3. R-A-M-D-H-A-N-I-E

4. bowls

5. shirt

6. Thursday

7. library
8. sandwich

9. Station

10. 0217856361

Manager: Bramley Community Centre. How can I help?


Woman: Oh hi. I’m calling about the art classes that the centre off ers. I had a quick look
at your website. I think one of the classes is called something like ‘Movement and
Light’?
Manager: That’s right. The focus is on painting in the style of the French Impressionists.
Woman: Yes, I saw that in the description. But I’ve got a few questions if you don’t mind.
Manager: Sure, go ahead.
Woman: Well, do I need to bring anything to the class? Or is everything supplied?
Manager: Not quite. What you’d need to do is get yourself some brushes Q1 – I’d suggest a
range of them, you know, in diff erent sizes.
Woman: All right. Yes, I guess it would make sense to bring your own. I’ll make a note of
that.
Manager: And you know what the fee for the class is? Woman: For one term –
it’s $170 isn’t it?
Manager: That’s right. But if you sign up for two terms, it actually works out cheaper.
Woman: How much would that be?
Q2 Manager: $285. And we do find that a lot of people sign up for a couple of terms – it’s
such a great class.

Woman: OK, I’ll give it some thought. And that’s on Monday evenings, isn’t it?

Manager: Yes, from 6p.m. to 8p.m. In room 15. Woman: And who’s the
tutor?

Manager: The tutor? He’s a local artist. You might have heard of him. Steve
Ramdhanie.

Woman: Oh possibly. How do you spell that surname?

Q3 Manager: It’s R-A-M-D-H-A-N-I-E. Got that?

Woman: Yes, thanks. Now, you’ve also got a class called ‘Clay Basics’. It’s a pottery
class, right?

Manager: Yes, that’s right.


Woman: Do we get to use a pottery wheel? Manager: Yes you do.

Woman: Oh great. But we’d be making something simple, I hope. I can’t imagine
producing a vase or anything with a handle.

Q4 Manager: No, in the first term, you’d just be producing two or three bowls. That
means learning how to shape and glaze them.

Woman: Sounds great. I guess it would be a bit of a messy activity. You wouldn’t want to
wear your smart clothes, would you?

Manager: Best not to. I’d recommend wearing something old – that you didn’t mind
getting dirty.

Q5 Woman: Well, I have a shirt like that that I could use. I’d just roll up the sleeves, I guess.

Manager: And the fee for that class is $180 per term. Woman: All right.

Manager: And the class runs on Wednesdays, 6.30 to 8.30p.m. Oh hang on, I’ve got that
wrong.

Q6 Woman: It’s on a Thursday, isn’t it?

Manager: Yes, my mistake. What else can I tell you? Oh yes, the tutor’s name is Theresa
Clark. Her works on display in reception here.

Woman: Oh great. I’ll have a look. Manager: Yeah, if you’ve got

time. Now listen and answer questions 7 to 10.

Woman: Um, now the other class I thought looked interesting was ‘Sketching
Architecture’.

Manager: Oh, yes. For that one everyone goes down to the local park, because it’s
surrounded by so many beautiful old buildings.

Woman: Oh, right. Like the old post office.

Q7 Manager: Well, I think you begin by drawing the library. There’s a good view of it from the
park, I believe.

Woman: OK, nice. Actually, come to think of it, I think I’ve seen the sketching class down
in the park when I’ve driven past. Don’t they all take a fold-up chair with them?

Manager: I believe so.

Woman: Is there anything else I ought to know?


Manager: Well, the people who do the class - they tend to make

Q8 a sandwich for themselves, and bring that along. So, I’d recommend you do that too.

Woman: Good idea. I get grumpy when I’m hungry. 3


Manager: All right, that’s $160 per term for that class, and it’s on Fridays, from 11 a.m. till
1.

Woman: So, if the classes are in the park – that’s quite a big area. Where do we actually
meet?

Manager: Good point. Everyone usually heads along just before 11a.m.,

and they meet each other at the top of Victoria Street, at the Station Q9
Road end.

Woman: So, on the corner? Great. And the tutor is ...? Manager: Annie Li.

Woman: You know what, I’ll have to think about the other two, but I’d definitely like to
enrol for Annie’s class. I’m happy to pay now.

Manager: In that case, you’re going to need Annie’s cell phone number in case you’re
running late or the class gets cancelled.

Woman: I see. OK, what’s the number, please?

Manager: It’s 021 785 6361. Just text her if there’s a problem. Now, how Q10

would you like to pay? We can...

Exercise 50:

Example Answer
Name of agent: Flagstone
Areas dealt with: 1 .................... and north suburbs
Rent: from 2 £.................... to £ .................... a
month

Depends on: the area


availability of 3 ....................
garage
Properties available: West Park Road
rent 4 £.................... a month
including 5 ....................
Tithe Road
rent £380 a month
including 6 .................... rental
Viewing meet at office on 7 .................... at 5.00 pm
arrangements:
Need: letter from bank
reference from 8 ....................
Must: give 9 .................... notice of moving in
give deposit of 10 .................... pay for
contract

Answer

1. city centre

2.  250-500

3. garden

4. £325

5. water bill(s)

6. telephone

7. Wednesday afternoon

8. your employer
9. 2 weeks’

10. 1 month’s rent

JANICE: Hello ... Flagstone.

JON: Oh hello; is that Flagstone Properties?

JANICE: Yes that’s right. Flagstone here. How can I help you?

Hello. I’m ringing just to make enquiries about renting a house. My name’s
JON:
Jon Anderson.

JANICE: Yes, Mr Anderson. What sort of thing were you looking for?

JON: Two-bedroomed house with garden.

Well. .. yes, sir, that shouldn’t be any problem ... just to let you know that our
JANICE:
main areas, the main areas we deal with, are the city centre itself...

JON: City centre ... uh-huh.

JANICE: And the north suburbs.

JON: Oh well... we were most interested in the Northern areas actually.

JANICE: Right... yes... What sort of price were you thinking of?

JON: Well... could you give me some idea?

JANICE: Certainly. It really ranges from £250 per month.

JON: Only £250?

JANICE: Yes, to about £500 depending on a number of different factors.

JON: What does it depend on?

JANICE: Well, obviously the quality of the area. And then whether there’s a garden.

JON: Well, as I said, we’d want a garden.

JANICE: And a garage pushes up the price.

JON: Right... well, we wouldn’t necessarily need one. I think about £350 a month
would be our limit.

JANICE: OK. Well... would you like to have a look at a couple of properties, sir?

JON: Yes, that’d be great.

JANICE: Looking at our files ... I think we’ve got two which might suit you ...

JON: Hang on. I’ll just get a pen. Right.

JANICE: OK. Well, there’s one on West Park Road which is £325 a month.

JON: Are the bills included?

JANICE: Well, that one just includes the water bill.

JON: OK, right.

JANICE: And the second house is in Tithe Road. I’ll just spell that for you ... OK?

JON: T-I-T-H-E Road.

JANICE: Got that. And how much is that one?

JON: That’s £380.

JANICE: 380. Is that including water?

JON: No, I’m afraid not, but it does include the telephone rental.

JANICE: Oh well, that’s not too bad then. So, ...

JANICE: So, when would you be available to see them?

JON: Well, I’ll be in town next week... say... Thursday?

No, I’m sorry we don’t have any availability for Thursday. How about
JANICE:
Wednesday afternoon?

JON: OK. That’s fine. Would 5.00 be OK?

JANICE: Yes, fine. 5.00 it is. Just come to the Flagstone Offices.
Oh, before I forget. What sort of things do I need to get done... to rent with
JON:
you?

JANICE: Well, the most important thing is a letter from your bank...

JON: No problem...

JANICE: And then a reference letter from your employer.

JON: Yes, that’s OK.

JANICE: Great, and then we would need you to give 2 weeks’ notice of moving in...

JON: Right... 2 weeks’ notice. And what about a deposit?

JANICE: That’s one month’s rent, whatever the amount is.

JON: OK. One month. Is that it?

JANICE: No, sorry, one more... you will have to pay for the contract.

Oh yes. I’d forgotten about that. OK, fine. So I’ll start arranging those, and
JON:
I’ll...

JANICE: ... I’ll see you next week.

JON: Yes. Thanks very much. Bye.

Exercise 51.

Questions 11–15

Match the creatures to their behaviour.

Write the correct letter, A, B, C or D next to each description.

A swifts
B bats
C corkroaches
D centipedes

11 feed on the guardrail beside the path

12 are dangerous to people


13 collect in large groups

14 fly only at night

15 live on the roof of the cave

Questions 16 and 17

Answer the questions below. Write ONE WORD ONLY for each answer.

16 What is the guano from the caves used for?

………………………………….

17 What are the birds’ nests from the cave used for?

………………………………….

Questions 18–20

Complete the sentences below. Write NO MORE THAN TWO WORDS for each answer.

In the cave, you should:

18 carry a .…………………

19 wear a .………………… and a hat

20 keep to the .………………… at all times

11 C
12 D
13 B
14 B
15 A
16 fertiliser / fertilizer
17 soup
18 (working) light / torch
19 (waterproof) jacket
20 (marked) trails / path

Good morning, everyone. Welcome to our visit to one of the most famous caves in Vietnam.
As you know, this cave is famous for its wildlife, and one of the creatures you will observe in
here is the small cave cockroach. They live mostly on the bird and bat droppings that are so
plentiful in the caves. The guardrails along the trails are covered with these droppings, and
this makes a feast for the cockroaches, so be careful where you put your hands. They will
not harm you, but it can be a shock if you touch them!
Once you are in one of the main caves, look out for the green centipedes. They will not be
on the trail but can often be seen on the wall close by. They feed on other insects and are
fascinating to look at because of their colour, and, of course, their many legs. Please, please
do not try to pick one up though; these centipedes have a very nasty poisonous bite.
There are also deep red millipedes. These have a fully rounded shape, and they look like a
streamlined elongated train with a hundred or so closely packed legs extending right and left.
When you get to the large high caves, you should look right up above you for the swifts and
bats. The bats in this cave are mostly a type of dwarf bat, which are common in this part of
the world. They will be clustered high up against the walls – maybe a hundred or two
hundred together. They look like shadows high on the walls of the cave. They are likely to
be very quiet right now, but because there are so many of them together you will have no
difficulty identifying them. They sleep all day until they all leave the cave in a massive flock
on their nightly hunt for flying insects.
The swifts are the creatures you can see flying around during the day, especially if they have
young ones to feed. They can navigate in the darkness here, and will fly outside in ones and
twos at dusk to catch small winged insects like mosquitoes. However they tend to return
before it is pitch black outside, and they do not hunt at night. The swifts make nests, usually
higher up on the ceiling of the cave.

The paths tend to run around the edges of the large caves. Mostly this is because the centre
is a mound of guano – the bird and bat droppings. This is also the source of the strong smell
inside the caves. You may not like this smell, but the locals know its economic value. They
have harvested the products of these caves for centuries. The guano is very valuable as
fertiliser, and so it’s collected each year, once the young birds have grown and the swifts
have abandoned their nests.
The guano is not the only valuable by-product of the wildlife here. As you travel through the
caves, you will notice some bamboo structures. These very flimsy looking sets of poles that
go a full hundred metres, right up to the roof, are what the locals climb up to gather the
swift’s nests. These are even more valuable than the guano, as they are the main ingredient
in bird’s nest soup.
Before you begin, it’s time for some safety instructions. As you probably know, this is a huge
limestone cave that goes about one kilometre back into the hills and in places it’s a hundred
metres in height and three hundred metres wide. There is no need to crawl around in here
as you do in other caves, but it is dark inside, of course … that’s why I insisted that you bring
a working light – please check that it shines brightly, and ensure that you stay together with
others who have a good torch. In one of the larger areas of the cave, the roof is pierced so
some sunlight will get through. It is best to turn your torches off if you can see well, and save
your batteries.
It is a good idea to put your waterproof jacket on now. The walls may be wet but that is not
the main reason for the jacket. The bats and birds do excrete, and they are above you, so
just in case … and of course, your hat or hood also keeps you safe from animal droppings.
It’s not advisable to use the guardrails as handholds. There are lots of droppings on those
rails, and don’t forget the cockroaches!
You absolutely must follow the marked trails. The guardrails on either side are put there so
that you cannot mistake them. We take no responsibility for your safety if you go over or
under the rails into other cave areas.
Keep your torches shining on the path whenever you are moving, just to be sure of your
footing, and don’t try to go too fast. You might trip, and you will certainly miss some of the
fascinating wildlife in the cave.

Exercise 52.
Questions 11–15
Choose the correct letter A, B or C.
11 The guided bushwalk is suitable for
A adults only
B children over 12 and adults
C children over 8 accompanied by a parent
12 On the bird observation outing, it is recommended that you have
A waterproof footwear
B a bird identification book
C binoculars
13 For the trip to the sand dunes, a company will donate
A water
B tools
C gloves
14 The bush tucker excursion will cost (per person)
A $15
B $12
C $7
15 The deadline to register for the bush tucker outing is
A 25 November
B 15 November
C 10 November
Questions 16–20
Complete the table below.
Write NO MORE THAN TWO WORDS AND/OR A NUMBER for each answer.
Activity Leader Date Venue Time
Bush walk Glenn Ford 16 …………… Springvale 17 …… - 1pm
Bird watching Joy Black, club 10 September Camford 4.30 – 6.30 pm
18 ……………
Sand dunes Rex Rose 26 November 19 …………… 8.30 – 10.30 am
Bush tucker Jim Kerr, ranger 3 December Carson Hills 10 am - 20……

11. B
12. A
13. A
14. B
15. C
16. 2 July / 2nd July
17. 9.15/nine fifteen am/a.m.
18. president
19. observation hut
20. 2/two pm/p.m.

Hi everyone. It’s good to see such a big turnout at our Nature Club session for
June. Just before we start this evening’s workshop, I’d like to draw your attention to
what we have in store for you in the second half of the year.
First of all, the guided bushwalk – this is always a favourite – starting out on the
Springvale plateau and continuing down into a section of the state conservation area.
Last year, we invited children aged 8 and over if they came with a parent, but the track
has been washed out in a few places since then and it can be quite rough, so this year we
considered restricting it to adults only … however, on reconsideration, the committee
has now decided to recommend it for all bushwalkers who are over the age of 12.
Another very popular option is the bird observation walk. We’ll be searching for
both migratory and native birds as we walk through tidal marshlands and mangroves
and you can expect to get your feet uncomfortably wet and muddy if you don’t wear
rubber boots – these are a must. The leader will have a strong pair of binoculars, so
we’ll rely on her to name the species for us … and we’ve ordered some bird
identification books that you may wish to purchase at a later date.
From the bush to the swamp, and now to the sand dunes … Our leader will help
us identify plants native to the local area as well as some invasive weed species. We’ll
be asking for volunteers to help pull out the weeds where possible, so a pair of sturdy
gardening gloves is essential. Spades and other tools will be provided. It could get very
hot and you’ll need water – plenty of it – but a local business owner is willing to
provide bottled water free of charge.
The next outing, bush tucker, is a new one – have you ever wondered what life
in this country would have been like two hundred to two thousand years ago? Well,
come on this trip and you’ll find out how the indigenous inhabitants used local plants as
food and medicine.
Because lunch is included in this trip, there will be a small charge per person.
We had originally thought 7 dollars would cover the basics – sausages and bread
followed by tea and coffee, but then we thought a few different cuts of meat and salad
would be nice and that brought the price up to 12 dollars a head. At one stage we even
contemplated including seafood, but that would have been a bit too expensive – around
15 dollars – so meat and salad it is. We expect this to be a popular event, so we’ll need
advance bookings to organise the catering. Please let us know your intentions by the
10th of November, and be aware that we’ll require pre-payment by the 15 th of
November. You can still change your mind and get a refund up to the 25 th of November
but after that date, if you pull out, you’ll forfeit the money paid.
Well, now … if you can give me a few more minutes of your time, I’ll fill in a
few details for you. The bush walk, led by Glenn Ford, is first up in July – on the
second of the month. It’ll start from Springvale as usual but this year we’ll be setting
off in the morning, at nine fifteen, and we’ll get back at one in the afternoon.
The bird watching expedition is on the tenth of September at Camford and the
leader is the president of the Nature Club, our very own Joy Black. If you have any
questions at all about bird life, Joy is the person to ask. This is a twilight outing from
4.30 to 6.30.
Next up is the trip to the sand dunes on 26 November with Rex Rose. A bit of an
early start – especially for those of you with a fair way to travel – but we’ll meet at the
observation hut at half past eight. That’s the observation hut, 8.30 till 10.30, and even
at that time of the morning it’ll be very hot, so come prepared!
The last trip on the programme is the bush tucker excursion on the third of
December with ranger Jim Kerr. This will be at Carson Hills and the presentation and
demonstration will take place from 10 am till 11.30 but be prepared to stay on for the
barbecue and bush tucker lunch at 12 o’clock. I expect we’ll wind up at two and you
can head for home at that time.
Well, that’s all I have to tell you. A booklet will be mailed out to you later with
those events, dates and times – but don’t wait, put them on your calendar now.

Exercise 53.
Questions 11-14
Complete the notes below.
Write TWO WORDS AND/OR A NUMBER for each answer.

Royal Victoria Nursing College Factsheet


- Opened in 11 ……………
- 7-storey building
- 85 individual rooms, including 4 12 ………… and 8 wards.
- Cost at time of construction: 13…………… pounds.
- Most popular period: 1920s. Students from all over the world, especially Canada, New Zealand,
India and 14 ……………
- Decision to close college made in late 1990s; new college located in Derby.
Questions 15-17
What is the main area of responsibility of each of the following people? Choose THREE
answers from the box and write the correct letter, A-G, next to questions 15-17.

Area of responsibility
A Marketing
B Media
C Sales
D Museum maintenance
E Exhibitions
F Human Resources
G Arts

People
15. Susan
16. Vanessa
17. Gordon

Questions 18-20
Complete the notes below. Write ONE WORD AND/OR A NUMBER for each answer
NATIONAL MUSEUM OF NURSING FREE MONTHLY LECTURES
Subject Data/Time Speaker
The history of nursing 18 September 15th, 7pm Dr. Mark Fisher
…………… at the college
Nursing in 19 …………… October 10th, 7pm Mrs. Margaret Hall
The 20 …………… of nursing November 12th, 7pm Mrs. Nigella Smith

11. 1857
12. lecture theatres
13. 2 million / 2m
14. Australia
15. B
16. F
17. E
18. research
19. India
20. future
Hello everyone, and thanks for coming on this very exciting day as we launch the U.K.'s first
National Museum of Nursing here in London. My name's Mark Fisher and I'm the Director of the
museum.
As most of you will know, this building began life as the Royal Victoria Nursing College. As
you might have guessed from the name, it was officially opened during the reign of Queen
Victoria in the 19th century. At that time, there was a strong movement calling for increased
professionalism in the field of nursing, so in 1855 the relevant government department granted
permission for this institute of education and research to be established, here in London on the
bank of the River Thames. It was officially opened two years later.
At seven stories high, it was one of the tallest buildings in London at the time of construction. It
contained 85 individual rooms, including 25 classrooms, 4 lecture theatres and 8 wards for
nursing practice. Most of the other rooms were used as offices and for storage space. The
construction of the building cost 2 million pounds, around 800 million pounds in today's money,
which shows that the development of the nursing profession was a top priority at the time.
The golden age for this institute was between the two World Wars. In the 1920s it was
considered the leading nursing college in the world. Nurses from all over the world came here to
study, especially from India, Canada, Australia and New Zealand. The U.S.A. and some of our
European neighbors like France also sent trainees, but not in the same numbers.
So, that's some information about the history of the building and why it was so important to
continue its legacy. Now I'd like to move on to introduce some of our new department managers
who will be in charge of the day-to-day running of this museum, and explain what their roles will
be.
First, please welcome Susan Jones, who will be responsible for media relations. Her degree
was in Marketing but she has spent most of her career working for some of our most prestigious
national newspapers as an Arts correspondent. Susan has a deep knowledge of the museum
world and will be making sure that our institution maintains a high profile and hopefully draws
in the crowds.
Next up, where's Vanessa? Oh, there she is. Well, Vanessa Redmond has had a long and
colourful career in a number of fields, starting off in Sales and eventually going back to
university to get a postgraduate degree. As our Director of H.R., she will be ensuring that we
attract the most dynamic people to work in our museum and contribute to our success.
Last but by no means least, I'd like to introduce Gordon Timms...well, as such a well-known
figure in the U.K. arts world, Gordon really needs no introduction. Having worked in museum
maintenance for most of his career, I am very glad that he has now decided to join us as our
exhibitions manager. This will be an important focus for our museum, especially during the
initial period as we try to get as many people through the door as possible and spread the word.
We already have a number of exciting projects in the pipeline and Gordon will be telling you
more about those in due course.
Before 1 finish today, I'd just like to point you in the direction of our new monthly lecture event.
Starting from next month we will be inviting prominent speakers to give talks on a nursing-
related topic, always beginning at 7pm. To kick things off, I will be giving a talk on the college's
achievements in nursing research, dating back to the 19th century. That will be held next month,
September the 1511. The following month on the 10th Mrs. Margaret Hall, who has spent most
of her career in Singapore, will be reporting to us on a fascinating year she recently spent in the
Indian countryside. And on November the 12th, Mrs. Nigella Smith, who has written a number
of well-known books on nursing, will be giving us her views on nursing's future, which is also
the topic of her new publication. Do come along, tickets are free on a first come first served
basis.
Well, that's all I'd like to say today. As I said, thanks very much for coming today and I hope you
will continue to support the museum in the future. Now, before we finish, does anyone have any
questions?

Exercise 54:

Write NO MORE THAN THREE WORDS for each answer.

11  Who is Mrs Sutton worried about? .................... .

12  What is the name for a group of family doctors working in the same building
together? .................... .

Questions 13-18

Complete the form below.

Write NO MORE THAN THREE WORDS AND/OR A NUMBER for each answer.


Name of Health Number of Information about
Other information
Centre doctors doctors

Dr Jones is good
Appointment with 16  ....................
Dean End 13 .................... system 15 ....................patients.
than South Hay Dr Shaw is good with
small children.

Dr Williams helps
Building less modern
South Hay 14 .................... people
than Dean End
with 17 ....................

Questions 18-20

Question 18

Write NO MORE THAN TWO WORDS OR A NUMBER.

Doctors start seeing patients at the Health Centre from .................... o’clock.

Question 19

Choose TWO letters A-E.

Which TWO groups of patients receive free medication?

A people over 17 years old

B unemployed people

C non-UK residents

D people over 60 years old

E pregnant women

Question 20

Write NO MORE THAN TWO WORDS OR A NUMBER.

The charge for one item of medication is about £ .................... .


Answer:
11. her daughter
12. practice
13. 6
14. 4
15. better
16. elderly
17. back
18-19. B-E
20. 5 pounds
MRS SMITH
Hello, Mrs Sutton. Come in. How are you settling in next door? Have all your things from
Canada arrived yet? I thought I saw a removals van outside your house yesterday afternoon.

MRS SUTTON
Yes. They came yesterday. We spent all day yesterday arranging them. It’s beginning to feel a
bit more like home now.

MRS SMITH
That’s good. Look, come in and sit down. Are you alright? You look a bit worried.

MRS SUTTON
Well, I am a bit. I’m sorry to bother you so early, Mrs Smith, but I wonder if you could help me.
Could you tell me how I can get hold of a doctor? Our daughter, Anna, isn’t very well this
morning and I may have to call somebody out. She keeps being sick and I am beginning to get a
bit worried. I just don’t know how the health system works here in England. All I know is that
it’s very different from ours back in Canada.

MRS SMITH
Well, I don’t know really where to start. Let me think. Well, the first thing you have to do is find
a family doctor - sometimes we call them general practitioners as well - and register with him or
her. If you live here, you’ve got to be on a doctor’s list. If you’re not, things can be a bit difficult.
Nobody will come out to you if you’re not registered. Anyway, they work in things called
practices. Sort of small groups of family doctors all working together in the same building. Now
what you’ve got to do this morning is register with one of them.
There are two practices near here, so we’re quite well off for doctors in this part of Manchester.
There’s the Dean End Health Centre about ten minutes’ walk away and there’s another practice
in South Hay. That’s about five minutes away going towards the town centre. We’re registered at
the Dean End one, but they’re both OK. There are about six doctors in our practice and four in
the other. So ours is quite big in comparison. And the building and everything’s a bit more
modern. South Hay is a bit old-fashioned but the doctors are OK. Their only problem is that they
don’t have a proper appointment system. Sometimes you have to wait for ages there to see
someone.
Anyway, you go to the receptionist in whichever health centre and ask her to register you with a
doctor there. You have to fill in a form, but it doesn’t take long. Ours is called Dr Jones and
we’ve been going to him for years - ever since we moved here fifteen years ago. I wouldn’t say
he’s brilliant but I suppose he’s alright really. We’re used to him now. They say he’s very good
with elderly people, but he does tend to get a bit impatient with children. Listen, the one who’s
supposed to be really good with small children is Dr Shaw. I’ve heard lots of people say that.
She’s young and she’s got small children of her own. So you could try registering with her.
And if her list is full, I heard somebody say the other day that there’s a really nice young doctor
at South Hay, a Dr Williams. He holds special clinics for people with back trouble. But that’s not
really your problem, is it?
MRS SMITH
If you want a doctor to visit you at home, you have to ask for a home visit. You’re supposed to
do that before 10.30 in the morning, but obviously, if it’s an emergency, you can phone at any
time, night or day. It might not be your doctor that comes, though. It’s quite often one of the
other doctors in the practice. It doesn’t really seem to make much difference.
Otherwise you make an appointment to see your doctor at the health centre. You usually get seen
the same day. Not always of course, but usually, as I say. They hold surgeries between 9 and
11.30 every weekday, and from 4 to 6.30 Monday to Thursday. Saturdays are only for
emergencies.
When the doctor sees you, he gives you a prescription. He writes what medication you need on it
and you take it to a chemist’s shop. There’s one opposite the centre.
If it’s for a child under 16, you don’t have to pay. So if it’s for Anna. there’s no problem. The
same thing goes if you’re unemployed or retired, or if you’re pregnant. Just as well because it’s
not cheap. You pay the same price for each item the doctor has prescribed. At the moment it’s
something like £5 per item. So you pay for the medication but the consultation with the doctor
doesn’t cost you anything. It’s completely free as long as you’re a resident here. You’re going to
be here for three years, aren’t you? So there shouldn’t be any question of you paying anything to
see the doctor. So that’s one less problem to worry about.
Look, Mrs Sutton. If you want, I’ll sit with your daughter for half an hour if you want to go
down to the health centre to register. It’s no trouble really, don’t worry.

MRS SUTTON
Are you sure you wouldn’t mind? That would really help me a lot. I’ll ask them if they can send
someone round later to see Anna. I think I’ll try the Dean End Centre.

MRS SMITH
Good idea. Don’t worry about Anna.

MRS SUTTON
Right. I’ll be back as soon as I can.

Exercise 55:

11) The next meeting of the soccer club will be in the .................... in King's Park on 2 July.

12) The first event is a .....................

13) At the final dinner, players receive .....................

Questions 14-17

Complete the table below using NO MORE THAN THREE WORDS AND/OR A
NUMBER for each answer.

Training Session
Competition Number of Team Games Begin
(in King's Park)

Junior 14 ......................... 8.30 am 15 .........................


Senior 16 ......................... 2.00 pm 17 .........................

Questions 18-20

Complete the table below using NO MORE THAN THREE WORDS AND/OR A
NUMBER for each answer.

Name of Office Bearer Responsibility


Robert Young: President to manage meetings
Gina Costello: Treasurer to 18 ....................
David West: Secretary to 19 ....................
Jason Dokic: Head Coach to 20 ....................

Answer:
11. clubhouse
12. picnic
13. prizes
14. 10
15. Wednesday afternoon
16. 4
17. Sunday afternoon
18. collect the fees
19. send out newsletters
20. supervise the teams
Good evening everyone and thank you for coming to the Soccer Club meeting. It's good to see so
many parents and children here tonight, and I know you are looking forward to a great football
season. Now I'd like to take a few minutes to tell you about some changes to the Soccer Club for
the coming season.

Now, this season we'll be playing all our matches for both the junior and senior competitions at
King's Park, instead of Royal Park, which was used last season.
Now for meetings, we're going to use the clubhouse in King's Park, and the next meeting will be
held in the clubhouse on the 2nd of July.

As usual, we hope to begin the season with a picnic next Saturday at the clubhouse. Please try
and come to the picnic as it's always good fun. The last week of the season we usually have a
dinner and presentation of prizes to the players, and more information about this will be given to
you later in the season.

This season we have more teams than ever. We hope to have ten teams, instead of five, in the
junior competition and they will play on Saturday mornings, beginning at 8.30 a.m..
Training sessions will be held in King's Park on Wednesday afternoons for the juniors and they
will be wearing red shirts again this year.

In the senior competition, there'll be four teams, the same as last year, and their games will be
played on Saturday afternoons starting at 2.30, no sorry, it will be a 2 o'clock start, and the
training session for seniors is planned for Sunday afternoons.

Now I'd like to introduce you to the new committee for the Soccer Club for this season. Firstly,
let me welcome Robert Young, the new President, who will manage the meetings for the next
two years. Robert's son has been playing football with the club for over five years now, and
many thanks to Robert for taking on the job of President.

Next we have Gina Costello, she's the treasurer, and she will collect the fees from you for the
season. Please try and give Gina your fees as early as possible in the season, as the club needs
the money to buy some new equipment.

Then there's David West who has volunteered to be the club secretary, and one of the many jobs
he will have is to send out newsletters to you regularly. If you have any information that may be
useful, please let David know so that it can be included in these newsletters.

Also I'd like to introduce you to Jason Dokie who is the Head Coach. For all the new members
here tonight, this is the third year thet Jason has been with us as Head Coach, and we are very
lucky to have such an experienced coach and former player at our club. He will continue
to supervise the teams, at training sessions and on match days.

Now before we finish and have some refreshments, does anyone have any questions they'd like
to ask the new committee?
Exercise 56:
Questions 11-13

Complete the sentences below. Write NO MORE THAN THREE WORDS for each answer.

11) .................... are often known by their famous bridges.

12) The speaker compares a bridge to a cathedral or .....................

13) Sydney Harbour Bridge is nicknamed .....................

Questions 14-18

Complete the form below.

Write NO MORE THAN THREE WORDS AND/OR A NUMBER for each answer.

Date Event

1916 (14) .................... agreed to finance bridge


Contract signed with engineering firm

(15) .................... 1926 Construction involved:


• knocking down (16) ....................
• creation of many jobs

1932 Bridge completed at a cost of (17) £....................

Opening ceremony Ribbon cut by a man riding


March 1932
a (18) ....................

Questions 19 and 20

Complete the questions below.

Write NO MORE THAN THREE WORDS AND/OR A NUMBER for each answer.

19  How long is the tunnel? ....................

20  Name ONE thing the tunnel can withstand. ....................

Answer:
11. cities
12. a palace
13. the coat hanger
14. government
15. 1924
16. 800 houses
17. 9.5 million
18. horse
19. 2.3 kilomitres
20. a ship /an earthquake
Announcer
The Sydney Harbour Bridge is nearly three-quarters of a century old and, to help celebrate this
important occasion, our reporter Sarah Chambers has compiled this brief history of her favourite
bridge.

Sarah
A bridge is more than just a crossing over a river or a waterway - it is a landmark in its own
right; a landmark which allows us to identify one city from another. Think, for instance, of the
Bridge of Sighs in Venice, or the magnificent Charles Bridge in Prague. Each of these cities can
be recognised by their famous bridges. The Golden Gate Bridge in San Francisco is another
example of a city known by its bridge. But in addition to this, a bridge is a kind of ornament for a
city, similar, if you like, to a cathedral or a palace.

Here in Sydney we may not have our own palace, but we do have our famous and much loved
bridge -The Sydney Harbour Bridge, which is sometimes affectionately known as 'the coat
hanger' because of its arched shape. It was built back in the 1930s, and so the bridge is coming
up for a significant birthday. Let's have a little look at its history.

[Pause]

Although the idea of building a crossing over Sydney harbour had been discussed many years
earlier, it wasn't until the year 1916 that the state government agreed to allocate some money for
the construction of a bridge.

The chief engineer for the bridge was a man tailed Dr John Bradfield, a brilliant engineer who
supervised the entire project from beginning to end. First they had to decide on a design, so he
organised an international competition to choose a design, and ultimately got the one he wanted.
The job went to a British engineering firm and the contract was sinned in 1924. The design he
chose was the single-arch bridge that you see today, made of steel, with a tower at either end.

In 1926, construction finally began. The first thing they had to do was demolish 800
houses around the site where the towers were to be built. The poor families, however, never
received any compensation for this! But the project created thousands of jobs - much needed in
those difficult times. Of course, like all projects of this size, it took much longer to build than
originally planned - it was supposed to have been finished by 1930 - but actually it wasn't
completed for another two years. It also cost twice as much as the original quote, coming in
at £9.5 million instead of the agreed contract price of £4.2 million! But what's new?

The opening ceremony took place on 19 March 1932, and a large crowd gathered for the
occasion. The Premier of the State was just about to cut the ribbon when suddenly a man rode
through the crowd mounted on a horse and slashed the ribbon with his sword. He wanted to be
the first to cut the ribbon. Anyway, they tied the ribbon back together and the ceremony
continued. The man on the horse was fined £5 for his offensive behaviour!

Since then, millions of cars have crossed the bridge, each paying a toll to do so. By the early
1980s the government had paid off the loan #or the money they'd borrowed al! those years
before, but motorists continued to pay to cross from north to south. This money was
subsequently used to build a tunnel under the harbour to reduce the amount of traffic on the
bridge.
[Pause]

The tunnel was opened in 1992 and cost $544 million. It is 2.3 kilometres long and is equipped
with all the latest technology, including closed circuit television to monitor any problems. And it
has most definitely reduced the load on the bridge, as it carries around 75,000 vehicles each day
which would otherwise have to use the bridge. And it's apparently strong enough to withstand
the impact of a ship or even the impact of an earthquake.

The tunnel has been a welcome solution to Sydney's traffic problems, but, of course, a tunnel
could never compete with a bridge as a landmark for any city. So let's wish the bridge a very
happy birthday!

Exercise 57:

Question 11

Choose the correct letter, A, B or C.


11.    According to the speaker, the main purposes of the park are

A. education and entertainment.

B. research and education.

C. research and entertainment.

Questions 12-14

Label the plan below. Write NO MORE THAN TWO WORDS for each answer.
12. …………………….

13. …………………….

14. …………………….

Questions 15-20

Choose the correct letter, A, B or C.

15.    When are the experimental areas closed to the public?

A. all the year round

B. almost all the year


C. a short time every year

16.    How can you move around the park?

A. by tram, walking or bicycle

B. by solar car or bicycle

C. by bicycle, walking or bus

17.    The rare bread animals kept in the park include

A. hens and horses

B. goats and cows

C. goats and hens

18.    What is the main purpose of having the Rate Breeds Sections?

A. to save unusual animals

B. to keep a variety of breeds

C. to educate the public

19.    What can you see in the park at the present time?

A. the arrival of wild birds

B. fruit tree blossom

C. a demonstration of fishing

20.    The shop contains books about

A. animals

B. local traditions

C. the history of the park.


Answer:
11. B. research and education.
12. (the) Forest
13. Fish Farm(s)
14. Market Garden
15. C. a short time every year
16. A. by tram, walking or bicycle
17. C. goats and hens
18. B. to keep a variety of breeds
19. C. a demonstration of fishing
20. A. animals
Welcome to Green Vale Agricultural Park. As you know, we have only been open a week so you
are amongst our first visitors. We have lots of fascinating indoor and outdoor exhibits on our
huge complex, spreading hundreds of  hectares. Our remit is to give educational opportunities
to the wider public as well as to offer research sites for a wide variety of agriculturists and
other scientists (Q11).

Let’s start by seeing what there is to do. As you can see here on our giant wall plan, we are now
situated in the Reception block…here. As you walk out of the main door into the park there is a
path you can follow. If you follow this route you will immediately come into the Rare Breeds
section, where we keep a wide variety of animals which I shall be telling you a little more about
later. Next to this…moving east…is the large grazing area for the rare breeds. Then further
east…in the largest section of our Park is the Forest(Q12) Area. South of the grazing area and in
fact just next to the Reception block is our Experimental Crop Area.  In the middle of the Park…
this circular area is our lake…These two small rectangular shapes here …are the Fish Farms
(Q13) where we rear fish for sale. To the east of those is the marsh, is our Market
Garden(Q14) area, growing vegetables and flowers.

All these areas can be visited by the general public for almost all the year…although …please
take note of  the large signs at the entrance to each area which tell…which tell you when certain
areas are being used for particular controlled experiments and are therefore temporarily out of 
bounds(Q15) to the public.

You can see for yourself  what a huge area the park covers and a key question is always, how can
we move around? Well you have a choice of means …all environmentally friendly …cars are
banned in the park. We have bicycles (Q16) which you can hire behind the Reception block…
here…the healthy ones of  you can go on foot(Q16)and finally there’s our electric tram (Q16),
powered from solar cells. You find more information about this at the front entrance.

A good place to start on your tour is the Rare Breeds section. We keep goats (Q17), sheep
and hens (Q17)and other kinds of poultry. We are also thinking of bringing in cows and horse
but we do not, as yet, have facilities for these bigger animals. The animals are fed in public twice
a day and a short lecture given on their feeding habits and nutritional needs. These are very
popular with the public but of course we mustn’t lose sight of the main purpose of  having this
section, not as such to preserve rare animals but to maintain the delivery of  breeds (Q18)to
broaden the gene pool for agricultural development. Green Vale changes with the seasons with
different events happening at different times of the year. May will be perhaps our most
spectacular month with the arrival of the Canada geese and when our fruit trees will be in full
blossom, but there are interesting events on all year round ….for example John Havers, our
expert fly fisherman, is currently giving displays(Q19) on the lake. Each of the sections has
its own seasonal calendar…please consult the summary board at the main entrance. And the final
section, as we return to the Reception blocks, is the orchard.

Do take time to browse round our shop…there is a wide selection of books on wildlife (Q20),
some of  them written by local authors, and the history of farming, including organic farming,
something which the park  will be diversifying into the coming months.
Exercise 58:

Question 11-15

Complete the sentences below. Write NO MORE THAN TWO WORDS AND / OR A
NUMBER for each answer.
 The Dinosaur Museum

11. The museum closes at ……………… p.m. on Mondays


12. The museum is not open on ………………
13. School groups are met by tour guides in the ………………
14. The whole visit takes 90 minutes, including ………………minutes for the guided tour.
15. There are ……………….behind the museum where students can have lunch.
Question 16-18

Choose THREE letters, A-G


Which THREE things can students have with them in the museum?

Target : 16  17   18

Characteristics

A. food

B. water

C. cameras

D. books

E. bags

F. pens

G. worksheets

Answer:
16: ………………………

17: ………………………

18: ………………………

Questions 19 and 20

Choose TWO letters, A-E.


Which TWO activities can students do after the tour at present?

Characteristics

A. build model dinosaurs

B. watch films

C. draw dinosaurs

D. find dinosaur eggs


E. play computer games

Answer:
19: ………………………

20: ………………………

Answer:
11: 1.30 

12: 25 December

13: car park / parking lot

14: 45

15: tables

16: C   17: F   18: G

19: B  20: E
Hello, and thank you for asking me to your teachers’ meeting to talk about the Dinosaur Museum
and to tell you about what your students there.

Well, let me give you some of the basic him formation first. In regard to opening hours, we’re
open every day of the week from 9.00 am to 8.00 pm except on Mondays when we close at
1.30 pm (Q11) . And, in fact the only day in the year when we’re closed is on the 25th of
December (Q12). You can book a guided tour for your school group any time that we’re open.

If you bring a school group to the museum, when you arrive we ask you to remain with your
group in the car park. One or more of  the tour guides will welcome you there (Q13) and brief
you about what The tour will be about. We do this there because our entrance is quite small and
we really haven’t got much room for briefing groups in the exhibition area.

As far as the amount of time you’ll need goes, if you bring a school group you should plan on
allowing a minimum of 90 minutes for the visit. This allows 15 minutes to get on and off  the
coach, 45 minutes (Q14) for the guided tour and 30 minutes for after-tour activities.

If you’re going to have lunch at the museum you will, of course, have to allow more time. There
are two café in the museum, with setting for 80 people. If you want to eat there you’ll need to
reserve some seating, as they can get quite crowed at lunch time. Then outside the museum at
the back there are tables (Q15), and students can bring their own lunch and eat it there in the
open air.

When the students come into the museum foyer we ask them to check in their backpacks with
their books, lunch boxes, etc, at the cloakroom before they enter the museum proper. I’m afraid
in the past we h have had a few things gone missing after school visits so this is a strict rule.
Also, some of the exhibits are fragile and we don’t want them to be accidentally knocked.
But we do provide school students with handouts with question and quizzes on them (Q16) .
There’s so much that students can learn in the museum and it’s fun for them to have something
to do. Of course they’ll need to bring something to write with for these (Q17). We do allow
students to take studentsto take photographs(Q18) . For students who are doing projects it’s
useful to make some kind of visual record of what they see that they can add to their reports. And
finally, they should not bring anything to eat into the museum, or drinks of any kind.

There are also a few things the students can do after the tour. In the threatrette on the ground
floor there are continuous screenings of short documentaries about dinosaurs which they can
see (Q19) at any time. We used to have an activity room with more interactive things like
making models of dinosaurs and drawing and painting pictures, even hunting for dinosaurs eggs,
but unfortunately the room was damaged in a bad storm recently when water came in the roof, so
that’s closed at the moment. But we do have an  IT centre where students have access to CD
ROMs with a range of  dinosaur games (Q20). These games are a lot of fun , but they also
teach the students about the lives of dinosaurs, how they found food, protected their habitat,
survived threats, that kind of things.

And …..I think that’s all I have to tell you. Please feel free to ask any questions if you would like
to know anything else…
Exercise 59:
Question 11-20

RIVENDEN CITY
THEATRE

A doubled in number

B given separate entrance

C reduced in number

D increased in size

E replaced

F strengthened
G temporarily closed

Part of the theatre

11 box office .................... .

12 shop .................... .

13 ordinary seats .................... .

14 seats for wheelchair users .................... .

15 lifts .................... .

16 dressing rooms .................... .

Questions 17-20

Complete the form below.

Write NO MORE THAN TWO WORDS AND/OR A NUMBER for each answer.

Play Dates Starting time Tickets available Price

October 13th to for 19 ...............


Royal Hunt
18 ............... pm 20 £...............
of the Sun
17 ............... and ...............
Answer:
11.B
12.G
13.C
14.A
15. E
16.D
17. October 19th
18. 7pm
19. Monday-Thursday
20. 18

And here on Radio Rivenden we have Lynne Rawley, the Public Relations Officer of our own
Rivenden City Theatre. Hello, Lynne.LYNNE:Hello.MAN:Now, the theatre is reopening soon
after its three-year redevelopment programme, isn't it?LYNNE:That's right, and there are a lot of
improvements. The first thing people will see when they go in is that the foyer has been
repainted in the original green and gold. Then the box office has been reoriented, with its own
access from the side of the building instead of through the foyer, which means it can be open
longer hours, and has more space, too.
The shop that used to be in the foyer, which sold books and CDs, is the one part of the
redevelopment which isn't yet complete. The plan is to find new premises for it near the theatre,
and we've had difficulty finding somewhere suitable. We hope to reopen the shop in the next few
months.MAN:Will audiences find any difference in the auditorium?LYNNE:Yes, we've
increased the leg-room between the rows. This means that there are now fewer seats but we're
sure audiences will be much happier. And we've installed air conditioning, so it won't get so hot
and stuffy.
We already had a few seats which were suitable for wheelchair users, and now there are twice as
many, which we hope will meet demand. Something else that will benefit audiences is the new
lifts. The two we used to have were very small and slow. They've now gone, and we've got much
more efficient ones.MAN:Anything for the performers?LYNNE:Yes, we've made a number of
improvements backstage. The small, dark dressing rooms we used to have have been converted
into two large airy rooms, so they're much more comfortable now. And the state of-the-art
electronic sound and lighting systems have been installed.  MAN:OK, so what's the first play that
audiences can see when the theatre reopens?LYNNE:We've got a very exciting production of
Peter Shaffer's Royal Hunt of the Sun, which is currently touring the country. That starts on
October the 13th and runs till the 19th. We're experimenting a bit with the time the curtain goes
up.
We used to start all our performances at 7.30, but that made it difficult for people to go home by
public transport, so instead we're beginning at 7, because at 9.45, when it finishes, there are still
buses running. Tickets are already selling fast. The Friday and Saturday performances sold out
almost immediately and, in fact, now there are only tickets for Monday and Thursday.MAN:How
much are they?LYNNE:We've introduced a simpler price structure. Ticket prices used to range
from £6 to £30 but now they're all £18. They're available from the box office, in person, by
phone, fax or post, or online.MAN:OK, Lynne, now if you'd like to give the contact details for
the theatre...
Exercise 60:

Questions 11 – 15
Choose the correct letter, A, B, or C.
11   The company deals mostly with:
A   Big cities.
B   Nature holidays.
C   Nepal.
12   The overseas consultants deal mostly with:
A   Asia.
B   North America.
C   Europe.
13   For deserts and gorges, customers should come in the:
A   Morning.
B   Afternoon.
C   Night.
14   Trips to regional locations are good because:
A   The buses are comfortable.
B   There is storage for suitcases.
C   They can be seen quickly.
15   SleekLine buses are particularly known for their:
A   Service.
B   Size.
C   Comfort.

Questions 16-20
Identify the rooms in the office plan.
Write the correct letter, A-G, next to the questions.

16   Local Tours


17   Interstate Tours
18   International Tours
19   Asian Region
20   General Office
Answer:

11 B
12 A
13 B
14 C
15 A
16 D
17 F
18 B
19 C
20 A

You will hear a representative from EasyTravel Travel Agency explaining to some customers the
benefits of her company.

Hello everyone. Now, you’re here because you’re interested in travel, right? And you’re in the
right place, for at EasyTravel, we have the best deals for the best locations. We specialise in eco-
travel, or holidays designed to get you amongst nature, ignoring the hustle and bustle of big
cities. So, whether you want to hike in Nepal, as many people do, or follow some jungle paths in
the rainforests of Queensland, we can give you the best deal.

If you look at our office here, you’ll see our overseas consultants. That’s for trips overseas,
obviously, to Europe, to North America, but primarily for the Asian market, which is generally
more popular in this part of the world. Most people like that touch of the orient, right? But they
also like the domestic market, since this country offers its fair share of beautiful natural vistas.
What about the deserts, anyone? What about some striking red-rock gorges? Then, talk to our
domestic consultants, who can arrange anything you want. But they deal with the accounts in the
morning, so you’ll need to talk to them in the afternoon. And remember, our office doesn’t open
at night, sorry to say.

Now, around the outskirts and outer regions of this city, there are many beautiful places that you
might not be aware of. And the advantages of these regional locations are many. Mostly, by
being closer, some of them can be done in a day tour. Yes, it’s fast and convenient, with none of
those long-haul bus trips which often leave you cramped and uncomfortable, and stuffing heavy
suitcases into luggage racks.

But, I should tell you about our buses, those that are needed for the somewhat more distant
holiday destinations. They definitely do not suffer from those problems I just mentioned. We use
the services of the famous Sleek Line Company, whose buses are known as the very best. Yes,
they are big, yes, they are comfortable, but what makes them especially different is the personal
attendant, who accompanies the driver, ready and willing to serve you , and ensure that your trip
is the very best. So, whether it’s overseas or local, we can certainly give you what you want.
Now, do you have any questions?

Right, let me orient you to our main EasyTravel office here. On this table right beside us are
travel magazines for you to browse through, and on the wall next to that are many more, for all
parts of the world. Our four travel consultants sit over there, on the other side of that long
counter. That’s right, four of them, side by side, all serving various regions.

Now, let me tell you their specific functions. Firstly, the consultant on the left, next to the plant,
is the ‘Local Tours’ consultant, serving tours in the immediate vicinity of this city. Next to her is
what we call ‘Regional Tours’, targeting the state-wide options. -Next- to her is the ‘Interstate
Tours’, and that can involve either buses or planes, in the former case, utilising the SleekLine
Bus service, as you know. And finally, next to her, in the corner, is ‘General Enquiries’, which is
self-explanatory. If you have questions of a general nature, rather than one relating to specific
destinations, you can go there.

Now, as I said, we can do international tours, and for that you need our big office, just through
that door - the one between those two plants. However, if your international tour is in
the Asian region, which is generally our most popular option, then we deal with that in a separate
room the one opposite International Tours, but not; the corner one. Just go through that door on
the left - the one next to that cupboard. The door next to that is, in fact, our General Office, so
please don’t go through there. That’s reserved for staff members only.

Exercise 61.
Questions 21–23
Choose ONE letter, A, B, or C.
21 Mary has been called in for a meeting because:
A she has been working too hard
B she has offered to lead an activity
C she is going on a training programme
22 The most important part of planning a field trip is:
A to organise activities for everyone
B to know about the official rules for field trips
C to think about and plan for problems
23 Leaders must develop:
A a tailored trip plan
B an adventure activities plan
C a hazard management plan
Questions 24 – 30
Complete the sentences below. Write NO MORE THAN TWO WORDS for each answer.
24 Mary’s plan must consider .…………………, such as local rain or wind, the land and the
party members.
25 Mary’s plan does not need to consider things such as tropical storms or serious illnesses,
which are known as .………………… .
FIELD TRIP PLAN:
WEATHER:
• everyone needs warm clothing and a 26.…………………
ACTIVITIES
• hiking – first aid kit, map and a 27.…………………
• avoid routes where an avalanche, mudslide or 28 .………………… is possible
GROUP MEMBERS
• think about the fitness and 29………………… of the group
• do not allow 30 .………………… to be responsible for transport.

21 B
22 C
23 C
24 (significant) factors
25 (unlikely) events
26 raincoat
27 compass
28 flash flood
29 experience
30 student drivers
STUDENT: Hello, Mr Hadstone. Is this the right time for our meeting?
TUTOR: Yes, it is. Thanks for coming in at such a late hour, Mary. I know you’ve had a busy
day studying and will be keen to get home … and thanks for volunteering to run this project.
It’s going to help you develop and practise skills needed by teachers today. Field trips are getting
more and more a part of school life, so as a student of education it’ll be wonderful training for
you. It’s really a great opportunity.
STUDENT: We did loads of field trips at school, so I’ve got a good idea of what sort of things
we could do.
TUTOR: Hmm, I expect so, but we’re here to go through the basics of planning one, and the trip
leader carries a load of responsibility. Right now, you’re focusing on activities, but your main
job is to consider the dangers, and come up with ways of countering or avoiding them.
There are lots of government regulations you won’t have been aware of on your school trips, but
they are just a guideline for your own planning….some of those school trips you went on would
have been pretty adventurous, right?
STUDENT: Yeah.
TUTOR: OK, and your plan needs to be tailored to the kind of trip you’re doing. On a well-
planned and successfully led adventure trip, we don’t often hear of problems … even though
sometimes there’s bad weather, for example, that a school party has managed to combat. That’s
because the leader created a well thought out hazard management plan.
STUDENT: Oh, I thought I’d just be taking my mates out on a trek – now it’s all paperwork!
TUTOR: Yes, well, that’s why I called you in. We’ll work on this together over the next few
days – I just wanted to give you a heads-up on what you’ll need to think about. There are some
aspects that every trip needs to consider. What do you think they might be?
STUDENT: Uh ... well …heavy rain, or high winds, I guess, and any dangers in the terrain...
TUTOR: Yes, we call those the significant factors … and another important one is the make-up
of your group. But, you don’t need to go overboard. There are some kinds of hazard you won’t
need to think about at all: things like hurricanes, earthquakes, radioactivity, or major diseases
such as cancer. The official name for those is unlikely events, because they almost certainly
won’t happen.
TUTOR: OK, so let’s consider the hazards seen as most likely on a field trip into the
countryside. Weather causes real problems – overexposure to the sun or the cold – even the wind
can have a big impact. And, of course, the weather can change very suddenly and without
warning.
STUDENT: Yes, people can get into trouble in the hills if they don’t bring extra layers of clothes
and a jacket – even if they start walking on a hot day. Oh … and a raincoat, too, of course.
Umm …what’s next then?
TUTOR: Well, let’s think about possible activities and what you might need.
STUDENT: Yes, OK … Well, for hiking of course we need a first aid kit. Oh … and a decent
topographic map of the area. And we need to make sure that more than one person can read it.
I’ve run into lots of difficulties in the past with people who can’t identify even major features,
like rivers. And some people have no idea about contour lines. Ah….and I suppose a compass,
too.
TUTOR: You’d need to list those. Then there are things that may be obvious, but must be written
down and considered seriously. For example, if there’s a possibility of falling more than 2.5
meters, that’s considered life threatening, and I’m sure you would be aware of problems near the
sea, like tides or high waves – and the trouble you can get into where there’s a possibility of an
avalanche or a mudslide … or a flash flood, if you’re anywhere near rivers.
STUDENT: Yes, well, I was thinking of an adventurous route for this trip … you know, that’s
always more fun, and it’s such a cool feeling when you’ve achieved something really difficult.
TUTOR: Yes, OK, but then you need to consider who’s going to be in your party…. don’t go
and plan things that are beyond the reach of most people, or you’re asking for trouble. You need
to take into account the physical strength and experience of the party as a whole. When you
make your groups, make sure there’s at least one person in each one who’s been hiking a few
times before.
STUDENT: Wow, there’s a lot to write down, isn’t there? I’m really keen to get started now.
TUTOR: Well, good, because there’s a lot more detail to consider. For now, I’ll just mention two
more of the common hazards for high school trips, in particular.
STUDENT: Yes?
TUTOR: The Ministry of Education website says – don’t use inexperienced volunteers, and
don’t allow student drivers to bring their own cars, or to drive anyone else’s car, for that matter.
STUDENT: Well … now I really have something to think about! Thanks, Mr Hadstone

Exercise 62.

Questions 21–25
Complete the sentences below.
Write NO MORE THAN TWO WORDS for each answer.
21 Students must follow ……………………. to prevent accidents in the lab.
22 The students have not been using ……………………. while in the lab.
23 Students cannot eat or drink until ……………………. is finished and they have washed their
hands.
24 Tessa should tie her hair back to avoid danger when she is working with a
……………………. or chemicals.
25 Students must wear long sleeves and shoes made of ……………………. in the lab.

Questions 26–28
Choose the correct letter A, B or C.
26 Which student is currently using an appropriate notebook?
A Vincent
B Tessa
C Neither student
27 The tutor says that writing observations in complete sentences
A is often not a good use of time
B makes them easier to interpret later
C means that others can understand them
28 The students must write dates
A next to each drawing
B next to each written section
C next to each drawing and written section

Questions 29 and 30
Choose TWO letters, A–E.
Which TWO things must be included in the conclusion to the experiment?
A the questions investigated
B the solutions to the questions
C the student’s own thoughts about the experiment
D the length of time spent on the experiment
E the student’s signature

21. safety procedures/directions/safety rules


22. eye protection / safety glasses/goggles
23. clean-up / clean up / cleanup
24. naked flame
25. leather
26. C
27. B
28. C
29 &30. C, E (in either order)

TUTOR: Now Vincent and Tessa, I’ve asked the two of you to come and see me because I’m a
bit concerned after that incident in the science lab last week. I realise that neither of you have
had much experience in a laboratory before …
VINCENT: Well, we mostly just studied theory at high school …
TESSA: and we rarely got the opportunity to carry out any experiments.
TUTOR: Fair enough. But we must all abide by certain safety procedures – the last thing we
want is for one of our students to get hurt.
TESSA: We understand that.
TUTOR: Our priority is to make sure that the chemistry laboratory is a safe place and, actually,
accidents can easily be prevented if you just think about what you’re doing at all times.
TESSA: It sounds simple enough.
TUTOR: It is if you always use good judgement, observe safety rules and follow directions.
VINCENT: We’ve read the rules on the poster inside the lab.
TUTOR: And yet last week you were seen working in the lab without eye protection.
TESSA: What do you mean? I was wearing my glasses.
TUTOR: Prescription glasses are not safety glasses – you must always wear the goggles
provided – you’ll find they fit quite comfortably over your ordinary glasses.
VINCENT: Oh, I see.
TUTOR: Just make a habit of putting them on before you start and keep them on until you are
finished. And another thing, never eat or drink while in the laboratory.
TESSA: What – not even water?
TUTOR: Not even water – at least not until after clean-up. Then, be sure to wash your hands
thoroughly with soap and hot water and dry them on a clean towel first. And Tessa, your hair
should be tied back when you’re in the lab.
TESSA: It’s not that long.
TUTOR: Still, it poses a hazard when you’re working with chemicals or a naked flame. If you
can’t tie it back or pin it up, see if you can tuck it into a cap or something.
TESSA: Yes, I can do that.
TUTOR: Thank you. Now, Vincent, last week you wore a tee-shirt and trainers in the lab. The
rules clearly state that long-sleeved shirts and leather shoes must be worn.
VINCENT Oh, yes, I remember – I was late getting back from sports practice and I didn’t have
time to change.
TUTOR: Well, it mustn’t happen again.
VINCENT: Okay, I’ll see that it doesn’t.
TUTOR: Good. As for the rest of the safety precautions, refer to the safety poster inside the lab
and you shouldn’t have any problems.
TUTOR: Now, before you go, a word about record-keeping.
VINCENT: Oh, good – I was going to ask you about that.
TESSA: What’s the best way to keep track of what we’re doing in the lab?
TUTOR: Well, obviously, all your observations should be written down – I know you think you
won’t forget stuff and you’ll be able to recall it later but generally this turns out not to be the
case. Written data, however, are a permanent record. And you must be thorough. Organise and
record everything in a bound notebook.
TESSA: I use a spiral notebook.
VINCENT: And I use a large note pad.
TUTOR: That won’t do. A book with binding ensures the pages are not easily removed or lost.
Oh, and be sure to write your entries in complete sentences.
TESSA: Isn’t that a waste of time?
VINCENT: Surely notes are good enough.
TUTOR: You might think so but brief notes can be hard to decipher at a later date,
whereas with full sentences you are less likely to misinterpret data.
VINCENT: I make sketches, you know, simple drawings.
TUTOR: That’s a good idea, Vincent, but be sure to date them.
TESSA: You want us to write the date next to each drawing?
TUTOR: Yes, every sketch and every entry must be dated.
TESSA: What about headings?
TUTOR: Use the title of the experiment as your first entry. When you have completed your
observation entries, answer any questions that have been posed and then, finally, write your
conclusion.
VINCENT: How do we write a conclusion? Do we need to repeat things like the questions and
our findings, or the time it all took?
TUTOR: Just write your own ideas or feelings about the experiment as the conclusion. Oh
… and remember to sign it! Well, that’s all I have time for today. If you have any questions,
ask the lab assistant or come back to me.

Exercise 63.

Questions 21-26: What did findings of previous research claim about the personality traits
a child is likely to have because of their position in the family? Choose SIX answers from
the box and write the correct letter, A—H, next to Questions 21-26.

Personality Traits
A outgoing
B selfish
C independent
D attention-seeking
E introverted
F co-operative
G caring
H competitive

Position in family
21 the eldest child
22 a middle child
23 the youngest child
24 a twin
25 an only child
26 a child with much older siblings

Questions 27 and 28: Choose the correct letter, A, B or C.

27 What do the speakers say about the evidence relating to birth order and academic success?
A There is conflicting evidence about whether oldest children perform best in intelligence tests.
B There is little doubt that birth order has less influence on academic achievement than socio-
economic status.
C Some studies have neglected to include important factors such as family size.

28 What does Ruth think is surprising about the difference in oldest children's academic
performance?
A It is mainly thanks to their roles as teachers for their younger siblings.
B The advantages they have only lead to a slightly higher level of achievement.
C The extra parental attention they receive at a young age makes little difference.

Questions 29 and 30: Choose TWO letters, A—E. Which TWO experiences of sibling
rivalry do the speakers agree has been valuable for them?

A learning to share
B learning to stand up for oneself
C learning to be a good loser
D learning to be tolerant
E learning to say sorry
21 G
22 F
23 A
24 E
25 B
26 C
27 C
28 A
29&30 IN EITHER ORDER B D

RUTH: Ed, how are you getting on with the reading for our presentation next week?
ED: Well, OK, Ruth — but there's so much of it.
RUTH: I know, I hadn't realised birth order was such a popular area of research.
ED: But the stuff on birth order and personality is mostly unreliable. From what I've been
reading a lot of the claims about how your position in the family determines certain personality
traits are just stereotypes, with no robust evidence to support them.
RUTH: OK, but that's an interesting point — we could start by outlining what previous research
has shown. There are studies going back over a hundred years.
ED: Yeah — so we could just run through some of the typical traits. Like the consensus seems to
be that oldest children are generally less well-adjusted because they never get over the arrival of
a younger sibling.
RUTH: Right, but on a positive note, some studies claimed that they were thought to be good at
nurturing — certainly in the past when people had large families they would have been ex
ected to look after the oun•er ones.
ED: There isn't such a clear picture for middle children — but one trait that a lot of the studies
mention is that they are easier to get on with than older or younger siblings
RUTH: Generally eager to please and helpful — although that's certainly not accurate as far as
my family goes — my middle brother was a nightmare — always causing fights and envious of
whatever I had.
ED: As I said — none of this seems to relate to my own experience. I'm the youngest in my
family and I don't recognise myself in any of the studies I've read about. I'm supposed to have
been a sociable and confident child who made friends easily — but I was actually terribly shy.
RUTH: Really? That's funny. There have been hundreds of studies on twins but mostly about
nurture versus nature ...
ED: There was one on personality, which said that a twin is likely to be quite shy in social
situations because they always have their twin around to depend on for support.
RUTH: My cousins were like that when they were small — they were only interested in each
other and found it hard to engage with other kids. They're fine now though.
ED: Only children have had a really bad press — a lot of studies have branded them as loners
who think the world revolves around them because they've never had to fight for their parents'
attention.
RUTH: That does seem a bit harsh. One category I hadn't considered before was children with
much older siblings — a couple of studies mentioned that these children grow up more quickly
and are expected to do basic things for themselves — like getting dressed.
ED: I can see how that might be true — although I expect they're sometimes the exact opposite
— playing the baby role and clamouring for special treatment.
RUTH: What was the problem with most of these studies, do you think?
ED: I think it was because in a lot of cases data was collected from only one sibling per family,
who rated him or herself and his or her siblings at the same time.
RUTH: Mmm. Some of the old research into the relationship between birth order and academic
achievement has been proved to be accurate though. Performances in intelligence tests decline
slightly from the eldest child to his or her younger siblings. This has been proved in lots of recent
studies.
ED: Yes. Although what many of them didn't take into consideration was family size. The
more siblings there are, the likelier the family is to have a low socio-economic status —
which can also account for differences between siblings in academic performance.
RUTH: The oldest boy might be given more opportunities than his younger sisters, for example.
ED: Exactly.
RUTH: But the main reason for the marginally higher academic performance of oldest children
is quite surprising, I think. It's not only that they benefit intellectually from extra attention at a
young age — which is what I would have expected. It's that they benefit from being teachers
for their younger siblings, by verbalising processes.
ED: Right, and this gives them status and confidence, which again contribute, in a small way, to
better performance. So would you say sibling rivalry has been a useful thing for you?
RUTH: I think so — my younger brother was incredibly annoying and we fought a lot but I think
this has made me a stronger person. I know how to defend myself. We had some terrible
arguments and I would have died rather than apologise to him — but we had to put up with
each other and most of the time we co-existed amicably enough.
ED: Yes, my situation was pretty similar. But I don't think having two older brothers made me
any less selfish — I was never prepared to let my brothers use any of my stuff ...
RUTH: That's perfectly normal, whereas ...

Exercise 64.

Questions 21 and 22
Choose TWO letters, A—E. Which TWO groups of people is the display primarily
intended for?
A students from the English department
B residents of the local area
C the university's teaching staff
D potential new students
E students from other departments
Questions 23 and 24
Choose TWO letters, A—E. What are Cathy and Graham's TWO reasons for choosing the
novelist Charles Dickens?
A His speeches inspired others to try to improve society.
B He used his publications to draw attention to social problems.
C His novels are well-known now.
D He was consulted on a number of social issues.
E His reputation has changed in recent times.
Questions 25-30
What topic do Cathy and Graham choose to illustrate with each novel?
Choose SIX answers from the box and write the correct letter, A—H, next to Questions 25-
30.
Novels by Dickens
25 The Pickwick Papers ______________
26 Oliver Twist ______________
27 Nicholas Nickleby ______________
28 Martin Chuzziewit ______________
29 Bleak House ______________
30 Little Dorrit ______________
Topics
A poverty
B education
C Dickens's travels
D entertainment
E crime and the law
F wealth
G medicine
H a woman's life

21&22 IN EITHER ORDER B D


23&24 IN EITHER ORDER B C
25 G
26 B
27 D
28 C
29 H
30 F

CATHY: OK, Graham, so let's check we both know what we're supposed to be doing.
GRAHAM: OK.
CATHY: So, for the university's open day, we have to plan a display on British life and literature
in the mid-19th century.
GRAHAM: That's right. But we'll have some people to help us find the materials and set it up,
remember — for the moment, we just need to plan it.
CATHY: Good. So have you gathered who's expected to come and see the display? Is it for the
people studying English, or students from other departments? I'm not clear about it.
GRAHAM: Nor me. That was how it used to be, but it didn't attract many people, so this year it's
going to be part of an open day, to raise the university's profile. It'll be publicised in the city, to
encourage people to come and find out something of what goes on here. And it's included in
the information that's sent to people who are considering applying to study here next year.
CATHY: Presumably some current students and lecturers will come?
GRAHAM: I would imagine so, but we've been told to concentrate on the other categories of
people.
CATHY: Right. We don't have to cover the whole range of 19th-century literature, do we?
GRAHAM: No, it's entirely up to us. I suggest just using Charles Dickens.
CATHY: That's a good idea. Most people have heard of him, and have probably read some
of his novels, or seen films based on them, so that's a good lead-in to life in his time.
GRAHAM: Exactly. And his novels show the awful conditions that most people had to live
in, don't they: he wanted to shock people into doing something about it.
CATHY: Did he do any campaigning, other than writing?
GRAHAM: Yes, he campaigned for education and other social reforms, and gave talks, but I'm
inclined to ignore that and focus on the novels.
CATHY: Yes, I agree.
CATHY: OK, so now shall we think about a topic linked to each novel?
GRAHAM: Yes. I've printed out a list of Dickens's novels in the order they were published, in
the hope you'd agree to focus on him!
CATHY: You're lucky I did agree! Let's have a look. OK, the first was The Pickwick Papers,
published in 1836. It was very successful when it came out, wasn't it, and was adapted for the
theatre straight away.
GRAHAM: There's an interesting point, though, that there's a character who keeps falling
asleep, and that medical condition was named after the book — Pickwickian Syndrome.
CATHY: Oh, so why don't we use that as the topic, and include some quotations from the
novel?
GRAHAM: Right, Next is Oliver Twist. There's a lot in the novel about poverty. But maybe
something less obvious ...
CATHY: Well Oliver is taught how to steal, isn't he? We could use that to illustrate the fact
that very few children went to school, particularly not poor children, so they learnt in other
ways.
GRAHAM: Good idea. What's next?
CATHY: Maybe Nicholas Nickleby. Actually he taught in a really cruel school, didn't he?
GRAHAM: That's right. But there's also the company of touring-in actors that Nicholas joined.
We could do something on theatres and other amusements of the time. We don't want only
the bad things, do we?
CATHY: OK.
GRAHAM: What about Martin Chuzzlewit? He goes to the USA, doesn't he?
CATHY: Yes, and Dickens himself had been there a year before, and drew on his
experience there in the novel.
GRAHAM: I wonder, though. The main theme is selfishness, so we could do something on
social justice? No, too general, let's keep to your idea — I think it would work well.
CATHY: He wrote Bleak House next — that's my favourite of his novels.
GRAHAM: Yes, mine too. His satire of the legal system is pretty powerful.
CATHY: That's true, but think about Esther, the heroine. As a child she lives with someone she
doesn't know is her aunt, who treats her very badly. Then she's very happy living with her
guardian, and he puts her in charge of the household. And at the end she gets married and
her guardian gives her and her husband a house, where of course they're very happy.
GRAHAM: Yes, I like that.
CATHY: What shall we take next? Little Dorrit? Old Mr Dorrit has been in a debtors' prison for
years ...
GRAHAM: So was Dickens's father, wasn't he?
CATHY: That's right.
GRAHAM: What about focusing on the part when Mr Dorrit inherits a fortune and he
starts pretending he's always been rich?
CATHY: Good idea.
GRAHAM: OK, so next we need to think about what materials we want to illustrate each issue.
That's going to be quite hard.

Exercise 65:
Box Telecom

Problem: been affected by drop in 21 .........................


growing 22 .........................
delays due to a strike

Causes of problems: high 23 .........................


lack of good 24 .........................

Questions 25-27

Choose the correct letter, A, B or C.

25) What does Karin think the company will do?


A look for private investors

B accept a takeover offer

C issue some new shares

26) How does the tutor suggest the company can recover?

A by appointing a new managing director

B by changing the way it is organised

C by closing some of its retail outlets

27) The tutor wants Jason and Karin to produce a report which

A offers solutions to Box Telecom's problems.

B analyses the UK market.

C compares different companies.

Questions 28-30

Which opinion does each person express about Box Telecom?

Choose your answers from the box and write the letters A-F next to questions 28-30.

A its workers are motivated


B it has too little investment

C it will overcome its problems

D its marketing campaign needs improvement

E it is old-fashioned

F it has strong managers


28) Karin  
29) Jason  
30) The tutor  

Answer:
21. sales
22. competition
23. interest rates
24. training
25. A
26. B
27. A
28. C
29. B
30.D

TUTOR: Right, Jason and Karin, now I asked you to look at the ease study for Box Telecom as
part of your exam assessment. It's interesting because they are in the middle of problems at the
moment and I want you to track how they deal with them. Let's start with you, Karin. Having
read through the ease study, can you just summarise what the problems were that Box Telecom
had to take on board?

KARIN: Um, yeah ... Well of course what first came to their attention was that, despite a new
advertising campaign, they were suffering from falling sales - and this is something that had
many causes. On top of that immediate problem, what had also happened over the last two years
was that, although they had invested in an expansion plan, they had to face up to
increased competition. And, before they had a chance to get to grips with the effects of that, they
were stalled by a strike and it was just when they were thinking about making a colossal
investment in new machinery for their plants. So they were really in trouble.

TUTOR: Yes, I think that's fair. And Jason, you contacted the company, didn't you? What did
the company define as the reasons for these problems?

JASON: Well, I think they've hit on the right things - it would be easy to say they had invested
too heavily, or at the wrong time, but in fact the signs were good and what they were set back by
was high interest rates. At the same time, their longer-term problems, which were affecting their
market share, were eventually credited to poor training - and having looked at the details in their
last report I think that's right.

TUTOR: SO, onto the larger issues then. Karin, what do you think the company will do?

KARIN: Hmm ... Well, obviously they have the choice of accepting the very favourable terms
that another company - KMG Plc - have given them to buy them out. That would mean creating
a new company with a new image. Or they could decide on a bolder move and offer some new
shares if they wanted. But I think they're much more cautious than that and expect they will
start trying to find individuals who'd be prepared to back them with some of the capital they
need.

TUTOR: Well, you mustn't always assume that dramatic problems require dramatic solutions.
Sometimes there's a simple fix such as changing the guy at the top. If they truly are cautious,
then I suspect they will seek to shut down some of their shops. But a more ambitious approach,
and one which I think would have more chance of success, would be to alter how they're running
things - the management layers and the processes. So in your analysis try to think of all the
options. Jason?

JASON: Yes. it's interesting because I've found it a really useful company to study. Its problems
cross all types of industries and it's lucky it's so big - a smaller or even medium-sized company
would have gone under by now.

TUTOR: Ah well in fact, what I want you two to do is to go away when we've finished our
discussion today and write a report. We've looked in general at the telecommunications market in
the UK over the last few sessions and I want you to take Box Telecom as an example
and suggest some ways in which they might overcome their problems. And outline the reasons
why you think as you do - but try and keep it intrinsic to the company rather than dragging in
other examples. Is that OK, Karin?

KARIN: Yes, I think I can do that. Personally I've got great hopes for it. I think it will recover.
That advertising campaign they did was very strong and they're very innovative with their
products - they set new trends. The company's got to recover, don't you think, Jason?

JASON: Hmmm - I'm not sure. I think it can but it's not a foregone conclusion unless they
manage to attract the right level of investment. The company definitely needs a boost and to
attract more highly skilled workers if their recovery is to be long-lasting. When I was talking to
the marketing manager he said to me that he thinks the company had got a great management
team - but he would say that, wouldn't he? - but they are suffering from having to work with
outdated production machinery and that could cost a lot to put right.

TUTOR: Well, personally I think the stock market is to blame. I think they were expecting too
much of the company and then inevitably it looked bad when it didn't perform. The market
should have had more realistic expectations.
And I disagree with you about the advertising campaign Karin. That's where they could do with
some innovation - to get sales kick-started. Anyway, let's see what you come up with ... [fade]

Exercise 66:

Questions 21-24

Choose the correct letter, A, B or C. Honey Bees in Australia

21.    Where in Australia have Asian honey bees been found in the past?

   A. Queensland

   B. New South Wales

   C. several stales

22.    A problem with Asian honey bees is that they

   A. attack native bees

   B. carry parasites

   C. damage crops.

23.    What point is made about Australian bees?

   A. Their honey varies in quality.

   B. Their size stops them from pollinating some flowers.


   C. They are sold to customers abroad.

24.    Grant Freeman says that if Asian honey bees got into Australian.

   A. the country’s economy would be affected.

   B. they could be used in the study of allergies.

   C. certain areas of agriculture would benefit.

Questions 25-30

Complete the summary below. Write ONE WORD ONLY for each answer.

Looking for Asian honey bees

Birds called Rainbow Bee Eaters eat only 25……………………… , and cough up small bits of
skeleton and other products in a pellet.

Researchers go to the locations the bee eaters like to use for 26………………………

They collect the pellets and take them to a 27………………………  for analysis.

Here 28………………………  is used to soften them, and the researchers look for


the 29………………………  of Asian bees in the pellets.

The benefit of this research is that the result is more 30………………………  than searching for
live Asian.
Answer:
21. A. Queensland
22. B. carry parasites
23. C. They are sold to customers abroad.
24. A. the country’s economy would be affected.
25. insects
26. feeding/eating
27. laboratory
28. water
29. wings
30. reliable/accurate

PROFESSOR:      Good morning everyone. In today’s seminar, Grant Freeman, a biologist who
specialises in identifying insects, and who works for the Australian Quarantine Service, has
come to talk to us about his current  research work. Right, well, over to you, Grant.

GRANT:               Good morning, everyone. I’m sure that you know that the quarantine service
regulates all food brought into Australia. Well, obviously they want to protect Australia from
diseases that might come in with imported goods, but they also want to prevent insect pests from
being introduced into the country, and  that’s where I have a particular type of  them around in
various states of Australia. We discovered a few of them in Queensland(Q21) once and
eradicated them. Now, we’re pretty keen to make sure that there aren’t any more getting in,
particularly to New South Wales and other states.

STUDENT 1:     What’s wrong with Asian Honey Bees? Are they so different from Australian
bees?
GRANT:               Well, in fact, they look almost the same, but they are infested with
mites(Q22) – microscopic creatures which live on them, and which can seriously damage our
own home – grown bees, or could even wipe them out.

PROFESSOR:       Well, what would happen if  Australian bees died out?

GRANT:                Well, the honey from Australian bees is of excellent quality, much better than
the stuff  the Asian bees produce. In that, Australia exports native Queen bees to a large
number of countries because of this (Q23). When the European Honey Bee was first
discovered out in the bush, we found they made really unpleasant honey and they were also too
big to pollinate many of our native flowers here in Australia.

STUDENT 2:            That must have had a devastating effect on the natural flora. Did you lose
any species?

GRANT:                    No, we managed to get them under control before that happened but if
Asian bees got in there could be other consequences. We could lose a lot of money
(Q24) because you might not be aware, but it’s estimated that native bees’ pollination of flower
and vegetable crops is worth 1.2 billion dollars a year. So in a way they’re the farmer’s friend.
Oh, and another thing is, if  you’re stung by an Asian Honey Bee, it can produce an allergic
reaction in some people; so they’re much more dangerous than native bees.

PROFESSOR:       How will you know if Asian bees have entered Australia?

GRANT:                We’re looking at the diet of the bird called the Rainbow Bee Eater. The Bee
Eater doesn’t  care what it eats, as long as they’re insects(Q25). But the interesting thing
about this is that we are able to analyse exactly what it eats and that’s really helpful if  we’re
looking for introduced insects.

PROFESSOR:       How come?

GRANT:                Because insects have their skeletons outside their bodies, so the Bee Eaters
digest the meat from the inside. Then they bring up all the indigestible bits of skeletons and, of
course, the wings in a pellet – a small ball of waste material which they cough up.

PROFESSOR:        That sounds a bit unpleasant. So, how do you go about it?

GRANT:                In the field we track down the Bee Eaters and find their
favourite feeding(Q26) spots, you know, the place where the birdsusually feed. It’s here that we
can find the pellets. We collect them up and them back to the laboratory(Q27) to examine the
contents.

PROFESSOR: How do you do that?

 
Exercise 67:

Questions 21-26

Choose the correct letter, A, B or C. Latin American studies

21.    Paul decided to get work get work experience in South American because he wanted

   A. to teach English there

   B. to improve his Spanish

  C. to learn about Latin American life

22.    What project work Paul originally intend to get involved in?

   A. construction

   B. agriculture

   C. tourism

23.    Why did Paul change form one project to another?

   A. His first job was not well organized.

   B. He found doing the routine work very boring.

   C. The work was too physically demanding.


24.    In the village community, he leant how important it was to

   A. respect family life.

   B. develop trust

   C. use money wisely.

25.    What does the Paul say about his project manager?

   A. He let Paul do most of the work

   B. His plans were too ambitious.

   C. He was very supportive of Paul.

26.    Paul was surprised to be given

   A. a computer to use

   B. so little money to live on

   C. an extension to his contract

Questions 27-30

What does Paul decide about each of the following modules?

Write the correct letter, A, B or C, next to questions 27-30.

A.  He will do this.

B.  He might do this.

C. He won’t do this.

Module

27. Gender Studies in Latin America

28. Second Languages Acquisition

29.Indigenous Women’s Lives


30.Portuguese Language Studies

Answer:
21 C. to learn about Latin American life
22 C. tourism
23 A. His first job was not well organized.
24  B. develop trust
25 C. He was very supportive of Paul.
26  A. a computer to use
27 C   28 A   29 B   30 C

WOMAN:                I’ve been reading your personal statement, Paul. First, let’s talk about your
work experience in South America. What took you there? Was it to gain more fluency in
Spanish?

PAUL:                      Well, as I’m combining Spanish with Latin American studies, my main idea
was to find out more about the way people lived there (Q21). My spoken Spanish was already
pretty  good in fact.

WOMAN:              So you weren’t too worried about language barriers?

PAUL:                      No. In fact, I ended up teaching English there, although that wasn’t my
original choice of work.

WOMAN:                I see. How did you find out about all this?

PAUL:                      I found an agency that runs all kinds of voluntary projects in South
America.

WOMAN:                What kind of work?

PAUL:                      Well, there were several possibilities.


WOMAN:                You mean construction? Engineering work?

PAUL:                      Yes, getting involved in building projects was an option. Then there was
tourism –taking tourists for walks around the volcanoes – which I actually chose to do(Q22),
and then there was work with local farmers.

WOMAN:              But you didn’t  continue with that project. Why not?

PAUL:                      Because I never really knew whether I’d be needed or not. I’d thought it
might be difficult physically,  but I was certainly fit enough…no, I wanted to do something
that had more of a proper structure to it (Q23), I suppose, I get de-motivated otherwise.

WOMAN:                What do you think you learned from your experience? It must have been a
great opportunity to examine community life.

PAUL:                      Yes, but it was difficult at first to be accepted by the locals. It was a very
remote village and some of them were reluctant to speak to me – although they were always
interested in my clothes and how much I’d had to pay for them.

WOMAN:                Well, that’s understandable.

PAUL:                      Yes, but things soon improved. What struck me was that when people
became more comfortable with me and less suspicious (Q24) , we really connected with each
other in a meaningful way.

WOMAN:                You made good friends?

PAUL:                      Yes, with two of the families in particular.

WOMAN:                Good. What about management. Did you have a project manager?

PAUL:                      Yes and he gave me lots of advice and guidance. (Q25)

WOMAN:                And was he good at managing too?

PAUL:                      That’s wasn’t his strong point! I think he was often more interested in the
academic side of things than filling reports. He was a bit of a dreamer.

WOMAN:                 And did you have a contract?

PAUL:                      I had to say for a minimum of three months. My parents were surprised
when I asked to stay longer – six months in the end. I was so happy there.

WOMAN:                 And did anything on the administration side of things surprise you? What
was the food and lodging like?
PAUL:                      Simple…but there was plenty to eat and I only paid seven dollars a day for
that which was amazing really. And they gave me all the equipment I needed…even a laptop.
(Q26)

WOMAN :                  You didn’t expect that then?(Q26)

PAUL:                       No. (Q26)

WOMAN :                Well, I’ll look forward to hearing more.

WOMAN :                But now let’s look at these modules. You’ll need  to start thinking about
which ones you’ll definitely want to study. The first one here is Gender Studies in Latin
America.

PAUL:                       Mmm…

WOMAN :                It looks at how gender analysis is reconfiguring civil society in Latin
America. Women are increasingly occupying positions is government and in other elected
leadership positions in Latin America. I think you’d find it interesting.

PAUL:                       It is was to do with people in the villages rather than those in the
public sphere, I would (Q27) .

WOMAN :                Okay. What about Second Language Acquisition?

PAUL:                       Do you think I’d find that useful?

WOMAN :                Well, you’ve had b some practical experience in the field, I think it would
be.

PAUL:                       I hadn’t thought about that. I’ll put that down as a definite, then. (Q28)

WOMAN :                Okay. What about indigenous Women’s Lives. That sounds appropriate.

PAUL:                       I thought so too, but I looked at last year’s exam questions and that
changed my mind,

WOMAN :                Don’t judge the value of the course on that. May be, talk to some other
student first and we can talk about it again later. (Q29)

PAUL:                       Okay.

WOMAN :                Yes, And lastly, will you sign up for Portuguese lessons?

PAUL:                       My Spanish is good, so would I find that module essay?


WOMAN :                Not necessary. Some people find that Spanish interferes with learning
Portuguese…getting the accent right too.It’s quite different in a lot of ways.

PAUL:                       Well, I’d much sooner do something else, then. (Q30)

WOMAN :                Alright. Now, what we need to do is…

Exercise 68:

Questions 21-25
Choose the correct letter, A, B or C.
21   Which part has the tutor already read?
A   the introductory chapter
B   the procedure section
C   the results and discussion section
22   Which part of the paper did the tutor like?
A   introduction
B   layout
C   background information
23   Kathy and the tutor both agree to continue to
A   refer a lot to the example received in class.
B   copy the information.
C   conduct further research in the library.
24   Kathy asks the tutor for help with the ………….. section.
A   abstract
B   bibliography
C   appendix
25   What will Kathy do next?
A   try out software
B   work on the bibliography
C   make an animation

Questions 26-30
What is the desired outcome to each of the following course of action?
Choose FIVE answers from the box and write the correct letter, A-F, next to questions 26-30.
Desired outcomes
A   practical experience
B   publish the work
C   join Machine Engineer Society
D   give suggestions
E   stay up to date
F   make important contacts
26   Make a good grade
27   Meet engineering professionals
28   Visit the factory
29   Seek summer internships
30   Present dissertation

Answer:
21 A
22 B
23 C
24 B
25 A
26 C
27 F
28 E
29 A
30 B
TUTOR:    Before we start, Jimmy and Kathy, thanks for coming in today to talk about your
current research paper! Well, I will also give you some suggestions for your future presentation
later.

JIMMY:    That’s great!

TUTOR:    Okay, I have read the introductory chapter, and so far I like where you’re going with
your research, you two.

KATHY:    Thanks! What did you think of the procedure section?

TUTOR:    I haven’t gotten there yet. I will get to that and the results and discussion section in a
bit.

JIMMY:    Oh. If you haven’t read the rest, are you just saying you like the introduction?

TUTOR:    No - the layout is really well done. You have each section clearly marked and have
the header and footer perfectly formatted, and your title page is right on the money. A lot of
students have trouble with that one.

JIMMY:    To be honest, we did refer a lot to the example we received in class.

TUTOR:    That’s good to do for spacing and layout, as long as you’re not also copying the
information. The background information is a little sparse, though. You may want to add to it.
KATHY:    You think so? I was more worried about whether I had enough data.

TUTOR:    You definitely need more background information. I would think about finding some
more online articles or doing more research in the campus library.

JIMMY:    That’s a good idea - we can go tomorrow. I find it too tough finding the subject
matter in the online journal database.

TUTOR:    I also like being able to flip through the physical journal as opposed to trying to scroll
down on a computer.

KATHY:    Me too. Oh, I almost forgot. I’ve included all of my citations in the abstract, but
could you help me with the bibliography? I should be using a bibliography, right? Not an
appendix?

TUTOR:    Sure, I can help with that. Yes - for this type of scientific research paper, list all
sources that you cite in the body of your paper in a bibliography. Go to the website I gave you
last time to see the exact way to list each source.

KATHY:    Okay, thanks. I’ll do that. We still have a lot of things to fix up.

TUTOR:    Yeah, but there’s a lot of good stuff here to work with. So enough about the paper,
how is the presentation going?

KATHY:    Well, it’s alright. I am going to go try out the new presentation software while
Jimmy’s working on the bibliography.

JIMMY:    Yeah, we are hoping to make an animation of an actual pump but still have a lot to
learn about how to do that.

KATHY:    Who would have thought before we started this project that we would be able to
recreate the motion of a pump? This stuff is just so interesting.

TUTOR:    So glad to hear it!

JIMMY:    Yeah, I am glad I took engineering this semester. I would definitely like to keep up
with it.

TUTOR:    You know, there’s an organisation called the Machine Engineer Society. You should
look into joining it. You would need to score well in your engineering class to qualify, but I think
you can do it.

KATHY:    Hm, interesting. I will definitely check it out. I would really like to get in contact
with some professionals in the engineering field to find out more. I don’t really know anyone in
the field now, though.
JIMMY:    I think if you keep meeting people in your classes and professors you’ll be able to get
in contact with some really helpful people.

TUTOR:    Well said, Jimmy. If engineering pumps is something you both are specifically
interested in, make sure you stay up to date on new developments. In fact, you could visit the
local water treatment facility periodically to see what new developments are going on.

KATHY:    Hm, that may be a good way to get some practical experience.

TUTOR:    Well, I don’t think they would let you handle any equipment by just visiting the
facility. If you really want to get your hands dirty, so to speak, I would recommend instead
seeking a summer internship.

KATHY:    Wow, you have so many helpful suggestions for getting a leg up. Now if only you
could tell me how to get my work published!

JIMMY:    Haha, wouldn’t that be nice.

TUTOR:    Well, honestly, all you really need to do is once you have a dissertation, present it.
Present it often and to many audiences, and once you get feedback, adjust it. You’ll get
published one day.

KATHY':    Wow, this meeting has been truly inspiring. Thanks for your help!
Exercise 69:

Questions 21-23
Choose THREE letters, A-G.
Which THREE factors should the student consider while selecting courses?
A   class time
B   course topic
C   amount of homework
D   ease of course
E   relevant to future career
F   course structure
G   professor reputation

Questions 24-27
Choose the correct letter, A, B or C.
24   The tutor recommends against taking Human Physiology because it would not be the best
A   time management.
B   chance at earning an A.
C   topic for a research paper.
25   The student decides to do a dissertation because
A   he takes it to boost his GPA.
B   he likes to develop more supportive details.
C   he wants to conduct more interviews.
26   The student thought the research paper was
A   already completed.
B   worth finishing.
C   too complicated.
27 The method of data collection was
A   interviews
B   lab studies.
C   questionnaires

Questions 28-30
Complete the sentences below.
Write ONE WORD ONLY for each answer.
First draft should be finished by the end of 28……………
Dissertation should be registered with the 29…………… in the Department Office.
The student can get the relevant database from the 30…………… Office.

Answer:
21-23 B, E, F
24 A
25 B
26 C
27 A
28 March
29 secretary
30 computer

STUDENT: Hi, Professor Jamison, I was hoping we could talk this week about choosing courses
for next semester.

TUTOR:    That sounds great! Any idea yet what you would like to take?

STUDENT: Well, I was hoping you could kind of tell me. There are so many to choose from. I
don’t even know where to start!

TUTOR:    Sure. Well, first thing’s first. Do you know what you will major in? It is important to
take courses that are relevant to your career path.

STUDENT: Well, I think I want to major in Biology. I want to go to medical school, so it seems
like it would make sense.

TUTOR:    I agree - if you like Biology, it is definitely a good idea to focus on that since so
many of thè requirements overlap.

STUDENT: Okay, so I should take a few Biology classes then?

TUTOR:    Yes. Let’s start there. You should pick one or two Biology classes. Not just any
Biology classes, though! If possible, read online or talk to senior fellows and find out about the
structure of the courses. You don’t want to end up signing up for two classes that require labs in
the same semester - you’ll spend so much time in the Biology Department that you won’t see the
light of day.

STUDENT: Oh right, good idea. I heard labs can be as long as 4 hours.


TUTOR:    That is true. And another thing - make sure the topic is either relevant to your major,
or something you are interested in learning about. It sounds obvious, but do not just take a class
because you heard it was easy, or because it does not require attendance.

STUDENT: Of course not. Though it would be really nice to have at least one class, that’s a little
bit less rigorous than the ones I’m taking this semester. I barely sleep as it is with all this
studying.

TUTOR:    That may have more to do with your study habits, though I don’t disagree that your
schedule is really difficult this semester.

TUTOR:    So any idea which Biology classes you may decide on for next semester?

STUDENT: Well, I was thinking of taking Human Physiology - it sounds relevant and
interesting.

TUTOR:    I think it is an interesting class, but I would recommend against it. You are already in
Human Anatomy, which also covers Physiology so it would be repetitive, and probably not the
best use of your time.

STUDENT: If I already know some of the stuff, I could take it to boost my GPA a little bit.

TUTOR:    Um, you could maybe do that, except the professor that teaches it is famous for
giving out the fewest A’s of any professor.

STUDENT: Oh wow. Never mind.

TUTOR:    Now, changing subjects, have you decided yet on your research topic?

STUDENT: Well, I actually decided. I don’t want to do a research paper. I’m going to do a
dissertation instead. I think I will much more enjoy coming up with a thesis and finding all the
details to support it.

TUTOR:    I see. Didn’t you already do some of the research though? What happened to that?

STUDENT: Well, I started doing research. Then the more data I collected, the more complicated
things got. I realised I would have to take a lot more steps to randomise the sample, and then I
realised I would need to control for more factors.

TUTOR:    I see. What kind of data collection did you do?

STUDENT: I interviewed employees at the water treatment facility after Professor Dickinson
recommended it.

TUTOR:    I understand. Well, the research paper could be much more outside work, but it may
be worth it if that is what you are interested in.
STUDENT: No, I am a lot happier just doing the dissertation.

TUTOR:    That’s fine with me. Just remember, you need to finish your first draft and send it in
soon. It’s already the middle of February!

STUDENT: Oh wow, it is, isn’t it? I just have to finish it by the end of March, right?

TUTOR:    Yep, but don’t forget! The date will really sneak up on you.

STUDENT: OK, I’ll make sure to set reminders in my calendar. And who do I talk to about
registering my dissertation?

TUTOR:    You should go to the Department Office, and then talk to the secretary.

STUDENT : I thought I needed to talk to the Department Head.

TUTOR:    Not for registering. If you need help developing your dissertation, that’s when you
should go to the Department Head.

STUDENT: Oh, I see. And who would I see about getting access to the database of past research
on my topic?

TUTOR:    That you can find in the computer lab, specifically in the office - you’ll have to ask a
lab technician to give you access.

STUDENT: Okay. Well, I’ll go ahead and get started on that then. Thanks!
Exercise 70:

Questions 21 – 25
Complete the flowchart.
Write ONE WORD ONLY for each answer.

Foundation for Essay Writing


Decide on 21……………. you like.

Focus on 22……………. area of interest.

Write 23……………. statement.

Create 24…………….

Ensure this 25……………. clearly.

Questions 26-30
Complete the summary.
Write NO MORE THAN TWO WORDS OR A NUMBER for each answer.
There are several 26……………. involved in producing a good essay. The writer must think
independently and give 27……………. examples as support, each one with a reference (which is
a 28…………….). The formatting must follow the 29……………. issued by the university, as
well as the word count decided by the lecturer, although it can vary by 30…………….
Answer:
21 Topic
22 Current
23 Thesis
24 Outline
25 Progress
26 Aspects
27 Real-life
28 Necessity
29 Style guide
30 10%

Eric: Hi, Ms Harris. Here we are, talking once again.

Ms Harris: Well, I’m always willing to help out wherever possible. Is anything troubling you?

Eric: Well, your advice last week about writing summaries was very useful, and I’m using these
summaries a lot in order to prepare my final master’s thesis.

Ms Harris: But you’re having problems, right?

Eric: Yes, I have a few issues to discuss. Obviously I want a good essay, and I want to  achieve
high marks. But I’m not sure on the best way to start.

Ms Harris: That’s an easy question. Start with a topic, but not one that I necessarily suggest, but
one that you want to explore. You will always write better when doing so on a topic you are
interested in, not one imposed upon you by others.

Eric: Well, I’m interested in management theory.

Ms Harris: Then pursue that. However, that alone is far too broad. Break it
into various current areas of discussion and relevance, then look more carefully at one of them
— say, management and cultural differences, or management and motivation, or other aspects,
such as the role of salary, group cohesion, or leadership.

Eric: I would say I’m interested in group cohesion—that is, how people interact in
the workplace.
Ms Harris: Well, that’s a start, but you can’t then just write planlessly, without defining exactly
what you intend to do within the area you’ve chosen. You’ll have to think of a thesis, and this
statement could be of several types.

Eric: Such as what?

Ms Harris: Oh, you could argue a point, something that you believe in; or discuss an issue,
looking at its various perspectives; or critique the opinions of others, pointing out the pitfalls and
flaws. The thesis statement will make that very clear because it will say, in simple terms, what
you intend to achieve in your essay.

Eric: I see. And then I can just begin writing, right?

Ms Harris: Wrong! [Uh?] If your essay is going to be clear, it needs to be logical and organised,
and this means you’ll need an outline. This could be written as a flowchart, or spider graph —
that is, a series of connected lines, but whatever shape the outline takes, there must be a sense
of progress, in, more or less, a straight line, towards a goal.

Eric: And then my essay will be good? 

Ms Harris: With such progression, definitely—as long as you do achieve everything that you set
out to do, as specified in the beginning, okay? 

Eric: Well, Ms Harris, your advice about essay writing all sounds very useful: solid, and step by
step, so I’ll certainly follow that. But can you tell me in more general terms, what constitutes the
best essay?

Ms Harris: Oh, there are many aspects to consider, but one of the most important is certainly,
original thinking. The best essays are written by people who think for themselves, and not just
copy or imitate established views.

Eric: I think I can do that.

Ms Harris: But remember, you can’t just give a list of unsupported assertions. There needs to be
support as well — a chain of logic linking each step in your argument. [Right] Yet that alone is
not enough. Your argument may be logically sound, but is it practically so? For that, you need
examples, from real-life, to illustrate your points or sub-points.

Eric: I’ve got lots of examples from my course readings. I suppose I could use them. 

Ms Harris: But remember, whenever you use an example, or facts or figures that are
not commonly known, you must give a reference. That’s an academic necessity, without which
your writing will automatically fail.

Eric: So, it’s necessary to give these. Right.


Ms Harris: Absolutely—and with that, your essay will be fine, but always ensure that you format
it clearly.

Eric: What do you mean?

Ms Harris: I mean the practical considerations, as written in the university style guide, such as


leaving appropriate margins, using double spacing, Arial font, and so on. You see, your writing
must look good, as well as be good.

Eric: I can certainly do that.

Ms Harris: And, finally, related to the previous point, you need to respect the number of words
required. Your individual lecturers will give you a word-count figure, so follow what they say,
making your essay neither too long, nor too short—perhaps 10% either way should be
acceptable. 15% was common when I was younger, but it’s a lot stricter now, and 20% would be
pushing the boundaries just a little too far.

Eric: I guess I can do all that. Thanks for your advice.


Exercise 71.
Questions 1-5: Sentence completion
Listen and complete sentences 1-5 below. Write no more than three words for each answer.
When talking to babies adults 1.___________ several times.
Motherese provides a 2.__________ which allows language to develop in children.
In comparison with a 3.___________, a human baby is relatively helpless.
Human mothers used their voices to 4.___________ their young.
Language began as sounds became standardized into 5.___________.
Questions 6-9: Multiple-answer questions
Choose two letters A-E
6-7 Why might mothers have put their babies in slings?
A to transport them
B to protect them
C to control them
D to keep them warm
E to communicate with them
Choose two letters A-E
8-9 Linguists say a comprehensive theory would not explain
A how grammar developed
B how language is learnt
C how sounds got meanings
D how sounds are produced
E how speech developed
Question 10: Multiple-choice question
Choose the correct letter A, B, C, or D.
10 What is the speaker's main purpose?
A to describe how mothers talk to their babies
B to compare how humans and animals look after their young
C to explain a new theory of origin of human language
D to contrast the views of linguists and anthropologists
1. repeat phrases
2. (kind of) framework
3. baby chimpanzee
4. pacify
5. words
6-7. A,D
8-9. A,C
10: C

Hi, I'm Emma Bailey, and today I'm going to be talking `baby-talk'. Hopefully, you'll find the
subject interesting rather than infantile. I'd like to start by getting you to imagine a scenario.
You're in an office or at a family gathering when a mother comes in with her young baby. Like
everyone else, you want to see the mother and baby and you probably want to talk to the baby.
How do you do this? What kind of language do you use? Recent research has shown that adults
all talk to babies in similar ways: they repeat phrases over and over again in a high-pitched
'sing-song' voice with long vowel sounds. And if they ask questions they exaggerate their
intonation. Researchers have discovered that this kind of language, which they have called
motherese, is used by adults all over the world when they talk to babies. And according to a new
theory, motherese forms a kind of framework for the development of language in children. This
'baby talk', so the theory goes, itself originated as a response to another aspect of human
evolution: walking upright. In contrast to other primates, humans give birth to babies that are
relatively undeveloped. So, whereas a baby chimpanzee can hold on to its four-legged mother
and ride along on her back shortly after birth, helpless human babies have to be held and carried
everywhere by their upright mothers. Having to hold on to an infant constantly would have made
it more difficult for the mother to gather food. In this situation, researchers suggest, human
mothers began putting their babies down beside them while gathering food. To pacify an infant
distressed by this separation, the mother would 'talk' to her offspring and continue her search for
food. This remote communication system could have marked the start of motherese. As mothers
increasingly relied on their voices to control the emotions of their babies, and, later, the actions
of their mobile juveniles, words emerged from the jumble of sounds and became
conventionalized across human communities, ultimately producing language. Not all
anthropologists, however, accept the assumption that early human mothers put their children
down when they were looking for food. They point out that even modern parents do not do this.
Instead, they prefer to hold their babies in their arms or carry them around in slings. They
suggest that early mothers probably made slings of some kind both for ease of transportation
and to keep their babies warm by holding them close to their bodies. If this was the case,
they would not have needed to develop a way of comforting or controlling their babies from a
distance. It is not only anthropologists, but also linguists who challenge this explanation for how
language developed. They say that although the motherese theory may account for the
development of speech, it does not explain the development of grammar. Nor, they say, does
it explain, how the sounds that mothers made acquired their meaning. Most experts believe
that language is a relatively modern invention that appeared in the last 100,000 years or so. But if
the latest theory is right, baby talk - and perhaps fully evolved language - was spoken much
earlier than that. We know that humans were walking upright one and half million years ago.
This means that mothers may have been putting their babies down at this time, and
communicating with them in `motherese'. We can be sure that this is not the end of the story, as
anthropologists and linguists will continue to investigate the origins of this most human of
abilities -language.
Exercise 72.
Questions 31-40
Choose the correct letter, A, B or C.

31. The symptoms of synaesthesia


A are the same for everyone with the condition.
B cannot be controlled.
C can be harmful.

32. People who discover that they have synaesthesia


A often say they thought everyone experienced it.
B express negative feelings about their condition.
C wonder what it is like to be normal.

33. One research project looking at synaesthesia


A produced different results from other research.
B was able to estimate the proportion of people with the condition.
C found that a small number of people saw different colours.

34. The condition means some people see numbers or letters


A as either red or blue.
B as days or months.
C as colours or in particular relative positions

35. The way people experience colours


A will be exactly the same for everyone.
B can lead to disagreements.
C differs when a large number of experiments are carried out.

36 When seeing certain words, people with word-taste synaesthesia


A will differ in the way they taste a word.
B generally get a sweet taste in their mouth.
C have similar experiences.

37 What does the speaker say about synaesthesia?


A Infants may all have the condition.
B It is difficult to prove that the condition exists.
C The condition becomes more noticeable during childhood.

38 Family members with synaesthesia


A are not always closely related.
B may not have the same symptoms.
C make up forty per cent of the extended family.

39 People with synaesthesia


A are advised to take up a hobby or interest.
B are often talented artists.
C often take an interest in the arts.

40 What attitude do scientists have towards synaesthesia nowadays?


A It has little scientific worth.
B It may help them better understand how the brain functions.
C They need more proof that it exists.
31 B

32 A

33 B

34 C

35 B

36 A

37 A

38 A

39 C

40 B

Today we're going to look at a fascinating condition that challenges the idea that we all see and
experience the world around us in a similar way. For example, what do you see when I mention a
day of the week or a month? What colour is the letter A? Or the number 10? If you often find
yourself having more than the normal sense sensations, you too could have a condition known as
synaesthesia.

Synaesthesia is a harmless but fascinating condition which is often described by psychologists as


the joining of the senses. We normally experience our senses individually, so we see a colour or
hear a word, whereas people with synaesthesia will find two or more senses being stimulated at
the same time by a single timulus. Some people will see or feel a colour when they hear a sound.
Others will experience a taste or smell when another sense is stimulated. This happens
automatically — the sensation can't be managed.

People often go through life unaware that they have the condition. A common response from
individuals who learn for the first time that they have synaesthesia is one of surprise to
discover that other people don't experience the same thing. It's a normal part of life for them,
and they will rarely describe the symptoms negatively.
To estimate the numbers of people with synaesthesia, one group of researchers sat people in front
of a computer and showed them letters and numbers in black. Participants were asked to choose
a colour for each character they saw. A small proportion of participants, namely those with
synaesthesia, consistently described the same characters as having the same colours. On the
basis of the results, researchers were able to predict that synaesthesia affects about one per
cent of the population. This number has been confirmed in other research.

Synaesthesia takes many different forms, but the most common is to see or feel a colour in
relation to letters and numbers. It's commonplace for people to identify A with red, B with
blue, and so on. Some people will actually see a colour, but in most cases it's a question of
feeling or sensing the colour. However, it's just as commonplace to see days months, letters,
and numbers spatially, that is in lines or circles, for example. People might say they see
Monday up high, Tuesday just below Monday, Wednesday on the left, Thursday on the right,
and so on.

This doesn't mean that people with synaesthesia always agree on what they sense. Two
synaesthetes will often argue over the colour of a letter, for example. But patterns emerge if a
large enough sample of people are observed, providing clear evidence of this condition despite
individual variations.

Colour and spatial synaesthesia are amongst the most common forms of the condition, but they
are by no means the only way people experience it. One of the more interesting combinations is
word-taste synaesthesia.This occurs when words lead the person to experience tastes or certain
taste sensations. So a person's name might have the flavour of a particular sweet places might be
associated with the taste of particular snacks. Taste needs to be seen in a wider context here. The
sensation may be a feeling on the tip of the tongue or at the back of the throat and will
differ from person to person.

Some researchers believe we are all born with the condition and that it's most prevalent in
our early years, but it then tends to become less noticeable as we enter childhood. It's a
fascinating thought that as infants we experience the world around us through our senses in a
different way than as adults. However, testing this hypothesis will be challenging, bearing in
mind the difficulty of getting feedback from young infants!

Research also points to the fact that synaesthesia runs in families. In fact, as many as 40 per cent
of synaesthetes, as they are called, know of someone in the family with a similar condition. This
won't necessarily be a close family member, and the condition may be traceable back to
previous generations or to an extended family member such as a cousin or uncle.

There is evidence that synaesthetes are often creative and will often have artistic hobbies or
interests. Researchers think this is not necessarily because synaesthesia makes them naturally
more talented in this area but the fact that they have multiple sensory experiences generates
an interest in, for example, art or music.
So that's synaesthesia. Apart from its intrinsic interest, for psychologists its a fascinating
indication that we may all experience the world around us in different ways. Once upon a time,
these findings would have been regarded as highly subjective, lacking evidence, and not of any
scientific worth. However, we now have a much greater interest in how the brain helps us
make sense of the world, and the study of synaesthesia is one way for us to discover more
about this.

Exercise 73.

Time Perspectives
Time Zone Outlook Features & Consequences
Past Positive Remember good times, e.g. birthdays.
Keep family records, photo albums, etc
31 ___________ Focus on disappointments, failures, bad
decisions.
Present Hedonistic Live for 32___________; seek sensation, avoid
pain
Fatalistic Life is governed by 33 ___________, religious
beliefs, social conditions. Life ‘s path can’t be
changed.
Future 34___________ Prefer work to play. Don’t give in to temptation.
Fatalistic Have a strong belief in life after death and
importance of 35___________ in life.

Questions 36–40
Choose the correct letter, A, B or C.
36 We are all present hedonists
A at school
B at birth
C while eating and drinking
37 American boys drop out of school at a higher rate than girls because
A they need to be in control of the way they learn
B they play video games instead of doing school work
C they are not as intelligent as girls
38 Present-orientated children
A do not realise present actions can have negative future effects
B are unable to learn lessons from past mistakes
C know what could happen if they do something bad, but do it anyway
39 If Americans had an extra day per week, they would spend it
A working harder
B building relationships
C sharing family meals
40 Understanding how people think about time can help us
A become more virtuous
B work together better
C identify careless or ambitious people

31. negative

32. pleasure

33. poverty

34. active

35. success

36. B

37. A

38. C

39. A

40. B

Today, I’m going to be talking about time. Specifically I’ll be looking at how people
think about time, and how these time perspectives structure our lives. According to
social psychologists, there are six ways of thinking about time, which are called
personal time zones.
The first two are based in the past. Past positive thinkers spend most of their time in a
state of nostalgia, fondly remembering moments such as birthdays, marriages and
important achievements in their life. These are the kinds of people who keep family
records, books and photo albums. People living in the past negative time zone are
also absorbed by earlier times, but they focus on all the bad things – regrets, failures,
poor decisions. They spend a lot of time thinking about how life could have been.
Then, we have people who live in the present. Present hedonists are driven by
pleasure and immediate sensation. Their life motto is to have a good time and avoid
pain. Present fatalists live in the moment too, but they believe this moment is the
product of circumstances entirely beyond their control; it’s their fate. Whether it’s
poverty, religion or society itself, something stops these people from believing they
can play a role in changing their outcomes in life. Life simply “is” and that’s that.
Looking at the future time zone, we can see that people classified as future active are
the planners and go-getters. They work rather than play and resist temptation.
Decisions are made based on potential consequences, not on the experience itself. A
second future-orientated perspective, future fatalistic, is driven by the certainty of life
after death and some kind of a judgement day when they will be assessed on how
virtuously they have lived and what success they have had in their lives.
Okay, let’s move on. You might ask “how do these time zones affect our lives?” Well,
let’s start at the beginning. Everyone is brought into this world as a present hedonist. No
exceptions. Our initial needs and demands – to be warm, secure, fed and
watered – all stem from the present moment. But things change when we enter formal
education – we’re taught to stop existing in the moment and to begin thinking about
future outcomes.
But, did you know that every nine seconds a child in the USA drops out of school?
For boys, the rate is much higher than for girls. We could easily say “Ah, well, boys
just aren’t as bright as girls” but the evidence doesn’t support this. A recent study
states that boys in America, by the age of twenty one, have spent 10,000 hours
playing video games. The research suggests that they’ll never fit in the traditional
classroom because these boys require a situation where they have the ability to
manage their own learning environment.
Now, let’s look at the way we do prevention education. All prevention education is
aimed at a future time zone. We say “don’t smoke or you’ll get cancer”, “get good
grades or you won’t get a good job”. But with present-orientated kids that just doesn’t
work. Although they understand the potentially negative consequences of their
actions, they persist with the behaviour because they’re not living for the future;
they’re in the moment right now. We can’t use logic and it’s no use reminding them
of potential fall-out from their decisions or previous errors of judgment – we’ve got to
get in their minds just as they’re about to make a choice.
Time perspectives make a big difference in how we value and use our time. When
Americans are asked how busy they are, the vast majority report being busier than
ever before. They admit to sacrificing their relationships, personal time and a good
night’s sleep for their success. Twenty years ago, 60% of Americans had sit-down
dinners with their families, and now only 20% do. But when they’re asked what they
would do with an eight-day week, they say “Oh that’d be great”. They would spend
that time labouring away to achieve more. They’re constantly trying to get ahead,
to get toward a future point of happiness.
So, it’s really important to be aware of how other people think about time. We tend to
think: “Oh, that person’s really irresponsible” or “That guy’s power hungry” but often
what we’re looking at is not fundamental differences of personality, but really just
different ways of thinking about time. Seeing these conflicts as differences in time
perspective, rather than distinctions of character, can facilitate more effective
cooperation between people and get the most out of each person’s individual
strengths.
Exercise 74.
Questions 31-33
Choose the correct letter A, B or C.
31 The local business people who had approached the Centre had all encountered
A enormous problems.
B few problems.
C many obstacles.
32 The main focus of the Centre is now
A large national companies.
B technology companies.
C businesses that have just started up.
33 Snapshot research was carried out
A over the Internet.
B by telephone.
C by personal contact.
Questions 34 and 35
Answer the questions below. Write NO MORE THAN THREE WORDS AND/OR A
NUMBER for each answer.
34 How much higher are local business rents compared to those nationally?
___________
35 How many local businesses close a year after they have started working with the Centre?
___________
Questions 36-40
Complete the table below. Write NO MORE THAN THREE WORDS for each answer.
Size of business Companies Help being given
Start-ups O-foods improving the 36___________ turnaround
Innovations support to attract business partners and
achieve 37___________
Small Sampsons Ltd business 38___________
Vintage Scooter product monitoring scheme after sales
customer service
Medium Build Ltd extension of 39___________
Jones Systems conflict management and 40___________
31 A: Listen when the speaker says: We established the Centre in response to approaches from
several business people ... Moreover, they had all without exception come up against B is
incorrect because it is the opposite. C is incorrect because although obstacles is mentioned, the
speaker says: ...enormous bureaucratic obstacles — we do not know how many.

32 C: Listen for the words centre and focus. The other two are incorrect because neither are
mentioned.

33 B: Listen for snapshot research and conducted. The speaker does not mention the Internet (A)
or personal contact (C).

34 33/thirty-three %/per cent/percent: Listen for: The most common reasons given for the
businesses closing were: first, high rents ,..

35 2/two: Listen for: Since the centre came into existence three years ago, we have helped to
change this climate ("failure. The current statistics ...

36 stock

37 production targets

38 expansion plans

39 (company's) product range

40 team building
Exercise 75.
Questions 31–40
Complete the notes below.
Write NO MORE THAN TWO WORDS AND/OR A NUMBER for each answer.
Climate change
HUMAN FACTORS
• Cutting down trees for 31 ……………………
• Industrial Revolution
• 32 ……………………
• Increase in population  deforestation
KNOWN EFFECTS
• Over previous 130 yrs: temp has increased by 0.6 ° C
• Since Ind. Rev.: CO2 has increased by 30% & Methane has increased by 33
…………………… (from mining, animals, rice paddies)
• N2O has increased (from 34 …………………… esp. fertiliser; waste management; car
exhausts)
• Greenhouse Effect: gases form 35 …………………… → heat trapped → Earth warms up
FUTURE EFFECTS
1. Rise in sea levels → ice melting
Sea level Number of people at risk
1998 levels 36 ……………………
+50 cm 92 million
+1 metre 37 ……………………

2. Change in 38 ………………… → more arid areas → population movement to cities


3. Increase in pests and 39 …………………… e.g. malaria
4. Change in ecosystems:
*shift in 40 …………………… – some die, others multiply
*deserts get hotter & bigger
31. fuel

32. combustion engine

33. 145% / 145 per cent

34. agriculture

35. (a) barrier

36. 46 million

37. 118 million

38. vegetation zones

39. (certain) disease(s)

40. species composition

This lecture in Environmental Studies is on the topic of human influence on


climate change. First, I’ll outline some of the factors affecting climate, then go on to
discuss what has already occurred, and finish up by speculating on the effects.
Previously, we’ve covered how factors such as ocean currents and prevailing
winds affect climate change naturally. However, the influence of human activity on
climate is what I’ll talk about today. At first, the effect on the climate was relatively
small; trees were cut down to provide fuel for fires, and, as we know, trees absorb
carbon dioxide and produce oxygen so the amount of carbon dioxide in the atmosphere
would have increased – but not noticeably.
So, in what ways has human activity really impacted on the climate? A major
contributor was the advent of the Industrial Revolution at the end of the 18 th century,
combined with the invention of the combustion engine. In addition, Earth’s burgeoning
population has had a marked effect on climate. The first two factors saw increased
amounts of carbon dioxide being released into the atmosphere from the burning of fossil
fuels, such as coal and oil. The final one, human expansion, has resulted in deforestation
on such a scale that the extra carbon dioxide in the air cannot be soaked up and
converted into oxygen by the remaining trees.
Okay – so what has already happened? Well, global temperatures have risen by
0.6 degrees Celsius in the last 130 years. Levels of carbon dioxide, methane and nitrous
oxide gases have escalated. Carbon dioxide concentrations have climbed by 30% and
methane levels have increased by 145% since the beginning of the Industrial
Revolution. Gas produced by fossil fuel extraction, livestock and paddy fields is
primarily responsible for the growth of methane levels. Nitrous oxide, or N2O, comes
from natural sources – wet tropical forests, for instance – but it is also produced by human-
related activities such as agriculture, which uses synthetic nitrogen fertilisers,
rubbish disposal systems and vehicle emissions.
How do gases like carbon dioxide and methane affect the climate? Well, this is
what we call the Greenhouse Effect. Under normal conditions, the sun’s rays hit the
earth and some are reflected back into space. However, these gases (CO2 and methane)
create a barrier in the atmosphere which prevents a proportion of the sun’s rays from
being reflected back into space – and, instead, the gases become trapped in the
atmosphere. It’s simple really – because the sun’s rays can’t escape, the Earth heats up.
What are the possible effects? Firstly, a rise in sea levels: we already know that
the Arctic ice cap has melted and shrunk considerably and great chunks of ice have been
lost from Antarctica. In 1998, it was reported that 46 million people lived in areas at
risk of flooding … and the number of people at risk will increase significantly if sea
levels rise. It is estimated that a rise of only 50 centimetres would put that number at 92
million. Further projections would see a rise of one metre put 118 million people in
danger of losing their homes and livelihoods – not to mention the loss of prime, fertile
farmland. Experts predict a rise of at least 50 centimetres over the next 50 years or so.
Secondly, there would be a modification of vegetation zones with changes in
the boundaries between grassland, shrub land, forest and desert. This is already causing
famine in arid areas of north-eastern Africa, and has instigated – and will continue to
instigate – mass movements of people away from dry regions. What we are seeing now
is only the first stage, with temporary camps for climate refugees already at
overcapacity; in the future, there will be significant migration resulting in extreme
overcrowding of towns and cities.
Another potentially disastrous effect of climate change is an increase in the
range and distribution of pests which could bring about an increase in the prevalence of
certain diseases. If we think of the malaria-carrying mosquito, for example, which
thrives in warmer regions – at the moment, about 45% of the world’s population is
exposed to malaria – but with an increase in temperature, there will be many millions
more cases of malaria a year.

The last effect I’m going to mention today is the change in ecosystems. Global
warming will influence species composition – for both fauna and flora – such that some
animal species will disappear and others will multiply; and it’ll be the same for plants
and trees. It is predicted that around two-thirds of the world’s forests will undergo major
changes of some kind. Scientists also expect deserts will become hotter and, of course,
desertification will continue at an increasingly worrying rate and will become harder, if
not impossible, to reverse.
What can we do to stop the process? Well, that’s the subject of next week’s
lecture – so I hope to see you all there.

Exercise 76.
Questions 31-40
Complete the table below.
Write ONE WORD ONLY for each answer.
Herzberg's Two Factor theory
A model for understanding what causes employee satisfaction and dissatisfaction with their job.
- Two factors
a. Factors that lead to increased satisfaction: 31. ______________
b. Factors that lead to increased dissatisfaction: 32. ______________ factors
- Factors leading to satisfaction and dissatisfaction are considered to be 33. ______________,
hence ‘two factor' theory
- Factors that increase satisfaction:
a. Achievement
b. 34. ______________
c. Enjoyment of work
d. More responsibilities
e. Personal 35. ______________
- Factors that increase dissatisfaction:
a. Salary (never enough)
b. Company policy and administration
c. Relations with 36. ______________
d. Working conditions
e. Relations with colleagues
- Possible combinations
Combinations Characteristics
- High H, High M — Staff have desire to succeed; few 37. ______________
- High H, Low M — No big issues, but not driven to improve; focus on salary.
- Low H, High M — Job stimulating but conditions are 38. ______________
- Low H, High M — Job is uninteresting and conditions are poor.
- According to Herzberg, it is necessary to eliminate job dissatisfaction before job satisfaction
can be increased.
- Ways to eliminate job dissatisfaction
a. Pay fair salaries
b. Guarantee employee 39. ______________ in the workplace
c. Create positive company culture (team building activities)
- Ways to increase job satisfaction:
a. Provide more challenging and 40. ______________ work
b. Pathway to promotion

31. motivators
32. hygiene
33. independent
34. recognition
35. advancement
36. supervisor
37. complaints
38. inadequate
39. security
40. rewarding

Morning everyone. This semester, we are focusing on different approaches for understanding
how employees can be motivated to perform well at their jobs. Today we are going to be looking
at a theory developed in the 1960s by a German psychologist named Frederick Herzberg, which
has been very influential on management training courses. However, it is not without its
drawbacks, as we shall see later.

Now, the basic idea that Herzberg developed is that, in a given workplace, there are two types of
factors at play. On the one hand, there are those factors that create increased job satisfaction,
which Herzberg called the 'motivators'. On the other hand, there are those aspects of a job that
lead to increased dissatisfaction, which are labeled 'hygiene' factors. Having identified these two
factors, these motivators and hygiene factors, Herzberg went on to state that a consequence of his
theory was that the underlying reasons for job satisfaction and job dissatisfaction must be
understood as independent of each other, hence the common name of this approach, the 'two-
factor theory'.

Let's firstly consider the former set of factors in more detail. What aspects of a job tend to
motivate employees and thus bring about greater job satisfaction? Well, Herzberg's own
conclusions were not based on armchair speculation, but rather supported by research carried out
within a variety of corporations. Perhaps unsurprisingly, he found that achievement was the
number one factor leading to increased satisfaction. Obviously, everyone likes to be a success!
This was followed by recognition, that is to say, having one's contribution valued. The work
itself, having more responsibility, and personal advancement, such as getting a promotion,
completed the list.
Now, some of you may have noticed something missing from the list that usually gets included
when people give their main reasons for working hard - salary. So, why isn't it there? Well, the
answer is that, for Herzberg, salary is not a great motivator because1 any pay rise will never
completely satisfy. Instead, it will eventually give rise to a demand for more money, while
insufficient salary will contribute to the employee dissatisfaction.

Apart from salary, what are some of the other factors leading to employee dissatisfaction? Well,
again, based on Herzberg's research, the findings tell us that company policy and administration,
relationship with one's supervisor, work conditions and how well the employee gets on with
colleagues are also key 'hygiene' factors. However, it is perhaps not surprising that how well an
individual interacts with the people around them will affect their degree of satisfaction with their
job.

Ok, so we have looked at the two different types of factors and discussed why, according to the
theory, some factors increase employee satisfaction and some contribute to dissatisfaction.
Before we move on, are there any questions?

Thus, according to the Two-Factor theory, we can find that there are four possible combinations.
The ideal situation is high hygiene and high motivation, which means that employees are
stimulated by their work and have few complaints. If only it were so easy! The second scenario
is high hygiene and low motivation. Employees have no major issues with their work, but are not
driven to improve themselves, and their focus is on the level of salary they receive. The third is
low hygiene and high motivation. The job is stimulating and challenging, but the salary and
working conditions are inadequate, which means that the employees have a lot of complaints.
The fourth and worst set of circumstances is where the employees have both low hygiene and
low. This means both that the staff are not motivated and also that they have many complaints.

Right, so having discussed the distinction between motivators and hygiene factors, what are
some of the consequences of the theory? Well, in terms of increasing employee well-being,
Herzberg thought that it was important to eliminate job, dissatisfaction first before going onto
creating conditions for job satisfaction. This is because otherwise they would work against each
other. There are several ways to decrease dissatisfaction, but some of the most important ways
are those discussed earlier, such as paying reasonable wages, ensuring employees job security,
and creating a positive culture in the workplace, for example by using team building activities.

However, eliminating dissatisfaction is only one half of the task of the two factor theory; the
other half would be to increase satisfaction in the workplace. This can be done by improving on
motivating factors, which are needed to stimulate an employee to achieve a higher level of
performance. As previously mentioned, these often relate to opportunities for increased
recognition, responsibilities and respect within the workplace, such as being assigned more
challenging and rewarding tasks, or by achieving a promotion to a higher level position.
Ok, so in summary we can see that, according to this theory, what makes the employee happy in
their job and what makes them unhappy exist as two separate sets of factors. However, is it really
the case that factors relating to employee satisfaction in a workplace can be divided so neatly
into two categories? Some critics have suggested otherwise. Let's now move on to look at some
of these criticisms, starting with...

Exercise 77.
Questions 31-40
Complete the notes below. Write ONE WORD ONLY for each answer.
Agricultural programme in Mozambique
How the programme was organised
• It focused on a dry and arid region in Chicualacuala district, near the Limpopo River.
• People depended on the forest to provide charcoal as a source of income.
• 31 ______________ was seen as the main priority to ensure the supply of water.
• Most of the work organised by farmers' associations was done by 32 ______________.
• Fenced areas were created to keep animals away from crops.
• The programme provided
— 33______________ for the fences
— 34______________ for suitable crops
— water pumps.
• The farmers provided
— labour
— 35______________ for the fences on their land.
Further developments
• The marketing of produce was sometimes difficult due to lack of 36 ______________.
• Training was therefore provided in methods of food 37______________.
• Farmers made special places where 38______________ could be kept.
• Local people later suggested keeping 39 ______________.
Evaluation and lessons learned
• Agricultural production increased, improving incomes and food security.
• Enough time must be allowed, particularly for the 40 ______________ phase of the
programme.
31 Irrigation

32 women

33 wire(s)

34 seed(s)

35 posts

36 transport

37 preservation

38 fish(es)

39 bees

40 design

I'm going to report on a case study of a programme which has been set up to help rural
populations in Mozambique, a largely agricultural country in South-East Africa. The programme
worked with three communities in Chicualacuala district, near the Limpopo River. This is a dry
and arid region, with unpredictable rainfall. Because of this, people in the area were unable to
support themselves through agriculture and instead they used the forest as a means of providing
themselves with an income, mainly by selling charcoal. However, this was not a sustainable way
of living in the long term, as they were rapidly using up this resource.

To support agriculture in this dry region, the programme focused primarily on making use of
existing water resources from the Limpopo River by setting up systems of irrigation, which
would provide a dependable water supply for crops and animals. The programme worked closely
with the district government in order to find the best way of implementing this. The region
already had one farmers' association, and it was decided to set up two more of these. These
associations planned and carried out activities including water management, livestock breeding
and agriculture, and it was notable that in general, women formed the majority of the workforce.
It was decided that in order to keep the crops safe from animals, both wild and domestic, special
areas should be fenced off where the crops could be grown. The community was responsible for
creating these fences, but the programme provided the necessary wire for making them. Once the
area had been fenced off, it could be cultivated. The land was dug, so that vegetables and cereals
appropriate to the climate could be grown, and the programme provided the necessary seeds for
this. The programme also provided pumps so that water could be brought from the river in pipes
to the fields. However, the labour was all provided by local people, and they also provided and
put up the posts that supported the fences around the fields.

Once the programme had been set up, its development was monitored carefully. The farmers
were able to grow enough produce not just for their own needs, but also to sell. However, getting
the produce to places where it could be marketed was sometimes a problem, as the farmers did
not have access to transport, and this resulted in large amounts of produce, especially vegetables,
being spoiled. This problem was discussed with the farmers' associations and it was decided that
in order to prevent food from being spoiled, the farmers needed to learn techniques for its
preservation.

There was also an additional initiative that had not been originally planned, but which became a
central feature of the programme. This was when farmers started to dig holes for tanks in the
fenced-off areas and to fill these with water and use them for breeding fish — an important
source of protein. After a time, another suggestion was made by local people which hadn't been
part of the programme's original proposal, but which was also adopted later on. They decided to
try setting up colonies of bees, which would provide honey both for their own consumption and
to sell. So what lessons can be learned from this programme? First of all, it tells us that in dry,
arid regions, if there is access to a reliable source of water, there is great potential for the
development of agriculture. In Chicualacuala, there was a marked improvement in agricultural
production, which improved food security and benefited local people by providing them with
both food and income. However, it's important to set realistic timelines for each phase of the
programme, especially for its design, as mistakes made at this stage may be hard to correct later
on.

The programme demonstrates that sustainable development is possible in areas where...


Exercise 78:

Questions 31-40
Complete the notes below
Write ONE WORD ONLY for each answer.
The Gherkin Building
Commissioned by: 31………….. firm called Foster and Partners
The features of its appearance:
• Its shape is like a 32…………..
• It can reduce the carbon 33………….. of the citys.
• It lets 34………….. pass through the building, both reducing heating costs and brightening up
the workspace
• One false story claimed that the exterior of the building is partly made of 35…………..
Architectural concept:
• links 36………….. with the workplace.
• relies less on 37………….. for temperature control than other similar buildings.
The features of its interior:
• The atria that let fresh air pass through the interior are known as 38…………..
• There is a place for entertainment called the 39………….. at the top of the building.
The future of urban planning and architecture:
• It is likely that the entire 40………….. will be designed with more similarly ecofriendly
buildings in future.
• A new building will be constructed aiming to produce zero waste and remove carbon dioxide
from us as much as possible.
Answer:

31 law (offices)
32 cigar
33 footprints
34 light
35 grass
36 nature
37 air-conditioning
38 lungs
39 clubroom
40 city

Today I’d like to tell you about how U.K. architects are playing their part to address the issue of
global warming. You have seen many of these iconic buildings while going about your everyday
life, but you may not know how they are affecting your tomorrow.

In 2003, construction was completed on the famous Swiss Re Building, or more informally
called the Gherkin, a true masterpiece commissioned by the law offices of Foster and Partners.

This is not the first ambitious endeavor of the firm - they are renowned for their various
philanthropic, environmental efforts. The Gherkin, with its cutting edge green initiative and
sharp design, is gaining recognition as an icon in modern architecture. You can pick it out of the
London skyline by its unorthodox cigar shape.

While its appearance is the obvious attribute at which to marvel, there is far more to this building
than meets the eye. And let’s face it - there’s a lot about this building that meets the eye. The
building helps reduce the city’s carbon footprints in a number of ways.

Just a quick note - in case you’re not familiar with the term ‘carbon footprints,’ get used to it! It’s
a buzzword you’ll hear relentlessly to talk about reducing emissions. Think of it as the amount of
harmful greenhouse gases that are given off into the environment by a single person,
organisation, or product.
So going back to the Gherkin Building - perhaps the most obvious as well as the most significant
eco-friendly feature is the glass windows, which allow light to pass through the building, both
reducing heating costs and brightening up the workspace.

The ingenuity behind the various eco-friendly aspects of the Gherkin has seen its fair share of
publicity both from serious and silly sources. In a recent April Fool’s Day edition, one e-
publication printed a story detailing plans to replace 50% of the current exterior with grass,
which would not only make large steps in the name of sustainability, but also give the building
the green hue that would truly earn it the nickname of the Gherkin. The only drawback is, as you
may have guessed, that this story was an April Fool’s Day joke and completely made up.

In all seriousness though, the building is setting a new standard of design that other architects
and city planners just cannot ignore. The building’s bold and cost-efficient design has won a
number of architecture awards, including the Stirling Prize, the London Region Award, and the
Emporis Skyscraper Award among others. The design comfortably accommodates a large
number of offices while keeping maintenance and operation costs down, striking a superb
balance between nature and the workplace.

Nature is well arid good, as long as the weather is nice outside. Given London’s notoriously bad
weather, the architects knew they must devise a quality temperature regulation system, and that
they did. A special system designed to reduce the building’s reliance on air-conditioning was
devised that cuts consumption in half compared to standard office buildings. There are atria that
link each floor vertically to one another, forming spiraling spaces up the entire building. They
serve not just as social common spaces but also act as the building’s lungs, distributing clean air
from the opening panels in the facade through the entire building.

The building isn’t all business though - it has its fair share of fun as well. At the very top,
a clubroom offers a picturesque entertainment spot for company functions, private parties, etc.,
with a breathtaking panoramic view of the city.

The creation of such an innovative structure has many wondering what the future of urban
planning and architecture may be. Well, if the other projects currently commissioned by Foster
and Partners are any indication, the entire city constructed with similarly eco-friendly buildings
is not far in the distance. The Masdar City development aims to create a desert city that produces
zero waste and removes as much carbon dioxide from the atmosphere as it puts in - a huge feat in
protecting our earth.

The Gherkin is a truly impressive feat, yet it is not the only one worth noting. Now to move on to
another green initiative, I’ll tell you about the Eden Foundation Building, found in Cornwall...
Exercise 79:

Questions 31-40
Complete the notes below.
Write NO MORE THAN TWO WORDS for each answer.
Advertising Effect
The important factor to consider
• The 31………….. that customers must travel affects the probability that they will buy the
product.
Methods of communication
• Advertising slogans are easier to remember if there is a 32………….. played with them.
• Mandy’s Candy Store appeals to people’s sense of 33………….. to draw in customers.
• To an ad campaign for digital products, it is 34………….. that is extremely important.
Effect on your product sales
• The customer’s 35………….. after he or she experiences the ad is most important.
Marketing strategies
• On international flights, it is wise for advertisements to be displayed in the
common 36………….. of most passengers.
• Very few young people buy 37…………..
• The UNESCO website would be a good place to advertise for companies aiming to improve
the 38…………..
• One good location to place ads for suntan lotion is the 39…………..
• A good scene for a water purification commercial would be wonderful sights of a
Answer:

31 distance
32 sound
33 smell
34 (the) flexibility
35 reaction
36 native languages
37 (a) newspaper(s)
38 environment
39 swimming pool
40 national park

Hello class, and welcome back to Marketing Strategies. This week I will expand upon last
week’s lecture by talking about factors you should consider when creating advertising materials
and the effects they can have on your product sales.

Lesson one: limit your advertising to the geographic area of your target market. Though you may
have a product that people want in a large area, the distance that customers are willing to travel is
a significant factor in their choice of where to purchase that product.

Take this example - if you are really hungry and decide you want a burrito, would you choose
the restaurant that is a block from your apartment, or the one that is just as good - or even
slightly better - across town? Of course you’ll pick the closer restaurant.

Next, there’s the method of communication to your target market. How do you decide among
radio ads, TV commercials, fliers, or even word of mouth?

While we often think of the visual presentation of ads, there is much more to advertising than the
look. Studies show that consumers are much more likely to remember advertising slogans if
there is also a sound played. Did you know that your sense of smell is closely linked with
memory? Think about Mandy’s Candy Store up the road. Every time you walk past it, you can
just smell the chocolate, right? I bet you can almost smell it now. Just mentioning the name
brings about the smell memory, and in turn a chocolate craving. What better way to
sell chocolate bars?

Obviously sometimes appealing to the senses isn’t the most practical way to advertise. For
example, it is a good idea to come up with a marketing strategy that adapts to the product,
especially digital products. The flexibility of this kind of product is extremely important, so it is
very common for advertisers to form one single layout for all of their ads - the visual, the
medium, even the majority of the content - and simply update the ad each time they come out
with a new version.

Remember, advertising is all about stirring up the right feeling in your potential customers,
whether by stimulating the senses, appealing to intellect, and so on. Once the customer
experiences the ad, the important thing is his or her reaction. Someone could love the ad you
made, but unless he or she considers buying the product, you have failed to get the reaction you
were looking for.

So once you have successfully reached your target customer and you have his or her business,
often you will want to expand to a larger market. More often than not, the same marketing
strategies you used in your small campaign may not work for a larger audience. The larger you
scale your product, the more factors you must consider. For instance, Apple operates worldwide,
so they must tailor their advertising for each market they enter. Often you’ll see Apple ads on
international flights that appear not only in English, which is the lingua franca of most regions,
but also in the native language of the majority of passengers. I traveled to Russia last week,
and it was really interesting to see the same Nike ad that I have seen a hundred times - except
this time it was in Russian!

Okay, going back to the medium of the advertisement, even after choosing to create print ads
(instead of radio announcements, television commercials, etc) there is more to consider. If you
print your ad in a newspaper, it will be read by a far different audience than if you print your ad
in a popular magazine. Would you put an ad for the new Justin Bieber album in a newspaper?
Probably not, because that product is most suitable for youths. Let’s face it - do you
know anyone under the age of 25 that buys a newspaper? No!

Now let’s try a few strategy exercises. Imagine you are a company that is aiming to improve
the environment by making products that reduce human waste. How would you advertise your
product? Clearly it would send the wrong message if you put up fliers or other materials that
cause lots of waste paper. Consider instead putting commercials on the Health Channel, or
buying ad space on websites like UNESCO.

Or here’s another example: What is one great place to advertise suntan lotion? How about
a swimming pool? It has the exact group of people that need the product.
Alright, one last thing. Let’s say you’re filming a commercial for a water filter pitcher. What
would be good scenery to use for the background? Think about somewhere calm and relaxing
with clean, fresh water. Can’t you see how much more effective a commercial with the beautiful
scenery and flowing rivers of a national park would be than, say, water dripping from a tap?

So to wrap things up today, think about the geography of your target market, the type of
marketing material you should use, and the most effective way to appeal to the customer in order
to make a successful ad campaign.

That is all I have for you all today. Make sure to read through chapter 8 for Monday if you have
not done so already. OK, now I’d be happy to answer any questions you may have...

Exercise 80:

Questions 31-40
Complete the notes below.
Write NO MORE THAN TWO WORDS for each answer.

The role of sleep in humans and animals


Importance of sleep in animals
• Compared to those without food, animals without sleep are more likely to suffer
from 31……………
• Sleep is necessary for all animals, whether they are reptiles, 32…………… or fish.
Differences in animals’ sleep and reasons for their ways of resting
♦ Dolphins
• swim to the surface when sleeping, because they need to 33……………
♦ Birds
• are constantly 34…………… in the presence of numerous predators.
♦ Horses
• do most of their sleeping standing up.
• do occasionally take short naps lying down. Lying in one position for a long time could well
injure a horse, because their 35…………… are delicate.
Potential problems encountering
• Animals can also have 36……………, the same as humans.
Importance of sleep in humans
• It helps us to organise our 37…………… of the day.
• It plays a key role in 38……………
e.g.
• Because of the similar sleeping pattern to that in humans, 39…………… are studied in order to
increase our knowledge of human physiology.
• Scientists choose to study the 40…………… of the fruit fly in order to know the function of
the human gene and understand developmental processes in humans.
Answer:
31 (an) attack
32 mammals
33 breathe
34 on edge
35 bones
36 sleeping problems
37 memories
38 learning
39 rats
40 genetic structure

Good morning, everyone. In today’s lecture, I’m going to talk about the role of sleep in humans
and animals. Of all the biological processes in the animal kingdom, sleep is perhaps the most
important. A human can survive for almost two weeks without eating, but did you know that one
week without sleep can be fatal? It’s even worse for animals, especially for those who must
avoid predators. Without sleep, an animal is much less capable of avoiding an attack - this is the
case for all animals, whether they are reptiles, mammals or fish. Let us look now at how different
animals sleep, reasons for their ways of rest, and the potential problems they might encounter.

In marine life, sleep must be balanced with breathing. For example, the dolphin must float to the
surface as it sleeps in order to breathe. Like other large sea mammals, they keep one eye open
and one half of the brain awake at all times to maintain some amount of consciousness required
to breathe and to watch out for possible threats. They sleep with only one brain hemisphere in
slow-wave sleep.

Birds also have unusual sleeping patterns, mostly due to being constantly on edge in the presence
of numerous predators. They usually sleep quite lightly -for example, Swainson’s Thrush, also
called Olive-backed Thrush, is a medium-sized thrush that takes hundreds of naps during the
day, each of which lasts just a few seconds. While migrating, migratory birds tend to function
well on micronaps.
Horses, on the other hand, do most of their sleeping standing up. Scientists think that horses
develop their habit of sleeping upright as a defense mechanism, a way of protecting themselves
against predators, and a standing position keeps a horse in a constant state of readiness to race
away if danger should approach.

Also, horses do occasionally take short naps lying down. Horses are heavy animals with big
muscles, but their bones are surprisingly delicate, so lying in one position for a long time could
well injure a horse.

Just like humans, animals can also have sleeping problems. Dr. John Hedricks and Adrian
Morrison from the School of Veterinary Medicine, University of Pennsylvania in Philadelphia,
determined that certain diseases were primarily associated with the sleep states in animals. In
their research they emphasised that because so much in this area still remains unclear, animal
models were very important for studies of sleep disorders. The physiology of sleep in animals
is similar to that of humans.

But why do we humans sleep? Researchers and scientists believe it helps us organise
our memories of the day - that sleep acts as a kind of filing system for the brain. Without it, our
thoughts become disorderly and confused, which leads to increased likelihood of accidents and a
tendency to say and do bizarre things. Researchers also believe that sleep plays a key role
in learning. We sleep so that the brain can integrate new knowledge and form new associations.
Because of the similar sleeping pattern to that of humans, rats are often studied in order to
increase our knowledge of human physiology. In one study, rats were kept awake for almost two
weeks, and their behavior was observed. Researchers found the sleep-deprived rats could hardly
remember anything of what they had been taught that day. For example, one rat had been taught
to recognise pictures of various Parisian landmarks in order to receive food - pressing a button
below a picture of the Louvre would result in food being released and so forth. However, when
deprived of sleep, they would press buttons seemingly at random.

In addition to rats, the fruit fly, a small insect that feeds and breeds on spoiled fruit, also has been
used as a model organism and thousands of scientists around the world work on it. But why was
the fruit fly chosen to be studied? It was for practical reasons. The most important one is that the
relationship between fly and human genes is so close that the sequences of newly discovered
human genes, including genes that show a susceptibility, can often be matched against their fly
counterparts. This provides an indication of the function of the human gene and could help in the
development of effective drugs to help people with sleeping disorders. Therefore, many scientists
today choose to study the genetic structure of the fruit fly, which could make a particularly
important contribution to the understanding of developmental processes in humans.

In conclusion, sleep is a necessary part of life not just for humans, but for the entire animal
kingdom. Now, I’d be happy to answer any questions you may have...
CHAPTER 2: MULTIPLE CHOICE

Exercise 1:

For questions 11-15, listen to a radio interview in which a psychologist, Colin Fraser, talks
about cultural identity and choose the correct answer A, B, C, or D which fits best according
to what you hear. Write your answers in the corresponding numbered boxes provided.
11. When discussing his own cultural identity, Colin reveals
A. his resilience to changing cultures.
B. his unorthodox family background.
C. his ability to adapt.
D. his feeling of alienation.
12. What does Colin regard as the defining aspect of a person’s cultural identity?
A. the sense of birth right
B. the emotion it generates
C. the physical proximity to heritage
D. the symbols of tradition
13. What is the influence of a culture attributed to?
A. the dissemination of wisdom
B. connection between societies
C. knowledge of one’s background
D. the practice of archaic rituals
14. According to Colin, what makes a culture successful on the global scene?
A. its capacity for tolerance
B. its isolation from the mainstream
C. its aptitude for resolving conflicts
D. its ability to be self-effacing
15. During the conversation, Colin is
A. distinguishing between birthplace and residence.
B. advocating the celebration of heritage.
C. highlighting the differences in societies.
D. addressing the issues raised by conflicting cultures

11. C
12. B
13. C
14. A
15. B

Interviewer: Hello listeners and welcome to the programme. Later we’ll be speaking to


community leader Jaqueline Epping about efforts to incorporate the wide diversity of residents in
her area, but up first we have psychologist Colin Fraser who is going to give us an insight into
why this incorporation is both necessary and beneficial. Welcome Colin.
Colin: Thank you, I’m happy to be here.
Interviewer: Now Colin, you’re a bit of a mixed bag aren’t you? You were born in Canada,
grew up in Scotland and now you reside in the States. So, if we’re talking about cultural identity,
what identity do you relate to?
Colin: I’ve also lived in England and France, and you’re right that I wasn’t born in Scotland but
my parents and my siblings were. In fact, my heritage goes back a number of years in that
country, so, even though I’m not Scottish by birth, I would have to say that I identify
predominantly with the Scottish culture. Having said that though, I think I’ve managed to
incorporate aspects of each of the cultures I’ve been exposed to. (11)
Interviewer: So what makes you predominantly Scottish?
Colin: Ooh, that’s not an easy question to answer. There are so many facets that make up a
person’s identity that I don’t think there’s one physical factor that I could point to and it’s
difficult to categorise culture. Some might say it’s determined by birth, but I think that’s too
simplistic. There’s more to it than that but without going to particulars, I would have to say I just
feel Scottish. And that, to me, is what’s important. I feel a sense of belonging, even when I’m
not there. (12) I went to a highland games recently, in America. You know, the outdoor event
with lots of kilts and caber tossing. It was great. Watching all the athletic events and dancing was
an amazing sensation. Even though it was a long way from Scotland, while I was there I just felt
at home. As soon as the mass pipe band started playing I was transported to what I consider my
homeland. And that’s key. Home is where the heart is as they say.
Interviewer: But how can a highland games in America make you feel at home if home is
thousands of miles away?
Colin: Because it’s not about the country, it’s the culture. Some of the biggest games are in
Canada and America and they’re a testimony to the importance of cultural identity. They were
born of the people who emigrated from Scotland to those countries, people who kept their
cultural roots alive and passed them down through the generations. The strength of their ties
stayed with them even as they were absorbed into another culture. Some people might suggest
that clinging to a bygone practice of cultural heritage is obsolete in today’s global society but I
think it is now in particular, with the interconnectedness of society, that the importance of
knowing who you are and where you come from becomes paramount. (13) There must be a
way of discerning oneself from the masses.
Interviewer: But wouldn’t that alienate a person from the culture they reside in?
Colin: On the contrary. In the global community, culture has to be accommodating. If it
excludes newcomers they will separate themselves from the indigenous population and that’s
when you get fractures in society, splinter groups of isolated people which can lead to conflict.
In order to avoid this, both the interloping and the native culture have to accept each other.
(14) There’s give and take as with any symbiotic relationship. But that doesn’t mean abandoning
your roots. It’s not just about how you see yourself but how the rest of the world views your
culture. If your culture translates well, you’re in luck. You can go anywhere just being yourself
and when you know yourself, the ability to accept others for who they are becomes much
simpler. You can appreciate their idiosyncrasies with greater ease.
Exercise 2. For questions 10-15, listen to a listen to a recording about someone giving advice
on how to ask their boss for a pay rise and choose the correct answer A, B or C which fits best
according to what you hear. Write your answers in the corresponding numbered boxes
provided.
10. The key factor when asking for a pay rise is
A. voicing your demands in a convincing way.
B. making it clear you feel undervalued.
C. proving you are an asset in the business.
D. comparing yourself to the rest of the staff.
11. If you have any failings, you should
A. check that no one knows about them.
B. put them right gradually so that it is not too obvious.
C. accentuate your strengths, such as punctuality.
D. make sure your boss likes you as a person.
12. When preparing what to say in your salary negotiation
A. put yourself in your superior's shoes.
B. do not forget that you really need that extra money.
C. make a list of all the points in your favor.
D. focus on what you can do for the company in the future.
13. What should you do if your boss raises objections to your pay rise?
A. pre-empt them by raising them yourself and giving a counter argument
B. make sure you can quote company rules to him or her
C. appreciate that your boss is only doing his or her job
D. accept any offer as it is better than nothing
14. During salary negotiations, it is important to
A. mention that the company is very successful.
B. ensure your boss is aware that you are taking these negotiations seriously.
C. arrange to see your boss early in the day when he or she is fresh.
D. try not to put your boss in an awkward position.
15. What should you do if you do not get a pay rise or as much as you wanted?
A. be prepared for a long drawn-out conflict
B. know that you might have to resign as a matter of principle
C. either have an alternative or ask for constructive criticism
D. either get a colleague to back you up or talk to your boss again soon

10. C 11. B 12. A 13. A 14. B 15. C

One of the most important situations in our professional life is when we feel we have to ask for a
pay rise. It can be awkward but if you aren't assertive and say what's on your mind, it may lead to
you feeling undervalued and having a negative attitude to your work and workplace.

A positive attitude, forward planning and perfect timing are the keys to getting a pay rise. You
may be asking for a number of reasons, ranging from a bigger workload or the increased cost of
living to the fact that you've found out that a colleague is getting more than you. But these
arguments will be secondary to your worth to the company.

Start by taking an objective look at your career. Are you good at your job? Are you punctual and
reliable? Do people know who you are, and for the right reasons? Are you worth more than
you're getting paid? If so, how much?

Are there any problems that you need to address? If so, make the changes subtly, over a
period of time. Bosses are not stupid, and sudden bouts of punctuality just prior to a pay
negotiation will seem like the worst type of creeping.

When planning your negotiation, don't base it on your gripes. Even if you think your future
in the company doesn't look too rosy, hear in mind the 'what's in it for me?' factor. You may
want extra money for all those things that are on your want list, for a holiday or a car, but your
boss will be more convinced by an argument based on your quality of work and dedication.

To strengthen your viewpoint, plan for potential objections. If your boss is going to resist,
what points is he or she likely to bring up? You could raise some first, along with
arguments in your defence. For example, the sort of line you could take is, 'I know most pay
rises are linked to set grades in this company, but I believe that my job has changed sufficiently
to make this an exceptional case.
Bartering can be embarrassing, but you will need to feel and sound confident. Remember that
negotiations are a normal part of business life. Never pluck a sum out of the air. Know exactly
what you will ask for and what you will settle for.

The timing of your communication can be crucial. Keep an eye on the finances and politics of
the company to avoid any periods of lay-offs or profit dips. If your boss can he moody, get an
appointment for his or her most mellow time of the day. Never approach the subject casually. An
on the hoof approach will make your boss twitchy.

There's always the chance that you won't get what you ask for. This is often the point at which
reasonable demands and negotiations can turn into conflict. Never issue ultimatums, and don't
say you'll resign if you don't mean it. Boost your confidence and your argument by having a
backup plan (that is, what you'll do if you don't get the pay rise you want). Plan for the future
by staying positive, asking when you could next apply and what can be done in the
meantime to help your case.
Exercise 3:

For questions 10-15, listen to a recording about rudeness and choose the correct answer A, B,
C or D which fits best according to what you hear. Write your answers in the corresponding
numbered boxes provided.
10. What is the main point of the anecdote Jeff tells?
A. That the young seem to lack social skills.
B. That it’s an all too familiar sight these days.
C. It’s the same thing as using your phone in the theatre.
D. They are no better than the Internet trolls.
11. What does Jeff say about the “death of deference”?
A. People no longer care what others think of them.
B. Teenagers didn’t want their behavior to be disapproved of.
C. For the most part, it had positive effects on society.
D. It made people disrespect those in authority.
12. What does Laura imply about teenage behavior?
A. There’s nothing that can be done about it.
B. That it’s not entirely their fault.
C. It’s worse nowadays than previously.
D. They are not taught how to behave.
13. How does Jeff respond to Laura’s opinion of teenagers?
A. He believes it is all their fault.
B. He thinks they are not properly brought up.
C. Those in charge of children should take responsibility.
D. He thinks she is just making excuses for bad behavior.
14. Both Jeff and Laura seem to agree that
A. A little bit of rudeness is a good thing.
B. Parents don’t teach their children how to converse.
C. The young don’t understand adult conversation.
D. The art of conversation has declined.
15. What does Laura say about modern technology and teenage behavior?
A. It has an effect on the development of social skills.
B. It is wholly responsible for today’s lack of social skills.
C. They become insulting and threatening.
D. They can hide from the world behind technology.

Your answers
10. A 11. C 12. B 13. C 14. D 15. A

Int: Today we’re going to discuss the topic of rudeness and if people today are ruder than they
were, say, thirty years ago. My guests are Laura Barnes and Jeff Swain, who edit the society
pages of their newspapers. So, Jeff, are we ruder these days?

M: Well, we’re all familiar with it, aren’t we? Loud conversation on mobile phones on buses and
trains and even theaters and cinemas; queue – barging, road rage – I could go on. And there are
those dreadful people, Internet trolls I think they are called, who send revolting messages, even
death threats, to people whose opinion they disagree with. Another thing, the other day I was in a
café, and there were three or four teenagers sitting at one of the table and they didn’t say a word
to each other the whole time they were there – just playing with their mobile phones. Do they
not know how to engage socially?

Int: So, what are the reasons for this new rudeness, and is it mainly confined to young people?

M: Young people are certainly much ruder than I would have dared to be when I was a teenager.
I say “Dared” because there were rules, if you like, and you risked disapproval if you broke
them. Perhaps that’s the problem; people don’t know what inappropriate behavior is anymore – r
they don’t care. Or the rules have changed. As to how it all happened, well, there are lots of
things to factor in. But I believe it began about the 1980s and 60s with the death of deference.
You know, automated respect for all elders and betters. We began to question the wisdom of
their decisions and their competence, and wonder how such fools got to the top of the pile
in the first place. All this was for the better, I think, as it broke down barriers in society
and was a move to greater equality. But I suppose once the floodgates are opened…
F: You may be right about that, but I don’t entirely agree with you about teenagers. I am not
saying they can’t be taught manners and social skills, but they have always been rude, ruder than
other age groups. Social ineptitude, lack of confidence, shyness, self-absorption – these are the
afflictions of the young. They just don’t have enough experience, awareness of others and
knowledge to cope. And certainly, these traits are not helped by the wired-up society we
live in.

M: Just a minute. Previous generation had no problems with manners and appropriate social
behavior. If teenagers have been rude throughout the ages, why are they worse now than ever
before? I was brought up with the basic good manners of ‘please’ and ‘thank you’, and thank-
you notes for gifts received, all that, so by the time I was a teenager, it was as natural as
breathing. I think parents and schools play a big part in all this.

F: Mm. I think a lot of parents would rather be their child’s friend rather than parent, which
sends out confusing messages and lessons in social behavior go by the wayside. Good manners
have to be taught, talked about and put into practice, and not a lot of that goes on these days.

Int: This being England, do you think there’s a class aspect to this?

F: You mean, are some classes ruder than others? I think it’s more of a pandemic, across all
classes and types. In fact, if you ask around in other countries, you’ll find that the whole world is
getting ruder.

M: It reminds me of a saying I heard about how a gentleman is never rude by accident. We’re not
allowed to give offence anymore. The English, or certain classes of the English, used to be very
good at being rude, or hurtful, by being polite. It’s an art form that still lives here and there,
but seems not to be passed on the younger generations anymore. So often they miss the point
– and irony is a no – go area.

F: But that’s often for the best of reasons. They think irony is exclusive, and they don’t want to
hurt people’s feelings. Which, despite all we’ve said, is good manners, if a little unexciting. But
then, political correctness has done away with a lot of things, not least wit, nuance and irony in
conversation, or the understanding of them.

Int: You mentioned social media earlier – how big a part does this play in the new
rudeness?

F: Quite big, I would say. Some people believe it’s entirely technology-driven. There are two
main strands to their argument: First, that it allows you to be anonymous, so you can insult or
threaten anybody at any time without anyone knowing who you are; secondly, that people are so
addicted to their electronic devices they are oblivious to the world around them. This, I think,
makes them miss the opportunity to learn how to interact with others of all types and ages.
Int: It’s all a bit worrying, isn’t it? But I’m afraid that’s all we’ve got time for today, so my
thanks to Laura and Jeff, and don’t forget to tune in on Friday

Exercise 4

For questions 10-15, listen to a radio discussion in which two writers, Tom Blake and Sally
Beauchamp, talk about their careers and choose the correct answer A, B, C or D which fits
best according to what you hear. Write your answers in the corresponding numbered boxes
provided.
10. What does Tom say about wanting to be a writer?
A. He had no idea that he would become a writer.
B. It was something he’d made up his mind to be.
C. It was always a possibility as a career.
D. He had no idea how one became a writer.
11. How does Tom feel about his contemporaries’ career choices at school?
A. He was baffled by their certainty.
B. He thought they lacked experience.
C. They hadn’t made their own decisions.
D. They were all following in their fathers’ footsteps.
12. What did it take Tom a long time to understand about American writers?
A. the importance of romance in their daily lives
B. the fact that their lives mirrored his own so closely
C. the influence that other people had on their writing
D. The practical considerations of time and money
13. How did Sally begin writing?
A. She was urged to write by her parents.
B. She became a journalist like her parents.
C. Early on she started to keep a journal.
D. Negative experiences created a need to express herself.
14. What does Sally say about journalism?
A. It provides her with story ideas.
B. It gives her less time for fiction.
C. It doesn’t bring in enough money.
D. It interferes with family life.
15. What aspect of the writing life do they both agree on?
A. It’s important for writers to have wide experience.
B. Writing fiction doesn’t provide a reliable income.
C. Writers tend to become self-absorbed and egocentric.
D. Journalism is the best way into becoming a writer.

Your answers
10. C 11. A 12. D 13. C 14. B 15. B

Int: Good morning. In the studio with me today are two writers who have both made a name for
themselves and won some of the top prizes, but became professional writers by quite different
routes. First you, Tom. What made you want to become a writer?

Tom: I could say that I always wanted to be a writer, but for a long time I had no idea of
what I wanted to do as a career. But it was always there in my mind. I was an avid reader, as
all writers are. But I didn’t get down to any serious writing until after I left school. I couldn’t
understand those children at school who knew from the age of, say, thirteen or fourteen,
exactly what career they wanted to follow, be it a lawyer, doctor, banker, soldier – and there
were several of those: I went to the kind of school where middling income and military types
sent their sons. Many were going into the family business. I thought, you’re too young to decide
now, you need to go out there and try different things before you decide.

Int: And did you?

Tom: Well I have done several kinds of job, many menial, as did some of my writing heroes, but
that was from economic necessity. You see, at first I had this romantic notion of what a writer’s
life was like. I had been reading the American writers of the early and middle twentieth century,
Hemingway, Fitzgerald, and so on, and they seemed to have a wonderful time. If you read their
biographies it seems that all they did was have parties and travel to exotic places and meet
interesting people. That’s the life for me, I thought. I was slow to ask questions like, when they
found the time to write, and where the money came from, but I got there in the end.

Int: What about you, Sally? Is it something you always wanted to do?
Sally: Yes, it was. I’m a bit like one of Tom’s school mates who followed in their father’s, both
parents in my case, footsteps. They were journalists, not novelists, but writing was part of our
lives. I just seem to have absorbed it, and started writing a diary as soon as I could put pen,
or rather crayon, to paper. Soon it felt like a bad day if I hadn’t written something. At the age
the world was a marvel, full of wonders and strange, to a child, goings – on. It still is.

Int: Now, you’ve both been journalists at one time or another. Was that helpful to you as a fiction
writer?

Sally: I wasn’t a journalist who went out hunting for stories. I was asked to review one or two
books after my first novel was published. The editor liked what I’d done and I got a weekly
column. It’s helpful in that it keeps me in the loop about what’s going on in contemporary
fiction, but the downside is it takes up time that should be spent writing my own stuff. And
time is at a premium, because I have two young children, and that’s pretty much a full-time job
in itself. On the plus side again, it keeps my critical faculties sharp, because you have to be a
good critic of your own work. And the money helps, of course: you can’t live off what your
fiction earns you unless you’re a best seller. You need a regular income, especially with a
family.

Int: Tom?

Tom: Yes, you need a steady flow of cash, even though I’ve managed to avoid the pram in
the hallway, a famous enemy of promise. Was journalism helpful? I think so, yes; and I still
take on assignments when offered them when I need the cash. I think it helps in that with news
stories you should keep out the personal, it’s not about you or how you feel, but about others and
the world at large. You get to know a lot more of the variety of life and other cultures first hand,
than you ever could if you just sat at your desk conjuring up imaginary worlds. Don’t get me
wrong, some great novels have been written that way, but I think writing is enriched by greater
experience. But I think too many writers these days are self-absorbed and their books shine a
light on little more than their personalities - which may or may not be interesting.

Sally: Well, we each do what we can cultivate the patch of ground we’ve been given to cultivate.
I went straight from school to do a creative writing course at university, and my first novel was
published shortly after that. So do I haven’t seen as much of the world as Tom or experienced the
dangers he has covering disaster areas…

Tom: I wasn’t including you in that category. You have the talent to go with the craftsmanship of
the true writer. I’m still dreaming of parties and exotic travel.

Int: I’m afraid I’m going to have to stop you there as we’ve run out of time. Thank you both for
joining us at this early hour.
Exercise 5:

For questions 10-15, listen to an interview in which two people, Dana Singleton and Joe
Fahey, talk about transport and choose the correct answer A, B, C or D which fits best
according to what you hear. Write your answers in the corresponding numbered boxes
provided.
10. What does Dana say about her reason for joining the Accident Prevention Board?
A. She did it after being involved in an accident herself.
B. She felt strongly about what they were doing.
C. She was close to someone who had been an accident victim.
D. She felt pressure from people she worked with.
11. The interviewer mistakenly believes that the number of serious road accidents
A. has recently overtaken the number of accidents on public transport.
B. is presently at an all-time high.
C. is a direct result of more aggressive drivers on the roads.
D. has risen in proportion to the number of cars on the roads.
12. According to the statistics Dana quotes, there were more serious accidents per year
A. before 1926.
B. between 1926 and 1970.
C. between 1970 and 1990.
D. after 1990.
13. Which of the following facts about public transport does Joe not state?
A. Public transport has a relatively poor safety record.
B. People sometimes feel intimidated when using public transport.
C. The high cost of public transport puts a number of people off.
D. Most complaints are about unreliable services.
14. According to Joe, public transport would improve if the government
A. improved the road network.
B. made it free to travel by bus.
C. re-allocated public spending.
D. banned certain cars from roads.
15. What best sums up Dana’s and Joe’s opinions on driverless cars?
A. Joe feels more strongly that they could help the situation.
B. Dana disagrees with Joe that they are the transport of the future.
C. They share the same views on introducing them gradually.
D. They agree that the poor infrastructure will cancel out any benefit.

Your answers
10. B 11. D 12. B 13. A 14. C 15. D

Int: If you’re listening on the bus or train, or in your car, this next item might be of special
interest to you. I’m joined by Dana Singleton from the Accident Prevention Board, or APB, and
Joe Fahey, who is a long-time campaigner for greater use of public transport. Dana, I’d like to
start with you. To become a member of a board such as the APB, you must have felt strongly
about road safety. What made you take a stand?

F: Well, unlike many people, I’m very lucky in that I didn't suffer a personal injury…. that's
often what motivates people to campaign and join movements, isn't it? But in a way, that’s
exactly the reason…. I was one of the lucky ones…. And I’ve since worked with many
colleagues who were victims. Every day, we read about accidents, and I just happened to ask
myself, ‘Who’s doing anything to stop them?’ So, when I found the answer, I joined them
because …..stand up for what you believe in, and all that.

Int: The APB deals with everything from industrial accidents to safety in the home, am I right?

F: Yes, and I‘ve worked in several fields but, for the last year, I‘ve been compiling information
for a report specifically on road safety. It’s going to make for horrific reading, with around
200,000 accidents a year leading to injury or death.

Int: I suppose with the increase in the number of cars on the roads; it’s inevitable that the
number of accidents goes up too. You must feel like you’re fighting a losing battle
sometimes.

F: It’s actually not what you think. Yes, the number of cars on the road is increasing, but fatal
accidents are going down, and have been doing so far many years. If you look at historical
statistics, the number of serious road accidents after we started keeping records in 1926 has
routinely been three or four times higher than today. It was only in 1970 that a major
decline started, and since 1990, most years have recorded the lowest since records began…. so
something is happening.

Int: I must say I find that quite shocking. Not at all what I expected.

F: But encouraging at that same time. It’s still too many, and much more can be done, but it’s
nice to be able to present the public with something positive.

Int: Yes, indeed. Joe Fahey is sitting patiently here. Joe, if you had your way, more people would
be using public transport, wouldn’t they?

M: That’s right. But we’d still need people like Dana because, well, accidents will happen. What
I believe is that we need to make public transport an appealing and viable choice. Safety is only
one reason for this…we also need to take into account our carbon footprint and the effect on the
environment…. but also our quality of life in terms of cutting down noise and reducing stress.
We think that public transport can do all those things very effectively. So what’s stopping more
people from using public transport?

M: There are several factors. Above all, people complain about the service, and studies have
shown that delays and canceled services are the most likely to send people away from
public transport and back into cars. High fares are also quoted by many – for some
journeys it is cheaper to buy and run a car than it is to use the train, let’s say. And there is
a perception among many that their personal safety is somehow compromised on public
transport, with a typical complaint being from late-night bus travelers who feel a sense of
threat or intimidation from fellow travelers….Although I have to say that with the rise of
road-rage incidents among car drivers, this is pretty much a misconception – the danger is
perceived to be greater, whereas, in reality, it isn't.

Int: I think we’ve all experienced at least one of these problems. What can be done?

M: I think it comes down to better management, and that needs intervention from central
government. Yes, the road network needs improvement, but with greater investment in bus
services and train networks, we could manage very well with the roads we have. All the
problems I mentioned can be solved; we just need to have the political will to do so.
Unfortunately, the government doesn’t see that their massive expenditure on road projects can be
avoided simply by boosting the existing rail network. That would take cars off the roads, freeing
them up for buses.

F: You see, I agree with Joe. People are going to continue their attachment to cars unless we can
offer them a more attractive alternative. We’ve made a lot of progress in some cities and the
work of some transport departments is admirable. We just need to do more. From the safety point
of view, public transport has an incredibly good record. We can make cars and roads
safer….and we are doing so…..but they’ll never be as safe as public transport. It goes back to
what you said about the number of cars on the road.

Int: What about driverless cars….would they help?

F: I’m afraid I don’t have a lot of faith in them. I mean, technology goes wrong sometimes too…
and the transition period seems problematic because we’re not going to switch overnight so there
will be a mix of transport types on the same roads. That worries me. But it’s the road situation
that bothers me more. Time and time again, we have seen new roads built to handle way
more capacity, and yet within a few years, the traffic grinds to a standstill. It’s not a
solution, is it?

M: No, Dana’s right. Driverless cars are still cars, and unless we can build a future – roof
road network, the same problems of safety and congestion will always arise.

Int: We’re going to take a short break now, but we’ll be back in a few minutes when I’ll be
asking…..
Exercise 6.

For questions 10-15, listen to a radio discussion about children who invent imaginary friends
and choose the correct answer A, B, C or D which fits best according to what you hear. Write
your answers in the corresponding numbered boxes provided.
10. In the incident that Liz describes
A. her daughter asked her to stop the car.
B. she had to interrupt the journey twice.
C. she got angry with her daughter.
D. her daughter wanted to get out of the car.
11. What does the presenter say about the latest research into imaginary friends?
A. It contradicts other research on the subject.
B. It shows that the number of children who have them is increasing.
C. It indicates that negative attitudes towards them are wrong.
D. It focuses on the effect they have on parents.
12. How did Liz feel when her daughter had an imaginary friend?
A. always confident that it was only a temporary situation
B. occasionally worried about the friend's importance to her daughter
C. slightly confused as to how she should respond sometimes
D. highly impressed by her daughter's inventiveness
13. Karen says that one reason why children have imaginary friends is that
A. they are having serious problems with their real friends.
B. they can tell imaginary friends what to do.
C. they want something that they cannot be given.
D. they want something that other children haven't got.
14. Karen says that the teenager who had invented a superhero is an example of
A. a very untypical teenager.
B. a problem that imaginary friends can cause.
C. something she had not expected to discover.
D. how children change as they get older.
15. According to Karen, how should parents react to imaginary friends?
A. They should pretend that they like the imaginary friend.
B. They shouldn't get involved in the child's relationship with the friend.
C. They should take action if the situation becomes annoying.
D. They shouldn't discuss the imaginary friend with their child.

Your answers
10. B 11. C 12. A 13. C 14. C 15. B

Presenter: Today we're talking about children and their tendency to have imaginary friends. Liz
McManus has a daughter called Caitlin, who's eight now. When she was three, she had an
imaginary friend called Tytner. Liz, tell us about Caitlin and Tytner.

Mother: Well, give you an example. One day I was driving Caitlin and Greg, her baby brother,
home when she solemnly informed me that Tytner was hitting the baby. So I said: 'You tell
Tytner that if he does that again, he'll be walking home.' Fifteen seconds later came the
inevitable news: 'He's just done it again, Mummy.' So I found myself in the embarrassing
position of having to pull over, open the back door and say to this imaginary little boy.
`Tytner, out, now!' And of course, as we drove off, Caitlin started crying because her friend was
standing on the pavement all alone. I had to turn back and go through the rigmarole of
pulling over and opening the door to pick him up again.

Presenter: Wow, that's some story! But in fact, Caitlin is no different from many children
and her invented, make-believe friend is far from unusual. As many as 65% of children
have had an imaginary friend at some point in their lives. The latest research suggests that
invisible friends, far from being a cause for concern, should be welcomed by parents
because they can help children to be more creative, confident and articulate, and have
more advanced communication skills. It is thought that these findings will help reverse
misconceptions about children with imaginary friends and that they will come to be seen as
having an advantage, rather than a problem that needs to be worried about. Did it worry you,
Liz?

Mother: I know it does lots of parents but I never fretted about it, I think I was just amused. I'd
be reading to her and I'd say, 'Is Tytner around?' and she'd say, 'Yes, he's just sitting at the end of
the bed.' He became the centre of her life. She'd have tea parties with him, and he'd go to bed
with her. She was shy and this was her answer. I knew she would grow out of it.

Presenter: Now Liz is one of 15 people taking part in a study of imaginary friends at the
Institute of Education in London, run by Karen Majors, an education psychologist and lecturer at
the institute. Karen, should parents worry about it?

Expert: Well, parents sometimes think, 'Is this healthy and how long should it go on for?' But it
is a normal phenomenon for normal children. And it's very healthy.

Presenter: Why do children invent imaginary friends?

Expert: I think that children create pretend friends for many reasons: as safe, trustworthy best
friends at a time when they are just starting to make real friends; as someone to confide in; and
as someone to play with. Sometimes it is about wish fulfilment; children who cannot have a
pet, for example, will invent one. I interviewed one little girl, aged six, who had a pony called
Minty for several years. It went to school with her and the teachers knew all about it. It was a
really strong relationship.

Presenter: Presumably, when they get older, children no longer have these imaginary friends.
Karen?

Expert: Well, my most surprising finding is that children don't always stop having these
made-up playmates when they start school. The imaginary friends often stay with them
through their teenage years, providing comfort and escape - although in secret. One
teenager I talked to had invented a superhero to help him through tricky patches. When
things hadn't gone well at school, he would come home and play with the superhero, for
whom everything always went well.

Presenter: How should parents treat these invisible people, Karen?

Expert: Well, sometimes of course, parents get irritated by them - for instance, if a child insists
on having the playmate at the dinner table with an imaginary setting and glass. Actually I myself
had a friend called Tiger when I was young, who would sit beside me at mealtimes. But I don't
think parents should tell children off for this kind of thing, or tell them that their friends
are not real. Perhaps the best way is Liz's down-to-earth approach.

Presenter: How did you handle it, Liz?

Mother: Well, I patiently acknowledged Caitlin's playmate but I tried not to get involved. I
never used to have to get out of the friend's way or anything. Other than that one incident in the
car, Caitlin's imaginary friend didn't impinge on my life.
Expert: Yes, I agree that parents should recognize imaginary friends, but they shouldn't try to
overly influence the friendship. Parents who interfere too much risk driving their children's
playmates away. If they try to direct the friends, they could spoil the fantasy altogether.

Presenter: Fascinating subject, thanks for coming in to talk about it, Liz and Karen.

Exercise 7
For questions 10-15, listen to a radio discussion about writing a novel and choose the correct
answer A, B, C or D which fits best according to what you hear. Write your answers in the
corresponding numbered boxes provided.
10. What does Louise say about Ernest Hemingway's advice to writers?
A. It is useful to a certain extent.
B. It applies only to inexperienced novelists.
C. It wasn't intended to be taken seriously.
D. It might confuse some inexperienced novelists.
11. Louise says that you need to get feedback when you
A. have not been able to write anything for some time.
B. are having difficulty organizing your ideas.
C. are having contrasting feelings about what you have written.
D. have finished the book but not shown it to anyone.
12. Louise says that you should get feedback from another writer because
A. it is easy to ignore criticism from people who are not writers.
B. another writer may be kinder to you than friends and relatives.
C. it is hard to find other people who will make an effort to help you.
D. another writer will understand what your intentions are.
13. What does Louise regard as useful feedback?
A. a combination of general observations and detailed comments
B. both identification of problems and suggested solutions
C. comments focusing more on style than on content
D. as many points about strengths as weaknesses
14. What does Louise say about the people she gets feedback from?
A. Some of them are more successful than her.
B. She doesn't only discuss writing with them.
C. She also gives them feedback on their work.
D. It isn't always easy for her to get together with them.
15. One reaction to feedback that Louise mentions is that
A. it is justified but would require too much effort to act on.
B. it focuses on unimportant details rather than key issues.
C. it has been influenced by reading other people's novels.
D. it is not suggesting that major changes to the novel are required.

Your answers
10. A 11. C 12. D 13. B 14. C 15. A

Interviewer: OK, in our weekly spot about how to write a novel, I'm talking to novelist Louise
Doughty. Louise, this week we're talking about getting comments and feedback on your work
from other people. 'If there is anything in your own work you think particularly fine,' said Ernest
Hemingway, 'strike it out.' Is that good advice?

Novelist: Well, few would-be novelists aspire to be as plain and brutal as Hemingway, I suspect,
but his dictum is still worth tucking into a corner of your brain — not to be followed
slavishly, but as an antidote to that great curse of the inexperienced novelist: over-writing.

Interviewer: How do you know if you are over-writing?

Novelist: Well, an excess of adverbs and adjectives is a clue. Repetition under the guise of
emphasis is another, and extended metaphors should be rationed tightly.

Interviewer: Now, there comes a point in the writing of every novel when you just don't know
anymore. You've been immersed in it for weeks, possibly years. You've lost sight of the
original impetus behind the book and are plagued with self-doubt — yet at the same time
you know there is something there and are not ready to give up on it.

Novelist: Yes, and this is the point at which you should be getting feedback.

Interviewer: From whom?

Novelist: As a rule, I'd advise against getting it from your nearest and dearest. You will be
wounded by their criticism and suspicious of their praise. Instead, join a writing community of
some sort. You need comments from another writer. Those who have been logging onto my
website will know that such a community has formed there. Creative writing courses and book
groups are also good places to find like-minded souls. Through such contacts, you can find
someone who understands what you are trying to do — which is not the same as someone
who is uncritical of the way you do it. There is a time and a place for emotional support, but
that is not what we are talking about here.

Interviewer: What kind of comments are useful in your view?

Novelist: Well, you need someone who is prepared to say, 'I really like the opening
paragraph but I thought it went a bit wrong after that because ...' and, crucially, is
prepared to be specific. 'I just didn't like him' is not a helpful comment on a character. 'I lost
sympathy with him in the scene where he tells his brother the truth because I thought he was too
brutal. Maybe you should re-write it making his motivation clearer and his language softer.'
That is useful advice: you can choose whether to take it or discard it. Similarly, at the level
of prose style, some well-meaning person might say 'It's a bit boring', but a helpful critic would
say 'You have three paragraphs of description here before you tell us who is talking; maybe you
should consider starting the conversation first and weaving all the description in, instead of
having it all in one chunk.'

Interviewer: How do you personally get feedback on your work?

Novelist: The most fruitful relationships I have with other writers are with the novelists
with whom I swap work - usually the person whose novel is under discussion pays for
dinner. If you can find other novelists who are working at a similar level to yourself, with
similar interests, and who are frank and unafraid of frankness from you, then keep them close at
hand.

Interviewer: Now what about reactions to feedback? We've talked about getting it, but what
about when you've got it?

Novelist: Well, even with a trusted ally, there comes a point when you have to stick to your guns
and say, 'This person whom I really respect doesn't like this bit but I do and it's staying in.' And
of course, feedback can be annoying. For a start, a lot of the criticism you receive will be stuff
which you knew already in your heart of hearts - you were just hoping nobody else would notice.
And sometimes, you will bristle at a certain comment on your work, not because it is
untrue but because the consequences of righting that particular defect are too daunting to
contemplate at that particular time - accurate criticism is the most painful of all. On other
occasions, you will have a gut feeling that the person reading your work just doesn't `get' it, or
wants you to write a different novel entirely. Frequently, the only way to work out if criticism is
useful is to nod sagely, then file it in a drawer. When the wounds have healed over and your ego
is not under immediate threat, then you will be able to assess its true worth.

Interviewer: Right. Now let's move on to talk about ...


Exercise 8.

For questions 10-15, listen to a radio interview with a chef about the process of eating and
choose the correct answer A, B, C or D which fits best according to what you hear. Write your
answers in the corresponding numbered boxes provided.
10. Heston mentions eating fish from a paper plate with a plastic knife and fork
A. because it is something listeners may have done.
B. because doing so made him think about the process of eating.
C. as an example of an unpleasant eating experience.
D. as an example of what influences the eating experience.
11. What does Heston say about taste?
A. Fat should be considered a taste.
B. Taste and flavor are separate from each other.
C. The sense of smell is involved in it.
D. The number of taste buds gradually decreases.
12. The experiment involving salt and other food shows that
A. it is possible to taste something that you can't smell.
B. the sense of smell is not as powerful as other senses.
C. food can taste better when you can't smell it.
D. the flavor of food can change as you eat it.
13. The story about the trainee waiters illustrates that
A. certain colors are more appealing than others.
B. something can seem to taste good because of its appearance.
C. one sense can strongly influence another.
D. some people can perceive taste better than others.
14. What does Heston say about bitterness?
A. It can give a false impression that something is harmful.
B. It can become the main reason why people like something.
C. Reactions to it can change over time.
D. Its function is widely misunderstood.
15. The problem with the dish Heston describes was caused by
A. its appearance.
B. the taste of it.
C. its combination of flavors.
D. the fact that people ate it repeatedly.

Your answers
10. D 11. B 12. A 13. C 14. C 15. A

Interviewer: I'm talking to chef Heston Blumenthal. Now, Heston, most of us think that the
business of eating is pretty simple, don't we? We eat things and, we like the taste of them or we
don't, but you reckon it's more complicated than that, don't you?

Chef: Yes, eating is a process that involves all the senses. Any notion that food is simply about
taste is misguided. Try eating a beautifully cooked piece of fish off a paper plate with a
plastic knife and fork - it is not the same.

Interviewer: So how does taste operate then?

Chef: The sense of taste can be broken down into five basic categories, all of which happen in
the mouth and nowhere else. These categories are salt, sweet, sour, bitter and umami - the most
recently identified taste. There is a current theory that fat is a taste but this has yet to be
proved. We have up to 10,000 taste buds on the tongue and in the mouth. These regenerate,
so the receptors we use today will not be the same as were used a couple of days ago. When
we eat, taste buds on our tongue pick up taste but not flavor. The molecules in food that
provide flavor pass up into the olfactory bulb situated between the eyes at the front of the brain.
It contains hundreds of receptors that register molecules contained in everything that we eat and
smell. This is where the flavor of the food is registered.

Interviewer: OK, so our sense of smell is connected with flavor rather than taste? Is that what
you're saying?

Chef: That's right. Smell and taste are registered in different parts of the head. There is a
simple but effective and enjoyable way of demonstrating this. Have ready some table salt and
biscuits, fruits or anything easy to eat. Squeeze your nostrils tightly enough to prevent breathing
through them, but not so tight as to hurt. Take a good bite of biscuit or fruit and start chomping,
making sure the nostrils remain clenched. You'll notice that it is impossible to perceive the flavor
or smell of the food being eaten. Now, with nostrils still squeezed and food still in the mouth,
lick some salt. Although it was impossible to detect the flavor of the food that was being
eaten with clenched nostrils, the taste of the salt is unhindered. Finally, let go of your
nostrils and notice the flavor of the food come rushing into your headspace.

Interviewer: I'll definitely try that sometime. So what you're saying is that all the senses can
affect your experience when you eat?

Chef: Yes, the brain has to process the information given to it by other senses while we are
eating, sometimes with surprising results. Here's another example. A few years ago at a
sommelier school in France, trainee wine waiters were put through a routine wine tasting.
Unknown to them, a white wine that they had just tasted had been dyed red with a non-
flavored food dye, then brought back out to taste and evaluate. Something very interesting
happened. They all made notes on the assumption that the wine was what it looked like -
red. In this case, the eyes totally influenced taste perception.

Interviewer: OK, so it's not just about taste, all the senses are coming into play in different
ways.

Chef: Yes, and as well as allowing us to enjoy food, the senses act as warning systems, taste
being the last of the sensory barriers, and bitterness the last of the taste barriers. A natural
aversion to bitterness can prevent us from eating foods that could be harmful, although it
appears that we have the ability to modify such basic likes and dislikes. For example, we
generally grow to like bitter foods such as tea, coffee and beer as we grow older.

Interviewer: What got you interested in this business of the role played by various senses in the
experience of eating? Was it just professional curiosity?

Chef: Well, I began thinking about this whole subject a couple of years ago when I noticed that
more and more customers at my restaurant were commenting on the fact that the red cabbage
with grain-mustard ice cream served as an appetizer just got better each time they ate it. This was
the only dish on the menu whose recipe had not changed over the past year. It seemed that the
barrier being presented with this dish was the vivid purple color of the cabbage, a color not
normally associated with food. To some diners, the initial difficulty of accepting this color
interfered with the appreciation of the dish, but as they got used to it, they lost their
inhibition and simply enjoyed its flavor.

Interviewer: I see. Now, of course, the sense of smell must come ...
Exercise 9. For questions 10-15, listen to an interview with an IT consultant called Paul about
how he started his own business and choose the correct answer A, B, C or D which fits best
according to what you hear. Write your answers in the corresponding numbered boxes
provided.
10. That does Paul say about the first years of his working life?
A. He got a teaching post in the university where he had studied.
B. His first job began to feel rather monotonous as time went by.
C. The kind of tasks he was instructed to do gradually changed.
D. The work became less enjoyable when some colleagues moved away.
11. What does Paul say made him decide to start his own consultancy business?
A. He was persuaded to do so by some friends.
B. He read a book which inspired him to take action.
C. He thought he could take advantage of a growing trend.
D. He realized he had the skills to make a success of his own company.
12. What does Paul suggest is his favorite type of client?
A. people he knew when he was a student
B. people recommended by agents
C. people he has met at conferences
D. people he has worked with previously
13. What does Paul find the most difficult aspect of running his business?
A. gaining a quick understanding of an unfamiliar situation
B. setting fees that match the effort involved
C. making a good impression on a new team of people
D. getting reliable advice from financial experts
14. What does Paul say about the way he uses a business mentor?
A. He uses his mentor differently from the way others use theirs.
B. He finds his mentor often comes up with fresh ideas for his business.
C. He gets information about the latest business theories from his mentor.
D. He likes to gets his mentor's reactions to plans that he is considering.
15. Paul explains that in the coming year he is going to
A. employ some new consultants.
B. have more variety in his work.
C. move to a more convenient office.
D. learn some additional skills.

10. B 11. C 12. D 13. A 14. D 15. B

Interviewer: So, Paul, you run your own IT consultancy business now. How did your career
start?

Paul: Well, after graduating in computer science, I got a job in the technical support team of the
university where I'd studied. I really enjoyed both the company of my colleagues and the
technically challenging and interesting tasks I had to deal with, but after a few years, I began
to see the same issues coming up again and again. Several members of the team left, as it was
all getting much less stimulating.

Interviewer: So what made you actually decide to set up your own consultancy?

Paul: Well, it seemed to me that there was going to be a trend among many organizations
towards taking on short - term external consultants for IT rather than employing a
permanent in-house team. And I thought that would be an interesting way for me to go. I'm
friends with a couple of guys who have a small start-up and they'd been operating for a few
years. I wasn't sure I could do it, but they helped me get started. They gave me much more
sensible advice than I've ever seen in any book or article for would-be entrepreneurs.

Interviewer: How do you get new clients?

Paul: Well, work is certainly erratic. For months, I might only have recurring clients, and then
suddenly could get four new ones in the same week. There's no one reliable source of work, so I
diversify. I get a fair amount of work from recruitment agents, but some of the jobs they provide
are not terribly challenging. And I get some work from people I was at uni with. I suppose
about half of my work is repeat business from people I've helped before, and I like that.
You start out from a position of knowing and trusting each other. Also, I go to conferences
to meet people and to promote myself and my business. I've been told that there's at least a two-
to three-year lag between presenting at conferences and getting work from them, so I'm not yet
sure how much business is going to come to me that way.

Interviewer: What is the hardest part of running your business?

Paul: At first, I found it extremely difficult to know what to charge clients. I ended up drastically
undercharging on a number of occasions before I realized I needed to get some financial advice
from an expert. That helped me enormously. It's often said that the first week in business is the
hardest. In my case, as I'm always moving on to provide support in a different company
nearly every week is in the sense the first week. I often have just that week to make a
difference to that team, and that's a challenge but, fortunately, I like challenges!

Interviewer: Would you say you have a business mentor?

Paul: Sure. My business mentor is an old friend of mine. He recently finished a Master's in
Business Administration and we talk every week about how I might develop my company. I
wouldn't say I have a clear idea about how other people use a business mentor - and it's quite
possible they do things very differently - but I have found our conversations very useful. It's
great to have someone experienced to sound ideas off. I have great faith in his instincts.

Interviewer: What are your plans for this year?

Paul: Well, we've just moved to a great new office in the center of town, so we'll enjoy being
based there. I'm considering taking on a couple of new staff to keep things ticking over there
while I deal with clients. I've got some very different new projects lined up over the next 12
months, so I am looking forward to getting my teeth into those. I'm looking forward to
having the chance to use some technical skills that I haven’t had to call on for a while, in
fact, so that will be good.

Interviewer: Well, I wish you all the very best with that.
Exercise 10. For questions 10-15, listen to a radio interview with a travel writer called Marina
Vardy and choose the correct answer A, B, C or D which fits best according to what you hear.
Write your answers in the corresponding numbered boxes provided.
10. What made Marina start traveling?
A. a sudden desire to overcome her fear of the sea
B. a difficult problem in the life she had at the time
C. an unexpected opportunity to escape a boring routine
D. a friend's wish for a traveling companion
11. What is Marina’s main reason for being happy about her decision to go on that first journey?
A. It led to a successful career as an author.
B. It gave her a more positive outlook on life.
C. It offered her some exciting adventures.
D. It taught her to cope with difficulties.
12. How did Marina first get into writing?
A. She wanted to describe the different exotic places that she visited.
B. She found it satisfying to write her private feelings in a teenage diary.
C. She enjoyed the reaction of others to something she wrote as a young girl.
D. She was keen to be like her father, who was an enthusiastic writer.
13. What does Marina say is her greatest challenge?
A. feeling uncertain about the quality of her work
B. coping with some lack of support from her family
C. having to face danger for the sake of a story
D. finding things to write about that will interest her readers
14. Marina says that aspiring travel writers must ensure that they
A. work hard to make their writing style as good as they can.
B. try to make their readers experience strong emotions.
C. offer much more than an account of their own adventures.
D. keep their own grandmother in mind as they write.
15. What does Marina say she finds particularly rewarding about being a travel writer?
A. It has developed her powers of observation.
B. It offers her the chance to take revenge of unkind people.
C. It provides her with an adventurous lifestyle.
D. It gives her a satisfying psychological detachment.

Your answers
10. C 11. B 12. C 13. A 14. C 15. D

Interviewer: With me in the studio today I have travel writer Marina Vardy. Marina, how did
you get started traveling?

Marina: People often think that those who travel are running away from something. They're
right. Aged 24, I decided to run away from a dull job, and life that had grown stagnant. I set
out to find something more, though I had no idea what 'something more' looked like.
Thanks to a chance encounter, I met a woman in a cafe who had a humble sailboat and a
dream of exploring the world. Against my better judgment, I decided on the spur of the
moment to accept her spontaneous suggestion to join her, despite the fact that I've always had a
morbid fear of deep water.

Interviewer: And you didn't regret it?

Marina: At times I did, especially at the beginning, but, rather to my own amazement, I got used
to putting up with all sorts of physically difficult situations. That voyage changed the course of
my life. It turned me into a travel writer and an adventurer, but above all, it made me an
optimist. I couldn't have asked for more.

Interviewer: Mm. What made you get started in writing?

Marina: Oh, that happened a good while before I set off traveling and I was inspired to write by
what I saw. At about 12, I wrote a little verse that included the line: 'People say I take after
my dad. He went bald and grey in his twenties. Great!' My mother stuck it on the fridge,
people laughed, and so began my passion for entertaining people with words. I've never
been one for keeping my thoughts private, I'm afraid.

Interviewer: As a travel writer, what's your biggest challenge?


Marina: My greatest passion in life is a midday sleep in a hammock, but people don't want to
read about that! So in order to get a good story, I end up doing things that most people avoid due
to their ridiculously high risk. But, to be honest, the worst thing's the fact that I constantly
battle self-doubt over whether or not my work is any good. You really don't know till you get
it done and others read it and pass their verdict. When you're penning your life story, it can seem
like you're being very self-indulgent. 'She's supposedly writing a masterpiece, but she hasn't
bothered to brush her hair in weeks!' my family says in hushed whispers behind my back. Or
they do in my imagination, at least.

Interviewer: What advice would you give to someone who is considering going into travel
writing?

Marina: Always assume that your travel experiences are only going to be interesting to your
grandma. Attention spans are slim, and there has to be more than a description of what you saw.
Nowadays, with all the other calls on people's attention, you have to work harder than ever to
keep your readers engaged. How can you give a universal appeal to your story? How can you
make it a page-turner? Can you make your readers laugh? Cry? Think of your travels like the
backdrop to a greater story that grips the reader, not the story itself. After all, you're trying
to grab the attention of an internet-obsessed generation, which means you've got a big job on
your hands.

Interviewer: Mm. What's the biggest reward of life as a travel writer?

Marina: Writing teaches you to experience life as an observer. No matter what situation
you're in, there's always that tiny author sitting in the back of your head, narrating the
events around you. Bad encounters make good stories, and they're cathartic to write about. Did
the same horrible person treat you badly? Not to worry - put him in your next story. And I'm nice
about kind people too, of course. Channeling your experiences into art is deeply enriching, I find.

Interviewer: Thank you very much, Marina.


Exercise 11.
You will hear a radio interview in which a choreographer, Alice Reynolds, discusses
a dance programme. For questions 1-5, choose the answer (A, B, C or D) which fits
best according to what you hear.

1 How is the programme designed to help youngsters?


A by getting them to talk about their feelings
B by encouraging them to loosen up
C by enabling them to convey their thoughts.
D by giving them a way to entertain themselves
 
2 When talking about the nature of communication, Alice reveals that
A teenagers are quick to react to a number of emotions.
B people who learn to show how they feel can articulate better.
C shy youngsters find the programme more useful than others.
D young people have a lot of pent up negative emotions.
 
3 What aspect of the programme encourages teenagers to face their troubles?
A the social side of dance
B the freedom of the movement
C the obligation to interact
D the release of feelings
 
4 Alice contrasts professional and amateur dancers in order to
A highlight the usefulness of the programme.
B emphasise the use of emotions in dance.
C illustrate the difference between teaching styles.
D explain the ability to recognise feelings.
 
5 What point does Alice make about the study into a person’s personality?
A It found that certain types of people dance better than others.
B Personality has a bearing on people’s willingness to participate.
C Who people are can be recognised through their movements.
D It revealed that most people try to hide their true nature.

1C
2B
3D
4A
5C

Interviewer: Good morning listeners and welcome to Window on the World. Today


we’re continuing our segment on communication. Yesterday, we took a look at music and
the way in which we express ourselves through song. Now, we’re going to delve into
something that’s very closely related and that is the medium of dance. With us in the
studio is Alice Reynolds, a choreographer who has worked with the London Ballet and
the Royal Ballet School and is running a project with the local youth centre to encourage
teenagers to express themselves through dance. Welcome to the show.
Alice: It’s a pleasure to be here.
Interviewer: Why don’t you tell our listeners something about what it is you do?
Alice: Well, as you said I’m a choreographer. I’ve worked with some amazing dancers
through the years and it’s always struck me how well emotions can be portrayed through
movement. Even when simply loosening up at a rehearsal, the dancers showed how they
were feeling. Dance has been around for centuries. Human cultures have used it
throughout that time as a means of communication in ceremonial rites and celebrations as
well as entertainment. So we decided to put this unique ability to communicate to
good use in helping young people express themselves.
Interviewer: In what way?
Alice: It can be difficult for people, especially young people and teenagers, to talk about
how they feel. There can be a number of reasons for that, they may be shy or just simply
not be able to find the appropriate words, but for those that can’t find their voice, dance
offers a way of getting those feelings out. Whether they are feelings of anger, love,
remorse, it doesn’t matter. Dance allows you to express these emotions in a positive way
instead of keeping them locked away. And we’ve found that, once you can express
yourself through movement, it’s easier to open up verbally.
Interviewer: Mmm, teenagers are not exactly known for their communication skills.
Alice: No, they can be quite socially awkward. But that’s another side to the dance
programme we run, it forces them to interact and by the end of it they usually come
out of their shells. Also, because we don’t suppress emotions, we let them loose,
they’re forced to deal with whatever is ailing them, any anxieties they may have. It
can be quite cathartic. It’s almost a form of therapy but without the therapist. It offers a
level of freedom unlike anything else. Afterwards the children seem to be more
emotionally well-balanced.
Interviewer: So, do you think it would work for anyone?
Alice: Of course. When I worked with professionals, I noted that, through their
training, no matter what type, they had learnt to express their emotions but not
betray their personalities. With the teenagers it was the opposite. We can already
see their personalities but their emotions are suppressed. It was this insight that
showed me how effective this venture could be. Our work with them is designed to
bring those emotions out for all to see. And they’re recognisable emotions. Dancing
conveys subtle messages about what people are thinking and feeling that other people can
read. After all, communication is a two-way street. This ability to read these messages is
an inherent human trait that has its roots in our evolutionary history. The power of
movement to tell a story is quite exceptional.
Interviewer: I suppose there are certain types of people, personalities, that take to dance.
Find it easy to get up and let loose?
Alice: Actually, it’s not whether someone will dance or how well they dance that’s
determined by their personality but rather how they dance. There was a study
conducted to see if you could tell a person’s personality from the way they moved
and the findings were emphatically yes. It seems we can’t hide who we are when we
dance. Be we an extrovert, neurotic, conscientious, open-minded or agreeable, it all
comes out on the dance floor. Having said that, certain personalities are better able to
convey emotion without assistance.
Interviewer: I wonder what my dancing would show. Probably just that I should really
have some classes. Well, thank you for taking the time to come in and talk to us.
Alice: Thank you for asking me.
Interviewer: If that has got any of you wanting to put on your dance shoes, ...
Exercise 12:

You will hear an interview with someone who consulted a 'life coach' to improve her life.
For questions 16-20 choose the answer (A, B, C or D) which fits best according to what you
hear.

16 Brigid says that she consulted a life coach because


A she had read a great deal about them.
B both her work and home life were getting worse.
C other efforts to improve her life had failed.
D the changes she wanted to make were only small ones.

17 What did Brigid's coach tell her about money?


A It would be very easy for Brigid to get a lot of it.
B Brigid's attitude towards it was uncharacteristic of her.
C Brigid placed too much emphasis on it in her life.
D Few people have the right attitude towards it.

18 What does Brigid say about her reaction to her coach's advice on money?
A She felt silly repeating the words her coach gave her.
B She tried to hide the fact that she found it ridiculous.
C She felt a lot better as a result of following it.
D She found it difficult to understand at first.

19 What does Brigid say happened during the other sessions?


A She was told that most people's problems had the same cause.
B Her powers of concentration improved.
C Some things she was told to do proved harder than others.
D She began to wonder why her problems had arisen in the first place.
20 What has Brigid concluded ?
A The benefits of coaching do not compensate for the effort required.
B She was too unselfish before she had coaching.
C She came to expect too much of her coach.
D It is best to limit the number of coaching sessions you have.

16. D

17. A

18. C

19. A

20. B

Interviewer: My next guest is Brigid McConville, a journalist who decided to get herself a 'life
coach'. Brigid, what made you do it and what is a 'life coach'?

Brigid: Well, all was not entirely well with my life. Nothing drastic. I just felt 'stuck' and in
need of change, both on the work front - too much to do, too little time - and at home -
ditto, I wasn't miserable enough for therapy or counselling. I simply wanted to get a little
more from life. Until recently, the options for someone in my situation would have been
extremely limited. Now, however, legions of 'life coaches' are out there, ready and waiting to
come to the aid of the frustrated and down-at-heart. For about £40 a session, your personal coach
will telephone you once a week, and spend half an hour talking to you in an effort to help you
sort your life out

Interviewer: But isn't this just another self-improvement fad? Like all the self-help books and
tapes?

Brigid: Well, I was a bit dubious myself, but I decided to try it. I booked a course with Fiona
Harrold, a leading British coach. She identified my anxieties almost immediately. Within half an
hour of our first conversation, I found myself agreeing that the first thing I had to tackle was my
deeply ambivalent relationship with money. Yes, of course it was rooted in childhood - but what
could we actually do about it? Fiona is a passionate advocate of self-belief and, with her
characteristic verve, she told me I had to carve out a whole new way of thinking about myself. I
must see myself as 'a magnet for money', she said. And she told me: 'Consider yourself
someone to whom cash flows effortlessly. Why shouldn't you have an easy life, an
abundance of pleasure, leisure and luxury - and all without feeling any guilt?'

Interviewer: How did you react to that?

Brigid: Well, it seemed such a preposterous idea that I laughed out loud down the telephone. But,
undeterred by my scepticism, Fiona told me to suspend my disbelief, and gave me a clutch of
positive affirmations with which to brainwash myself into readiness for riches. She told me to
repeat the following words whenever possible: 'I, Brigid, am now ready to have the ideal life that
I deserve.' Doing this, I found, cheered me up no end.

Interviewer: What else did she tell you?

Brigid: Well, subsequent sessions were more practical. First came the mandatory de-cluttering -
she told me to throw out as much unnecessary jumble and rubbish as possible, clearing space for
all the goodies to come - once the money started to roll in. Then we began trying to cure my
personal finance phobia; I dutifully did my sums, and started saving something, however small,
every month. My work also came under close scrutiny, too, as I made up my mind to concentrate
on jobs that really interested me. Exactly which issues you tackle during coaching is up to you.
According to Fiona, most people want to get organised at home and at work, make the most of
their abilities and sort out money problems. She reckons that building up confidence is vital. She
really does believe that people are capable of doing anything they want to do, and that all
that stands in their way is childhood conditioning.

Interviewer: So what did you get out of it all? And would you recommend it?

Brigid: Well, coaching makes you get on and do all those things you've put off for so long,
because there is the deadline of the next session. If you don't act in time, your coach probably
won't want to speak to you. So coaching is hardly a soft option. But for me, it has provided a
great boost. There have been no instant miracles, but things are looking up at work and
financially, money and I are definitely on better terms. I still have my doubts about the 'me
first' approach - but, then again, it is a healthy counterbalance to the `me last' way of
thinking I am used to.

Interviewer: Thanks, Brigid. Now, if you want to find out more about life coaches ...
Exercise 13.

You will hear an interview with someone whose family spent a year living without
television. For questions 16-20 choose the answer (A, B, Cor D) which fits best
according to what you hear, in the exam you will hear the piece twice.

16 One reason why the family decided not to have a television was that
A the reception from the communal aerial was often poor.
B they did not think the satellite technician would do the job properly.
C linking up with the communal aerial was complicated.
D they preferred to enjoy the beauty of their new surroundings.

17 One thing that Miranda enjoyed about not having a television was
A telling other people about what they did instead.
B returning to hobbies they had previously given up.
C observing the reaction of others when they found out.
D feeling more energetic during the evening.

18 Miranda says that one disadvantage of not having a television was


A the fact that they could not follow their favourite series.
B a constant desire to be more up-to-date with the news.
C being unable to discuss topics they had previously discussed.
D feeling out of touch with what other people talked about.

19 What does Miranda say about getting connected again?


A She felt it would be of some benefit to the whole family.
B She agreed because her attitude towards television had changed.
C She initially disagreed with her husband about doing so.
D She felt that they were doing so because they were lazy people.
20 Miranda says that since they got a television again, her children
A are more able to distinguish good programmes from rubbish.
B sometimes refuse to watch it when she suggests they do so.
C have decided not to return to the habit of watching it.
D never watch it simply because they are feeling lazy.

16 A

17 C

18 D

19 A

20 B

Interviewer: Miranda Ingram and her family were avid TV watchers until the day when
they found themselves without a television. Miranda, how did that come about?

Miranda: I would love to be able to say that this was because I flung the set through the
sitting room window or sold it, but the truth is that circumstances deprived us. We moved
to the middle of nowhere, surrounded by mountains, to an ancient cottage, which had
never had a TV point. Unbelievably, perhaps, in the 21st century, our options for getting
plugged in were remarkably sparse. We could have spent a fortune laying cables to the
nearest village and joining their communal aerial, which sent fuzzy pictures every
time it rained - and we're talking Wales here, so rain is not a rare occurrence. And any
time strong winds or stray animals knocked it out of kilter, the entire system went
down for days. Or we could have got satellite television, but when a satellite technician
arrived, he looked round at our mountains arid saw not breathtaking natural beauty but
obstacles. So neither option seemed worth the trouble,

Interviewer: So what was it like to be a family without a television?

Miranda: Well, we trained ourselves not to look at the TV listings so we wouldn't sigh
over what we were missing and started to revel in our moral superiority. `Did you
watch .., ?' people would begin, and we would watch their jaws drop as they wondered
what on earth we did, half way up a mountain with two small children and no
television. At the risk of sounding unbearably smug, we did indeed read more books,
listen to more music, and play more board games. And we sat outside and watched the
sun set or merely had an early night. Most significant, however, was simply discovering
the untold long, pleasant and potentially fulfilling hours there are in an evening.

Interviewer: Surely, there must have been some downside?

Miranda: At times, I must admit, we did feel like cultural oddities. Television enters the
language and we didn't know what people meant when they compared someone to
an apparently well-known character, or when they used what was presumably a
catchphrase from a popular programme. And my husband and I are confirmed news
junkies, so we really missed the television when it came to big news events. There are
certain stories where television pictures tell more than any amount of radio and
newsprint. But like any mild addiction, after an initial withdrawal, before long you hardly
give it a second thought.

Interviewer: So why, since you were evidently enjoying life without television, did you
get connected again!

Miranda: Minm, you may well ask. Well, it was my husband who persevered with the
satellite option. Not, I'm convinced, because he missed the broadcasts so much. More
because he missed playing with the remote control in the way that men love to. Anyway,
I went along with it because I'm certainly not one of those anti-TV types that believes the
box to be the source of all modern evil: there are lots of interesting and rewarding
programmes for both adults and children, and television is a perfectly good
ingredient of a well-rounded life. But its insidiousness lies in its being an easy option -
like a ready meal - which seduces you into forgetting the rewards that come from putting
a bit more into life. So I must say that when the day arrived for our connection, I was
apprehensive, terrified that this thing in the corner would dominate our lives.

Interviewer: So how have things turned out? Are you and the children TV addicts again?

Miranda: Well, amazingly, now we have our TV back, the children can take it or leave it.
Inadvertently, it seems, our year's abstinence must have coincided with their habit-
forming years, so it's a habit they don't have. Occasionally they slump, but often they'll
switch on for ten minutes before announcing it's `boring’ and rushing off to do something
else. I even find myself proposing half an hour's viewing as an activity, but if they
suspect it's because I want to sneak off and do something without them, they are
very unlikely to agree. We do watch television again, of course we do, but it is no more
than an option among others. We even watch rubbish from time to time, but now it's
because it has been one of those days when deciding to vegetate is a deliberate choice,
not just a habit.

Interviewer: That's interesting. Thanks, Miranda. After the break, we'll be discussing the
subject of television and its impact on our lives with .,.

Exercise 14.

You will hear an interview with someone who reviews hotels. For questions 16-20
choose the answer (A, B, C or D) which fits best according to what you hear. In the
exam you will hear the piece twice.

16 What does Paddy say about some readers of her column?


A They suspect that she enjoys criticising hotels.
B Her attitude to hotels has changed because of their response.
C Her comments match their experiences of hotels.
D They prefer reading about hotels they would not want to visit.

17 What does Paddy say about some hotel-keepers?


A They sometimes have to force themselves to have a sense of humour.
B They would be more suited to a different profession.
C They expect to receive negative comments about their hotels.
D They are surprised that they become friends of hers.

18 Paddy says that some hotel-keepers she has contacted about the book have
A realised that she does not really have an assistant called Emily.
B corrected inaccuracies that were in her review of their hotels.
C responded favourably despite criticisms she had made.
D made her wonder whether her reviews of their hotels were unfair.

19 Paddy says that one hotel-keeper she spoke to told her that
A other people are unlikely to be treated in the same way in hotels as she is.
B he was unwilling to discuss some of the comments in her review.
C her reviews did not have as much influence as she believed.
D he no longer wanted his hotel to appear in the book.

20 The same owner also told her that


A he had passed information about her to other hotels.
B he resented her description of him in her review.
C he did not understand why she wanted to put his hotel in her book.
D there was nothing distinctive about her physical appearance.

16 D

17 B

18 C

19 A

20 D

Interviewer: I'm talking to Paddy Burt, who has a weekly hotel review column in a
national newspaper and who has just compiled a collection of those reviews for a
forthcoming book. Paddy, when you go to a hotel to review it, what's your attitude?

Paddy: I always have high hopes - a 'bet this one's going to be good' feeling. But you
never can tell. Hotels that look so idyllic in one of the guides can be a terrible letdown,
which is why readers who say they enjoy the column invariably add 'particularly the
bad ones'. For example, I recently got this letter from a reader, who says: 'It used to
be every other week that you gave some poor hotelier a bashing. Now it's a rare
treat to read about one you've been severely critical of, and that's a pity since I love
it when you lay into a pretentious but bad one. Of course, it's helpful when you
recommend a good hotel, but, for entertainment's sake, do try to find some awful
ones, too.

Interviewer: So are you always aiming to find fault? Are you glad when you find
something you can be critical of?

Paddy: I don't have to try. And while I'm always happy to slam into any pretentious hotel
that doesn't come up to scratch, it's a different matter when the people are nice and their
hotel isn't. I still have to write about it and sometimes it hurts. Hotel-keeping, it has been
said, is akin to show business and, in the ones I like best, there is always a leading man
or woman who is sometimes so good I think he or she has missed their true vocation.
Such hoteliers usually have a sense of humour. They may not like what I have written
about them, but will respond in a good-humoured way. They are professionals. Many of
them have become friends.

Interviewer: What kind of hotels do you prefer? Is it possible to generalise about that?

Paddy: Well, I admit I have a penchant for owner-run hotels; they are more personal than
the chains. With a few exceptions, I like the owners of small hotels. Which is why I've
had such fun researching my book of review pieces that have appeared in the newspaper -
calling them if they haven't responded to the questionnaire I sent them and either telling
them who I am or, if I think they're going to shout at me, pretending to be the assistant I
haven't got, Emily. 'She didn't give us a very good review, did she?' some said, Well,
no - but maybe they have since made improvements and would like people to know
about them? Thus encouraged, the majority of these hoteliers have entered not just
into the book but into the spirit and have contributed interesting behind-the-scenes
stories.

Interviewer: So some of the hotels you reviewed and wanted to put in the book haven't
been included?

Paddy: That's right. There's one, for example, where the owner said - I recorded all the
calls - 'After insulting us and lying in her article, there is no way we would help her
perpetuate her grievances against the world in a publication.' To specify the lies, he
pounced on a remark I had made expressing surprise on being served certain vegetables
in his restaurant. 'She doesn't understand proper food,' he said. I was enthusiastic about it,
actually, and if he wasn't being so disagreeable, I would have liked to include his hotel in
the book. On and on he went. 'Since her visit, we've noticed that a lot of people read her
articles and then cross hotels off their potential list as a result of what she's said. They
then go to hotels where she's been fawned over and where they probably won't be
fawned over. We've also noticed she prefers staying in hotels that are almost empty
because that's when they have time to make a fuss of her.' Actually, being fawned over is
the last thing that I want.

Interviewer: So your column can provoke quite a reaction, then?

Paddy: Oh, yes. In fact, the same owner also said 'After she stayed here, we had four
hotels asking for her description. They wanted to know what car she was driving and
what credit card she had. Unfortunately, we couldn't give a description because she's
fairly nondescript.' But the peculiar thing is that when it finally clicked that being in the
book wasn't going to cost him a penny, he said he wanted to be included. Maybe it was
because he remembered that I had remarked on his resemblance to a much-loved
comedian, sadly now dead. I declined his kind offer.

Interviewer: I can see why. Paddy Burt, thanks for talking to me.

Exercise 15.

You will hear part of a radio interview with two academics, Anne Trowell and
Jonathan Sims, on internet technology. For questions 16 – 20, choose the answer (A,
B, C or D) which fits best according to what you hear.

16 Anne and Jonathan agree that people become anxious when technology
A begins to control their children’s lives.
B establishes unsustainable connections.
C forces a redefinition of social relations.
D introduces feelings of inadequacy.

17 What point is made about changes in our relationship with technology?


A The changes will require creative thinking.
B Technology will have to be more tightly controlled.
C People must learn to trust technology.
D The changes will involve more attention to detail.

18 When talking about internet search engines, the speakers agree that
A they are improving social interaction.
B much information is oversimplified.
C they sometimes lead to confusion.
D basic knowledge is widely available.

19 How does Jonathan define the problem of increased connectivity between people?
A It facilitates the communication of false emotion.
B It decreases the likelihood of agreement.
C It allows only a few people to compete for attention.
D It makes human interaction dependent on machines.

20 What does Anne find striking about international news in the digital age?
A the limited focus of the public
B the diversity of the media used
C the risks involved in reporting
D the need for sensitivity in editing

16 C

17 A

18 D

19 B

20 A

M: Hello, this is Future Net. Our topic in this week’s programme is the reach and influence of
internet technology. With us to discuss it are two prominent internet scholars, Anne Trowell and
Jonathan Sims. So Anne, what do you think about the way people will use and engage with
technology in the next few years?

A: When I think about the future reach of internet technology at a global level, I’m convinced
that it is always going to be asymmetrical. Not that technology won’t be present everywhere but
that it will be a layering effect. So imagining that patchiness is for me as a researcher actually
what makes it an interesting world.

J: Another point, I think, is that many people feel a persistent anxiety about what it means to be
constantly connected, whether it’s manifested in ‘my child has turned six, should I get them a
mobile phone’, or ‘my grandchild knows how to use the laptop and I don’t and she’s three’.

A: We worry about what people are connecting to, and to whom. But there is always a moral
anxiety that accompanies technology…

J: …because it threatens to rearrange the way we interact with other people. And any
change like that worries people.
A: Exactly. The accounts of when rock ‘n’ roll came along identify exactly the same current
don’t they, a sort of moral panic, with phrases like ‘have you seen what the young people are
doing?’

J: Yes. But for me, what’s interesting is the shift in our relationship with technology, this notion
that we are moving from a world of technology that we have to do all the work for, to plug in,
update, give passwords and networks … it’s just a lot of palaver
looking after these machines.
A: For me it’s about how we empower everyone. And for me that is not just about the
technology, it’s about how we come up with the possibilities and imagine how it
might be different, what we might become with technology as we move forward.
M: Jonathan, you’ve been exploring the effect of search engines.
J: Yes, I would argue if you look at the effect of search engines … in that sense of the
internet putting out enormous amounts of human knowledge and making it
accessible, we are growing up with a generation of people who’ll never have that
experience of having a long conversation with their friends about how big is this,
where is this … that won’t happen anymore.
A: Right. On complicated issues, we’ve still got a way to go but as far as that sort of
information goes, we’re there. But all the same, when you look back at the 1990s
there was a sort of naïve belief that, because everyone could be on these networks,
we would be connected to everybody on the planet and we would have this
incredible wealth of deliberation. And I don’t think that’s happened.
J: No, in fact, it turns out that being more connected to one another, more people have
voices, probably makes it worse rather than making it better in terms of coming to a
consensus. You’re trying to listen to everybody, everyone wants a turn to speak, all
sorts of dynamics, whoever’s loudest, whoever’s the most passionate ends up
having more power. That turns out to be a deeply human problem, not a
technological problem.
M: And what about the role of technology in providing us with news about the rest of the
world?
A: What’s funny is that despite the fact that reporting on the rest of the world has got so
quick and easy, we have less and less concern in what’s going on elsewhere
compared with our own back yards. So historically, we got our knowledge of
nternational affairs through a variety of edited media. And whether that was through
newspaper, whether that was through television broadcast, someone went out and
said here’s what’s out there and what’s important, and this was often a really difficult,
expensive, dangerous thing to do.
J: And then someone else edited it and rewrote it before it finally reached the public.
But now that’s all so much easier, maybe as you say it’s taking us in ways we
wouldn’t have predicted.
M: Jonathan and Anne, thank you… [fade]
Exercise 16.

You will hear a discussion in which two people, Sam Williamson and Kathleen Smithson,
talk about strategies to help the poor in resource-rich developing countries. For questions
16-20, choose the answer (A, B, C or D) which fits best according to what you hear.

16 According to Sam, the 'paradox of plenty' results in


A a vastly unequal distribution of wealth.
B the discovery of resources in developing countries.
C a reduction in the variety of products reaching the market.
D communities losing interest in government activities.
17 The company Sam worked for in Ghana
A caused severe problems for local companies.
B was criticised for not being trustworthy.
C adopted an approach that was innovative to the region.
D did not receive full cooperation from the authorities.
18 What does Kathleen think about contracts between governments and companies being made
public?
A It is not something business leaders want to do.
B It is only part of the process of reducing poverty.
C It is ineffective because of the public indifference.
D It will never lead to any great changes.
19 What problem does Kathleen highlight about original government documents?
A They are incomprehensible to ordinary citizens.
B They are sometimes altered to hide the truth.
C It can take years for them to be released.
D They cost a great deal to distribute.
20 The agreement made in Australia that Sam mentions
A focused solely on how to spread wealth.
B involved talks between a company and the public.
C resulted in compensation for pollution being accepted.
D was made after citizens became ill.

16 A
17 C
18 B
19 A
20 B

Interviewer: In 2008, oil and mineral exports from developing countries were worth about $393
billion, which was five times higher than those of farming products and foreign aid put together.
If used correctly, this revenue could be used to free millions from poverty. Why hasn't there been
extensive relief from poverty and what can be clone to rectify the situation?
Williamson: The answer to your first question is quite simple. It's because those who benefit
from the oil and mineral resources in developing countries are the elite from those countries and
the companies involved in extracting them. When this happens, we have a situation known as
the 'paradox of plenty'. This term is used to describe the phenomenon whereby countries
rich in natural resources like oil, gas and minerals end up poorer than those without them,
And by poorer, I don't mean the financial worth of the developing country and its citizens,
but the way in which the revenue from those resources is spread, with those at the top
becoming much richer and the gap between them and the poor growing ever wider.
Because of the corruption that is caused by the government and other elite groups placing their
interests above their responsibilities to the people, there is injustice and instability. Democracy
and basic human rights are undermined and there is a decline in competitiveness of the economic
sector as well as volatility on commodity markets. As for tackling the problem, the only way
forward is to draw up social contracts with communities through transparency to ensure that
they, too, benefit from the revenue from the resources.
Smithson: I agree that transparency is the key and it's been proven. In 2009, the Sierra Leone
government signed a deal with a foreign mining company, but when the contract was made
public, there was widespread condemnation of it, as it did not comply with the law. Eventually,
there were some changes made, but it would have been better if there had been an open
contracting process in the first place.
Williamson: That's exactly what happened when the mining company I was working for in
Ghana insisted that the Ghanaian government officials make the contract public before it was put
before parliament for ratification. When we requested such openness, I think they were taken
aback because this process had never been adopted in West Africa before. And it set the bar
really high for other foreign companies doing business in the area, as it became a precedent that
helped build trust and credibility. It was a win-win situation for the country as a whole and the
investor.
Smithson: Those situations in Sierra Leone and Ghana, however, seem to be the exception rather
than the rule. The community leaders I've spoken to say that it's great to have access to revenue
information, but all too often there is little impact or change at ground level. In other words, new
schools, roads and public services have not materialised. So, in my opinion, transparency, in
which the public is made aware of the nature of contracts drawn up between the
government and companies, must only be treated as a single, prerequisite step in the fight
against poverty. If that transparency results in accountability and people don't turn a blind eye
to the terms of the contract, then money can be spent on sustainable development.
Williamson: That's a good point. When contracts are open to public scrutiny, government
officials know they will be accountable for the terms of the deal and will be under public
pressure to sign deals that are acceptable for citizens. Then, systematic transparency will be
adopted as the norm and all stakeholders will be involved in the negotiating process.
Smithson: Again, that's a good starting point, but it's one thing to make government
documents available to the public in their original form and it's quite another to ensure
that communities understand what the contracts entail. It takes years of professional
training to be able to plough through legalese - the complex language of the law - and grasp
the meaning precisely. Rather than exploiting this to maintain the communication gap, some
governments have changed the form of the documents using new tools that simplify the contracts
in order to distribute information that can be readily understood. Then, communities can gauge
the real-life impact of contracts drawn up by governments and companies involved in extracting
natural resources.
Williamson: And when communication channels are clear, negotiations can be conducted
without government intervention. Take, for example, the agreement made between Rio Tinto
and the indigenous community in Australia. The meeting not only involved discussing the
best way to distribute revenue from the mining operations, but also how to reduce poverty. And
there were other issues on the agenda, too. Discussions centred around the mining not
compromising the quality of the soil, air and water so that the operations would not lead to
serious health concerns. I only hope that such negotiations set a precedent for other operations
related to natural resources and they became the norm rather than the exception.
Interviewer: I think ...
Exercise 17.

You will hear a discussion in which two authors, Emma Jameson and Philip Cross, talk
about writing a novel. For questions 16-20, choose the answer (A, B, C or D) which best fits
according to what you hear.

16 What does Emma say about the process of writing a novel?


A No two writers use the same approach.
B It cannot be put into words.
C There is no universal formula for it.
D Most writers are unwilling to discuss it.
17 According to Philip, a novel is likely to fail when the opening
A contains plenty of little-known vocabulary.
B pays too little attention to detail.
C doesn't introduce enough main characters.
D reveals too much about the plot.
18 Emma warns that having a flexible outline for the plot
A creates more difficulties than having a set outline.
B limits the writer's options for the ending.
C is a strategy only experienced writers can adopt.
D may result in the storyline becoming unclear.
19 Emma's tip for getting thoughts down into writing immediately
A means not necessarily writing the novel in sequence.
B increases an author's speed of writing.
C helps the writer meet deadlines more easily.
D makes the plot seem more realistic.
20 Philip believes that asking for advice before the novel is completed
A shows a lack of judgement.
B can lead to confusion.
C is likely to cause misunderstandings.
D should only be done as a last resort.

16. C
17. A
18. D
19. A
20. B

Interviewer: According to the National Novel Writing Month and the Telegraph Short Story
Club, the number of budding writers is on the increase. And from what I can gather, they're
always on the lookout for advice that will help them get their work published. Can you give our
listeners some pointers, Emma?
Emma: I’ll try, but I must emphasise that there is no single set of rules that applies to all
writers. So even if you take the advice of those writers who are willing to reveal their
approach to moulding their ideas into a novel, there's no guarantee the same method will
work for you. So, it's up to each individual writer to find a way to transform an idea for a
novel into the published article.
Philip: That's true, but there is a kind of template writers can use as a basis to develop their story.
By that I mean selecting the setting after the basic plot has been formulated, for example. Very
often the plot dictates the setting as it would in, say, a story about rival gangs. Such a story
would best be set in an urban environment where the physical aspects of the setting are more
realistic with regard to the plot.
Emma: Then you can follow up with points of view by deciding whether it should be written in
the first or third person and how much the reader should know at any particular point in the
story. As for the protagonist, he or she must be a character that the reader will root for because if
that doesn't happen, the reader will lose interest.
Philip: That's a good point, and it's also relevant to the conflict aspect of a novel that is
fundamental and independent of genre. If the readers aren't drawn into the story
immediately, either because the opening is too drawn out, or they are forced to use the
dictionary too often, they may be tempted to put down the book for good. So, hooking the
reader as early as possible is of paramount importance. One feature of an enticing opening is
revealing what is at stake for the protagonist. In other words, what will the consequences of
failure be? These must be made clear, without necessarily revealing every detail, as soon as
possible.
Emma: Once these have been established, the difficult decision relating to outline has to be
made. Some writers prefer to have a set direction for the plot with little room for manoeuvre. In
general, they say this approach brings clarity, whereas those who favour a more flexible outline,
in which they can introduce twists and turns on impulse, state that a lack of restriction enhances
their creative input. Personally, I adopt the latter approach, but if you do the same, be
careful you don't lose sight of the original plot altogether because you'll probably lose your
readers, too. Is there anything you'd like to add, Philip?
Philip: No, I think you've hit the nail right on the head. What I would like to mention, though, is
content. Traditional wisdom states that you should write what you know, but that can lead to a
limited perspective. So, I think research is in order to enhance interest which will not only
benefit the reader in terms of enjoyment but may also provide you, the writer, with ideas you
would not otherwise have thought of.
Emma: And whenever you get a great idea, you should write it down and work on it immediately
so that it doesn't get lost. I know this might mean not writing straight through from
beginning to end, but I know of writers who have come up with a brilliant idea for an ending,
write it and then work towards it. So, make use of inspiration at every opportunity. Of course,
you can't just wait around to be inspired as there are time constraints which involve setting
deadlines. You can't avoid these, but be realistic and don't force yourself to write a set number of
pages every day because you'll end up sacrificing quality for quantity.
Philip: And that brings me on to my final point - asking others for feedback. I know its difficult
to rely on your own judgement of quality, but my advice is that you should keep your work to
yourself and resist the temptation of seeking emotional support until you have finished. I know
that anyone you ask for an opinion will probably try to give you encouragement and make
constructive comments. But, more often than not, input from a third party is likely to leave
you bewildered and uncertain. So, work through your novel and when you're ready, enter a
competition. There are many of them out there for first-time novelists like you.
Interviewer: Well, thank you ...
Exercise 18.

You will hear part of a discussion in which two consultants, Abbie Dale and Ryan
Richardson, are talking about two different small businesses that have achieved success.
For questions 16-20, choose the answer (A, B, C or D) which best fits according to what you
hear.

16 What does.the company Ryan has chosen encourage its customers to do?
A buy every kit it produces
B create new designs and submit them
C personalise their online purchases
D tell others how good the products are
17 According to Abbie, the social clothing company's customers
A like to draw attention to themselves.
B are willing to help in publicity campaigns.
C show creativity in the videos they send in.
D have become fiercely loyal to the brand.
18 Ryan believes that the main difference between the two companies is
A the number of employees they have taken on.
B the way in which they have developed.
C the variety of problems they have to solve.
D the age range they cater for.
19 What does Abbie find most astonishing about the company she talks about?
A its skill in collaborating with partners
B the owner's ability to keep developments secret
C the fact that it has grown so fast
D the number of products it aims to launch
20 According to Ryan, people who buy and assemble the electronic kits
A should start their own businesses.
B are surprised by their simplicity.
C find the experience rewarding.
D aren't frightened of making mistakes.

16 C

17 A

18 D

19 D

20 C

Interviewer: Now, you've chosen businesses that seem worlds apart with regard to product and
production and yet have startling similarities in terms of strategy. Could you tell us a little about
these businesses created by young entrepreneurs?

Abbie: Yes, I decided to go with a clothing company, or rather a social clothing company as the
owner, Chris, prefers to call it. This particular online company functions more like a blog than a
conventional web store, which allows it to utilise the input from the blogger communities and the
news cycles directly. Then, the elements highlighted by the members of those communities are
incorporated into its designs. So, social engagement is key to its success with the emphasis on
listening to people rather than dictating to them.

Ryan: Yes, it's definitely a case of having your finger on the public's pulse. And I believe the
company I've selected does exactly that, but with an entirely different product - DIY electronic
hardware kits. Not so long ago, consumers would demand their electronic gadgets ready made
and ready to use. Now, however, devoting time to assembling the gadgets is perfectly acceptable.
The kits come with detailed instructions for assembly, but customers are actively encouraged
to modify the product so that it matches their particular taste. Then, customers have the
opportunity to embrace the culture of sharing online to feed off each other's creativity.

Abbie: That's another similarity with the company I've chosen. Its customers don't just want to
buy clothes; they want to be seen wearing them, too. After all, the target market is the
younger generation whose social media profiles reflect the 'look at me' mentality. The
company has taken this into account and now invites customers to submit photos or videos of
themselves wearing the company's products. Not surprisingly, there has been an overwhelming
response from which the company has dearly benefitted. Firstly, there is the publicity aspect,
which allows the company to advertise without spending huge sums. Secondly, and more
importantly, this initiative has given the company access to information only available on social
media and that is the speed of change in what the customer wants. Then, it's just a case of
keeping pace with that change.

Ryan: I'm glad you brought up the element of speed and online companies having to adapt
accordingly. Right from the outset, the founder, Limor, who did her master's at HIT, realised she
would have to come up with new ideas all the time. As a result, she made sure she embarked on
at least one new product per week mainly based on such technology as gyrometric sensors and
solar panels. Nowadays, her products, developed with her troubleshooting team of three
engineers, cover a much wider range of technologies that appeal to every generation. There
are kits for primary schoolchildren who can build robotic arms through to grandmothers
who can sew clothes with working LED displays with conductive threads and washable
hardware. This is probably the biggest difference between the two companies.

Abbie: As you inferred, it's not only recognising the need for change but it's also adapting to it.
At the moment, Chris's social clothing company aims to come up with three creations a week so
as not to fall behind. Incredibly, he plans to increase this rate to a mind-boggling twenty a
week at least, with about half designed through crowd-sourcing and collaborations with outside
designers. With this in mind, he has formed manufacturing partnerships that ensure new
inventory arrive in a matter of weeks, not months. As for managing the online catalogue, he aims
to develop it in keeping.with what works online. So again there are going to be novel ways of
presenting items, I'm not sure what novel ways he's referring to, but I'm certain they'll be as
creative as his clothing.

Ryan: And all this points to rapid development. Limor's company has sold over half a million
kits in seven years, Over the same period, she has moved into a large warehouse that has enough
space for storage and to accommodate equipment like laser engravers and mills. Therefore, every
part can be manufactured on site. If you look at the business now, and think back to how it
started - the notion of providing people with the opportunity to gain a sense of achievement
from making something on their own, which they obviously do - you'll be amazed. But don't
just stand there open-mouthed. Come up with your own idea and don't be afraid to go ahead with
it.

Abbie: I'd like to echo that sentiment and ....


Exercise 19.

You will hear part of a discussion in which two educationalists, David Shaw and Anoushka
Patel, talk about education standards in the UK. For questions 16-20, choose the answer
(A, B, C or D) which best fits according to what you hear.

16 What criticism does Anoushka make of primary school education?


A It has remained the same for too long.
B The teachers are inadequately prepared.
C It is not sufficiently varied.
D Pupils no longer find it enjoyable.
17 According to the statistics David quotes, the secondary school exam results pupils achieve
A have caused the national average to fall dramatically.
B cannot be directly compared to those of foreign pupils.
C do not reflect the true level of education in the UK.
D can be accurately predicted when they leave primary school.
18 Anoushka says that pupils who perform poorly do so because
A they lack the ability to concentrate in class.
B education is not a top priority for them.
C teachers are unable to provide them with enough attention.
D it is impossible for their parents to buy books.
19 David claims that universities are guilty of
A ignoring the advice given by the Department of Education.
B making it easier for students to get good degrees.
C accepting too many applications from school leavers.
D exaggerating the quality of teaching they offer.
20 Anoushka makes the point that universities in the UK
A have adapted to meet the needs of society.
B should not have to answer questions about their policies.
C have maintained a traditional approach to education.
D are still considered to be the best in the world.

16 C

17 D

18 B

19 B

20 A

There's been a great deal of criticism levelled at education standards in the UK recently. is this
criticism justified?

David: If statistics are anything to go by, the answer is a definite 'yes'. There was some progress
made between 1995 and 2005 in literacy and reading, but since then it has stalled. The result has
been that we've been overtaken by countries improving at a faster rate. So, we've now slipped
down to a dismal 23rd in the global league table. It's a worrying statistic.

Anoushka: It is, and it's one that doesn't seem to be about to change in the near future.
Unfortunately, the current system places too much emphasis on preparing primary school pupils
for the transition to secondary school. As a result, there is an overemphasis on tests at primary
school, which in turn means that the curriculum is significantly narrowed. It seems to be a
case of testing rather than teaching, with those pupils who are good at passing tests coming out
on top when they leave primary school.

David: And those who lag behind at the age of 11 seldom catch up. By 'behind' I mean not
reaching level four at the end of primary school. Only 6.5% of pupils who fall into this
category obtain the benchmark five good GCSE exam passes at secondary school when the
national average is up at 58.2%. And by good GCSE's, I mean from grades A to C. Obviously,
this figure of just over one in twenty does not compare favourably with the 46% and 95% who
reach the benchmark after leaving primary schools at levels four and five respectively. These
statistics show exactly why progress has stalled. It's simply because those performing poorly
have been unable to improve.
Anoushka: No, they haven't, but I'm not sure that the system is entirely at fault in this instance. If
we identify those who are under-performing, we can see that the vast majority are from
disadvantaged backgrounds. Because of their situation, they have bigger fish to fry, so to
speak. The pupils I'm referring to are those on free school meals from families facing tight
financial constraints, which can mean no heating at home in the cold winter months, for example.
Such children find it almost impossible to concentrate on their work when their teeth are
chattering. Teachers do all they can to compensate and they do try to give underprivileged pupils
equal opportunities, but I'm afraid in most cases, they're fighting a losing battle.

David: That's a fair point and it does show why the Department of Education's initiatives like
phonics training for new teachers have proved ineffective in re-establishing the progress in
literacy witnessed from 1995 to 2005. What it doesn't explain, though, is the criticism of falling
university standards. As you know, it is claimed that universities have been guilty of
dumbing down - a claim they have categorically denied. However, there is unequivocal
evidence to suggest that degrees are now worth substantially less than they were. Over the
past decade, the proportion of first-class degrees awarded in UK universities has risen by 50%,
while over the same period student numbers have risen by 20%. These figures either point to a
sudden phenomenal success story - I don't think so - or a sharp decline in the standards students
are expected to reach.

Anoushka: I'm not so sure about the relevance of that argument because the question of whether
standards have fallen or not cannot be answered with any degree of certainty. That's because
there is neither a universal standard over time nor a baseline for making comparisons. What I
believe to be a more suitable question is whether higher education is functioning appropriately;
for this day and age. So, what we have to ask is: Has the purpose of higher education moved
with the times? In my opinion, it has as school leavers who would never have had the
chance to go on to higher education now have something to aim at, They're not being
written off as they were in the past.

David: That's all very well, but ...


Exercise 20.

You will hear a discussion in which two biologists, Ian Cartwright and Angela Sharpe, talk
about conservation and the public's perception of it. For questions 16-20, choose the
answer (A, B, C or D) which best fits according to what you hear.

16 According to Ian, why aren't people easily convinced of the importance of protecting
endangered species?
A They find it difficult to understand the concept.
B They are presented with information that is too vague.
C They do not believe human activity causes extinction.
D They think that conservationists are exaggerating the situation.
17 Angela says that eco-tourism has been successful because
A it provides locals with a long-term source of income.
B it has been properly managed by governments in developing countries.
C it encourages people to have more respect for nature.
D it is affordable for a large number of people from developed countries.
18 The story Ian tells about how economists determined the value of the environment highlights
A what a complex subject economics can be.
B the ease with which false promises are made.
C why companies get away with polluting lakes.
D how ignorant people are of the role nature plays in their lives.
19 What explanation does Angie give for people being indifferent to the destruction of the
ecosystem?
A They believe scientists will fix the problem.
B The vast majority do not suffer too much when it happens.
C They consider the exploitation of environmental resources necessary.
D They think the cost of replacing unrecognised benefits has been overestimated.
20 The species Ian refers to
A indicate how fast an ecosystem is likely to collapse.
B only live in one specific ecosystem.
C are extremely sensitive to environmental change.
D appear to be in the greatest danger of extinction.
16 B

17 A

18 D

19 B

20 C

Interviewer: Today, we'll be talking to two biologists, Ian Cartwright and Angela Sharpe, about
conservation and the public's perception of it. Now, the message of how important it is to
conserve biological diversity is one that has been passed on to the public through education and
the media for some time and yet very little seems to have changed, Do you think you are fighting
a losing battle in trying to change the public's mindset, Ian?

Ian: I wouldn't go that far. But I will admit that it's been an uphill struggle to convince people of
the value of endangered species and what can happen if they become extinct because of human
activity. You see, although we know that the loss of a single species can trigger a chain reaction,
the impact of that loss is not always immediately apparent and can be unpredictable. As such,
it's an imprecise science, which makes it difficult to provide the public with the kind of
information they will be able to grasp and hopefully act on.

Angela: I agree it is difficult, but not impossible as we've been able to witness through eco-
tourism, which is particularly important in developing countries that cannot afford conservation
programmes. Because of its direct link to economic and social development, eco-tourism allows
local populations to see clearly the benefits of exploiting biological diversity to create a
permanent source of wealth rather than destroying it to satisfy short-term needs. The
success of eco-tourism shows that it is possible to conserve diversity when the benefits are clear
and within reach.

Ian: I think that's a fair point. Very often in the developed world, people who live in urban
sprawls are too far removed from nature to be able to appreciate the extent to which we
rely on what it has to offer. This was illustrated when economists were first requested to
include the environmental factor in their complicated calculations to determine whether a
project was economically feasible or not. To put a value on the environment, they visited some
residents of a city with a lake nearby and informed them that a company was interested in
purchasing land next to the lake to build a factory. They were also told the factory would
probably pollute the lake and in doing so kill every living thing in it. Then, they were asked how
much they were willing to give to prevent the company from building a factory and therefore
conserve the area. The amounts were noted, but when the researchers returned to the households
to tell the residents the company wanted to go ahead with the purchase, hardly anyone was
prepared to pay the sum of money they had pledged. As a consequence, the economists
concluded that the environment was virtually worthless and could not be factored into their
analyses.

Angela: That story ties in with public perception of what are aptly known as the unrecognised
benefits of conserving biodiversity. These are the services we receive when ecosystems function
normally. By that I mean such benefits as chemical cycling - the oxygen produced by rain forests
for example, water purification and flood protection. These are the kinds of things we take for
granted, so we tend not to consider them until an ecosystem together with its ability to provide
these services is destroyed and we have to foot the bill to replace them. Naturally, you might
expect that the high cost of fixing such problems would be enough to make people sit up and
heed the warnings about ecological disasters, but you would be wrong. This is because of the
relatively low proportion of the population that is severely affected by such catastrophes.

Ian: Yes, again it's a case of people not having to care unless they feel the full effect of an
environmental malfunction. What they don't realise is that there is always a wider impact of any
ecosystem collapse than just the localised one. So, keeping all ecosystems intact is obviously the
primary aim of all conservation projects with the focus on prevention rather than cure.
Fortunately, this can be achieved because within any single ecosystem there are species that act
as indicators of environmental quality. The decline of bald eagles, for instance, told us of the
dangers of DDT. And more recently, in Florida, largemouth bass have relayed the message that
there is mercury contamination in freshwater ecosystems. Likewise, the disappearance of
sawgrass in freshwater marshes has informed us of nutrient problems in the Everglades. Without
such environmental monitors, we would not have realised there were contaminants until even
more damage was done.

Angela: So, in answer to your question, I think I can speak for both of us when I say that the key
to putting the message across is to get people involved in conservation even if that just means
visiting a reserve.

Thank you both.


Exercise 21.

You will hear a discussion in which two people, Derek Grant and Lucy Wadham, talk
about the current state of mass tourism. For questions 16-20, choose the answer (A, B, C or
D) which fits best according to what you hear.

16 From the figures quoted by Lucy, it can be concluded that


A most of the revenue generated from safari packages goes to the airline.
B mass tourism inevitably leads to poverty.
C hotels in developing countries do not have high standards.
D there aren't sufficient local products to cater for tourists.

17 What does Derek say about governments whose countries are tourist destinations?
A They are constantly trying to increase their profit margins.
B They ignore the negative impacts of mass tourism.
C They have very limited bargaining power.
D They are unfamiliar with aspects of modern business methods.

18 Derek's example of excessive water consumption caused by mass tourism shows how
A ineffective the laws are in developing countries.
B local food production can be adversely affected by it.
C difficult it is to find sources of pure water.
D valuable a resource water has become in recent years.

19 Lucy says that when a resort loses its appeal,


A the locals are left to fend for themselves.
B money has to be spent on a new infrastructure.
C the cost of package holidays falls considerably.
D it becomes vulnerable to natural disasters.

20 According to Derek, tourism


A does more harm than good in the long run.
B is too big an industry to be tampered with.
C is largely responsible for global inequality.
D promotes cultural sensitivity most of the time

16 D

From every 1 pound spent on a safari package, 20 pence goes to the travel agent, 40 pence goes
to the airline, 23 pence goes to the hotel, 8 pence goes to the safary company and 9 pence ends
up in the Kenyan government’s coffers. Of that 9 pence, 15% is used to pay off debts and the
rest to import goods for those on safari.

17 C

That 11% may seem like a pittance, but governments whose countries play host to hordes of
foreign tourists are quick to emphasize the need for the foreign capital from the tourist industry
to promote economic development. In other words, they are prepared to take what is offered
rather than risk losing tourists if our operators decide to channel their customers to other
countries so that they can maintain or even increase their profit margins.

18 B

Benidorm is a case in point. This tourist destination has over 130 hotels and there are thousands
of swimming pool across the resort. As this place relies on an underground water source, water
has to be removed two or three times faster than it can be replenished to meet tourist demand.
The result is that the levels in the aquilfers drop, seawater seeps in and effectively poisons
the surrounding farmland.

19 A

Absolutely, and the upshot of this is that the locals have no choice but to satisfy tourist demand
as their livelihoods literally depend on tourism. They are caught between the proverbial rock and
hard place as when an area becomes tourist dependent and loses its potential to provide for
tourists, it is abandoned, leaving the locals with no means of support.

20 B

According to the World Travel and Tourism Council, travel and tourism employs 240 million
people and generates 0.4% of global GDP. Add to that the indirect employment created through
the multiplier effect and you have too many people dependent on tourism for it to undergo
any immediate radical changes.

Exercise 22.

You will hear part of a discussion in which a developer, Faye Mannia, and an
environmentalist, Peter Green, talk about a new government-run trial. For questions 16-20,
choose the answer (A, B, C or D) which fits best according to what you hear.

16 At the beginning of the discussion, it can be inferred that Faye believes the listeners
A are confident their government always acts wisely.
B are unfamiliar with current property values.
C may be biased against her point of view.
D have no knowledge of the trials whatsoever.

17 Peter says that the new government plan


A will lead to a reduction in the diversity of wildlife.
B has not been supported by a single scientist.
C should be delayed by at least ten years.
D may cause property prices to rise too quickly.

18 Faye mentions the iron ore mine in Brazil


A to point out that development is occurring on a global scale.
B to show the importance of increasing employment opportunities.
C to highlight the dangers of allowing poverty to spread.
D to illustrate the need to respond to changing priorities.

19 Peter fears that those involved in the new plan


A will break the law and go unpunished.
B cannot be trusted to pay for the habitats they destroy.
C will use it to their advantage.
D are unlikely to make any compromises during negotiations.
20 Faye accuses environmentalists of
A using the media to attack developers.
B having a lack of foresight.
C not appreciating historic buildings.
D ignoring the wishes of the general public

16 C
17 A
18 D
19 C
20 B
Interviewer: As the global population grows, a greater strain is being put on the planet to provide
for humankind. Very often, the struggle to improve standards of living results in having to decide
whether to exploit an area of natural beauty for its resources or leave nature alone. Here to
discuss the issue of money versus the environment are developer, Faye Mannia, and
environmentalist, Peter Green. Faye, could you tell us how you can put a price on the
environment?
Faye: First of all, I'd like to establish a level playing field for this discussion by dispelling the
notion that developers are the ruthless, insensitive individuals portrayed in the media. I
think that the newly-announced government plans to put a price on wildlife clearly shows that
developers are now considered responsible and the idea that developers just want to use nature
for their own ends is definitely a thing of the past. The initial trial will involve six local
authorities putting a price of, say, forty conservation credits on every ten homes developed. Each
credit could be worth £2,500, making a total of £100,000 to a broker who commissions another
landowner to convert fields or scrubland into meadows or woodlands to offset the developed
land. The remaining money is then paid to this landowner to maintain wildlife for two to three
decades.
Peter: I'm afraid I'm a lot less enthusiastic about the government plan than you are. The plan has
been drawn up without sufficient research into the type of artificial environment that
would substitute for the site torn up by developers. How, for example, can you think it's
acceptable to tear down existing woodland? One would have to wait at least a decade for new
woods to grow to be considered a replacement. In fact, it is highly unlikely that any artificial
environment will be able to support as many different species as one that has developed
naturally. I really feel that the whole idea of creating a new currency in the form of the
'conservation credit' is simply a way for developers to evade planning rules and to gain access to
prime greenfield sites so that they can make huge profits.
Faye: I think you're missing the point of the proposal trials. We shouldn't need to choose between
either improving the environment or growing the economy. We should aim to do both and these
trials should lead to ways of achieving this. And it is not simply a case of satisfying the wants
of developers as you have suggested — it is more a case of meeting needs. There is a parallel
in Brazil, where near the town of Carjass in the middle of the Amazon jungle lies a vast iron ore
mine. From the profits raised, the quality of life for the residents of Rio's favellas or slum areas
has been improved. Twenty years ago, these slums were neglected because they weren't
considered important. Now, perhaps because of global attention being drawn to Rio de Janeiro
through sporting events, improving the reputation of the city is top of the agenda. It's a case of
moving with the times.
Peter: But this comes at a cost. Once a specific environment has been developed, there is no
going back. Species disappear. England is losing its wildlife at an alarming rate. Every taxa of
birds, butterflies and bees has declined by around 40% and the country has lost 500 species in the
last 200 years. With these trials, losses would accelerate substantially. I'm afraid that this
system of using 'conservation credits' to destroy invaluable habitats is just a way of turning
wildlife into a tradable commodity with complex markets that could be exploited by
developers and bankers alike. It would inevitably lead to a lack of transparency, and
implementing the trials would mark the start of a slippery slope into the negotiation of numerous
murky deals.
Faye: I can assure you that everything will be above board. Details will be made available to the
public as they have been in the recent well-publicised deal, in which developers will provide the
£5 million so desperately needed to give a fortress used in the Napoleonic Wars a makeover in
return for permits to build a five-star hotel and 500 houses. The fortress is part of our national
heritage, which we developers will help retain for future generations. We're adopting a long-
term view, which environmentalists seem incapable of doing. Take what we did in the 1980s,
when we transported the earth dug from the Channel tunnel to an area near the White Cliffs of
Dover. Environmentalists criticised the idea at the time in exactly the same way as they are
attacking these trials. Now, three decades later, the area known as Sapphire Hoe nature reserve
attracts over 100,000 visitors every year.
Exercise 23.

You will hear a radio interview with a sports scientist talking about the effects of age on
ability to perform in endurance sports. For questions 16-20, choose the answer (A, B, C or
D) which fits best according to what you hear.

16 What point is Robert upholding when he says 'the Tour de France is living proof of
this'?
A Athletes should not be permitted to compete at these levels after the age of 40.
B No athlete is physiologically capable of winning once past 35.
C The vast majority of athletes reach a peak of fitness at the age of 35 and then decline.
D The average age of competitors in endurance sports is 29.

17 What was Robert's motivation for entering the Tour De France in his forties?
A so that the British Team had a competitor over 40
B to prove something to researchers
C to achieve a personal goal
D to finish his cycling career on a high

18 Why do the ages for best average performance in Marathons differ between men and
women?
A Female entrants are on average younger than male entrants.
B Women lose their lung and muscle capacity faster than men.
C Men demonstrate a faster average speed than women at different ages.
D Statistically, few women over 45 actually complete the Marathon.

19 How does qualitative analysis disprove the theory of age-related decline?


A It shows that decline is by no means uniform.
B It indicates that training time is a decisive factor.
C It highlights flaws in the theory of muscle loss.
D It proves that some people can halt the decline completely.
20 What conclusions does Robert draw from the research he has mentioned?
A Winning is possible at any age.
B Failure is inevitable when you don't follow the 10% rule.
C When winning is your goal you can achieve great things.
D Age should not be viewed as an obstacle to success.

16 C

Certain studies have shown that peak endurance performance is maintained until
approximately 35 years of age, followed by modest increases until 50 to 60 years of age,
with progressively steeper declines after that. And to be fair, the Tour de France is living
proof of this.

17 C

When I entered, winning wasn't my goal. Getting to the finish line was.

18 B

The scientific consensus is that age deterioration is the result of a decline in our maximal oxygen
capacity coupled with a tendency for our body fat to increase. The effects of these changes are
felt to a greater extent in endurance sports and unfortunately the deterioration seems to be
faster in women.

19 B

In fact, other scientific studies of athletes up to the age of 45 have shown that although, yes there
was a decline, the factors contributing to this were qualitative, not quantitative. For example,
rather than the decline being due to physiological factors affecting oxygen uptake, it was simply
the fact that older men trained less than younger men.

20 D

It means it's never too late to take up a sport and achieve something apparently miraculous.
Exercise 24.

For questions 11-15, listen to an interview with Kevin Langtree, who has just been voted
Best Young Chef of the Year and choose the answer A, B, C or D which fits best according
to what you hear. Write your answers in the corresponding numbered boxes provided.

11. How does Kevin account for his success?


A. He tries to love every minute of the work.
B. He learns a lot from the diversity of food and people he meets.
C. He envisions how takeaway meals will become in the future.
D. He learns to stop being unkind towards fast food.

12. What is Kevin's opinion about fish and chips?


A. He is ambivalent about having them every day.
B. He finds them indispensable in Britons' diet.
C. He thinks they do not bring the diversity that he loves.
D. He wants to incorporate them into food from different countries.

13. What does Kevin think about restaurants and takeaways in Britain?
A. Finding a parking slot to have meals is a major problem.
B. They should be restructured to display more variety.
C. Too many of them are from India and China.
D. They're all very difficult to get to without a car.

14. What does Kevin say about the American fast-food outlets?
A. They target children and young people.
B. They are cheaper than the other restaurants.
C. They make every customer look cool.
D. They are slowly taking over other fast-food outlets.

15. On mentioning root vegetables, what point does Kevin want to make?
A. British people are becoming more and more xenophobic.
B. There is greater choice, but imported foodstuffs are expensive.
C. The root vegetables produced in Britain in the past were best.
D. Cosmopolitan food reflects a generally less insular society.

11. B

12. C

1.3. B

14. A

15. D

Interviewer: This evening we're delighted to have been able to persuade Kevin Langtree into our
studio. Kevin, as you may remember, was the surprise winner of the Young Chef of the Year
competition. Kevin, what do you put your success down to?

Kevin: Hard work. I'm sure you expected me to say that, but I mean it. Except, for me, it isn't
really hard work at all; I enjoy every minute of it.

Interviewer: Okay, but you can't possibly enjoy all of it equally. Which bits do you like best?

Kevin: Learning. I'm learning all the time. There's such an immense variety of things —
and people — out there you can learn from. Do you know, I'm really glad to have been born
when I was. Just imagine — only a few years ago the staple diet was roast beef for Sunday
lunch, with the cold meat for sandwiches at teatime! And the only takeaway meal was fish and
chips. Sorry, I'm being unkind. You could, if you were feeling very adventurous, go in for steak
and kidney pie and chips instead! Rather different now, isn't it? In some places it's difficult to get
fish and chips at all. The fast-food market has been taken over by just about every nationality
you can think of. Did you know that Britons actually consume more pizzas than any other
nationality outside Italy? Also, as you know, kebab houses are enormously popular and, of
course, you can hardly move without tripping over Indian and Chinese restaurants and
takeaways. It's a very good thing, too.

Interviewer: So what's wrong with fish and chips?


Kevin Langtree: There's absolutely nothing wrong with well-cooked fish and chips. I love
them. But I also love variety. I love being able to eat food from a different country every
night of the week if I want to.

Interviewer: So, have you no criticisms of the way the restaurant scene is developing in Britain?

Kevin Langtree: I have. The scene I described earlier, of eating different food every night of the
week, is actually quite difficult unless you have a car and aren't bothered by the inevitable hassle
over parking. Why is it that all the curry houses crowd together like a wagon train under assault
by a war-party? It's the same with the Chinese restaurants. They're all together, several blocks of
them, and they're all selling the same thing. I'd like to take all the restaurants and fast-food
outlets in London and re-arrange them at random.

Interviewer: There are those who suggest that some very famous fast-food outlets, I won't
mention them by name (they're all American), are set to take over the world. What do you think?

Kevin Langtree: The answer is that they're simply not. They do what they do very well —
making very cheap meals which appeal to kids. But the kids grow up. It's not cool to be seen
there once you reach a certain age. They want something different. So, I welcome the Americans
just as I welcome everyone else. The greater the variety, the better.

Interviewer: Do you see any connection between our widening taste in food and us having, with
one or two exceptions, a more tolerant society than in the past?

Kevin Langtree: Oh, I'm sure that's so. I mean, just as an example,. it's difficult to feel anything
other than affection for the French while you're enjoying a hot baguette and a glass of Burgundy,
isn't it? And I'm sure that we must be more aware of other countries nowadays. We're more
cosmopolitan. After all, you go into any supermarket now and there it is, the world's
produce. In the past there were only the root vegetables of our cold, windswept island, and
that was it.

Interviewer: Well, it's certainly true that more and more people are willing to try more and
more things than in the past. I suppose that must make us less xenophobic. Now, I've just
got one last thing to ask you. It's something of a favour really. I've just had our producer on the
headset. She wants to know if you can possibly...
Exercise 25.

You will hear a discussion in which two historians, Matt Thomas and Sue Wilkins, talk
about a book they have written. For questions 16-20, choose the answer (A, B, C or D)
which fits best according to what you hear.

16 Sue says that


A until recently amateur archaeologists weren't allowed to dig on the Thames foreshore.
B official authorisation is needed to excavate along the river.
C any finds must be reported to the Port of London Authority.
D she had to be a member of a society to dig under any part of the Thames.

17 Matt says that, in the past, poverty-stricken children


A would hunt for birds along the river banks.
B used to try and sell things to people walking along the river.
C would burn things they found by the river to keep warm.
D discovered things in the mud at certain times of the day.

18 How did Sue use to feel about the Thames?


A nostalgic because she visited the river with her parents
B excited about finding rare jewellery and other treasures
C repulsed by the appearance of the water
D revolted by the thought that she would excavate in mud one day

19 Matt is intrigued by
A what can be learnt from the artefacts they find.
B the transport people used in the Middle Ages.
C how London's inhabitants used to dispose of their rubbish.
D the large expanses of mud which are exposed when the tide goes out.
20 What makes the discovery of the small-scale items so important?
A their surprising ornamental use
B what they reveal about family relationships
C the fact that the mud has conserved them remarkably well
D the fact that they confirm a widely held theory

16. B

17. D

18. C

19. A

20. B

Interviewer: This week, we're talking to historians Matt Thomas and Sue Wilkins, who've
recently published their book, Treasures of the Thames. Why did you and Matt decide to work
together on this book, Sue?

Sue: Well, I got to know Matt shortly after I became interested in the Thames and what a
treasure trove it is. As many of your listeners will know, the Thames is a tidal river. Twice a day,
when the tide goes out, large areas of mud are uncovered. In order to dig in these areas, called
the river's foreshore, you must have a permit from the Port of London Authority. In fact,
the only people allowed to dig along certain stretches are a group of amateur treasure hunters
called the Society of Thames Mudlarks, so I became a member, and that's where I met Matt.

Matt: I'd better explain to listeners that the word `mudlark', and we're not talking about the bird
here, originally referred to the youngsters who would scavenge in the mud along the sides of
the river in the 18th and 19th centuries. They'd search at low tide for anything that they
could sell, like pieces of metal, old rope or coal, for example. Nowadays, our members comb
the Thames foreshore, armed with metal detectors, and wearing overalls, gloves and rubber
boots.

Sue: Of course the river was very polluted years ago, and it can still be quite dangerous if you
don't know what you're doing. Matt has a lot more experience than I have in the field...

Matt: And Sue's a great writer, so we decided to join forces. By the way, the word 'treasures' in
the title of the book doesn't refer to gold, silver or precious stones, which are seldom found.
Treasures, true treasures, are the things that we find which teach us about the people who lived in
London in the past.

Sue: That's right, but valuable artefacts like gold necklaces or rings with precious stones in them
do occasionally turn up. Of course, the society has to report all historical finds to the Museum of
London... To be quite honest, when I was little and visited London with my parents, I'd look
at the brown water of the Thames and the muddy foreshore and think, 'How disgusting!' I
certainly wouldn't have dreamt of rummaging around in that filthy mud to look for
archaeological artefacts. Of course, the Thames is still an awful colour most days, but it's not as
dirty as it used to be.

Matt: No ... And the thrill of discovering rare or really old objects helps us to overcome any
squeamishness we might have ... What we're really interested in are artefacts that tell us about
how people lived hundreds of years ago. The Thames flows through the very heart of London
and, in the past, people relied on boats to travel up and down the river, and even across it.
Accidents would happen and things would fall overboard; of course, people's rubbish would end
up in the Thames, too. As much as seven metres of foreshore is exposed at low tide, and this is
actually one of Britain's most important archaeological sites.

Sue: Something we've covered in the book, and which I find very interesting, are small objects
believed to be mediaeval toys. The 'Mudlarks' have found tiny cannons, metal figures and
miniature household objects like jugs or stools made of pewter...

Matt: That's an alloy of tin and lead, by the way.

Sue: That's right, and ... and some of these objects date back to the 13th century. They're very
rare, actually, because pewter corrodes quickly under normal conditions, but the thick mud of the
Thames prevents corrosion and helps to preserve them. Some of these items are so beautifully
made, they may even have been ornaments. But experts are sure that a lot of them were toys.

Matt: This overturns some theories which suggest that people didn't get very attached to their
children in the Middle Ages because they had so many and because a lot of them died.

Sue: These discoveries suggest that, back then, parents cared just as much about their children as
we do now, and went out of their way to provide them with toys. And we're finding these toys
next to the River Thames, hundreds of years later.

Interviewer: Well, I'm sure... [FADE] [pause 10 seconds] Now you will hear Part 3 again. [repeat
part 3] [pause 5 seconds] That is the end of Part 3.
Exercise 26

You will hear part of a discussion programme where Florence, a marketing expert, and
Mark, a retail analyst, discuss impulse buying. For questions 1-5, choose the answer (a, b, c
d) which fits hest according to what you hear.

1. What does Mark imply when talking about the items made people purchase through
impulse shopping?
a. Many expensive items are often purchased in this way.
b. Impulse shopping can result in consumers buying unnecessary items.
c. Most impulse buys take place in clothes shops.
d. People very often purchase items that they cannot afford.

2. What point do both Mark and Florence make about the retail industry?
a. It actively encourages impulse-buying behaviours.
b. It maximises its profits by offering promotions on expensive items.
c. It has conducted extensive research into influencing people's way of thinking.
d. It often presents products of lesser quality as a good deal.

3. What do Mark and Florence agree has made impulse buying easier?
a. more disposable income
b. the availability of cheaper products
c. a wider variety of payment methods
d. advances in technology

4. When describing the relationship between stress and shopping, Florence says that
a. shopping may help to briefly reduce stress levels.
b. all impulse buys are done when the consumer is stressed.
c. the act of shopping can be stressful in itself.
d. consumers shop online to avoid stress.
5. What advice does Mark have for anyone wanting to curb their spending habits?
a. avoid the shops altogether
b. be aware that emotions guide purchasing decision
c. delay making a purchase
d. set a monthly budget for one’s spending.

1. B

2. A

3. D

4. A

5. C

Host: I'd like to welcome our two guests to the programme, marketing expert, Florence Jones,
along with retail analyst, Mark Collins. Both guests are here this evening to talk about impulse
buying. Mark, maybe you can begin by explaining to our viewers what exactly an impulse buy
is.

Mark: Well, simply put, an impulse buy is any purchase that a consumer makes which was either
unplanned or unintended. I think most people have experienced a time when they spontaneously
bought an item that was not originally on their shopping list when they left the house that day.
it's an all-too-familiar scenario, wouldn't you say, heading out to buy something specific, say a
new tie for work, and coming back with other random items, such as a new pair of trainers,
even though you seldom jog?

Floreroce: Yes, and we Brits are some of the most impulsive shoppers around. Admittedly, it's
something we see much less of with the more costly items, such as a car, where consumers are
more inclined to do some research before buying and do spend time thinking through their
purchasing decision before parting with their money. However, when it comes to items such as
clothes and cosmetics, DVDs, confectionery and all the various supermarket items, impulse
shopping Is a very common consumer behaviour. I think many people - if they actually took the
time to jot down all the impulse buys they'd made during a month - would be stunned at what
this amounts to.

Host: And what exactly is it that motivates consumers to shop in this way?
Florence: Well there's a whole list of possible reasons. One of the biggest consumer motivations
relates to the desire to actually save money. The retail industry knows only too well that the
price-conscious consumer is attracted to bargains and so it bombards shoppers with promotional
offers and reduced prices. Remember, as a profit-driven industry, the retailers' sole aim is to
maximise their profits. And they have as many strategies up their sleeves to persuade
consumers to part with their cash as there are items for consumers to spend their cash on.

Mark: And it certainly works, doesn't it? Shoppers walk into a department store only to be
greeted with eye-catching signs that read, 'Sale; Limited Time Only' or `Mid-Season Sale Ends
Today'. This has quite a big impact on consumers. Not wanting to miss out on what they perceive
to be a good deal, they make that unintended purchase, leaving with more items in their shopping
bags and less money in their wallets. It's through years of research into consumer behaviour that
the retail industry has come to understand the consumer mentality and, based on this, they have
developed a vast multitude of effective selling techniques.

Florence: At the same time, I don't think we can ignore the huge influence that online
shopping and smartphone devices have had on impulse buying. It has never been easier for
people to spend their disposable income, as the Internet allows us to shop at any time of the day
or night, and from any setting.

Mark: That's for sure. In fact, the credit card details for regular website customers are often
already stored in the system; in just a click of a button, the purchase is made. It is so effortless
that the consumer hardly has time to process their decision to buy something, let alone reconsider
or change their mind.

Florence: And let us not forget that aside from retailers, there are also some pretty strong internal
triggers that compel people to impulse shop. Researchers exploring consumer psychology
suggest that some of us might be more inclined to impulse shop as a way of alleviating our stress
levels. In other words, when a person is stressed, they shop more. And, momentarily at least,
this seems to offer a remedy for their stress. It is hardly surprising that consumers go on to
repeat this behaviour. Also, it just plain feels good to buy something new. This is especially so,
given the materialistic society we live in. People want to acquire, and take pleasure in acquiring,
more and more possessions.

Mark: But then this gratification subsides when a consumer realises that their spontaneous
spending has spiralled out of control. After that, it's extremely easy for someone to experience
regret, family disagreements, and all sorts of financial difficulties.

Host: And what advice would you give to consumers wishing to limit their spending?

Mark: It might sound obvious, but to avoid a situation where one ends up in debt because of
extreme spending, consumers must try to resist acting on these impulses. One way to do that is
to postpone buying something for a while. Even twenty-four hours later, a person may find
that they've had a change of heart, and that the urge to own that particular item has subsided.

Florence: I find it equally important to steer clear of the shops when you are feeling down. I
think if consumers were...

Exercise 27.

You will hear part of a discussion in which two people, Gillian and Angus, are discussing
the process of ageing after attending a talk on the subject. For questions 1-5, choose the
answer (a, b, c or d) which fits best according to what you hear.

1. When talking about the lecture, Gillian and Angus agree that
a. parts of the presentation were confusing.
b. the lecture gave them a different perspective on ageing.
c. the presentation was too focused on neuroscience.
d. the lecturer was very knowledgeable on the subject of ageing.

2. According to the discussion, what is true about the front part of the brain?
a. It undergoes significant development during our late twenties.
b. It is usually fully developed in early childhood.
c. It is in a process of development during adolescence.
d. It is the largest and most complex part of the brain.

3. What does Gillian imply about emotions?


a.The fact that teenagers may display excessive emotions can be justified.
b. Adults are better at recognising other people's emotions.
c. As teenagers become adults, their emotions follow a standard pattern.
d. Certain emotions are harder to control as we grow older.

4. What point is made when Gillian and Angus discuss the concept of maturity?
a. Biological factors are wholly responsible for a teenager's maturity.
b. Environmental factors influence maturity.
c. Some teenagers are more mature than others.
d. A teenager's level of maturity can be fully explained by science.
5. According to Gillian and Angus, what is one advantage of being an older adult?
a. greater feelings of self-worth
b. more wisdom
c. enhanced creativity
d. increased feelings of peace

1. B
2. C
3. A
4. B
5. D

Gillian: When you think about it, ageing is probably one of the most fundamental things about
being human. We all experience it - and it starts from the moment we're born.
Angus: Firm! And I don't know about you but, before today's lecture, I'd never given much
consideration to how a person's character changes with age. I've always associated ageing
with the more visible or physical signs of getting older.
Gillian: Same here, but it seems that the emotional and behavioural changes that take place
throughout a person's life are equally significant - especially, as we heard, during the period
when we move from adolescence into adulthood. What I find amazing is how much research
is being done on the brain nowadays to better understand these changes.
Angus: Yes. The lecturer did present loads of recent studies, some of which were a little
technical though; some background knowledge in the neurosciences would have come in handy.
What I hadn't realised is how much the brain is still developing and changing during
adolescence, and how this influences our behaviour as teenagers.
Gillian: Well, from what I understood, it's the... what was it called, prefrontal cortex... where
much of this change is taking place - the frontal area of the brain. It's fascinating to think that,
while around ninety-five per cent of our brain is in fact formed by the age of six, the brain may
not be fully developed until we're well into our mid-twenties. And that remaining five per cent
seems to develop mostly during our teenage years. I never knew that during this period,
neural connections within the brain are still being formed.
Angus: This does help us to shed light on why, as we get older and leave those years behind us,
we do seem to get better at, for example, planning and making decisions, and other tasks that this
part of the brain is thought to be responsible for.
Gillian: Precisely. We see a similar pattern with our emotions. I don't think anyone disagrees that
as we get older we're better at responding to situations with the appropriate level of emotion, so
we don't overreact or get overly emotional when things happen - we can control our feelings
more effectively. And it was explained that the way the teenage brain is formed and
develops often means that it's more likely for teenagers to be impulsive and risk-takers
compared to, say, their parents. Imagine, as teenagers, we're totally unaware of this huge amount
of change that's taking place within our brain.
Angus: Hmm, I suppose it goes to show that part of becoming a mature adult lies in changes
occurring in the brain during adolescence.
Gillian: To a degree, yes. Although, I'm not so sure that this can be fully explained by biological
changes alone. What about the role of our experiences in helping us to mature? Reducing
everything down to a biological level is a rather limited view.
Angus: You're right. Thanks to all our many life experiences, we do get wiser with age. We
make mistakes and we learn from them. Experience definitely influences the way we
perceive events, the way we behave and, ultimately, how we mature.
Gillian: From this perspective, getting older certainly has its value.
Angus: I'd say. And I imagine age brings with it far less stress too. I mean, look at our parents or
other people who have reached their fifties or sixties. They have managed to overcome some of
life's major challenges and achieved at least some of their life goals. The things that were once
the source of their anxiety are often eliminated by the time they reach that age.
Gillian: I find that one of the most challenging aspects of being a young adult is exactly this: all
the anxiety you have over the many, many things that you feel you must achieve in life, such as
having a successful career. I suppose with age, a more relaxed approach can be reached - a
person's view of the world and what is considered important certainly changes.
Exercise 28.
You will hear an interview with two experts on technology and learning. For questions 1-6,
choose the answer (A, B, C or D) which fits best according to what you hear.

1. What does Michael say about online learning?


A It appeals to a greater variety of learning styles.
B It enables learning to suit the needs of the individual.
C It may one day replace the traditional school system.
D It allows us to choose the subjects we study.

2. When discussing the Khan Academy, Michael and Helen agree that it
A focuses only on mastering a specific subject, like maths.
B offers a generous amount of teaching and practice.
C has brought education to huge numbers of people.
D is less effective in generating original thought.

3. TED serves as an example of a site


A whose speakers provide amusing, factual talks.
B whose original focus has changed over time.
C which offers extended lectures in various subjects.
D which presents the same speakers every year.

4. Both Michael and Helen agree that schools


A are too rigid in the choice of subjects taught.
B are not tolerant of independent thinkers.
C educate children for the present, not the future.
D underestimate the importance of failure in learning.

5. What conclusions about gaming do Michael and Helen reach?


A It provides relief from difficult everyday issues.
B It generates cooperation between participants.
C It can be a huge source of knowledge.
D Its competitive aspect can become addictive.

6. When discussing the potential of gaming, Helen reveals that she is


A surprised at the abilities it develops in gamers.
B excited about the endless possibilities.
C doubtful about our ability to see the bigger picture.
D concerned about the time gamers spend online

1. B
2. D
3. B
4. D
5. B
6. B
Interviewer: Today, I welcome two experts on technology and learning: Michael Bale and Helen
Brown. Good (Woman) morning to you both. Michael, how is technology influencing learning?

Michael: Well, one of the things technology has given us is the ability to share huge amounts of
knowledge online. The Khan Academy, for example, is an online school where you can take
courses for free. It doesn't restrict us to traditional classrooms. Students can learn at their
own pace at home. Having said that, the videos often assume prior knowledge of a subject, and
visual learning doesn't suit every student's learning style.

Interviewer: Could the Khan Academy potentially replace the traditional school?

Michael: It's doubtful it was ever meant to. Khan provides learning for a range of people, not just
students. But for a novice, the sheer volume of practice material can be overwhelming. Khan was
definitely a quantum leap in making learning accessible to everyone — even those who can't go
to school. But I doubt that it allows for the creativity which comes from interacting face-to-
face with other students and teachers.

Interviewer: What do you think, Helen?

Helen: Khan certainly instructs very effectively, but education isn't just the acquisition of
knowledge — it's questioning, evaluating ways of thinking and being open to new ideas and
problem-solving. One site that celebrates this is TED. You've probably heard of it. Interviewer
Yes, but could you give us an overview for our listeners? Helen: Certainly. TED is a non-profit
organization. Its motto is Ideas worth spreading. It offers short, online presentations by people
who are very inspiring for free. TED started in 1984, but really got off the ground in 1990, as
a conference covering three areas: Technology, Entertainment and Design, hence the name.
Later, TED expanded into other fields of interest to cover the full spectrum of topics:
science, art, business, poetry ... you name it!

Interviewer: OK, so how are presentations chosen?

Michael: Well, speakers have 6-18 minutes to present their ideas. Google the top 20 TED talks
and certain names come up time and again. Sir Ken Robinson is one of them. His highly amusing
talk focused on whether schools are killing creativity by expecting every child to learn the same
subjects in the same way. He believes schools must undergo a transformation to better prepare
children for a rapidly evolving world. An integral part of this, he says, is accepting that creativity
is a fundamental quality of all children, and that we must encourage it because the children of
today will be the designers and educators of tomorrow.

Interviewer: But we have no idea how the future will look. So how do you educate for that,

Helen? Helen: Well, for a start, you don't restrict creativity. Another point, which resonates with
many of us, is that schools instill a fear of making a mistake in pupils. And we have to get
away from that because an inventor who's afraid to get it wrong will never invent anything. The
road to success is paved with mistakes, Robinson says.

Interviewer: People obviously agree — his talk has been viewed 39 million times!

Michael: Exactly. And that fear of error has a knock-on effect as children grow. We seem to
educate them out of trying to be creative. And speaking of creativity, in another TED talk, Jane
McGonigal reflects on the power of gaming to change the world. Now I know some people
would say garners are just escaping into an imaginary world to avoid dealing with reality ... or
they're obsessed. But it's hard to dismiss what she says about thinking outside the box.

Interviewer: But what's the connection between gaming and changing the world? You've lost me
there.

Michael: Well, in her TED talk, McGonigal describes what happened in the kingdom of Lydia
(or Maeonia) in ancient times. Apparently, there was a terrible famine in the kingdom and people
were starving, so King Atys improvised a plan — people ate one day and played dice games the
next to divert their attention from hunger. In this way, they survived for 18 years, and according
to Herodotus, dice games were invented in Lydia. So what you have is a large number of
people finding a creative solution to the problem of being hungry by collaborating and
playing games.

Interviewer: OK, but do gamers today collaborate?

Michael: I think the figures speak for themselves. One popular game, World of Warcraft,
has millions of players in over 230 countries who collaborate in a variety of ways.

Interviewer: So do we underestimate the potential of gaming for learning?


Helen: Totally. Through gaming, we become better problem-solvers. Garners are optimistic
about winning, they concentrate intensely for hours, pool their knowledge and seem to enjoy
being part of an 'epic win" — working for something greater than yourself, like saving a world!
Applying players' abilities to world problems in a game could generate creative solutions.
The sky's the limit!

Interviewer: And there we must leave it. Thank you both.

Exercise 29.

Listen to an interview in which two trendspotters talk about their work. For questions 1-6,
choose the answer (A, B, C or D) which fits best according to what you hear.

1. What does Liz say about her career change?


A She had always been fascinated by youth-led trends.
B She moved as a result of something she had read.
C She wanted to improve her knowledge of design.
D She had no idea it was so complex a field.

2. What contrast is highlighted between macro and micro trends?


A Micro trends are influenced by current issues.
B Macro trends are of greater interest to high street retailers.
C Micro trends can occasionally outlast macro trends.
D Macro trends are harder to identify.

3. When Liz initially started trendspotting, she


A wasted too much time browsing - the Internet.
B photographed anything that triggered an idea.
C found street fashion less creative than today.
D made errors about what to photograph.

4. When asked about his work, Josh reveals that


A he prefers being freelance to agency work.
B his image on social media is one of his top priorities.
C only a small part of his day involves looking for ideas.
D trendspotting online requires different skills.

5. Liz and Josh agree a qualification in marketing is


A unnecessary to succeed as a trendspotter.
B valuable as a foundation for this career.
C helpful only if it covers consumer psychology.
D advisable as clients prefer qualified forecasters.

6. Liz and Josh both say that their work


A is instinctive by nature.
B requires knowledge of a specific subject.
C needs to be constantly updated.
D demands a good sense of timing.

1. B
2. D
3. D
4. B
5. A
6. A

Interviewer: Today, we're looking at careers related to marketing. My guests, Liz Bryant and
Josh Ramsey, are both trendspotters, who identify new trends in fashion and culture. They help
companies produce new products that will be both innovative and popular. Liz, you used to work
as a designer, right?
Liz: Yes, I worked in fashion design for five years, and I was always being told I had a good eye
for trends. Then one day, I came across a report on trends in youth culture published by a
global forecasting agency. It was fascinating; and since a grasp of up-and-coming styles is
paramount in trendspotting, I just knew it would suit me down to the ground as a career .
Trendspotting in fashion isn't just a question of "turquoise is in" this season and "grey is out".
You've got to tap into consumer tastes to learn why people think certain products are cool —
why some trends take off and others don't. It means being observant about micro and macro
trends.
Interviewer: Josh, can you tell us more about these trends?
Josh: Well, we make two types of predictions; the first is short-term and relates to micro trends
that may last only a year or two. In fashion, it's based on today's style on the runway, what's
being worn on the Streets of London or New York, the hottest Instagram images meaning trends
that people follow. The ideas are sold to high street stores, and they quickly appear in shop
windows. The second type of prediction has to do with macro trends — you know — long
term, more lasting changes in tastes. This is what trendspotters usually find most
challenging — it requires in-depth investigation and discussions with experts from a
variety of fields. We spot trends in architecture, communications, food, technology and lifestyle.
Look at the growth of the home espresso machine. Technology made them kitchen-friendly and
it's revolutionized how we drink our coffee. So our job is to second-guess how these changes will
affect our taste in consumer goods because these macro trends may be with us for years to come.
Interviewer: Now, Liz, where do you find your ideas?
Liz: I check hundreds of sites online, but it's easy to lose track of time and get bogged down, so I
go to shows and exhibits too. I always take a camera to record people, objects, colours ...
anything that triggers a new idea. I find my inspiration on the street, but it took me a while to
get a handle on the kind of style to target. Eventually, I learned to ignore "looks" already
there and avoid anything too off the wall or too quirky.
Interviewer: OK. So Josh, do you observe the public too?
Josh: Less so than Liz and less than I used to at the agency. I'm operating freelance from home
now and my work is mainly online. New clients find me through networking sites, so I
dedicate the first few hours of the day to maintaining my online presence. The rest is spent
answering emails, texting, chatting to clients and scanning blogs and images for inspiration. But
whether you're on the street or in front of a screen, identifying trends requires constant
observation.
Interviewer: Here's a question for both of you. What qualifications does a trendspotter need?
Josh: A degree in marketing isn't essential but some background is undeniably useful, and a
knowledge of psychology can help with predicting consumer behaviour.
Liz: Some trendspotters are qualified only in design, but training in advertising can be invaluable
and really, give you an edge when talking with companies, I think.
Interviewer: And finally, what makes a good trendspotter? Which skills are most important?
Liz: Broad interests in art, design, science, technology ... . Actual forecasting is a kind of sixth
sense, though. You can instinctively feel a trend developing on the street, and later, you see
it on the runway. Of course, you then need to bring it all together into a clear idea and get it to
your client and the market asap.
Josh: Yes, I think, at the root of it is intuition and that can't necessarily be taught . A
knowledge of design history goes a long way too — every trend is rooted somewhere in the past.
So while you're looking forward, you can't dismiss the past. I'm often surprised to see how past
styles influence current design. In time, fashion comes full circle, reinventing itself, but with a
modern twist.
Interviewer: Thank you both for coming today.
Exercise 30.

You will hear part of an interview with Miriam Baker, a psychologist, about how social
media has changed self-expression. For questions 1-5, choose the answer (A, B, C or D)
which fits best according to what you hear.

1 According to Miriam, what is the main reason people feel the need to create perfect
images of their lives on social media?
A They are hoping to attract more friends.
B They want their lives to seem more exciting.
C They are in need of validation.
D They have very unhappy lives.

2 In Miriam's view, the average user of social media is


A driven and only interested in self-gratification.
B lonely and looking for meaningful connections.
C curious and concerned with their surroundings.
D volatile and confused about what they want.

3 When discussing her own experience with social media, Miriam reveals
A her unexpected pleasure at the response.
B her desire to engage with it more.
C her worry of becoming addicted.
D her anxiety of not being accepted.

4 Miriam compares media in the past with social media today to


A highlight the reduction in consumption.
B focus on the differing subject matter.
C explain the difference in the audience.
D illustrate the variety of content.

5 What advice does Miriam give about combating `Digital Narcissism'?


A Be true to who you really are.
B Try to post more meaningful content.
C Focus on others and not yourself.
D Do things that increase your self-esteem

1C

2D

3A

4B

5A

Interviewer Welcome back to the show. With me is Miriam Baker, a psychologist specialising in
internet use and behaviour. Miriam, moving on, I'd like to talk about this explosion of selfies and
self-adoring posts documenting almost everything we do in life on social media. What Is this all
about?

Miriam: I'm so glad you brought this up, We are definitely seeing a large volume of content from
users about themselves. They thrive on how many likes they get. Social media is the perfect
platform for bragging and showing off. It's effortless and even celebrated. But users don't post
everything about themselves.

Interviewer: Well, it sure seems that way sometimes.

Miriam: I know, but seldom do we see the routine and boring parts of people's lives, like
cleaning up the house or paying their bills. Users portray only the interesting and fun stuff. They
even sometimes go so far as to stage content and posts that make their lives seem perfect; all in
the pursuit of likes. It's the never-ending search for confirmation that they are worthy.
Overall, they may have good lives with friends and so on but they are looking for
reinforcement from others. This unlimited 'look at me, look at me' content is what we
psychologists call digital narcissism and it's on the rise.

Interviewer: So what's wrong? Why are we so self-obsessed?

Miriam: It's the nature of the platform until it evolves into something more and it slowly will.
Think about it; It provides endless opportunities to focus on oneself and less on others and what's
going on in the world. This will definitely attract narcissists or bring out our narcissistic
tendencies. We are more connected than ever but less interested in other people. That said, most
social media users are not narcissistic necessarily but they tend to be unsure in what they
are looking for from the medium, causing sometimes unpredictable online behaviour.

Interviewer: I can relate to that. Sometimes I spend time scrolling and clicking on posts for no
real reason except that I'm bored.

Miriam: We all know that feeling. When I joined a social network some years back, I never
thought I would post much but when I did and got positive feedback, it was great. After some
time, though, I realised that in order for that to last I needed to post more. I could see how one
could get addicted or feel anxious about their posts. These are natural reactions but it's when we
take it to another level and start obsessing about how we are perceived that we are in danger of
becoming digital narcissists.

Interviewer: So, it's really a digital phenomenon?

Miriam: Basically, yes. Before social media, TV provided an escape from reality by giving us a
glimpse into the lives of fictional characters on TV shows and films. Then came reality TV,
turning everyday people into stars. As. online video and the first social media sites started
popping up, consumers became the content themselves. We are still spending the same amount
of time consuming and there is still lots to consume but what we are looking at has changed
considerably.

Interviewer: Well, we all know one of these narcissists or may even be one ourselves. Is there an
end to this online behaviour?

Miriam: You might expect me to say work on raising your self-esteem and that may help but the
real issue at heart here is: are you representing yourself truthfully? I say this often to all forms of
online engagement it’s important to focus on authenticity and just being yourself. It’s ok to
share your passions with the world but always keep in mind your real self. Seek out others with
similar interests and create more in-depth connections.

Interviewer: That makes sense to me. Can you tell me about other forms of online.
CHAPTER 3: CAE- CPE GAP FILLING

Exercise 1:

What happens in the Amazon has a 1)  ________________ on the planet as a whole.

In the 10 years up to 2009, over a thousand 2) ________________ of plants and animals were


discovered.

The plants and animals are in danger because the Amazon's 3) ________________ are at risk.

The region is using its vital resources to place itself in the 4) ________________.

The 5) ________________ of allowing even a small percentage of the carbon to escape would be
disastrous.

6) ________________, thanks to plans put forward by the government, has resulted in forest
clearances.

The demand for livestock means farming offers substantial 7) ________________.

Activity to obtain 8) ________________  and other natural resources are also problematic.

To safeguard the water requirements of farming, the 9) ________________  needs to be


protected
Hi, my name's Charlie Reid and I'm going to talk to you today about the Amazon rainforest and
what it means to each and every one of us. Now, you might be thinking, 'The Amazon? That's
miles away, how does that have anything to do with me?' Well, let me tell you that what goes on
there has a (1) massive impact everywhere around the globe and we should all be paying much
closer attention to the effect of what is happening to it.
The Amazon is a vast biome that contains the largest remaining tropical rainforest in the world,
home to at least 10% of the known species on Earth, some of which are endemic to the area. In
the decade between 1999 and 2009 alone, 1200 (2) new species of plants and animals were
identified. This variety of flora and fauna depend on the (3) ecosystems that exist amongst the
forests of the Amazon and they are under threat just as all the forests around the planet.
The threat comes, in essence, from globalisation and its resulting effect on the exploitation of
resources. In the Amazon, these resources are proving vital in the establishment of the region in
the (4) global market, but at what cost? In a word, that cost is deforestation, which is very bad
news. The Amazon contains between 90 and 140 billion metric tons of carbon. The exploitation
of the area over the last 50 years has already led to a loss of at least 17% of the forest. Now as
most of you know, we have a severe problem at the moment with CO 2 levels causing climate
change and the release of even a small fraction of the carbon in the Amazon through
deforestation could have devastating (5) consequences planetwide. Added to that is the loss of
photosynthesis, or the conversion of existing CO2 into oxygen, that would have been carried out
by these trees.
The reasons behind current levels of deforestation are wideranging. A number of government
initiatives in the region have been promoting (6) rapid development, which has led to hasty
action whereby vast areas of forest have been cleared for building material and to create land for
pasture or crops. These actions have been carried out in order to facilitate foreign investment in
the area. However, there is a lack of sustainable development and resource protection which is a
direct result of an absence of proper frameworks which need to be in place to implement such
procedures. Coupled with an inability of regional institutions to impose existing regulations to
protect the area, the results have been quite destructive. There is also a high level of poverty
experienced by the peoples who inhabit the Amazon which has only exacerbated the issue.
The (7) financial rewards of raising cattle on the land, due to the international demand for their
products, has led to many locals increasing their farmland at the expense of the forest. People
have to make ends meet, which is understandable, but it must be done in a sustainable manner. In
addition to the expanding farmland, new infrastructure such as roads, buildings and irrigation
measures have left their mark and operations to extract (8) minerals and other resources from
the land have contributed to the situation we find ourselves in today.
The effects cannot be overstated. Already we have seen a loss of biodiversity and some species
are threatened with extinction. Habitats have been eroded and a number of species are in terminal
decline. The loss of (9) forest cover has also had a negative impact on the water cycle of the
region. Vapour from the rainforest spreads over a great distance and is relied upon by agriculture
in other parts of the surrounding area. The loss of this rainforest vapour could result in drought
and have major environmental ramifications. The knock-on effect of all this is that the people of
the Amazon are suffering due to a reduction in natural resources. Approximately 350 different
ethnic groups call the area their home and they are struggling to cope with the damage that is
being done.
So, what can we do about all this? Well, firstly ...

Exercise 2:

You will hear part of a talk about community regeneration by a community liaison
officer called Dolores O’Reilly. For questions 1-9, complete the sentences with a
word or short phrase.

Deterioration of living standards in some areas has led to a drop in people’s levels of 1)
________________.

Not dealing with all the issues can result in the 2) ________________ not working.

Emphasis should be placed on the 3) ________________ in the area that need help.

Previously, the authorities looked for 4) ________________ to give the community a


boost.

The 5) ________________ of local people throughout the project is of the utmost


importance.

Looking to communities for solutions can reveal people’s hidden 6) ________________.

It is necessary to create a(n) 7) ________________ to support the process.

In order to facilitate equality, both parties involved in the process require 8)


________________.

Dolores felt 9) ________________ of what her liaison group achieved.


1. self-esteem
2. regeneration
3. local neighbourhoods 
4. corporate investment
5. involvement
6. talent
7. solid foundation
8. training
9. proud
Hi, my name’s Dolores O’Reilly and I’m going to talk to you today about community
regeneration. I’m a community liaison officer with Bramley city council and I was recently
involved in a regeneration scheme in my area.
A number of towns and cities around the country have experienced problems of decline in
certain areas with housing estates suffering high levels of poverty, unemployment, low
educational achievement, poor access to labour markets, crime and vandalism. The result has
been a decline in self-esteem in the residents in these areas which has only perpetuated the
problem.
It’s a vicious cycle. The area becomes run down and people lose hope and through their lack of
hope they allow the area to degrade even further. This cycle must be broken if these areas are to
flourish once more. There are a number of issues which need to be dealt with in order to achieve
this but take note, all of the issues must be addressed.
Neglecting even one of them has been shown to derail the process of regeneration. It therefore
needs to be tackled using a holistic approach at both local and national level to ensure an
effective framework is provided which has a clear focus. This focus should be on the local
neighbourhoods requiring attention.
In the past, projects were property-led meaning that the authorities believed that if an area looked
better, it would attract more corporate investment and the benefits of that would ‘trickle down’
to the wider community. Now we know that the more effective approach is to target the
problems in the community itself. If you don’t deal with them, no amount of outside investment
will work. Money can’t fix social exclusion and that’s a big problem in these areas.
The first step in this approach is to identify and understand the problems faced by the
neighbourhood and to highlight its assets. These assets should be ones that can be harnessed to
bring about necessary changes. At all stages local involvement is paramount, but no more so
than in this preparatory stage. It’s the locals that know the area and its history best. Neglect of
the areas and their residents has allowed problems to grow and multiply therefore, in order to
address any of the problems, you must get the community on board and motivated. In other
projects around the country, working with the locals has led to the discovery of a wealth of
untapped talent which proved instrumental in the success of these projects.
Once you have identified the areas which need the most attention you must work with the
community to establish a solid foundation to work from. After the preparatory stage the
community as a whole has to be informed of what is going on. Getting them involved late or in a
half-hearted manner will damage the program possibly beyond repair. This is vitally important
considering that community regeneration is a long process and the constant factor throughout is
the community. If you don’t have them on board, it simply won’t work.
Once they are involved they need to work closely with professionals and as such both parties
need to be supported with training. This enables the residents and professionals alike to play an
equal part in the process. This hopefully will lead to a productive partnership and positive
outcomes. One way to do this is to form a community liaison group which consults residents at
every stage. This was where I was involved in the project in my area. We presented every issue
to the community, had regular meetings where issues were discussed with community members
before going to the council and introduced a small grants fund which the liaison group were
directly responsible for.
These factors were especially important because they gave the community members a feeling of
control. And that’s something they felt had been taken away from them. That they were
somehow left behind, through no fault of their own, with no recourse. To succeed in life you
need to have a certain level of control over what happens to you and when it’s gone, it can have
devastating consequences. When I saw the results of our work in the liaison group, it made me
proud to see how the community rose to the challenge. It was quite inspiring.

Exercise 3.

You will hear a student, Sophie Reece, giving a short talk about a conservation
project. For questions 1-9, complete the sentences with a word or short phrase.

The state of the planet is the result of people’s need to make 1) ________________.

Sophie doesn’t want to delve into the subject of 2) ________________.

The volunteers offer the group whatever 3) ________________ they can spare.

The generosity of the public allows the group to take care of their 4) ________________.

Recently the group have been working on a(n) 5) ________________ involving local


businesses.

The group discovered that 6) ________________ in the way companies conduct


themselves can cut down on their environmental impact.

After the group left, the publishing company were almost a(n) 7) ________________ .

Volunteering to help the environment can help alleviate one’s 8) ________________.

Sophie believes everyone’s 9) ________________ is to do something positive.


1. progress

2. vehicular pollution

3. time

4. administration costs

5. scheme

6. minor (small) adjustments

7. paperless office

8. conscience

9. desire

My name’s Sophie Reece and I’d like to talk to you today about a conservation group that I’m
involved in. I work with a group of like-minded people who believe we owe a debt to nature for
everything mankind has done over the years in the name of progress, which has caused a lot of
damage along the way. I’m of course talking about the many different ways in which we pollute
this planet of ours through industry, ignorance and our negligent behaviour.

Anyone who has left the tap running while cleaning their teeth, left a light on in an empty room,
just thrown a plastic bottle away instead of recycling it or left the TV on standby has contributed
to the state that the planet is in today, and I’m not even going to talk about the impact caused by
the millions of cars on our roads. Vehicular pollution is too big a topic to tackle today. The
bottom line is that, any way you look at it, we’re all to blame and my group wants to make
amends.

I joined the group two and a half years ago. I was only planning to volunteer for a short period,
but the more I did, the more I wanted to carry on. Everyone in the group is a volunteer, and they
give as much time as they can. It’s a non-profit organisation but we still have to raise funds.
We’ve carried out a few fundraisers in my time there but mostly we rely on donations to cover
administration costs.
We’ve already completed a number of projects over the last couple of years. We’ve organised
clean-up days, recycling drives, tree plantings and much more but we wanted to go further so we
created a scheme to assist companies in the area to reduce their impact on the environment.

We go into participating businesses and work with them to identify areas where, by making
minor adjustments to working practices, they can make a difference. And sometimes it’s very
small adjustments that can have the biggest results. One company that we worked with recently,
a publishing company, had an extremely high level of paper usage. Although they did recycle
their paper waste, the cost to the environment was still high.

While recycling is always a positive, it’s better for the environment if you don’t use the natural
resources in the first place and recycling still uses power so we helped them find ways to reduce
the amount of paper they went through. And, can you believe that, by the time we left, they were
on their way to becoming a paperless office, which is quite an astounding accomplishment if
you think about it.

For those of you that might be interested in doing something similar, there are lots of ways you
can do your bit. You could sign up with us for one of our events, I’m not saying you have to join
my group, I didn’t come here to recruit members, no, you could just help out at one of our tree
planting or clean-up days. Alternatively, there are a number of other groups that run similar
ventures and you can volunteer with one of them. Anything you can do to help the environment
is great and it will ease your conscience and put your mind at rest that something is being done.

While it’s very commendable giving your time to one of these causes, you can always simply
start at home making small adjustments to how you carry out everyday tasks, like not doing the
things I mentioned at the beginning of my talk. Turning off the tap when you’re cleaning your
teeth saves precious water and turning lights off and not leaving things on standby conserves
energy. It’s not a question of having a passion for everything ecological, you just need to have a
desire to do some good, which I personally think we all have.
Exercise 4.

You will hear a zoo keeper called Gareth talking to members of the public about changes in
zoo practices over the last sixty years. For questions 7 – 15, complete the sentences with a
word or short phrase.

Gareth thinks attention has shifted from keeping enclosures free of (7) ________________ to
making them similar to animals’ natural environment.

He says it is difficult to provide natural surroundings while maintaining the animals’ (8)
________________ to the public.

At Hadley Zoo, keepers devise programmes which provide (9) ________________ to the
animals as part of their care.

In the 1980s, zoo animals were regarded as similar to (10) ________________ by members of
the public.

Gareth explains how the word (11) ________________ illustrates the attitude of the public to
zoo animals.

In one zoo, visitors can manipulate special (12) ________________, allowing them to interact
with the animals.

Gareth says that the apes bred in captivity have an attitude of (13) ________________ to
visitors.

The aim of training dangerous animals is to achieve (14) ________________ when handling is
required.

When the use of (15) ________________ was stopped, elephants became healthier and more
sociable.
7 disease

8 visibility

9 stimulation

10 works of art

11 connecting

12 toys

13 tolerance

14 stress reduction

15 chains

I’ve seen many changes over my years as a zoo keeper at Hadley Zoo and all of them have
brought benefits to the animals in our care. For example, the design of animal enclosures at zoos
is something that has undergone a radical transformation. When I started out, what was of
paramount importance wasn’t trying to create an environment which would allow animals to
behave as they would in the wild, which is what we try to do today, even if they do still have to
be kept behind bars – but it was actually making sure that disease was kept at bay. For example,
the floor of the ape house at Hadley Zoo, which was built in the 1960s, is made of concrete,
instead of natural materials because it’s much easier to hose out and bleach down. But we’ve
now tried to make the environment more ape-friendly by adding hammocks and ropes. But all
zoos have found that the problem with trying to build a natural environment for some of the
larger animals, is that when you provide a large area with plenty of shelter and vegetation,
visibility becomes an issue to the visitor. Trying to keep both animals comfortable and visitors
satisfied at the same time is a major challenge and requires a lot of creativity. At Hadley Zoo,
one of our basic tenets is to give the animals what we call ‘choice and control’, letting them
decide where to move and when – the idea is to encourage the animals to be as active and mobile
as possible – but we can’t always guarantee results that are visitor-friendly. As a zoo keeper, I’m
involved in developing programmes for the animals. We want to look after them as best we can,
but not to fully domesticate them. They’re still wild animals, so for example we might make
them forage for their food, or work for it in some way, which gives them the sort of stimulation
they’d miss if we just handed it over. We went through this phase in the 1980s as a profession
that because some of these animals were so rare, they were displayed to the public in zoos in
such a way as to make visitors feel that they were looking at works of art, because they were
looking at something like a tiger that’s a rare and endangered species. These days zoos are
having to fight harder for the public’s attention and we’ve learned what they don’t want is a
lecture on conservation issues, what they want is to experience something more personal and
emotional with an individual animal; they don’t want to treat it just as a rare spectacle. So the
buzzword is no longer ‘conservation’, although that remains the mission of all zoos, it’s
‘connecting’ – so it’s about relating people directly with nature. The idea of showing visitors
more of what the zoo does for animal well-being, while also giving them first-hand experience
with animals, is slowly spreading. So the orang-utans
can actually play with the public using toys which can simultaneously be controlled by the
public outside the enclosure, and the animals inside. And we’ve just opened a new tiger
exhibit with trails and corridors that will allow the big cats to roam through visitor space,
separated only by a glass wall.
At Hadley Zoo, we’re at the stage where all our big cats and apes were born in captivity
and not taken from the wild like previous generations. They’re still wild animals, but they’ve
grown up in the presence of humans in a controlled environment. From years of observing
the apes’ interaction with the public, I’d say they display tolerance rather than an attitude of
fear. Despite their ancestral roots in equatorial Africa they understand the benefits of
airconditioning and they prefer to stay inside, and who can blame them?
In the past handling the more dangerous animals was only possible if they were
immobilised or tranquilised which understandably they didn’t like. I remember the tigers
used to become super-aggressive every time the vets came anywhere near them. The
situation is different today because we can train the animals from a young age. One way
we do this is to train the animals to press against the mesh of their enclosures to receive
injections. The goal is stress reduction so that the lions and tigers become used to being
handled and can be given injections without the need for tranquilising.
When I began work as a keeper in 1987, the elephants were kept in chains in their cages at
night. And when the elephant manager made the decision ten years later to leave them to
move around freely in their cages, the fear was that it might make them harder to handle.
But in fact because their feet were in better condition, and because they could socialise in a
better way, it actually made them easier to handle.
Things have improved a lot, and I believe animals in zoos have a great life. In the wild
they’re… [fade]
Exercise 5.

You will hear a student called Mara Barnes giving a presentation about the language of the
Piraha people who live in the Amazon basin. For questions 1–9, complete the sentences
with a word or short phrase.

Mara defines the way of life of the Piraha people as fitting into the (1) ________________
category.

Mara explains that because most speakers of Piraha are (2) ________________, the language is
not under imminent threat.

Professor Everett was surprised to discover that the Piraha language has no words for ideas like
(3) ‘________________’ or ‘number’.

Mara says that common objects such as (4) ________________ were used to establish whether
Piraha people could count.

Mara thinks that the Piraha language sounds more like (5) ________________ than speech.

The (6) ________________ used in the Piraha language are thought to have originated in another
local language.

Mara uses the term (7) ‘________________’ for a common concept related to time that Piraha
people seem not to have.

Professor Everett gives the example of the Piraha’s superior (8) ________________ to support
his idea that they are not unintelligent.

Mara uses the expression (9) ‘________________’ to describe her attitude towards Professor
Everett’s theory of language
1 hunter(-)gatherer

2 monolingual

3 colour/color

4 batteries

5 humming

6 pronouns

7 collective memory

8 sense of direction

9 (keeping) (an) open mind

Hi. My name’s Mara Barnes and the subject of my presentation this evening is an amazing tribe
of people who live deep in the Amazon rainforest on the banks of the river. They are called the
Piraha and there’s about four hundred of them living in a scattering of small villages. The thing
about the Piraha is that, as well as living what we would term a hunter-gatherer lifestyle – that is
they’re not engaged in agriculture or animal husbandry, but live off the bounty of the forest
environment – these people have a unique language, that’s been studied by an ethnologist from
the University of Manchester called Professor Everett over a twenty-five year period.

Although the number of Piraha speakers is small, the language cannot be described as
endangered because most of its speakers are monolingual and have little contact with other
language groups. Similarly, few outsiders understand anything of Piraha, which isn’t related to
other existing languages. So the first thing that Everett had to do was learn the language himself.

From his first steps on Piraha land in 1977, Everett knew the tribe was remarkable. As far as he
could tell, the language had no words capable of conveying basic ideas like colour, although
words for light and dark existed, or more significantly counting. If this were true, then the
language would be unique – the world's only known language without numbers. A series of
experiments, using items that the tribe were familiar with, like batteries, established this to be the
case. But the Piraha had access to brazil nuts and were keen to set up trading relations with
neighbouring tribes, so Everett set out to try and teach some of them to count – with little
success. It seemed that in their everyday lives, these people had no need of numerical skills, and
so couldn’t even grasp the concept of number.

Everett had to wait months before coming to these conclusions, however, so indecipherable was
the language. It’s a kind of sing-song communication which some have compared to singing, but
which to my mind has more in common with humming than with the spoken word, and whistling
is also an important feature in communications in the jungle. Linguists have studied the structure
of the language and found that despite a very limited set of vowels and consonants and a lack of
complicated grammar, many ideas are conveyed through variations in pitch, stress and rhythm.
Although Piraha does have a set of personal pronouns, these seem to have been imported from a
neighbouring language, rather than being an original feature, and the language has no perfect
tense or way of reporting ideas such as ‘Mary said that John thought that Henry was happy.’
What are known as recursive sentences by linguists.

Because there’s no written version of Piraha, very few storytelling traditions and no tradition of

decorative art, the tribe seems to have a complete lack of what’s known as a collective memory –
in other words there’s little sense of history as people are focussed on their current needs. The
Piraha aren’t interested in either the distant past or the distant future, so don’t have the language
to express ideas related to those time periods. Having lived with the Piraha for many years,
however, Everett disputes the idea that they’re intellectually inferior to other peoples. He points
to their remarkable sense of direction as a skill that he himself has been unable to learn from
them, and says that their knowledge of local plants and animals and their behaviour patterns is
encyclopaedic.

Everett’s study of the Piraha is important for a number of reasons. Clearly, they’re a fascinating
people, but, most significantly, they call into question some of the most important
twentiethcentury theories regarding the link between language and thought – not least Professor
Chomsky’s ideas about a universal grammar that we all share. The evidence of the Piraha would
seem to suggest that this is not the case. Everett believes that it is the Piraha’s culture that
determines their language structure, rather than an innate system of grammar. As they have no
need to express certain ideas, then their language hasn’t developed them.

It’s a compelling argument, but like most people I’m keeping an open mind. Piraha is such a
difficult language to learn that few people have been able either to corroborate or refute
Professor Everett’s ideas. For the moment, at least, the secrets of the Piraha remain safely hidden
in the depths of the jungle.

Now before I go on to …
Exercise 6.

You will hear a practitioner talking about alternative medicine. For questions 7-15,
complete the sentence with a word or short phrase.

In spite of a long-established form of alternative medicine, homeopathy only (7)


________________ when it was discovered by Samuel Hahneman.

Alternative medicine to treat allergies is usually taken in (8) ________________.

The (9) ________________ in England is one person who severely criticised homeopathy.

Over two million people in (10) ________________ were treated with homeopathic medicine
before a hurricane struck.

In acupuncture, the intensity of stimulation caused by needles can be increased by passing an


(11) ________________ through them.

When acupuncture corrects the (12) ________________ through the body, the patient feels
better.

According to the practitioner, it is usually as (13) ________________ that patients go for


naturopathy treatments.

The primary aim of naturopathy is to change the patient’s (14) ________________ to effect a
cure.

The practitioner mentions (15) ________________ as two factors that can undermine health.
7. gained momentum

8. tablet form

9. Chief Medical Officer

10. Cuba

11. electric current

12. flow of energy

13. a last resort

14. lifestyle

15. poor diet and stress

Though there are several forms of alternative medicine used in the West, the most widely
adopted are homeopathy and acupuncture, and to a lesser extent naturopathy. Homeopathy dates
back to the time of Hippocrates, but it only gained momentum when it was only by Samuel
Hahneman, whose primary aim was to adopt a treatment that did not provoke the terrible side
effects linked with the conventional medicine of the time. Homeopathy revolves around the
principle of treating like with like. The treatment therefore involves providing the patient with
very small amounts of a substance that in large doses causes symptoms. The aim of this
procedure is to trigger the body's ability to heal itself naturally, and it is the homeopath's job to
match the most appropriate medicine to the programme of treatment. So, when treating an
allergic patient, the practitioner will prescribe small doses of an allergen, usually in tablet form,
to reduce the allergic reaction of that patient to that particular allergen. Although homeopathic
remedies do work, the mechanism by which they act on the body cannot be explained by science
- something which has led to scathing criticism from conventional doctors. Indeed, the fact that
the National Health Service now provides homeopathic treatments has irked England's chief
medical officer so much that she has categorically stated that homeopathic remedies are nothing
more than placebos. She has also gone on to warn of the dangers of using homeopathy to treat
malaria and other infectious diseases. To my mind, she is correct to point out the limits of
homeopathy, but there are limits to conventional medicine, too. However, I do not share her view
of homeopathy merely working like a placebo. After all, 300 million patients in 80 countries
have benefitted from this alternative treatment, including its adoption as preventative medicine.
For instance, in Cuba, 2.3 million people were given homeopathic medicine in advance of a
hurricane, the natural phenomenon which is known to trigger the disease leptospirosis. This
move significantly reduced the number of cases of the disease and made control of the epidemic
much easier. Now, I'd like to move on to acupuncture, which has been part of the traditional
medicine of China, Japan and other Eastern countries for thousands of years. Modern
acupuncture therapy is based on the discovery that when certain areas of the skin are stimulated
by inserting fine needles just under the surface, the functioning of specific organs in the body is
affected. This stimulation can be increased in intensity by passing an electric current through the
needles or by releasing heat from burning herbs onto the skin. The principle underlying the
practice is the belief that good health depends on a balanced flow of 'Chi'. 'Chi' circulates
through the bloodstream along energy pathways called medians that are linked to the organs. The
practitioner has to locate the blockages and apply needles to the appropriate points to correct the
flow of energy so that pain can be relieved and health restored.

The final form I just want to touch on today is naturopathy, which patients normally turn to as a
last resort. In a nutshell, naturopathy utilises natural healing practices like homeopathy,
acupuncture and herbal remedies and examines not only the symptoms of the patient but also
investigates the root cause of the illness. As such, the main goal of naturopathy involves
changing a person's lifestyle in order to facilitate a more permanent cure by treating the whole
person. By that I mean teaching the patient self-care to remove factors such as stress and poor
diet that can result in a degradation in health.
Exercise 7.

You will hear an architect talking about biomimicry, copying animal behaviour for human
purposes, in buildings. For questions 7-16, complete the sentences with a word or short
phrase.

When a (7) ________________ of termites enter a home, the occupants may have to leave
temporarily.

The way in which termites (8) ________________ in their mounds was the inspiration of a
building in Zimbabwe.

The African termite cultivates a (9) ________________ which acts as a primary food source.

It is the (10) ________________ in the enclosures that cools daytime air in the termite mounds.

The two buildings in Eastgate Centre are positioned (11) ________________ and separated by
an open space.

Cool air flows into the offices through (12) ________________ and special vents.

Stale air in the Eastgate Centre leaves through chimneys after entering the exhaust section of the
(13) ________________.

The owner of Eastgate Centre claims that regular fresh air changes can improve (14)
________________ by over 10%.

The CH2 building in Australia has (15) ________________ that open so that air can be taken in.

CH2 also uses (16) ________________ and thermal solar panels to further reduce its carbon
footprint.
7. colony
This is because many species of this insect feed on wood and if a colony moves in, the occupants
may have to move out until pest control has dealt with the problem.
8. regulate (the) temperature
When designer, Mick Pearce, investigated the building methods ed by termites to construct their
gigantic mounds, he was inspired by the way they are able to regulate the temperature in the
stuctures they build.
9. fungus
Instead, they feed mainly on a fungus which grow inside the mound.
10. wet mud
When, for example, it is hot outside, air is sucked in at the lower part of the mound from where it
is channeled down into enclosures whose walls are made of wet mud. The slightly cooled air
then flows up through channels to the peak of the mound.
11. side by side
Eastgate is not one building but two, side by side separated by open space covered by glass and
open to local breezes.
12. hollow floors
Like in the termite mounds, cool air begins to flow from the bottom of the building and is then
distributed into offices via hollow floors and baseboard vents
13. vertical ducts
Fresh air replaces stale air, which exits through chimneys after entering a exhaust section of the
vertical ducts.
14. worker productivity
The buildings receive a fresh air change every half hour, which the owner claims is able to
increase worker productivity by more than 10%
15. automatic shutters
Like Eastgate Centre, CH2 manages the internal and external temperature differences, but in this
case, a whole side of the building is opened up to a direct air intake through automatic shutters
16. photovoltaic cells
CH2 also uses photovoltaic cells and thermal solar panels to further reduce its carbon footprint.
Exercise 8.

You will hear a radio presenter called Tania Parades discussing the subject of whale
beaching along an area of the New Zealand coastline. For questions 7-15, complete the
sentences with a word or short phrase.

Tania wants to discuss the beaching reported this week not because whale beachings are unusual
in New Zealand, but because they have become (7) ________________ in the Golden Bay area.

Tania describes the mass stranding that occurred earlier in the week, as (8) ________________
on account of the sheer number of pilot whales that beached.

In spite of a rescue attempt by a large number of people, (9) ________________ of the whales
made it to the safety of deep water.

Tania believes that for once, it is the depth of water in the bay rather than
(10)________________ that is mainly to blame for the beachings.

The plentiful amount of (11) ________________ in the bay, combined with the gradual reduc-
tion in depth of the water, is thought to disrupt the whales' echo-Location methods.

The whales' navigation problems are compounded by the fact that the tide rapidly (12)
________________ of the bay.

However, the conditions in the bay do not explain why so many whales can all make an identical
(13) ________________ at the same time.

Scientists speculate that mass beachings may be linked to the fact that whales tend to maintain
(14) ________________ within their communities.

In this sense, a mass stranding could be explained as a failed (15) ________________ on the part
of the other whales in the pod.
7. an annual event

8. a (very) significant event / (very) significant

9. only a handful

10. human interference

11. soft sand

12. moves in and out

13. navigational error

14. strong social bonds

15. rescue effort


Exercise 9.

You will hear a radio presenter called Simone Poole discussing the origin of the
superstitions surrounding Friday 13'. For questions 7-15, complete the sentences with a
word or short phrase.

The word which describes people suffering from (7) ________________ about Friday 13" comes
from the Greek language.

When they go about numbering their floors, many hotels choose to (8) ________________ due
to its association with misfortune.

The Society of the Irish Motor industry brought in a (9) ________________ in 2013 in order to
accommodate people's superstitions, fearing sales of new cars would otherwise fall.

There are considerable (10) ________________ to be enjoyed by homebuyers prepared to


purchase a house numbered 13.

In Norse mythology, the (11) ________________ is said to have created mayhem on arriving
unexpectedly at the Valhalla banquet.

References to both the number thirteen and the day Friday in religious texts tend to have (12)
________________.

Lawson's book is about the negative effect on (13) ________________ that is experience when a
corrupt banker exploits the superstitions surrounding Friday 13'.

In certain parts of Southern Europe, Tuesday 13th is known as the greatest (14)
________________ of misfortune

The perception of the number 13 in Ancient Egypt differed from the negative one we tend to
have today because the 13th (15) ________________ was thought to represent a superior form of
existence.
7. a genuine phobia

8. skip 13 altogether

9. new registration system

10. savings

11. God of Mischief

12. negative connotations

13. the markets

14. calendrical omen

15. stage of life

Hello and welcome to the programme. I'm Simone Poole and what with the day being what it is -
this being Friday the 13th and all - I thought we'd start the show by taking a look at some of the
superstitions people have about today and where those superstitions come from. And it's no
laughing matter because some of us take Friday the 13th very seriously indeed! There is even a
word used to describe those who have a genuine phobia about the date -
paraskavedekatriaphobia. This is derived from Greek and literally means fear of Friday the 13th.
According to research conducted by a popular chain of hotels, almost 75% of adults in the UK
claim to have suffered from bad luck at some point in time on this day. And speaking of hotels.
given that the digit is associated with misfortune, did you know that many go so far as to skip 13
altogether in their floor-numbering system? So, for example, you may find that your hotel lift
can take you from Floor 12 direct to Floor 14, with Floor 13 simply not existing at all. So, as you
can see, the fear, or superstition, is very real. Indeed, the Society of the Motor Industry in Ireland
was so concerned about the public's discomfort surrounding the number 13 that it completely
changed the registration number plate system. For fear that the number 13 appearing on car
plates would hit sales, a new registration system was introduced for the start of 2013 and has
been in operation ever since. Further evidence of the public's fear of this unlucky digit can be
found in the fact that, according to one online property agency, house buyers can make savings
of up to £9,000 if they are willing to call Number 13 their new home. But why all the fuss - what
is it about the number 13 that makes us so superstitious? Well, there are many theories for the
origin of this illogical fear. For instance, in Norse mythology the superstition can be traced back
to the banquet at Valhalla where the God of Mischief caused chaos when he turned up
unannounced as the 13th guest. Then there are religious references. too. For instance, Friday was
the day Eve tempted Adam with the apple from the Tree of Knowledge and it also marked the
start of the great flood during the time of Noah. Numerous other references with negative con-
notations for both Friday and the number 13 can also be found elsewhere in religious texts From
the world of literature, Thomas W. Lawson's novel Friday the Thirteenth, published in 1907,
may have added fuel to the fire of and popularised the superstition surrounding the day,
Lawson's story tells of the deeds of an unscrupulous banker on Wall Street, who takes advantage
of the superstitions around the 13th to cause widespread panic and chaos in the markets. In parts
of the Mediterranean, though, whilst the number 13 is similarly associated with misfortune, it is
Tuesday and not Friday - in Spain and Greece, for example - which is synonymous with bad
luck, Tuesday the 13th then, would be regarded there as the greatest calendrical omen of
misfortune. However, the number 13 has not always been associated with misfortune. Indeed, the
Ancient Egyptians regarded it as a lucky number, believing that the 1 3th stage of life was related
to the afterlife, a sort of higher spiritual state of being. Indeed, although Friday the 13th is a
widely held superstition today, closer exami-nation at local level of perception of the day, and
date, in different cultures tends to be quite revealing. It really is interesting that the whole idea of
Friday the 13th ...
Exercise 10.

You will hear a radio presenter called Giles Franklyn discussing a weather phenomenon
known as a fogbow. For questions 7-15, complete the sentences with a word or short
phrase.

Fogbows only seem totally white (7)________________, if not properly examined, and that is
why the term white rainbow is misleading.

A fogbow is actually made up of a number of very (8) ________________ such as the subtle red
visible on its outer edge and the blue on its inner.

There is a clear connection between (9) ________________ and the strength of the colours that
form in a bow.

Cloud bows can regularly be seen by people travelling in-flight on airplanes when they are (10)
_______________ and cloud formations are below.

Water light which enters the water droplet undergoes _______________ and refraction,
producing the range of different colours of a rainbow.

A rainbow is formed when each of the seven waves of light gets reflected in a (12)
________________thus creating the familiar arc of the bow.

When light is diffracted by fog, the arc of the bow formed is (13) ________________ than that
of an ordinary rainbow.

Giles is not surprised that the photographer captured the image of the fogbow because fogbows
are frequently evident in (14) ________________ terrain.

Fogbows are most likely to occur in places where there are (15) ________________ such as on
elevated land or over Arctic waters.
7. from a distance

8. weak colours

9. (water) droplet size

10. looking downwards

11. dispersion

12. slighly different direction

13. broader and paler

14. moutainous and hilly

15. stark temperature contrasts

Giles Franklyn: ... and that's tomorrow's forecast for you, but, before I say cheerio, one of our
listeners sent in a marvellous photograph today of a very unusual phenomenon - a white rainbow
- inquiring as to what exactly it is. We've uploaded the photo to our website and social networks
and due to the immense amount of interest we've had since, by way of comments and likes, I just
wanted to take this opportunity to explain what exactly people are seeing in the image. Although
it is sometimes called a white rainbow, and, indeed, it is a similar weather phenomenon to that of
the rainbow in most respects, a fogbow, as the arc in the photo is more commonly known, is, as
the name suggests, notably different to behold. Fogbow is the preferred name because using the
term white rainbow is actually somewhat inaccurate. From a distance and without careful
examination, a fogbow may indeed appear completely white. However, this is not so. Whilst the
bow only consists of weak colours, it has more than one of them, including a reddish outer edge
and a blueish inner one. The reason the colours are so weak as compared with a typical rainbow
is due to the dimensions of the water droplets that cause fog, which are considerably smaller than
those produced by other forms of precipitation. Indeed, in many cases, the water droplets are so
small that the entire fogbow can appear white - there is a direct correlation between the droplet
size and colour vivacity, you see, so the smaller the droplets, the weaker the colours. However, a
closer inspection will reveal that, though faint, the colours are present. No fogbow is ever,
therefore, in the strictest sense, a white rainbow. Because fog is essentially low cloud, it is hardly
surprising that the phenomenon is also frequently visible from aircrafts, typically when they are
flying above clouds - looking downwards, if you like. Such fogbows are often given the
alternative name cloud bows, for obvious reasons. Another name for fogbows was given to them
by mariners, who tend to encounter the phenomenon regularly out at sea. To them, they are sea-
dogs. Fogbows are also occasionally visible at night time, when they are known as lunar
fogbows. But why does droplet size have such an impact on colouration? Well, as I may have
mentioned, fogbows are formed in much the same way as rainbows. Here are the basic steps:
First, white light enters the water droplet. As it does so, it is divided into its seven different
colours due to a process called dispersion. The different colours of light refract, or change
direction, after entering the droplet at slightly different rates. Therefore, when the seven separate
waves of light then get reflected off the inside of the droplet, they each go in a similar but
slightly different direction, emerging to form the familiar large, colourful arc of the rainbow.

The difference with fogbows is that the fight is not refracted, however: whilst refraction is at
work in large water droplets to change the direction of light, the tiny droplets found in fog cause
light to diffract instead. In simple terms, the process of diffraction involves breaking the light
waves up and spreading them out into many different directions. The diffracted light produces
weaker colours and the familiar broader and paler arc of the fogbovv. Knowing what we now do
about fogbows, it's unsurprising, therefore, that the photographer who captured the striking
image that provoked such interest when we posted it on our website and forums was in the
Scottish Highlands at the time of capture. After all, fog is a very frequent weather event in
mountainous and hilly regions where there are often stark temperature contrasts. Far the same
reason, spectacular fogbows are regularly seen over Arctic waters, too. Indeed, if you find you
now have a taste for fogbows and a hankering to find more, you'd do well to ...
Exercise 11.

You will hear a mountain rescue team Leader called Sean McKinley giving a talk on
avalanche awareness on a daily radio show. For questions 7-15, complete the sentences with
a word or short phrase.

Gravity is responsible for natural snowslips which are termed (7) ________________.

Natural snowslips can also be produced by seismic events as well as (8) ________________.

Avalanches caused by people account for a (9) ________________ of avalanche-related injuries


and deaths.

Powder avalanches only occur around times of (10) ________________ when the wind is
moderate or still.

The (11) ________________ process changes the consistency of powder snow once it has been
lying for a while.

When the snowpack has (12) ________________ in layers under the surface, an upper layer or
layers can break away and slide, resulting in a stab avalanche.

As well as the intensity of snowfall and the angle of slope, (13) ________________ , which can
lead to instabilities in the snowpack, can also be a very significant contributing factor to
avalanches.

A wetter, denser snowpack can result from heavy rain or (14________________ due to rapid
temperature increases. Such conditions may produce wet snow avalanches.

(15) ________________, and large snow-top cornices suggest that there has been a period of
heavy winds very recently.
7. spontaneous avalanches

8. rock falls

9. disproportionate number

10. intense snowfall

11. thawing and freezing

12. a vulnerability

13. sudden temperature change

14. thawing: tan chảy

15. raised footprints

Sean McKinley: Thank you for having me on the show today. My name is Sean McKinley and
I'm here to discuss avalanche awareness and safety, which is particularly important at this time of
year. I do not want to put people off exploring our wonderful hills and mountains, but, as a
mountain rescue team leader who has seen the worst consequences of ignorance, I would like
people to be well-informed so that they can make good decisions out there. So, first and
foremost, what causes an avalanche? Well, typically the culprit is the force of gravity. That is
what produces a natural avalanche in most cases when conditions are right. Such snowslips are
known as spontaneous avalanches. Other less typical causes of natural avalanches include rock
falls and, rarer still, of course, seismic events, such as earthquakes and volcanic eruptions.
Although such geological phenomena are extremely rare in most parts of the world, obviously,
given the unique set of geological conditions we are faced with here in the Pacific Ring of Fire,
they present a much more real and present threat to us, and one we can do little to control. What
we can control, however, is our own behaviour, which is to blame for a disproportionate number
of avalanches which result in human injuries or fatalities. I'm referring, of course, to human-
triggered avalanches, but more on this later. For now, let me just say that around 90% of
avalanche victims trigger their own avalanche, and let that sink in. There are two main types of
avalanche; powder avalanches and slab avalanches. The former occur during or shortly after pe-
riods of intense snowfall on days when winds are very light or non-existent. You will almost
never see one in the absence of snowfall because once lying snow has gone through the thawing
and freezing cycle. It takes on a different composition to fresh powder and becomes part of the
main snowpack. As a rule of thumb, watch out for more than 30cm of continuous build-up of
snow in one day or more than 2cm in one hour. The latter, on the other hand, can occur at any
time when there is a vulnerability in the existing snowpack. Snow builds up in layers and if there
are weak layers of poorly bonded snow beneath the surface, this can cause upper layers to break
away and slide. Slab avalanches most frequently happen on slopes of between 30 and 45 degrees,
so extra caution should be exercised at such angles. The risk of being avalanched is also highest
when you are positioned lower down ridges. So, already we have identified some of the major
risk factors, such as slope angle and snowfall intensity. Another major risk factor is sudden
temperature change. This can disrupt the snowpack and lead to instabilities. Heavy rain can also
sometimes destabilise the snowpack by making the snow wetter and consequently denser, with
the increased density raising the likelihood of wet snow avalanches. A similar problem can occur
without precipitation when temperatures rise suddenly and sharply, causing thawing; again, a
wetter, denser snowpack is the result. Another factor always at play is wind speed. Strong winds
can deposit more snow on lee or downwind slopes. And even when there is no new snow, such
winds can redeposit snow from the existing snowpack on the same lee slopes. During and
directly after such windy periods, great care should be taken in selecting slopes to traverse. Signs
that there have been gales recently can often be seen in the presence of large cornices or
overhanging snowtops on peaks and ridge edges, not to mention in raised footprints in the snow
itself. So, clearly, when considering a day out in the hills, forewarned is definitely forearmed.
Also, we should not forget the importance of ...
Exercise 12.

You will hear a financial expert called Alexandra Harrow talking about the influence of the
mobile phone on the way people transfer money on a radio show. For questions 7-15,
complete the sentences with a word or short phrase.

M-Pesa is a way of effecting (7) _______________ and making purchases and payments using
your mobile phone.

Although some could afford to repay small loans, poor people of Africa were generally ignored
by the tra-ditional banking system because their custom was not viewed as (8) _______________
to worry about.

The idea for M-Pesa was inspired by how Africans had started using (9)_______________ as a
form of currency and a new way of making payments.

The system developed by the DfID and Vodafone was only intended to be used to (10)
_______________.

People with relatives in (11) _______________ found the new system very handy for making
fund transfers back home from the big city.

In Kenya, M-Pesa shops now outnumber (12) _______________ dramatically, which


demonstrates the significant influence the new payment system has had there.

Despite its success in parts of Africa, (13) _______________ has really yet to start using mobile
money at all.

Mobile money contributes to tackling corruption in traditionally (14) _______________, which


is why it has become so popular.

Mobile money also helps to stop (15) _______________ because it makes it very hard to hide
payments and income.
7. money transfers

8. profitable enough

9. pre-paid airtime

10. repay small microfinance loans

11. remote villages

12. cash machines

13. the developed world

14. cash-driven economies

15. tax evasion


Alexandra Harrow: Now I want to turn to a service which is little known here in the UK but
which is revolutionising the way business is conducted in certain parts of the world. The service
I'm referring to is M-Pesa. Let's take a look at this M-Pesa revolution and how it is transforming
the way money is circulated in some African economies and elsewhere. First of all, what is M-
Pesa? It is simply a way of making money transfers and purchase or payment transactions via
mobile. The story of this money via mobile concept first began in 2002 at a conference in
Johannesburg on sustainable development. Present at the conference was the head of the UK's
Department for International Development, who had a fund at his disposal to invest in a project
that would help improve access to financial services. A lack of access to the banking system was
a particular problem for millions of poor people living in Africa. How many would-be
entrepreneurs were being denied their chance to contribute to the economy as a consequence?
Even if such people wanted only small loans, which they could afford to pay back, in reality,
they were too poor for the existing banking system to bother with; the loans would not have been
profitable enough to make them worthwhile. Therefore, what was needed was a new system
tailored to meet their needs - a soprce of financial services for entrepreneurs and small
businesses lacking access to the traditional banking system. This new system would become
known as rnicrofinance, and mobile money transfer would soon play a big role in it. The
Department for International Development (DifiD) had already noted a rising trend in African
countries whereby mobile phone customers were themselves developing a sort of currency (or
quasi-currency) by transferring pre-paid airtime to one another as a means of payment. It decided
to try to take things one step further and create an official payment system modelled very
similarly. To do this, it teamed up with the Vodafone network, with each partner committing one
million in funding to the fledgling project. Although neither partner envisaged the system serving
any purpose other than to repay small microfinance loans by SMS, users. however, had a
different idea and it was ordinary mobile phone customers in Kenya, where the project first
started, that drove its development into something much more influential. They started using it to
send money to one another as a means of payment for things other than loans - from everyday
purchases to utility bills. Not only that, but they also found it incredibly convenient for sending
money home from big cities where they worked to their families in remote villages. Within eight
months of launching, more than a million Kenyans had signed up to mobile money. Today, there
are well over twenty million users and about 100 times as many M-Pesa kiosks as there are cash
machines. In a country where cash used to be king. now mobile money does the talking. And the
mobile money revolution has now spread to around two-thirds of all developing countries,
though it has yet to be introduced or embraced in any sense in the developed world. However,
that may yet happen because, in actuality, mobile money payment is a far simpler and quicker
service than any existing banking app can offer. Why it has been embraced in the developing
world so quickly is clear, though. Cash-driven economies have a lot of corruption. Introducing
mobile money helped to tackle this and was an unexpected benefit of the service. For example, in
Afghanistan, policemen only started to realise how much of their basic salary was being taken
illegally by their superiors before being passed on to them when they started to receive their
payments by mobile instead of cash .- the amount of money they received was shockingly more
than what their superiors had been giving them. Meanwhile, back in Kenya, drivers there realised
that traffic officers harassing them for bribes would not be so keen to accept them through the
M-Pesa system, since such bribes were then trackable and could be used as evidence, so they
stopped carrying cash altogether and then the harassment petered out too. Another benefit of this
track ability, of course, is the fact that it helps prevent tax evasion. If payments and income can
be tracked, it becomes very difficult to avoid paying tax.
Exercise 13.

You will hear arboriculturist Marcella Im discussing her job on a radio programme. For
questions 7-16, complete the sentences with a word or short phrase.

Im likens her job to that of a (7) _______________ a term which people generally are more
familiar with.

lm says her role mainly involves the growing and maintenance of (8) _______________.

In built-up areas, Im says a big task is ensuring that trees exist (9) _______________ with the
concrete jungle around them and its inhabitants.

(10)_______________are what determine whether Im will use modern or traditional equipment


on any given job.

Safety issues and convenience are why Im would rather not use (11) _______________ if she
can help it.

Im's work frequently involves (12)_______________, whereby leaves and branches are removed
from mature trees.

An arboriculturist would be more inclined to be in a (13) _______________ or supervisory


position, rather than actually doing the hands-on work itself like an arborist.

The reason Im wanted to establish her own consultancy was (14) _______________.

Working on the projects associated with (15) _______________ is a far cry from her arborist
work which requires the ability to operate as (16) _______________, something she found most
difficult upon setting up her consultancy.
7. tree surgeon

8. trees, hedges and shrubs

9. in harmony

10. accessibility and height

11. traditional methods

12. thinning

13. managerial

14. mainly financial

15. vested interests

16. part of a team

Marcella Im: ... Thank you for having me on the show. Now, as I was saying, my job title is
arboriculturist, which is a bit of a mouthful I'll grant you! But perhaps my role is better known by
another name, so you can call me a tree surgeon if you like. I don't mind! Although the roles are
actually not quite the same. But don't call me a horticulturist: then you're into a closely related
but very different field. Not all of us 'culturists' do the same work, you see! The viniculturists, for
example, seem to have all the fun! Not that I'm jealous, mind you So what do I do? Well, mainly,
I cultivate and manage trees, hedges and shrubs. That is, in a nutshell, what my role involves. I
work in both rural and urban areas, and each present their own set of challenges. One of the
biggest challenges in urban areas, for example, is working out how trees can inhabit the concrete
jungle safely and in harmony with the buildings and people around them. My work will regularly
require me to use specialist equipment and machinery. For instance, I am very familiar with the
inside of a climbing harness! After all, it is often necessary to climb high up into trees to remove
loose branches and so forth. The choice of whether to use a rope and harness or modern
machinery, such as a crane, for a job is usually dependent on two factors: accessibility and
height. If I can get a lifting machine into the area and if it can take me up sufficiently high in the
tree, then I will, invariably, choose this option today because using modern machinery is much
safer and more convenient than employing traditional methods.
The most common activity I tend to do in my day-to-day work is thinning, which involves
making the branches and foliage on mature trees less dense, This could be to promote growth, for
safety reasons, or for a number of other purposes. Some would say that technically makes me an
arborist (an actual tree surgeon) not an arboriculturist. What's the difference? Well, an arborist is
mostly involved in planting, thinning and groundwork, while an arboriculturist tends to operate
in a supervisory role or a managerial capacity. They would, for example, select suitable trees for
a given environment and design landscaping schemes. However, I work in a hands-on role only
by choice. I also run my own consultancy business where I am at the top of the hierarchical
pyramid, involved in all major decision-making. Being honest, I prefer the hands-on work, but it
is my consultancy that pays the biggest bills at the end of the day, so my motivation for running
it is mainly financial. I value the business very much and the benefits I enjoy as a consequence of
owning it. There is a lot of paperwork and official correspondence involved, though, which I find
quite tedious. For example, I must carry out tree inspections and surveys and compile reports for
the likes of engineers, solicitors, mortgage brokers and insurance companies. I also do work for
the local council in terms of reviewing planning applications. There are a number of vested
interests in such projects, so, naturally, working on them is very different to my arborist work. I
need a very different skill set and I must function as part of a team and communicate effectively,
both of which are challenging when you work with people from all walks of life. That said, it is
the former which I struggled with most initially on opening up my consultancy, as l had been so
used to playing it solo on projects, as it were, for so long. Now, there are a number of different
factors which...
Exercise 14.

For questions 16-25, listen to a radio presenter named Finbarr Baird discussing reports in
the news about first-born children being the smartest siblings and complete the sentences.
Write NO MORE THAN THREE WORDS taken from the recording in each blank.

• Baird is not happy with deceptive (16) _______________ appearing in the news this week.

• According to research, the reason for first-borns' superior performance lies in more parental
care taken of them, rather than their (17) _______________.

• Baird suggests it is natural for parents to feel (18) _______________ on the birth of their
second child and subsequent children.

• The research confirmed that first-borns get (19) _______________ in the early days of
childhood and this gives them an advantage, which is evident from as early as one year of age.

• Such advantage is repeatedly manifested in (20) _______________.

• The research accounts for what scientists term (21) _______________, which sees children
born later into a family being more likely to earn and achieve less.

• As a matter of fact, it is only (22) _______________ that is missing for younger siblings at
times; the (23) _______________ they enjoy is more or less the same as that for first-borns.

• The conclusions drawn from the research make sense to Baird as (24) _______________ from
parents for each of their kids is disproportionate to the number of children they have.

• Much as Baird finds the research interesting and useful in confirming our intuitions, he is
annoyed by the (25)_______________, which is why he wants to explain the findings clearly.
16. headlines

17. higher innate intelligence

18. slightly more relaxed

19. extra focus

20. IQ tests

21. birth order effect

22. intellectual stimulation

23. emotional support

24. time and effort

25. (the) shabby reporting

Finbarr Baird: Welcome back to Today with Finbarr Baird. Now, next up, I want to have a look
at what's in the news, and one of the things that really caught my eye and, indeed, rubbed me up
the wrong way this week is the way the results of new research by scientists into child
performance have been reported. It is not the research itself that I take issue with, nor is it the
main body content of the news reports either; however, I find the headlines that have been used
extremely misleading. For instance. one paper leads with the following, and I quote: 'First-born
children are smarter than their siblings: This clearly implies that the first child is more intelligent
than any subsequent children born into the same family, yet that is not what the research suggests
at all. Contrarily, the research found that first-born children have a tendency to perform better in
their education. Note the distinction: perform better does not necessarily equate to being more
intelligent. Indeed, the research specifically cited not higher innate intelligence but the fact that
first-borns receive more attention from their parents as the reason for their superior educational
performance. And, actually, when you reflect on it, this makes perfect sense. After all, a first
child is a big deal for any new family, and young parents are often understandably nervous, so
they devote an extra special amount of time to the first-born and making sure they do the best
they can with its upbringing. When the next child comes along, they are perhaps. naturally,
slightly more relaxed as they have been there before, as it were, so, whether intentionally or
otherwise, they give a little less of themselves to the child-raising process and put slightly less
effort in. The data, which was collected from thousands of families over the course of more than
a decade, confirms this. It found that parents tend to do fewer activities with their second child
and subsequent children, reading less to them, not encouraging them to do art and crafts or music
as much, and so on. This extra focus that first-borns receive in their early childhood gives them
the edge over their younger siblings. The advantage is evident in children as young as one year
old and it shows up time and again in 10 tests. Indeed, the research goes a long way towards
explaining a phe-nomenon scientists call birth order effect, whereby children born earlier into a
family with numerous siblings tend to have a superior chance of enjoying better wages and
greater educational achievement in life. It must be stressed, however, that the report does not
accuse parents of being neglectful towards second-born and later children. Indeed, the research
suggests all children in a family tend to receive equal amounts of emotional support. It is just the
intellectual stimulation that is sometimes lacking for younger siblings. And, in fact, there are
logical explanations for why this might be so; after all. two children is twice the work of one for
parents, and so on. Clearly, parents cannot devote all of their time to a second-born child in the
same way they could the first-born when it arrived because they are still caring for the first-born,
so the time and effort must be shared between the children. As I said, I find the research
interesting, and I think it confirms a lot of what we already intuitively know and logically would
understand: however, what irks me is the shabby reporting, so I would like to clarify once and
for all: in no way does the research suggest that first-borns are innately more intelligent than
their later siblings. It only suggests that they perform better on average because they receive
more attention in their early childhood. In other words, the additional intellectual stimuli they are
exposed...
Exercise 15.

You will hear part of a radio programme about 'Bollywood', the Indian film industry. For
questions 9-17, complete the sentences with a word or short phrase.

All over Bombay there are enormous (9) _______________ advertising films.

In Indian films, the (10) _______________ often changes suddenly.

The films are (11) _______________ both European and Indian filmgoers.

The female stars are beginning to wear (12) _______________ rather than traditional clothing.

A Hollywood-style obsession with body shape means that a lot of people are now going to (13)
_______________.

The films often convey (14) _______________, such as a belief in family life.

Some critics argue that the films are superficial, often lacking a recognisable (15)
_______________

Recently there has been an (16) _______________ of fresh talent from different backgrounds

All these changes may well serve to keep Bollywood (17) _______________ for the foreseeable
future.
9. posters

10. location

11. drawing

12. miniskirts

13. gyms

14. traditional values

15. plot

16. influx

17. alive and well

Woman: There is no 'Bollywood' sign on the parched hills above Bombay, but the city has other
ways of reminding you that it is the world's most prolific film capital. At every junction in the
city's ramshackle road system the superstars of the film industry stare down from giant posters.
But, unlike the chaotic city from which it originates, Bollywood has its rules and regulations
firmly in place. Most of the leading players are from a handful of families. Violence is
acceptable, but kissing isn't. Sex is taboo, but suggestive hip rolls and pelvic thrusts in dance
routines are fine. Abrupt shifts of location during songs are frequent. And the song and dance
routines must be sumptuous affairs. But it's not just in India that these gorgeous films are finding
enthusiastic audiences. Because of the diaspora, the films were initially exported for expatriate
Indian audiences, but now they are drawing Western audiences as well. The industry seems to be
brimming with confidence as it undergoes a revolution. It's argued that the explosion in satellite
television and screen technology is transforming Bollywood and fast bridging the gap between
East and West. But can Western filmgoers really learn to appreciate three-hour boy-meets-girl
musicals in a language they don't understand? Meanwhile, things continue to change in the
Indian film industry. Saris are being swapped for miniskirts, and the traditional dancing is being
replaced by MN-style numbers. Bollywood's body facism now rivals Hollywood's, with skinny
actors often drawn from the ranks of models and beauty queens. Business is booming for
Bombay's cosmetic surgeons, and the gyms are full to bursting. However, the Western gloss is
only that. Beneath the surface there has been a resurgence of traditional values in the most
popular films. Values like the sanctity of the family and its importance over romantic love —
these values appeal to those living in the diaspora who have succeeded in terms of material
wealth, but still want to hold on to traditional values.

But the industry is under pressure from many critics, who argue that its obsession with style over
substance has squeezed out any semblance of a plot. The films often appear to be naive, lacking
in deep characterisation and light on subject matter. But slick marketing and stricter financing
may bring a new professionalism. The grip of Bombay's film dynasties has loosened, and a
recent influx of moviemakers from journalistic and advertising backgrounds has introduced new
vigour to the business. A wave of experimentation by a new generation of film-makers also
seems more likely to ensure crossover success. These new artists, often educated in the West,
want to 'tell it as it is'. Perhaps this heralds even greater success for Bollywood. Certainly the
movie industry is alive and well, and it seems ready to stay that way for a very long time.
Exercise 16.

You will hear part of a radio programme about the prospects for space travel. For
questions 9-17, complete the sentences with a word or short phrase.
To many people, the idea of space tourism may seem like the invention of 9 _______________
writers.
Up to $10 billion a year could be brought in by space travel, according to 10 _______________
research
In 2001, Denis Tito spent over a week 11_______________ the Earth.
We must learn not to always associate space travel with 12 _______________ funding.
Most people in 13 _______________nations would become space tourists if they could.
At the start of space tourism, the facilities might be 14 _______________.
Eventually, the accommodation would become much more 15 _______________and cater for
hundreds of guests.
In the last stage of the development of space tourism, 16 _______________would be lower.
It is important, economically, that the new vehicles should be 17_______________ , so that they
could make large numbers of trips.
9. science fiction
10. market
11. orbiting
12. government
13. industrialized
14. spartan
15. sophistcated
16. ticket prices
17. (fully) reusable
Woman: Space Tourism is the term that has come to be used to mean buying tickets at your local
travel agent, in order to travel into space and back. Your average person may find this idea
unlikely, more science fiction than science fact, but the truth is that over the past few years a
growing volume of professional research has been carried out and it is now clear that setting up
commercial space tourism services is a realistic target for business today. Market studies in the
United States and elsewhere suggest that, when fully developed, space travel and tourism could
become a new money-making opportunity, with annual revenues of as much as 10 billion dollars
per year. The first space tourist, Ameiican businessman Dennis Tito, paid a fortune for the
privilege of spending eight days, in April 2001, orbiting our planet in the International Space
Station. The technical know-how to make passenger launch vehicles and orbiting hotel
accommodation is available. And there is no doubt that a lot of people want to go into space.
Market research has revealed that the majority, at least in the industrialised countries, would like
to take a trip to space if it were possible. This prospect of mass space tourism creates the
potential for reducing the costs of space travel by large scale operations like airlines. The major
obstacle to the development of space tourism is the conservatism of the space industry. Since
Sputnik was launched in 1957, space activities have been funded by governments. Even science
fiction writers often assume that space activity will be government activity. So, we will need to
change the way we think about space. Space tourism services will not happen as a natural
consequence of present-day space activities. Entrepreneurs with vision and courage are called for
here! And, like any other business, once space tourism gets started it will develop progressively.
The first phase might be small-scale and highly priced, but the scale of activities will grow and
prices will fall as it matures. In the beginning there would be relatively few customers —from
hundreds per year to thousands per year. And the service would be nearer to adventure travel
than luxury, with spartan amenities prevailing. The second phase would see growing demand,
from thousands of passengers per year to hundreds of thousands per year. Tickets to orbit would
cost less, and flights would depart from various spaceports. Orbital facilities would grow from
being clusters of pre-fabricated modules to far more sophisticated structures constructed in orbit
for hundreds of guests, and offering a range of orbital entertainment. In the third and final phase,
ticket prices would fall and customer numbers would rise to perhaps millions of passengers per
year. Moves are already afoot to turn this dream into reality. For instance, one company is
developing technology to test the practicality of a new earth orbit transportation concept, capable
of carrying people into space in a fully reusable vehicle, operated very much like today's
commercial aircraft. Alongside this, half a dozen space transportation development companies
have been formed in the US. These companies expect that, within 4 — 5 years, they will be able
to carry people to an altitude 10 times higher than that reached by the highest flying commercial
aircraft. There seems little doubt that space tourism will become commonplace in our lifetime. It
is an idea whose time has come.
Exercise 17.
You will hear a radio report about photography. For questions 7-15, complete the
sentences with a word or short phrase.
Improved technology in the late 19th century meant that photographs could be printed in 7
_______________.
In the first half of the 20th century most news came via 8 _______________.
When TV became a more important means of giving news, photographs in periodicals focused
on 9 _______________.
Andy Warhol set out to comment on the widespread influence of mass-market images by taking
pictures of 10 _______________ things.
Steichen began the fashion for taking 11_______________ photographs of the rich and
glamorous.
Other photographers started the trend towards 12_______________ by depicting the injustices in
society.
August Sanders is well-known for his depiction of Germany's 13_______________.
In an era in which the function of photography has changed immensely, the popularity of
14_______________ has endured.
Nowadays, what we know as a photograph is often made from electronically combined
15_______________.
7. newspapers and magazines

8. photojournalism

9. make-believe

10. familiar

11. celebrity

12. social commitment

13. different classes

14. the family snapshot

15. digital images

7. Photographs could be transferred quickly, cheaply and in large numbers onto the pages of
newspapers and magazines.

8. In fact, until the 1970s, when TV began to take over its role, photojournalism was the
primary sources of news from around the world.

9. As TV took over the subject of hard news, magazines began to concentrate on make-believe.

10. Andy Warhol used familiar photographic images, from cereal packets to Hollywood pin-ups,
to point out how all-pervasive and manipulative mass-market images had become.

11. The cult of the celebrity photograph had begun decades before, when Steichen accepted a
contract to provide photographs for the rich and glamorous for Vanity Fair

12. But, while artist photographers had been gazing inwards, photographic reporters had begun
to look outwards, and used their cameras to document social inequality, and in this way the
tradition of social commitment in photography became established by people such as Jacob Riis
and Lewis Hine.

13. Atgets’ documenting of architectural details and neighbourhoods of old Paris or August
Sanders’ massive study of the different classes of the German people.

14. one that affects every one of us closely, and that is the family snapshot. We can measure our
lives in pictures by recording our personal great moments. This is the one aspect of photography
that has survived intact in a century where photography is being used less and less for
documentary and reportage.

15. What used to be called a photograph is more often a mixture of digital images that a
computer operator can paint in and out, with the dexterity of a painter with a brush.

Exercise 18.

You will hear an expert talking about seaweed and its uses. For questions 7-15, complete
the sentences with a word or short phrase.

Seaweed has been eaten since ancient times in 7 _______________.

Seaweed isn't fattening, but it contains various 8 _______________ as well as carbohydrates and
proteins.

Before nori is dried, it is 9 _______________ in colour.

A snack enjoyed by 10 _______________ is rice enclosed in a sheet of seaweed.

To cultivate nori, the Japanese put spores on 11 _______________ and wait for them to grow
into fronds of seaweed before harvesting them.

In Britain, factories making 12 _______________ used to burn kelp because it contains large
quantities of potash and soda.

In Scotland, kelp was gathered from the shore and used as 13 _______________ on fields.

13 _______________, which is traditionally eaten for breakfast in part of Wales, is made from a
seaweed that is very much like nori.

One day, seaweed may be used to produce 15 _______________ on long journeys in outer
space.
7. Asian countries

8. vitamins and minerals

9. red or purple

10. Japanese children

11. ropes or nets

12. glass and soap

13. (a natural) fertilizer

14. Laver bread

15. oxygen

I'd like to talk to you today about the aquatic plants commonly known as seaweed. Seaweeds are
a type of algae - very simple water plants with no real leaves or roots. Seaweeds may be simple
organisms, but they can actually grow quite large; in fact, Giant kelp is one of the largest plants
in the world! Algae are very important ecologically because they produce most of the oxygen in
our atmosphere, and they provide habitats for many marine creatures. As far as we know, there's
no poisonous seaweed. In fact, it's considered a delicacy in Asian countries, where its been
consumed for thousands of years. A Chinese poem written almost 3,000 years ago describes a
woman who cooks 'sea plants'. Why would anyone want to eat seaweed, you might ask. Well, it's
an acquired taste, but the fact is that seaweed is one of the most nutritious foods available. Low
in calories, it contains proteins and carbohydrates, but it's also rich in a variety of vitamins and
minerals. Interestingly, the minerals in seaweed are found in almost the same ratio as they are in
the human body. There are many different types of seaweed, of course. In Asia about 21 varieties
are used in everyday dishes. Most Westerners are familiar with the variety known as nori, the
dark, crunchy substance used to wrap sushi in. In its natural state, nori is red or purple, but
becomes darker when dried. It's an important part of Japanese cuisine. A popular lunchtime
snack for Japanese children consists of cold rice balls wrapped in a sheet of nori, and another
treat is toasted non flavoured with soy sauce and dipped in sesame seeds. The Japanese started
cultivating nod about 100 years ago because of the increase in demand. Ropes or nets suspended
in the sea are seeded with spores, and when the strands of seaweed develop, they're harvested.
Then they're washed and shredded before being pressed and dried to form paper-like sheets.
These days, Japan produces about 7 billion sheets of nori a year! Asia is not the only place where
seaweed is harvested, of course. For example, kelp has been used for many generations in
Scotland. In fact, the word 'kelp' was first used to refer to the ash produced from burning
seaweed and only later came to be used for the seaweed itself. The ash was used in the
manufacture of glass and soap, because of its high potash and soda content. In recent years, kelp
which has washed up on the coast of Scotland after a storm has been collected and sold for the
extraction of 'alginates', chemicals used for their thickening properties. Alginates can be found in
such diverse products as ketchup, postage stamps, ice cream and beer! In the past, kelp was also
collected by Scottish farmers, who would plough it into the soil as a natural fertiliser. In Wales,
seaweed has been part of the diet for a very long time. A seaweed similar to nori is cooked for
several hours until it becomes soft and jelly-like. Known as laver bread, it forms part of the
traditional Welsh breakfast when eaten with oatmeal and bacon. So, as you can see, seaweed is
very versatile. It's now used worldwide in cooking, but also has medicinal applications because
of its high iodine content. In the future, when conventional fuels become scarce, it may be used
as a biofuel. Hamburgers could be made healthier by having the fat in them replaced with
seaweed extract. Scientists are even looking into the possibility of using seaweed to provide
oxygen for astronauts. It seems that the possibilities are endless. Lowly seaweed should not be
underestimated - something that some cultures have known for centuries.
Exercise 19.

Judith Nixon is the manager of the Farthington Gallery in Russell Square, where an exhibition of
1)_____________ is taking place.

What particularly impresses Judith about many forms of long-distance communication is the
degree of 2)_____________ shown by people who devised them.

Ancient civilisations were not ignorant of the fact that an effective communication network was
a fundamental part of 3)_____________.

The Roman 4)_____________ was the most advanced of its time.

The Romans relied on a 5)_____________ of messengers for speed of delivery.

A boom in international trade in the Middle Ages brought about an increase in


6)_____________.

These reforms were later improved upon as a result of the invention of the 7)_____________.

The Thurn and Taxis family created an extensive network of 8)_____________ covering the
majority of Europe.

The failure of the Pony Express was attributed to the 9)_____________ of the telegraph.

Carrier pigeons proved them to be invaluable as 10)_____________.


1. antique postal items

2. ingenuity

3. governing the extensive empires

4. postal system

5. relay system

6. business correspondence

7. printing press

8. postal routes

9. ill-timed advent

10. war correspondents

Presenter: In our high-tech world of e-mails and text messages we have grown accustomed to
taking ease of communication for granted, and can't even begin to imagine a world where
someone in America or Australia is farther than a phonecall away. Of course, it wasn't always so.
With me in the studio is Judith Nixon, the manager of the Farthington Gallery in Russell Square,
where an exhibition of antique postal items is taking place this week. Judith, welcome to the
programme.

Judith: Thank you.

Presenter: Could you give us a brief summary of the evolution of postal services through the
ages? I gather it's rather a fascinating story?

Judith: Well, I certainly think it is. But then I have always found the various ways in which
civilisations throughout the ages have dealt with the problems of long-distance communication
fascinating, not least for the ingenuity that characterises many of the cases, especially when you
consider the means they had at their disposal in those times. Of course the most obvious way of
sending a message, and one of the first, was by human messenger. One of the most famous
examples of which was Phidippides, the runner who, according to legend, in 490 BC ran the 42.5
km from Marathon to Athens to enlist help from the Spartans against the Persians. Since- good
communications were essential for governing the extensive empires of the ancient world, it is not
surprising that references to runners or messengers on horseback are plentiful throughout history.
Records dating as far back as 4000 years refer to messenger systems in ancient Egypt and relay
stations situated on major roads in the first millennium in China. However, as is often the case
with anything useful, credit has to be given to the Romans for establishing the most highly
developed postal system of the ancient world. Obviously the sheer size and complexity of the
Roman Empire necessitated the need for a reliable and speedy method of communication with
provincial governors in far-flung places. The relay system established by the Romans allowed
their messengers to travel at a speed that could not be matched in Europe until the 19th century.
The next major development occurred towards the end of the Middle Ages when international
commerce began to flourish and with it the growth of business correspondence. In the 13th
century, messenger systems were again adopted to allow corporations to maintain contact with
their customers. In the late 15th century, the trend toward improved postal services was given a
further boost by the invention of the printing press in 1450. One result of that was the increased
profitablilty of letter carrying which in turn led to a growth in private enterprise, the most famous
and extensive by far, being that of the Thum and Taxis family who in the 16th century created an
extensive network of postal routes covering most of Europe. These systems were successful to
the extent that remnants of the system could still be seen in Germany as late as 1867. Presenter:
Well all of these developments are without doubt fascinating Judith, but you promised to tell us
something about the more off-the-wall forms of communication Judith: Yes, yes, I did, didn't I.
Well, one which I'm sure most of your listeners will have heard of is the Pony Express which
operated in the United States for a brief period, from April 1860 to October 1861.This mail
delivery service, like the Romans, also relied upon a relay system to ensure speed of delivery.
However, partly due to the hazardous nature of the territory the riders had to cover, but primarily
to the ill-timed advent of the telegraph, this daring, but ultimately financially disastrous
enterprise was short lived, although it did provide the backdrop for one of the most colourful
characters of the Wild West in the form of William Cody, better known as Buffalo Bill'. Perhaps
more inspiring than plain runners or men on horseback, but just as old, is the notion of the carrier
pigeon. The earliest reference to the domestication of pigeons was found, surprise, surprise, in
Egyptian records dating back to 3000 BC. One of the more romantic notions, although not
proven, is the belief that the ancient Egyptians announced the arrival of important visitors by
releasing pigeons from incoming ships. In more recent times, carrier pigeons were more
practically used during the two World Wars as war correspondents, carrying messages and
microfilm to and from battle headquarters.

Presenter: Well, I can certainly understand why you say that the evolution of the postal system
has been a long and arduous journey. Tell us a little about the actual exhibits visitors can see
when they visit the Farthington this week.

Judith: Well, the exhibition is basically divided into four distinct sections ...(fade)
Exercise 20.

Collisions between the objects in the Earth’s orbit will make everything here be
1)______________ .

Dr Novak describes space junk as being a result of our own 2)______________ .

Valuable orbits are 3)______________ that threatens the operation of satellites and the lives of
astronauts.

Government agencies are now joined by 4)______________ in space exploration.

Discarded spacecraft and equipment constitute a kind of 5)______________

In the 1960s, one astronaut 6)______________ on his camera in orbit.

The erosion of the paint on spacecraft generates a 7)______________ that may damage
everything in orbit.

Given time, 8)______________ or are hit and penetrated by debris.

The continuity of collisions leads to a process termed 9)______________.

Sustainable development means meeting the needs of the present generation without
10)______________ of future generations to meet their needs.
1. ground into celestial scrap

2. environmental negligence

3. peppered with debris

4. commercial enterprises

5. orbital time capsule

6. lost his grip

7. cosmic grit

8. fuel tanks deteriorate

9. collisional cascading

10. compromising the ability

Presenter: Now, as every school pupil can tell you, as long as an object is above the last traces of
Earth's atmosphere, it will stay in orbit for thousands or even millions of years. Eventually, a
month or a millennium after launch, it will hit one of the millions of other objects orbiting the
Earth. That collision will generate new fragments that will go whirling around the planet until
they too are involved in collisions. Over time everything in Earth's orbit will be ground into
celestial scrap. With me in the studio is Dr Frank Novak of the European Space Agency who is
going to tell us more about space junk. Dr Novak.

Novak: Well, venturing into space is inherently risky and orbital debris is just one of the many
hazards that a space traveller faces. But the debris hazard is unique in being a product of our own
environmental negligence. After a mere forty years in space we have seriously polluted the final
frontier. Valuable orbits are peppered with debris that threatens the operation of satellites and the
lives of astronauts. A small group of orbital debris experts have been concerned about this
problem for years and have slowly gained the attention of the government agencies and
commercial enterprises that are now leading the way into space. Yet every four days, on average,
another rocket will be launched into space making the problem worse.

Space may seem remote, but it's really not that far away. If you could drive your car straight up,
in just a few hours you'd reach the altitude at which the space shuttle flies. The popular orbits for
satellites begin twice as. far up - about 400 miles above Our heads. Since 1957 there have been
over 4,000 space launches. The leftovers from these launches - used up satellites, the rockets that
carried these satellites aloft, equipment from aborted scientific experiments - form a sort of
orbital time capsule, a mausoleum of space technology. In 1963, 400 million tiny antennas about
the size of needles were released into orbit in order to see if radio waves would bounce off them.
Communications satellites soon made the antennas obsolete but they still float in lethal clumps
1,500 miles overhead. In 1965, the astronaut Michael Collins lost his grip on a camera while on a
space walk. Many spacecraft shed debris - bolts, lens caps, equipment covers - the way children
shed toys. Even the paint on spacecraft has a tendency to erode in the harsh environment of
space, creating a cosmic grit that now pelts everything in orbit. Many of the objects released into
space, like Collins's camera, have fallen back to Earth. The upper atmosphere, where the space
shuttle flies, gradually slows objects down; they re-enter the atmosphere and burn up within a
few months or even years. But a few hundred miles higher the atmosphere is so thin that it is
ineffective for cleanup. Spacecraft that are launched into orbits at this height will stay in space
indefinitely.

Especially troublesome are pieces of the more than a hundred rockets and satellites that have
exploded in orbit. At the end of their useful lives spacecraft typically contain some fuel left over
from the launch. The fuel tanks deteriorate over time or are punctured by debris. The leftover
fuels mix together and explode. In the worst case on record, the explosion of a European Arlene
rocket produced more than 500 pieces of debris big enough to disable a spacecraft.

Eventually the number of explosions will diminish, but by then spacecraft will be breaking up for
another reason. As more objects go into orbit, spacecraft will begin colliding with - and being
shattered by - debris. Furthermore, collisions beget more collisions in a process known as
collisional cascading. Once cascading begins, the number of objects in a particular orbit will
gradually increase - and the risk to satellites and manned spacecraft will rise accordingly. A team
of researchers in Italy have calculated that enough objects are already present in two popular
orbits for cascading to begin. By the time the cascades have run their course, in a hundred years
or so, even small spacecraft will suffer damaging collisions after just a few years in orbit.

Human societies have done plenty of things that we, or our descendants, may some day regret.
At the beginning of the Atomic Age we seriously polluted vast tracts of and that will take years
to clean up. We have increased the amount of carbon dioxide in the atmosphere despite a
scientific consensus that global temperatures are rising as a result. In 1987 the World
Commission on Environment and Development defined sustainable development as meeting the
needs of the present generation without compromising the ability of future generations to meet
their needs. In space we are failing the sustainability test miserably. A hundred years from now,
when our descendants want to put satellites into orbits teeming with debris, they will wonder
what we could have been thinking. The simple answer is that we weren't thinking at all.
CHAPTER 4: CAE – CPE MATCHING

Exercise 1:

TASK ONE

For questions 1-5, choose from the list (A-H) what unexpected aspect each speaker
encountered while watching the film.

A the versatility of the cast

B the seamlessness of the images

C the authenticity of the scenes

D the film’s sense of humour

E the feeling elicited

F the scarcity of roles

G the impact of the soundtrack

H the intensity of the special effects

Speaker 1 1 __________

Speaker 2 2 __________

Speaker 3 3 __________

Speaker 4 4 __________

Speaker 5 5 __________
TASK TWO

For questions 6-10, choose from the list (A-H) what each person sees as the role of
special effects in films.

A setting the scene

B tricking the audience

C shocking the viewer

D augmenting the viewing experience

E crafting new realities

F affording harmony in the story

G enhancing the characters

H providing excitement

Speaker 1 6 __________

Speaker 2 7 __________

Speaker 3 8 __________

Speaker 4 9 __________

Speaker 5 10 __________
Speaker 1 1 B
Speaker 2 2 D
Speaker 3 3 F
Speaker 4 4 A
Speaker 5 5 E

Speaker 1 6 D
Speaker 2 7 H
Speaker 3 8 A
Speaker 4 9 F
Speaker 5 10 E

Speaker 1

I think it’s important for films not to go overboard with their special effects. It can be tricky, but
when crafted in the right way they can make the film. It’s a fine line but sometimes subtlety is
the key. Special effects should add to and strengthen the film but not take it over. One film
that used its effects well is Forrest Gump. You don’t usually think of it as a special effects film
and that’s down to the way in which the effects were married so well with the rest of the film’s
content. I was astounded by how smooth the scenes were, where Forrest met with past
presidents.
 
Speaker 2

Thor: Ragnarok was a bit of a surprise for me. While I was fully expecting to be blown away by
the special effects, as has become the norm with all the superhero movies, and I certainly wasn’t
disappointed on that score, I was really taken aback by how funny it was. I couldn’t help
laughing out loud at some points. And it wasn’t pointless humour, it added to the story – gave a
light-hearted contrast to the action, which there was plenty of. I especially loved the thrill of
watching the action scenes. The effects were thrilling, as they should be, made even more
breathtaking by the fantastic use of music as an accompaniment.
 
Speaker 3
Special effects have become so commonplace now that they are, I think, being overused. It’s
shocking how many films use them just because they can. To my mind, they should only be
used when they need to be, such as laying the groundwork for fantasies or adventures in
space like Gravity. The special effects in Gravity were excellent but they didn’t stand out, which
meant the director was free to simply tell the story. And the way he did that was exceptional. I
couldn’t believe how effective it was with only two main characters and for a lot of it there
was only Sandra Bullock’s Dr Stone on the screen.
 
Speaker 4

I love the Pirates of the Caribbean films. They’re so funny. I remember watching the first one. I
knew it was going to make me laugh, and it did. And you know what, that was down to the
actors. The character of Captain Jack especially was a revelation. I never would have thought
that an actor could capture a role so completely but … And the special effects also lent to the
humour. They were used at just the right time and at the right pace to allow the plot to flow.
And that’s what effects should do, they should be in tune with the film, not go against the
grain of it.
 
Speaker 5

I love watching films as a way of escaping the everyday normality of my humdrum life. That’s
why I enjoy science-fiction films and fantasies so much. To me that’s what these films are for.
The special effects create other worlds for us to explore through the big screen instead of
just showering us in huge explosions. One that definitely fits the bill is Avatar. It was amazing.
I was so excited when I went to see it, but I didn’t realise how emotionally invested I would
get in the characters. They were created on a computer screen yet I cared about what happened
to them. I laughed and cried with and for them. It was very moving.
Exercise 2.

You will hear five short extracts in which people are talking about an art exhibition
they went to. While you listen, you must complete both tasks.

TASK ONE For questions 1-5, choose from the list (A-H) the reason each speaker
attended the exhibition.

A a friendship with the artist

B a familial connection

C a recommendation from a friend

D a chance encounter

E the lack of an alternative

F the provision of disabled access

G a company trip

H a positive review

Speaker 1 1 __________

Speaker 2 2 __________

Speaker 3 3 __________

Speaker 4 4 __________

Speaker 5 5 __________
TASK TWO For questions 6-10, choose from the list (A-H) what each speaker found
most remarkable about the experience.

A the intricacy of the pieces

B the scale of the attractions

C the use of the space

D the variety of pieces

E the allure of the artwork

F the ability to interact with the exhibits

G the imagination of the artist

H the service of the attendants

Speaker 1 6 __________

Speaker 2 7 __________

Speaker 3 8 __________

Speaker 4 9 __________

Speaker 5 10 __________
Speaker 1 1 G
Speaker 2 2 F
Speaker 3 3 E
Speaker 4 4 D
Speaker 5 5 C

Speaker 1 6 E
Speaker 2 7 H
Speaker 3 8 A
Speaker 4 9 D
Speaker 5 10 G

Speaker 1

You know my idea of a good time is not freezing in sub-zero temperatures in the name of
bonding with colleagues so I wasn’t looking forward to it. But, you know what? I had an
amazing time. In fact, I was so mesmerised by the exhibits I hardly noticed how cold it
was even though it was -10. It was only when I couldn’t push the button on my camera
that I realised my fingers were frozen. Thank goodness I could still operate the camera
with my gloves on cause I didn’t want to miss a thing. I was mesmerised by each piece.
I just couldn’t take my eyes off them. I’m actually gonna go back with my friends. Best
work outing ever!

Speaker 2

We were in the city for the holidays and we’d seen the exhibition advertised online. We
wanted to take in a show while we were there but there were only a few we could go to
cause my mum has mobility issues. This exhibition seemed perfect because they
accommodated everyone and I was particularly impressed with the staff. They
couldn’t have been more helpful. One of them even fetched a chair when my mum
needed a rest. She went home and told all her friends about it she was so pleased. It’s a
sorry fact that not everyone is as helpful as the people at the exhibit and she has run into a
few problems in other places.

Speaker 3

My friend was dying to go to the exhibition but I wasn’t fussed, although to be quite
honest, there was nothing else I wanted to see so it was the best of a bad bunch, I
suppose. Having said that, I did enjoy it. Tiny sculptures that fit on the head of a pin
might not be everyone’s cup of tea, and I didn’t think they would be mine, but what got
me was the attention to detail. It must have been a really talented artist that could carve
that level of detail in something so small. You had to use a magnifying glass, that the
staff provided when you went in, just to see them.

Speaker 4

I’d read about the exhibition and people were talking about it at work, but I didn’t intend
to go until, out of the blue, I bumped into an old friend and can you believe it, he was
with the artist. They had studied at university together. Call it fate if you will but they
invited me to a special viewing and it would’ve been rude to refuse.The exhibition was
quite spectacular. I couldn’t believe all the different things it had. There was
everything from life-size sculptures to interactive exhibits where you could create
your own art no matter your artistic level. I painted a picture using a wheelchair while I
was sitting in it. That was interesting!

Speaker 5

It’s a wonder what kind of a mind can create pieces of art, big and small. I’m not
artistically inclined so it always amazes me. Whether they’re a painter, a sculptor or they
use some other method, I’m always curious as to how they come up with the idea for the
piece, what inspires them. My neighbour, who I’ve been close to for years, told me
about an exhibition of her brother’s work and I decided to check it out. The theme of
the exhibition was ‘space’ and I could see how each piece related but I couldn’t figure
out the thought process that led him to each piece. It was a great exhibition but I’m
still puzzled by that.
Exercise 3.

You will hear five short extracts in which different people talk about doing without
their mobile phones. While you listen, you must complete both tasks.

Task 1 For questions 1 — 5, choose from the list (A — H) what reason each speaker
gives for doing without their mobile phone.

A it was too distracting

B it disappeared

C it was broken beyond repair

D it was someone else's fault

E it was not waterproof

F it was stolen

G it was lost

H it needed expert attention

1 Speaker 1 __________

2 Speaker 2 __________

3 Speaker 3 __________

4 Speaker 4 __________

5 Speaker 5 __________
Task 2 For questions 6 — 10, choose from the list (A — H), what each speaker
missed most about their mobile phone.

A being connected

B making phone calls

C social media

D some personal pictures

E favourite apps

F getting woken up

G games and email messages

H a complete list of contacts

6 Speaker 1 __________

7 Speaker 2 __________

8 Speaker 3 __________

9 Speaker 4 __________

10 Speaker 5__________
Speaker 1 1 E
Speaker 2 2 B
Speaker 3 3 H
Speaker 4 4 A
Speaker 5 5 C

Speaker 1 6 D
Speaker 2 7 H
Speaker 3 8 B
Speaker 4 9 F
Speaker 5 10 A

Speaker One

It happened because my mother changed the sheets and, as she told me later, she heard a banging
sound in the washing machine. She stopped the programme because she was curious about the
noise and of course, there was my phone ! It had obviously got left on my bed, so it was my fault
really. We tried everything to dry it out, putting it in a bowl of rice overnight, you know, as they
suggest you should do, but the water had already done too much damage and the phone
never worked properly again. It was an old one, so it didn't matter too much but it was
annoying that I couldn't retrieve one or two things, some of the first photos I took of my
friends are gone forever. It took me a few days to catch up on my social media updates when I
finally got a new phone set up. Some of my friends thought I'd disappeared!

Speaker Two

I was late for work and in a bit of a rush. When I got to work, I realised I didn't have my phone,
and thought I'd probably left it charging at home. But when I got home that evening, it wasn't
there. Then I called the taxi firm, because I'd taken a cab to work that morning. But they said
nothing had been handed in. To this day, I don't know what happened to it, whether I lost it
or someone stole it. It was almost new and quite an expensive model. Luckily, I was able to
erase it remotely and then I knew that my personal information was safe. There were a lot of
contacts on there that I wasn't able to recover and l had to start again with a new device the
next day. I'm going to be a whole lot more careful with my mobile in future!

Speaker Three
Well, it's inevitable, isn't it, that technology will let you down? For me, it couldn't have happened
at a worse time. I was on a business trip and my mobile suddenly just froze. I tried everything
you are supposed to do, I tried taking out the battery and putting it in again, turning it off and
turning it on again and fiddling with the various cards, but nothing worked. It was really a
disaster because it meant that I could only contact people using my laptop, so I was a bit
lost for a day or two, not being able to call people. I had to wait till I was back home to get
it fixed by a specialist and those two days without it made me realise how much I rely on it.

Speaker Four

I wanted to be offline for a few days. I hate the idea that people expect me to be available at all
times. So, to relax properly on my fishing trip, I turned off my phone because I didn't want
any interruptions. I don't think anything is so important that it can't wait forty-eight hours and I
knew if something was really urgent, my family would alert my companion. Of course I forgot
that having turned off my phone, the alarm wouldn't ring and I didn't get up at the crack
of dawn as I was planning. But apart from that, yes, I can recommend turning your phone off
from time to time, to really recharge your own batteries.

Speaker Five

I panicked a bit when it happened. It just fell out of my hand and onto the kitchen floor. I
immediately knew it was broken, the screen was smashed to pieces. I was so cross with
myself. That thing was my constant companion; my whole life was stored in there, from work to
games to correspondence, videos and photos - everything. Thank goodness I had saved the
important stuff to the cloud, as they call it. I had to replace it, of course, but for the short time I
was without it, I felt lost and disconnected from the world. Terrible, isn't it, that we should be
so reliant on these devices?
Exercise 4

You will hear five short extracts in which different people are talking about the
breakdown of a friendship. You will hear the recording twice. While you listen, you
must complete both tasks.

Task One

For questions 21-25, choose from the list (A-H) the reason each speaker gives for
their friendship ending.

A not making time for each other

B a misunderstanding

C lack of reciprocation

D a betrayal

E character differences

F inability to adapt to change

G frequent arguments

H avoiding issues

Speaker 1 21 __________

Speaker 2 22 __________

Speaker 3 23 __________

Speaker 4 24 __________

Speaker 5 25 __________
Task Two

For questions 26-30, choose from the list (A-H) how each speaker felt about their
friendship ending.

A they felt blamed for it

B they were indifferent to it

C they were not surprised

D they were filled with anger

E they felt guilty

F they were puzzled

G they did everything to prevent it

H they wish it had happened sooner

Speaker 1 26 __________

Speaker 2 27 __________

Speaker 3 28 __________

Speaker 4 29 __________

Speaker 5 30 __________
Speaker 1 1 E
Speaker 2 2 F
Speaker 3 3 C
Speaker 4 4 H
Speaker 5 5 D

Speaker 1 6 C
Speaker 2 7 B
Speaker 3 8 F
Speaker 4 9 E
Speaker 5 10 D

Speaker 1:

It took another year or so before the inevitable happened and we finally stopped hanging out.

I guess everyone has a different tolerance level for negativity and I couldn’t just take Alex’s
gossiping..... I’m relieved I don’t have to be around her toxic words and constant eye
rolling anymore.

Speaker 2:

When Charlotte got married, she started spending less time with me...it was more that her
situation just didn’t match mine any more. We were doing different things ... we wanted to do
different things ... our social circles became completely different.

... but she seemed totally dispassionate about the whole thing. Made me realise I wasn’t
losing anything letting go.

Speaker 3:

It was always me there for him but not the other way round.

Sometimes you just can’t figure out why you start feeling differently about your friendship.

Speaker 4:

It may seem counterintuitive but it was actually the lack of conflict that was the sign that our
friendship was over. It meant Emma and I had both given up on our friendship and were ready
to move on. You have to confront problems when they arise. Arguing can actually be good.
Still, I should stress even though I was ready to let go, it didn’t mean that I didn’t feel bad
about doing so, or responsible. Because I did.

Speaker 5:

...at first I gave him the benefit of the doubt. (to decide that you will believe someone, even
though you are not sure that what the person is saying is true). After all, I hardly knew the person
who told me the story. But I was wrong to.

But at the time my only thought was how do I prevent this person from ever hurting me
again? And the answer was: don’t even speak to him again.
Exercise 5. You will hear five short extracts in which different people are talking about a
heated argument they had. You will hear the recording twice. While you listen, you must
complete both tasks.

Task One
For questions 21-25, choose from the list (A-H) what each speaker did to cause the
argument.
A told serious lies
B damaged personal property
C read a private letter
D made a false accusation
E revealed confidential information
F made someone feel embarrassed
G omitted to tell someone something
H borrowed something without asking
Speaker 1 21 __________
Speaker 2 22 __________
Speaker 3 23 __________
Speaker 4 24 __________
Speaker 5 25 __________

Task Two
For questions 26-30, choose from the list (A-H) what each speaker did to resolve the
situation.
A made a promise
B took someone out for a meal
C paid compensation
D made a joke
E begged for forgiveness
F wrote a letter of apology
G put the blame on someone else
H bought a present
Speaker 1 26 __________
Speaker 2 27 __________
Speaker 3 28 __________
Speaker 4 29 __________
Speaker 5 30 __________
21 F

22 D

23 C

24 G

25 H

26 E

27 F

28 A

29 H

30 B

I was doing a series of lessons on technological change and thought it would be a good idea to
take in an old phonograph that belongs to my grandmother so that they could appreciate the
advances we've made. The whole class found the demonstration and the old music I played both
fascinating and amusing, so I was satisfied. When I returned the phonograph, my grandmother
accused me of taking it without permission. She also said I was responsible for a crack in it,
which had been there when I got it. I should have been more sensitive, but I snapped and said
things about her forgetfulness. Immediately, I could see how distressed and uncomfortable she
was. Before I could make amends, my mother had leapt to her defence and we ended up in a
screaming match. Eventually, I accepted the blame, apologised and literally pleaded with
them to put an end to the resentment. They did, and we're back on speaking terms.

It all started when I got wind of her seeing someone else. I couldn't remember any details
because I just saw red. I didn't even stop to think about the possibility that someone had made it
all up so that we wouldn't stay together. Looking back, I probably should have done. So, there I
am, all worked up and I confront her head on without asking her whether there was any
substance to it. To cut a long story short, we found out that someone was jealous of us, but l still
had to take the blame. She said she would only forgive me if I admitted my mistake in black
and white. I had no idea why she insisted on that and I didn't ask. I just did it . Perhaps we'll
look at it one day and have a laugh about it.

I suppose one of my biggest faults is that if someone has a go at me when I've done something
wrong, I'll turn it into an argument by accusing them of having done something had to roe. That's
what happened with my flatmate, Andrew, the other day. Luckily, he knows what rm like, so he
said he'd forget about it if I gave him my word that I'd never do it again. Although he didn't
catch me red-handed, it was no use denying I'd done it because when I heard him come in, I
had to get it back on his desk so fast that I didn't put it exactly where he'd left it. And when
you're as neat and tidy as Andrew is, you notice when something is out of place. As for the
contents, my lips are sealed. After all, I value our friendship and I want it to stay that way.

I know it's contentious but I don't think lying includes not telling somebody everything. I mean,
where do you draw the line? How was I supposed to know what is important enough to reveal
and what isn't? Jennifer, on the other hand, thinks that keeping things back is tantamount to
telling a lie. There was no convincing her she was wrong and I wasn't prepared to argue any
longer. I didn't want to admit she was right either, so I got her a little something she'd had her
eye on for some time. Was I paying her off? Sure, but it was worth it to end the tension that was
building up between us.

I think the best way to appease someone is to do something you'll enjoy as well and to do it
quickly. That's what I did after I'd upset my flatmate, Christina. We went to that new Thai
place in town which she said she'd like to go to and where I could sample new delicacies.
Everything was or me, of course. She must have appreciated the gesture because she hasn't said a
word about what happened the night before since then. Thursday's usually her night off, so how
was I to know she needed it because she'd swapped shifts with one of her colleagues. I
admit she had a right to be annoyed because it is hers, but I think she went over the top
and treated me as if I'd scratched the paint on it or something. Anyway, it's all water under
the bridge now.
Exercise 6.
You will hear five short extracts in which different people are talking about their
postgraduate study courses. You will hear the recording twice. While you listen, you must
complete both tasks.
Task 1: For questions 21-25, choose from the list (A-H) what reason each speaker gives for
beginning the postgraduate course.
A. improve job prospects
B. make new contacts
C. facilitate a change in career direction
D. satisfy a personal interest
E. delay an inevitable job search
F. avoid redundancy
G. improve understanding of a particular subject
H. enter a completely different profession
Speaker 1 21 __________
Speaker 2 22 __________
Speaker 3 23 __________
Speaker 4 24 __________
Speaker 5 25 __________

Task Two
For questions 26-30, choose from the list (A-H) what benefit each speaker expects from
having that particular postgraduate qualification.
A to relocate permanently
B a greatly improved income
C more time for family
D a better benefits package
E a better idea of which area to specialise in
F an activity to fill his time
G a contact in academia
H a respectable qualification
Speaker 1 26 __________
Speaker 2 27 __________
Speaker 3 28 __________
Speaker 4 29 __________
Speaker 5 30 __________
21 A

22 E

23 C

24 D

25 G

26 B

27 A

28 C

29 F

30 E

Speaker 1

When I began my MBA, I had already been in employment for five years, and I was doing pretty
well, but I realised I could do better. There's just so much competition these days, so you really
need to have that extra edge that makes you more attractive to the employer. I estimate that
once I've completed my MBA, I will be able to command a 30-40% higher salary and I will
be much more desirable to blue chip companies. When I first graduated, I don't think I was
anywhere near as ambitious as I am now. But it feels good. It was no easy task getting on my
MBA course; I had to try two years running. But now I'm at the London School of Business and
I'm making great contacts — it's expensive, but it's worth every penny.

Speaker 2

Most of the people on my course are older than me and they already have experience in this
sector under their belts. For them postgraduate study is a way to improve their skill set, whereas
for me, it's an alternative to unemployment! Consequently, I think I'm finding the practical
side a bit tougher than them, but I'm sure I'll find my feet. I had expected to see more overseas
students on the course like me, doing a postgraduate course as a way to ride out the economic
crisis. It's almost impossible for graduates to find jobs at the moment, but maybe they struggled
with the fees. I was extremely lucky to get funding and I'm lucky to have a family to stay with
while I do this. I can't ignore the fact that being able to study like this abroad is a way for
me to actually get out of my own country and work somewhere with more opportunities.

Speaker 3
I'm not new to postgraduate study, this is actually my second postgraduate course, and a lot of
my friends thought going down this road was ridiculous since I'm already doing well teaching in
businesses. But for me a PGCE will solve a lot of problems. It will be a way to get into the
state teaching system, get a steady salary, a decent pension and move forward in a sector I think
I'll enjoy. I'm a single mum, and not being available for my son after school and during the
holidays is a big problem. A job in school teaching would put an end to that. And if I get a
job in a really good school, then I can get my son in too. I was able to get funding because
Modern Language teachers are in demand, so it was something I could achieve financially. The
greatest difficulty for me was that in order to get accepted onto the course, I had to have a basic
maths qualification, so I had to complete the GCSE that I failed all those years ago at school
myself!

Speaker 4

I began this postgraduate course for no other reason than a passion for learning. Actually,
I do have a specific interest — Byzantine History — I find it fascinating, and now I'm
retired, I was crawling up the walls with boredom. The way we are expected to study is much
more collaborative than when I was a young student, which proved a problem at first. I'm in a
group and as the oldest, everyone looks to me for guidance. Working alongside a group of young
people has made me feel at least ten years younger and made me realise how much I've missed
teaching myself. After the course I am going to offer my services at the adult education centre
free of charge — they're looking for a Byzantine specialist.

Speaker 5

I took it for granted that I would move straight into postgraduate study because I want to pursue
a career in academia and in order to do that I need to get a PhD under my belt. For me this
postgraduate course is a stepping stone towards that PhD. It will help me broaden and intensify
my subject knowledge in biochemistry in order to identify exactly what my research area
should be when I begin the PhD proper. I was very careful when choosing my university to be
at an establishment which would provide the appropriate environment, and Herriott-Watt does.
Excellent academic staff and I've already been offered a provisional place on a post-doctorate
research project. It seems far away but the kind of research I want to be involved in will take
years to undertake.
Exercise 7.

You will hear five short extracts in which students talk about the people who have inspired
them. You will hear the recording twice. While you listen, you must complete both tasks.
Task 1: For questions 21-25, choose from the list (A-H), why the speaker looks up to that
particular individual.
A They never gave up on their dream.
B They offered a safe haven and guidance during a troubled time.
C They are battling with a serious health issue.
D They possess a kind and charitable spirit.
E They have shown great courage and strength of character.
F They became hugely successful in their field.
G They have used their talent to do charity fundraising.
H They have shown great ambition.
Speaker 1 21 __________
Speaker 2 22 __________
Speaker 3 23 __________
Speaker 4 24 __________
Speaker 5 25 __________

Task Two
For questions 26-30, choose from the list (A-H) how that person has affected the speaker.
A They have given the speaker the strength to cope with severe adversity.
B They have made the speaker feel kinder towards people.
C They have guided the speaker's personal life choices.
D Their 'tough love' attitude made the speaker see sense.
E They have involved the speaker in charity work.
F They saved the speaker's life
G They have inspired the speaker to be a good parent
H The speaker aspires to emulate them in their professional life.
Speaker 1 26 __________
Speaker 2 27 __________
Speaker 3 28 __________
Speaker 4 29 __________
Speaker 5 30 __________
21 C
22 B
23 E
24 A
25 D
26 A
27 F
28 G
29 H
30 B
You now have forty-five seconds in which to look at Part 4.

PAUSE 45 SECONDS

[sound]

Speaker 1

PAUSE 2 SECONDS

When my husband was diagnosed with Parkinson's a few years ago, I was devastated. I couldn't
face the fact that we would possibly not grow old together. Then I saw the actor Michael J Fox
on TV. He's younger than my husband and has had to live with the same debilitating
disease for many years now. What impressed me was his unwavering optimism and
determination not to let it take over his life. I cried a lot that day, but it was for the last time. I
knew in my heart we had to make the most of the days we have together. and celebrate all the
good things about our life rather than mourn a future loss.

PAUSE 3 SECONDS

Speaker 2

PAUSE 2 SECONDS

Living in the inner city, when I was 17, I was hanging out with the wrong crowd and had
fallen into seriously bad habits. My dad would have none of it and when he saw I wasn't
going to straighten up, he packed me off to my uncle who owned a farm in the north .
During the first few weeks, I had the worst time of my life and I would have managed to run
away if he hadn't proved far cleverer than I was. With time, I came to respect him for being the
most genuine, caring, kind person I'd ever met. He taught me to respect myself and nature and
how to live in harmony with it. Honestly, but for him, I might not be standing in front of you
now.

PAUSE 3 SECONDS
Speaker 3

PAUSE 2 SECONDS

From what I once read in an article, we usually put our parents up on a pedestal when we're kids
only to bring them crashing down as teenagers, but for me, that was never the case. We lost my
dad when I was two and my mum had to raise us, that is, my two brothers, two sisters and
myself, all on her own. She was the most hardworking person I've ever known, a strong,
unbreakable spirit — never complaining, never indulging in self pity. I grew up to be a
fighter — as independent and responsible as she was. I just hope I'm able to teach my kids a
fraction of what she taught me.

PAUSE 3 SECONDS

Speaker 4

PAUSE 2 SECONDS

It might sound corny as, especially when she first became a success, the rags-to-riches story of
JK Rowling was sprawled all over the papers and magazines but, for me, it's not just that. Don't
get me wrong, I do admire how she rose from being a destitute single mother who had to resort
to writing her novel in cafés because she couldn't afford heating her flat to being one of the most
celebrated novelists of our time. But what I find absolutely fascinating is how she persisted
after countless agents and publishers turned down her work. She's a person who really
believed in herself and her talent and I hope that I can be the same when it comes to
dealing with the music industry.

PAUSE 3 SECONDS

Speaker 5

PAUSE 2 SECONDS

I sometimes quote some obscure philosopher when I want to impress at parties, but the two
people who single-handedly stop me from despairing of the human race are Tania and
Jeremy, whom, I may add, I've known since nursery school. My first recollection of them,
actually, is the two of them feeding their lunch to the birds on a day it had snowed because they
feared the poor creatures wouldn't be able to fend for themselves. Thirty years later, they run one
of the most ambitious food-bank projects, collecting unwanted food from restaurants and
supermarkets to distribute to the homeless or young families in need. Their unflinching
dedication and commitment have somehow rounded the sharp edges of the world view of
such a dyed-in-the-wool cynic as I think myself to be.

PAUSE 10 SECONDS
CHAPTER 5: AUTHENTIC LISTENING

Exercise 1:

https://www.npr.org/transcripts/795235653

Australia possesses an 1.________________ as it hasn't had a recession in nearly three decades.


But the deadly wildfires raging through large parts of the country are threatening key parts of the
country's economy.

The 2.________________ run by Fiona Austin is usually full in January, but tourists were
ordered to evacuate, and only a few people remain.

Things are becoming so volatile and people are still unsure as to whether they could
3.________________ again.

Australia calls itself the 4.________________, a nation so fortunate in geography and natural
resources that the economy has been growing steadily since 1991.

The wildfires have destroyed both residences and 5.________________. Besides, ferry service in
the city's world-famous harbor has sometimes been canceled because of 6.________________.

The Australian Open and the Tour Down Under 7.________________ are mentioned as major
events that may be rescheduled.

Housing prices, which have been 8.________________ in recent years, have fallen to a deep
low.
1. enviable economic record

enviable: causing envy

Collocation: enviable reputation/position/record/ability

Ex: Wellington College is one of the world’s leading co-educational boarding and day schools
and has an enviable reputation for excellence and innovation.

Japan is in the enviable position of having a budget surplus.

Matt Haig is a writer of admirable versatility (children's books, nonfiction, adult novels) and an
enviable ability to produce bestsellers.

Edinburgh boast the enviable record of being the only unbeaten team in Europe after five
rounds of the Heineken Cup.

2. campground

3. flare up : suddenly start again (disaster, disease, fire and anger)

The bushfire emergency in central Queensland flared up on Sunday evening with residents in
the path of the massive Deepwater blaze told to leave immediately.

4. lucky country

5. prime farmland

6. poor visibility

7. bike race

8. skyrocketing: to rise quickly to a very high level (prices, bills, costs and wages)

The industry's development costs will skyrocket. The time of cheap energy resources, cheap gas
is surely coming to an end.

Transcript

LULU GARCIA-NAVARRO, HOST:

Australia has gone nearly three decades without a recession. It's an enviable economic record.
But as NPR's Jim Zarroli reports, the Australian wildfires are threatening key parts of the
country's economy.

JIM ZARROLI, BYLINE: Normally, the campground run by Fiona Austin near the Australian
city of Shoalhaven is full in January, but tourists have been ordered to evacuate the area. And
with the fires still raging, she doesn't expect them to be back anytime soon.
FIONA AUSTIN: There is a lot of fear because they're being so changing and volatile. You
know, people are still unsure as to whether they could flare up again.

ZARROLI: As planes carrying water to the wildfires buzz overhead, Austin tells NPR her
campground is empty right now except for a few permanent residents.

AUSTIN: We're on 15 acres so - yeah. To only have a couple of tents here - here comes another
plane - is very unusual for us.

ZARROLI: Australians call themselves the lucky country. The economy has been growing
steadily since 1991, a remarkable run. Economist Justin Wolfers of the University of Michigan
says that's partly because the population has grown a lot. But, he says, the country's been
fortunate in some other ways, too.

JUSTIN WOLFERS: Not only did we start the last few decades a relatively rich country and in
the club of the first-world industrialized countries; we're also parked right next to Asia, which is
where much of the world's growth has come from over the past few years.

ZARROLI: As China has grown, it's been hungry for the kinds of commodities Australia has a
lot of, like coal, natural gas, wheat and wool. China sends more tourists to Australia than any
other country. But the rampaging fires are dealing a blow to the economy.

Martin North heads the research firm Digital Finance Analytics.

MARTIN NORTH: Just the area of Australia that's now impacted is unheard of. So we are in
uncertain territory.

ZARROLI: The fires have destroyed more than 1,800 homes, as well as enormous amounts of
prime farmland. Even in places far from the fires, work life is being disrupted. People with
respiratory problems are staying home. Hospital visits are up. And construction crews can't
work. In Sydney, ferries aren't running because of poor visibility in the harbor, says Katrina Ell
of Moody's Analytics.

KATRINA ELL: There was a few days late December when fire alarms were actually going off
in very large buildings within the city center just because of the poor air quality.

ZARROLI: As word of these conditions spreads around the world, tourism is taking a big hit.
There's even been talk of rescheduling some of the big events that draw in millions of visitors
each year, like the Australian Open and the Tour Down Under bike race. Martin North says this
is happening at a time when the Australian economy was already softening a bit.

NORTH: We were already looking, I think, pretty shaky. And that was before all the bushfires.

ZARROLI: China's economy has slowed lately, and Australia has felt some pain. Unemployment
ticked up last year. House prices, which have been skyrocketing for a long time, have taken a
hit. Katrina Ell of Moody's Analytics doesn't think Australia is headed for a recession, but it's
hard to know for sure.

ELL: What's really concerning to us is that this is still relatively early in our typical bushfire
season. So there's concern about how much longer this bushfire season will run for.

ZARROLI: The longer the fires last, the more damage they will do. And that means that after
almost 30 years of steady growth, the lucky country could finally see its luck run out.

Jim Zarroli, NPR News.

Exercise 2:

https://www.youtube.com/watch?v=_tcXZJd9vKM
Koalas are being particularly 1.________________ in New South Wales as their habitats are
being destroyed.

It is estimated that hundreds of koalas have died in 2.________________ raging in Eastern


Australia since September.

While the disaster has inflicted serious damage on the animal, the support from the public has
been 3.________________.

A large amount of money has been set aside to help 4.________________ koalas.

There are concerns that a new heat wave could further intensify the raging 5.________________.

Rising temperatures, which dry out their habitats, deforestation and disease are
6.________________.

1. hard hit: badly affected


Pensioners, and in particular those dependant on state benefits, have been hard hit by the
increase in oil and other commodity prices.

2. deadly bush fires

3. equally overwhelming

4. save and rehab

5. infernos

inferno: a very large dangerous fire that is out of control

Ex: 108 people have been killed in Australian inferno. Bodies are still being removed from the
burned-out shells as survivors describe the firefront as ferocious and fast.

6. taking a toll

If something takes its/a toll, it causes suffering, deaths, or damage

Bangladesh's road accidents take heavy toll on poor families, and cost Bangladesh almost as
much as it receives in foreign aid.

Transcript

HOWELL: In Australia, as bush fires rage on, we now know that they're having a devastating
impact on koalas. Koalas are being particularly hard hit in New South Wales as their habitats
are being destroyed. My colleague Natalie Allen explains, there is hope as experts fight to save
one koala at a time.

(BEGIN VIDEOTAPE)

NATALIE ALLEN, CNN CORRESPONDENT (voice-over): A koala hospital in Australia


overwhelmed with patients with wildfire injuries.

UNIDENTIFIED MALE: He seems to be quite dehydrated and probably quite hungry.

ALLEN (voice-over): Not all were lucky like this little one. Animal experts estimate hundreds of
koalas have died in deadly bush fires raging in Eastern Australia since September.

Australia's environment minister saying on Friday up to 30 percent of koalas in the country's


New South Wales region may have been killed in the ongoing fires. The region's Port Macquarie
Koala Hospital has admitted more than 300 of these adorable animals so far this year.

But the hospital's director says the support from the public has been equally overwhelming. The
hospital has received over $2 million in donations on the GoFundMe page site.
UNIDENTIFIED FEMALE: The number of visitors that are here at the Koala Hospital over the
last 5-6 weeks has been phenomenal. Everyone is so afraid that we're going to lose koalas that
they want to come and see them. That's the power of the koala worldwide.

ALLEN (voice-over): The government says it's working with koala experts and $6 million has
been set aside to help save and rehab them. A new heat wave is expected to hit parts of the
country over the weekend and into next week, raising fears that high temperatures and dry winds
could further intensify the raging infernos.

That's all bad news for the already vulnerable koala community.

UNIDENTIFIED MALE: Very little would actually survive in there unscathed. Wallabies,
kangaroos, deer would get out because they can run but koalas just really can't.

ALLEN (voice-over): The Australian Koala Foundation says the numbers are dwindling and the
species is at risk of extinction if the population continues to shrink. Rising temperatures, which
dry out their habitats, deforestation and disease are taking a toll. The staff here hope his fate will
be different in a time when the death of even one koala could be one too many -- Natalie Allen,
CNN, Atlanta.

Exercise 3.
There are signs that the 1.________________ impact of the Coronavirus outbreak is more
significant than initially expected.

Rapid selling of securities came in the wake of 2.________________ restrictions imposed in the
city of Wuhan.

The government extended the holiday's usual weeklong 3.________________ by three days.

Freya Beamish fears it might have been too late for the 4.________________ to contain the
epidemic.

Global stocks dropped heavily, which is a contrast with last week when investors were mostly
5.________________ by the outbreak.

According to Jay Bryson, as China's position on the world stage grows, its faltering economic
activity is likely to exert some 6.________________ to trading partners.

7.________________ are mentioned as types of businesses which could suffer heavily due to
lower demand from China.

On the other hand, the economic storm brings a host of opportunities for companies which
produce vaccine or 8.________________.

1. financial/economic
2. travel

Ex: The UK’s trade association for holiday operators and travel agents cast doubt on a
sufficiently rapid lifting of social distancing or travel restrictions imposed due to Covid-19.

3. factory shutdown

4. quarantine: a period of time when an animal or a person that has or may have a disease is kept
away from others in order to prevent the disease from spreading

French officials have announced that Britons and those from European Union member states will
not be placed in 14 days mandatory quarantine if they travel to France, as suggested by the
country’s health minister.

5. unfazed: not worried or surprised by something unexpected that happens

Global stock markets have been largely unfazed by the news of President Trump being
impeached, as investors widely expected the US Senate to vote against his removal from office.

6. spillover effects

Spillover effect refers to the impact that seemingly unrelated events in one nation can have on
the economies of other nations.

Ex: Even if an individual country is fortunate enough to escape widespread viral contagion, the
spillover effects from global developments or broken supply chains may still lead to faltering
economic activity.

7. Airlines and resort companies

8. protective masks

ARI SHAPIRO, HOST:

American financial markets are not immune to a new respiratory virus that has spread quickly
from China. Stocks fell sharply today on fears the coronavirus could take a larger economic toll
than initially expected. The virus has sickened thousands of people and killed more than a
hundred. As NPR's Scott Horsley reports, there are signs that financial fallout, like the virus
itself, might not be easily contained.

SCOTT HORSLEY, BYLINE: The sell-off came as China's government moved aggressively to
limit travel in and out of the region where the outbreak began. Tens of millions of Chinese
citizens are now blocked from traveling around the country at the height of the Lunar New Year,
typically a busy travel season. What's more, the government extended the holiday's usual
weeklong factory shutdown by an extra three days.
FREYA BEAMISH: Probably, investors are kind of waking up to that story.

HORSLEY: Freya Beamish is chief Asia economist for Pantheon Macroeconomics. She worries
the quarantine comes too late to stop the spread of the virus since numerous infected
people were already on the road. But she says the travel restrictions will put a serious damper
on holiday shopping and excursions.

BEAMISH: It seems like the worst of both worlds for the macroeconomy.

HORSLEY: Investors in Japan thought so. The Nikkei stock market suffered its worst drop in
five months today. European markets also slumped. Here in the U.S., the Dow Jones Industrial
Average tumbled more than 450 points. That's a contrast with much of last week, when U.S.
investors were largely unfazed by the outbreak. When I spoke with chief investment officer
David Kotok of Cumberland Advisors a few days ago, he warned investors were underestimating
the potential fallout.

DAVID KOTOK: The markets who are saying, in my opinion, this is nothing more than a cold
and sniffle - business as usual - are not evaluating the risk well enough.

HORSLEY: Part of the challenge for forecasters is so much about this virus is still unknown.
Many are looking for a model in the SARS outbreak of 2002 and 2003, which killed more than
700 people. At first glance, this virus appears to be less severe, but Jay Bryson, acting chief
economist at Wells Fargo Securities, says China is a much bigger player on the world stage now,
so any fallout will be amplified.

JAY BRYSON: What happened is the size of the Chinese economy has more than doubled over
that period of time, and so if it were to slow down significantly because of this, you know, that
could have some spillover effects to some of its trading partners.

HORSLEY: In 2003, China was still a newcomer to the global trading system. Todd Lee of IHS
Markit says today it's much more integrated with the world's economy.

TODD LEE: Obviously, it depends on, you know, whether or not the government can effectively
contain the outbreak, but in terms of the supply chain disruption, it will be much bigger than
before.

HORSLEY: Lee says China's economy is also more fragile today. The coronavirus emerged after
a period of slowing growth and a tense trade war with the United States. China's consumers play
a bigger role in the country's economy now than they did back in 2003, and so far, that's where
most of the costs of this outbreak have appeared. Airlines and resort companies have seen
their stocks fall in anticipation of reduced demand from China. The outbreak could also
make it harder for China to make good on the big purchases of U.S. goods that were promised in
the newly announced Phase 1 trade deal.
Every economic storm brings a silver lining of opportunity, though. Some of today's big winners
on Wall Street include a company that's working on a vaccine for the coronavirus as well
as a firm that makes protective masks. Scott Horsley, NPR News, Washington.

Exercise 4.
Preliminary findings from the UK showed that dexamethasone, a 1.________________, could be
lifesaving for COVID-19 patients on ventilators.

Kirsten Lyke, who runs a coronavirus vaccine trial at the University of Maryland, believes that
there are certain issues that need to be 2.________________.

As the press release only gives the brief results, she is waiting for more in-depth information
from the 3._______________. Also, it needs seven years to ascertain the patients get positive
results as hasty release and unreliable intervention might 4.________________.

In general, Lyke is 5.________________ about the outcome of the trial.

Doctors got into trouble with 6.________________ such as hydroxychloroquine and


azithromycin, but Lyke believes this is not the case for dexamethasone, which doctors are
familiar with.

People are becoming 7.________________ with the government’s inconsistent guidelines on


mandatory mask wearing.

America's 8.________________ is an Achilles' Heel when it comes to saving lives in a


pandemic.

1. low-cost steroid: một loại hormone tổng hợp


2. unwound

3. peer-review paper

4. erode public trust

5. cautiously optimistic

6. new interventions

7. exasperated: bực tức

8. individualism

Is dexamethasone good news in the treatment of coronavirus or another exaggerated or false


hope? Doctors in the U.K. say that dexamethasone, a low-cost steroid, reduced deaths in
COVID-19 patients on ventilators. But the findings are preliminary and come just as we've seen
several other prominent revisions in the advice that scientists have offered around the
coronavirus.
Kirsten Lyke runs a coronavirus vaccine trial at the University of Maryland and joins us now.
Thanks so much for being with us.
KIRSTEN LYKE: Thank you for asking, Scott.
SIMON: I gather this U.K. trial was led by a team at Oxford. They say using this steroid could
save one life for every eight people on a ventilator. But as I don't have to tell you, Dr. Lyke,
people want to hear, should I take it or not?
LYKE: You know, it's an easy answer to say yes or no, but these are very complex things that
need to be unwound. This is a press release, so they're going to basically give us sort of the
bottom line. But many of us would like to see the peer-review paper to understand how these
people were randomized, who was not randomized - that's important to know - and, you know, a
lot of other questions to give us a little bit more understanding as to the results.
SIMON: If we were living in a world without a pandemic, how long would it take to get the
results you need to be really confident about how to use this drug?
LYKE: Yeah, interesting that you ask that question because this year, there was a
groundbreaking publication that demonstrated if you start steroids early in the evolution of acute
respiratory distress syndrome, or ARDS, that you can get some positive results. It took them
seven years from...
SIMON: Yeah.
LYKE: ...Start to results. And so, you know, we're in unprecedented times, and people really
want to get results out quickly. But at the same time, if things are released too early or there's
harm that occurs from the intervention, that really erodes public trust.
SIMON: The researchers who led this trial, I gather, say that dexamethasone should immediately
become standard care in patients with severe cases of COVID. It sounds like you're
uncomfortable with that.
LYKE: Well, I - you know, I'm cautiously optimistic. This study - it was only people who were
on oxygen or people that were ventilated...
SIMON: Yeah.
LYKE: ...That got the significant improvement. And people that were not on oxygen actually
tended towards some harm.
SIMON: At the same time, Doctor, can you understand families who say, look; we have a loved
one who is very sick, and we're willing to take that chance?
LYKE: Yeah, I can totally understand that. And that's not just the patients; that's also the doctors.
And early on in this pandemic, everyone wanted to have an intervention. Everyone wanted to do
something. Sometimes doing something feels better...
SIMON: Yeah.
LYKE: ...Than just sitting back and waiting. And I think that's why we got into trouble with
hydroxychloroquine and the concomitant drug, azithromycin, which is an antibiotic. You know,
the two together really, probably, had a harmful effect, particularly with the conduction of the
electrical waves in the heart. So we'll be able to unravel this in retrospect, but I think we just
need to be extremely cautious when we implement any new interventions. Dexamethasone is not
a new intervention, so we do have a lot of background with dexamethasone. I think we just need
to know when's the best time to treat our patients and not induce harm.
SIMON: I wonder if you can help us understand something, Dr. Lyke. Can you see why people
get confused? At one point, we hear from the highest medical levels - I don't mean the White
House - you know, the average person will not have to wear a mask, to just a few weeks later,
everybody ought to wear a mask. Can you see how people begin to feel exasperated?
LYKE: Yeah, it's frustrating. It's frustrating from our side, too. This is a completely new virus
which is unprecedented. And it's true. Typically, we say that using a mask doesn't necessarily
protect you; it protects other people. And I think that's the consistent message. When we're
telling people to wear masks, it's really useless if you're the only one wearing it in a crowd. It has
to be the entire crowd. You know, there's a lot of individualism in the United States, but the
pandemic and the virus don't really respect the individualism. And I think we need to really be
stepping up as a group and protecting each other.
SIMON: Dr. Kirsten Lyke of the University of Maryland, thanks so much for being with us.
LYKE: Thank you, Scott.

Exercise 5.
A photo of the moment which has since gone 1.________________ shows a white man carried
to safety by a black protester after clashes broke out between supporters of the Black Lives
Matter Movement and 2.________________ in London.
Patrick Hutchinson says he wanted to act as a 3.________________.
When Patrick caught sight of the victim, Patrick 4.________________ him to go and pick him
up. Then, he put the man on his 5.________________ and carried him over to the police.
Patrick hopes people will break down the 6.________________.
As for Patrick, interpreting the idea meant keeping the peace between the two
7.________________.
In that moment, Patrick acted on 8.________________.

1. viral
2. far-right groups
3. peacekeeper
4. climbed underneath
5. shoulder
6. race barriers
7. viciously opposed groups
8. instinct

HOLMES: The powerful picture of a black protester carrying a white man to safety is going viral
on social media at the moment. This all happened during clashes on Saturday between Black
Lives Matter demonstrators and far-right groups in London.

Patrick Hutchinson says he picked the man up after noticing he was injured and carried him to
police nearby to keep him safe. For more on this pretty unforgettable image, I'm joined by CNN
Salma Abdelaziz at London's Waterloo station where these clashes broke out on Saturday. Really
is an amazing image. Tell us more about the man and his motives.

SALMA ABDELAZIZ, CNN PRODUCER: That's exactly right, Michael. And those steps just
behind me there is actually where this whole scene unfolded. There were right-wing
demonstrators clashing with supporters of the Black Lives Matter Movement.

Now, the Black Lives Matter Movement had actually canceled demonstrations for the weekend,
but Patrick Hutchinson says he knew that there would be young supporters of the movement that
would come out that they could potentially get into trouble, and that he wanted to act as a
peacekeeper. Take a look at our interview.

ABDELAZIZ: Is this you in the photograph?

PATRICK HUTCHINSON, PROTESTER: Yes, that is me in that photograph.

ABDELAZIZ: Can you describe to me what's happening in this picture?

HUTCHINSON: My friends and I sort of put a cordon around this man. He was on the stairs,
lying in the fetal position with, you know, anything was about to happen to him. The first time I
saw him was when I sort of climbed underneath him to go and pick him up.

ABDELAZIZ: And you could have looked at this man and thought he is my enemy. Why did
you choose to help him?

HUTCHINSON: There was a particular thought that I had that, you know, you have to show
some sort of, you know, love for your fellow man, okay, regardless because I was saying that if
the other three officers that were present when George Floyd was unfortunately murdered, if
they, just one of them had stepped in and stopped, you know, their fellow officer from doing
what he did, he'd be alive today.
ABDELAZIZ: And you put him on your shoulder, you carried him over to the police, then what
happened?

HUTCHINSON: I'm carrying him. My friends are surrounding me, protecting myself, and the
man on my shoulder. He was, you know, still sort of getting, receiving blows. You could still
feel people trying to hit him. I carried him over to the police and I said, here you are. And one of
the police officers said, thank you, you did a good thing there.

ABDELAZIZ: What do you want people to take away when they look at that picture?

HUTCHINSON: I think, hopefully, they will take away breaking down the race barriers and
realize and see that we're all one people, that we're all one race.

(END VIDEOTAPE)

ABDELAZIZ: And this is what is so extraordinary about the Black Lives Matter Movement,
Michael. It doesn't have one address, it doesn't have one leader, it is not a monolith, its how you
interpret the idea.

And for Patrick, interpreting that idea meant coming out on Saturday, even though he'd never
attended any of the demonstrations, coming out on Saturday, trying to keep the peace between
these two viciously opposed groups.

And helping a man who could have potentially held prejudices against him, although he did not
know who that man is, and he remains unidentified. And as you heard there, Patrick's hope is
that, yes, in that moment, he acted on instinct.

But that there is a message in that, and that everyone is equal, everyone is human. And he says
he hopes that that man he has rescued might see that picture and think that way as well. Michael?

[02:15:05]

HOLMES: It would be interesting to see the other man's reaction indeed.

Exercise 6.
https://www.youtube.com/watch?v=K30zVlSWJXw
For questions 1-5, listen to a report about how European countries are dealing with the
coronavirus pandemic and decide whether these statements are True or False.
1. Under the full lockdown in Italy, all grocery stores must be shut down.
2. There is a ban on entry for people from Austria into Italy.
3. Angela Merkel warned that Coronavirus could infect up to 17% of Germany's population.
4. Public gatherings still take place in the UK.
5. According to the professor, although the UK is taking more drastic measures than Italy, its
effectiveness is open to question.

1. False
Italy shut down all shops except for grocery stores and pharmacies.
2. False
Austria has banned Italians from entering the country (should be from Italy into Austria)
3. False
Angela Markel said as many as 70% of Germans could catch the virus
4. True
In the UK no mass restrictions on events or crowds
5. False
The UK’s approach is laid-back compared to Italy

Exercise 7.
Recruting minority volunteers in clinical trials for their coronavirus vaccines means overcoming
deep-seated and 1.________________ of the medical system.

Black Americans have reason to be suspicious. Beyond the well-known Tuskegee experiments,
where syphilis patients were 2.________________ for decades, they've also faced an ongoing
3.________________ by medical providers.
While mostly white people enrol in COVID vaccine trials, Meharry is one of the few places in
the country where Black patients are being 4.________________ with a personal invitation to
take part.

The National Institutes of Health has suggested minorities should be 5.________________ in


testing the COVID vaccine.

Mack says there are no 6.________________ if medical research is to reflect the diversity of the
U.S. It takes time to build trust and meaningful relationships with people who've been excluded.

The primary effort 7.________________ existing trial networks that were designed for HIV
research and convincing patients of color to help with COVID.

And there's a danger that lunging for big diversity goals could result in 8.________________.

Historically Black medical institutions in the country are uniquely positioned to do this work.
While they haven't been on the 9.________________ of the vaccine trial recruitment, they mean
to play an important role.

Convincing hundreds of thousands to sign up will be difficult. But even for those who don't
participate, researchers hope their 10.________________ efforts will at least result in more
minorities ultimately taking the vaccine when it's available.

1. well-founded mistrust
2. misled
3. exclusion and mistreatment
4. wooed: try to get the support of
5. overrepresented
6. shortcuts
7. taps into: manage to use something in a way that brings good result
8. less-than-willing participation
9. leading edge
10. outreach: bringing medical or other services to people at home or to where they spend time

SACHA PFEIFFER, HOST:

Black and Latino people in the U.S. have been disproportionately affected by COVID-19. That's
a big reason pharmaceutical companies want and need to include minority volunteers in clinical
trials for their coronavirus vaccines. Recruitment efforts are happening, but that often means
overcoming deep-seated and well-founded mistrust of the medical system. And as Blake Farmer
of WPLN in Nashville found, that's not something that can necessarily be done at warp speed.

BLAKE FARMER, BYLINE: Half a dozen patients snack on turkey sandwiches and potato
chips around a conference table. They're visiting with their doctor, Vladimir Berthaud, at
Meharry Medical College.

VLADIMIR BERTHAUD: So what's the best hope to get rid of this virus?

UNIDENTIFIED PERSON #1: Vaccination.

BERTHAUD: Vaccination. So raise your hand if you would like to take the vaccine.

FARMER: He senses some hesitation. All of these patients are Black.

LANETTE HAYES: I ain't going to be the first one, now.

FARMER: That's Lanette Hayes. Katrina Thompson says she does want to get a shot for
protection against the coronavirus. People in her apartment building aren't doing the basics of
covering their coughs.

KATRINA THOMPSON: The word vaccination don't scare me. The word trial do.

FARMER: Black Americans have reason to be suspicious. Beyond the well-known Tuskegee
experiments, where syphilis patients were misled for decades, they've also faced an ongoing
exclusion and mistreatment by medical providers. But Dr. Berthaud, who is Black and from
Haiti, appeals to a sense of duty. Plus, he's recruiting in Nashville and wants more than 300
people of color.

BERTHAUD: If you don't have enough people like you in those vaccine trials, you will not
know if it works for you. You will not know.
FARMER: For most of the COVID vaccine trials, recruitment is happening online, which often
results in mostly white people enrolling. But Meharry, which is a historically Black school, is
one of the few places in the country where Black patients are being wooed with a personal
invitation to take part. And this trial doesn't even start until October. Meanwhile, other
pharmaceutical companies are nearly done recruiting. Moderna is publicizing its demographic
statistics. They're somewhat better than the typical clinical trial but still not a good representation
of the diversity in the U.S. And the National Institutes of Health has suggested minorities should
be overrepresented in testing the COVID vaccine.

DOMINIC MACK: We say we want everybody to be included.

FARMER: Dr. Dominic Mack of Morehouse School of Medicine in Atlanta is working with the
NIH to make sure people of color are included in COVID research.

MACK: Really, the effort for the vaccinations, in a sense, are started the same way they always
been.

FARMER: Mack says there are no shortcuts if medical research is to reflect the diversity of the
U.S. It takes time to build trust and meaningful relationships with people who've been excluded.

MACK: Now, that being said, the only thing we can do is what we're doing.

FARMER: The primary effort taps into existing trial networks that were designed for HIV
research and convincing patients of color to help with COVID. Reverend Ed Sanders of the
Metropolitan Interdenominational Church in Nashville has helped educate Black clergy about
HIV. But he says it's not his job to preach trial participation from the pulpit.

EDWIN SANDERS: I am not going to do anything more than make sure people are able to make
an informed choice.

FARMER: And there's a danger that lunging for big diversity goals could result in less-than-
willing participation. Professor Rachel Hardeman studies health equity at the University of
Minnesota.

RACHEL HARDEMAN: I think there's a lot of potential for more harm because of the quick
timeline if we don't have the - again, the right people.

FARMER: Historically Black medical institutions in the country are uniquely positioned to do
this work. While they haven't been on the leading edge of the vaccine trial recruitment, they
mean to play an important role. The president of Meharry Medical College is himself an
infectious disease researcher. But instead of working on the vaccine trials being hosted on his
campus, Dr. James Hildreth plans to participate as a patient.

JAMES HILDRETH: I think my role is more important in advocating for people to be involved
in the vaccine studies than to be one of the leaders of the study.

FARMER: Back in that cramped conference room, Dr. Berthaud won over the holdouts.

ROBERT SMITH: Oh, yeah. Where's the line? Where do we sign?

UNIDENTIFIED PERSON #2: You don't have to yet. Wait until...

FARMER: That's Robert Smith with his young grandson in tow. And Smith says he'll participate
for no other reason than he trusts his longtime physician.

SMITH: He's not only my doctor, he's proven to me that he cares about me.

FARMER: Convincing hundreds of thousands to sign up will be difficult. But even for those
who don't participate, researchers hope their outreach efforts will at least result in more
minorities ultimately taking the vaccine when it's available. For NPR News, I'm Blake Farmer in
Nashville.

PFEIFFER: This story comes from NPR's partnership with Kaiser Health News and Nashville
Public Radio.

Exercise 8.
Dr. Hanan Balkhy is assistant director general for 1.________________ for the World Health
Organization, speaking from WHO headquarters in Geneva. Despite some skepticism, the WHO
says the coronavirus is usually not airborne, meaning it can't be suspended in the air and
2.________________ there where it might get breathed in. The organization believes that this
coronavirus mainly spreads through big droplets that would only be able to travel a few feet.
These guidelines are based on the knowledge of 3.________________. Its siblings would be
MERS coronavirus, SARS coronavirus which are falling under the 4.________________. While
it is a tiny piece of good news that it appears not to be airborne, WHO is not
5.________________ other studies or views on this. It is also important to reemphasize the
situation of the scenarios where a 6.________________could be done.
It is also inadvisable for those who are 7.________________ to wear a mask or a glove outside
in the community because it might give them the 8.________________. In addition, most
people are wearing masks incorrectly. They're not putting the metal over their nose. They're not
securing the 9.________________.
Compared to MERS coronavirus, this virus might affect or inflict children at a higher level.
Children, just like adults, might be more vulnerable if they had some 10.________________ or
underlying conditions.

1. antimicrobial resistance: kháng thuốc kháng sinh


2. hover
3. pathogen
4. droplet transmission criteria
5. discounting
6. risk assessment
7. asymptomatic: không có triệu chứng
8. false sense of security
9. leakage of air
10. comorbidities: bệnh lý đi kèm

MARY LOUISE KELLY, HOST:


The World Health Organization says the coronavirus is mainly spread by droplets of mucus or
saliva created when we cough or sneeze. These droplets can land on nearby surfaces and then be
touched by others, which is why you should wash your hands and try not to touch your face. The
WHO says the virus is usually not airborne, meaning it can't be suspended in the air and hover
there where it might get breathed in. Some experts on virus transmission dispute this, though,
saying it's too soon to know.

Well, joining me from WHO headquarters in Geneva is Dr. Hanan Balkhy. She's assistant
director general for antimicrobial resistance.

Dr. Balkhy, welcome to ALL THINGS CONSIDERED.

HANAN BALKHY: Thank you for having me on the program.

KELLY: Knowing how the virus spreads is obviously crucial to trying to figure out how to slow
it down. Why does the WHO believe that this coronavirus mainly spreads through big droplets
that would only be able to travel a few feet?

BALKHY: So we base these guidelines on our knowledge of the pathogen. So what family does
the emerging virus belong to? And this one belongs to the coronavirus. So its siblings would be
MERS coronavirus, SARS coronavirus and some other less prominent ones. The two pathogens
that we know a little bit more about, which is SARS and MERS, are falling under the droplet
transmission criteria.

KELLY: The question of airborne versus droplets - airborne would be - what? - a worst-case
scenario, meaning it might be able to linger in the air, and people could - would have a better
chance of breathing it in.

BALKHY: Absolutely. Not only will it be able to linger in, but actually, it can be disseminated
through air currents way much easier than the pathogens that are primarily transmitted through
droplet routes. So...

KELLY: So is this a tiny piece of good news that it appears not to be airborne, that it is spread
via droplets?

BALKHY: Definitely. That is definitely good news. However, we have to be very cautious and
hear where some of the critics out there might say, why are we not calling it airborne? I think
when you look at the sheer number of positive cases, they're happening with very clear mixing
and mingling, and they're very close with each other. So that does not indicate airborne
transmission.
KELLY: So if I hear you correctly, you're not discounting other studies or other views on this.
You're just saying until we have solid evidence otherwise, past similar coronaviruses and what
evidence we do have thus far points to it being mostly spread by droplets, not airborne.

BALKHY: Yes, absolutely. And I think - we do believe that the WHO has to give guidance for
the globe. And I want to reemphasize the situation of the scenarios where you need to do a risk
assessment where you would apply airborne isolation because you have specific high-risk areas
in your facility that does not allow for proper cleaning, proper hygiene. I cannot put a patient in a
negative-pressure room to do a bronchoscope, for example. So I have to apply certain measures
based on the scenario that I have at hand.

KELLY: For those who are not sick, to their knowledge, who are not showing any symptoms,
should everyone be wearing masks in public?

BALKHY: The immediate answer is they should not be. If you are asymptomatic, then you
should not be wearing a mask or a glove outside in the community because we know that it gives
you, first of all, the false sense of security. Number two, I can tell you by walking in the streets
and talking to my colleagues out there, most of the people who are wearing masks out there are
wearing them incorrectly. They're not putting the metal over their nose. They're not securing the
leakage of air.

However, what would be a good time to put on the mask? Let's say in the scenarios where you
have a patient who is home-isolated because of mild disease - that person has sputum, has
phlegm. The burden of the pathogen in his immediate environment needs to be minimized, and
putting a mask on at home might help him.

KELLY: You are a pediatrician, and I wonder what strikes you in terms of coronavirus and
young people because the early reporting was that children appeared to be much less vulnerable
than their parents...

BALKHY: Absolutely.

KELLY: ...And their grandparents. But now we seem to be seeing more and more children being
diagnosed and getting sick. What's happening?

BALKHY: Absolutely. And I think, again, this is how this virus is different than the MERS
coronavirus that we've experienced, where that one has really, really not affected or inflicted
children at any level close to what we see now already three months only into COVID. What I
don't have is the details of the comorbidities of the children.
KELLY: You're saying that children, just like adults, might be more vulnerable if they had some
underlying condition.

BALKHY: Yes, exactly, because the cases reported from the MERS corona - they were not
normal, healthy children, if you will.

KELLY: That's Dr. Hanan Balkhy. She is assistant director general for antimicrobial resistance
for the World Health Organization, speaking with us there from WHO headquarters in Geneva.

Dr. Balkhy, thank you so much for your time.

BALKHY: Thank you. Thank you very much, and have a great day.

Exercise 9:
https://www.youtube.com/watch?v=1oDyQP5Ycgw
For questions 1-5, listen to a report on five key global issues. What does the speaker say about
each of the issues? Choose five answers from the box and write the correct letter, A-J, in the
corresponding numbered boxes provided.

A. Clashes between forces within a newly-formed country cause heavy death toll and looming
starvation
B. Under a new zero-tolerance policy, children were forced to work in cages
C. A peace agreement was signed, ending years of civil war
D. The authority is reportedly detaining populations of an ethnic group in a bid to re-educate
them
E. World powers help to resolve a civil conflict by backing Saudi Arabia’s campaign
F. An inhumane policy faced immense public opposition and political pressure
G. Military forces commit atrocities against a minority group, resulting in mass migration.
H. The government attacks a religious minority group in an effort to crack down on freedom of
speech
I. Some countries place travel restrictions on civilians to prevent them from fleeing across
borders
J. A civil conflict broke out, followed by military intervention from neighbouring countries.

Global issues
1. China’s internment camps
2. Rohingya crisis
3. South Sudan’s civil war
4. Conflict in Yemen
5. U.S. family separation

1. D
China has been locking up the minority group in mass numbers for months, to politically
indoctrinate them
2. G
Myanmar’s alleged state-sponsored violence against the ethnic and religious minority group has
claimed the lives of at least 10,000 people and forced the over 700,000 others to flee across
international borders for safety. A recent report by the UN Human Rights Council detailed
systematic forms of violence perpetrated by security forces; including torture, imprisonment,
mass rape, arson, and indiscriminate killing. The group accuses the state of committing genocide
against the Rohingya.
3. A
South Sudan, the world’s youngest country, continues to be afflicted by civil war and violence.
Since 2013, the war has killed well over 50,000 people... The South Sudanese not only face the
threats of war, but also a food shortage that could put well over 1 million at risk of severe
hunger.
4. J
The conflict has devastated what was already the poorest country in the Arab world... The civil
conflict turned into an outright proxy war between regional ...
5. F
The Trump Administration’s policy has been criticized by multiple countries, human rights
groups, and political leaders within the US. The UN Human Rights Council has even strongly
condemned the U.S for the separation and detention of the families...

Exercise 10.
https://www.youtube.com/watch?v=HN9P8uHEtUg
For questions 1-5, listen to a report on education in several countries worldwide. What does
the speaker say about these countries in relation to their education? Choose five answers from
the box and write the correct letter, A-J, in the corresponding numbered boxes provided.
A. A decline in student performance on tests has been recorded.
B. Approximately two-thirds of students aged nine and ten fail reading tests.
C. Teachers tend to quit their jobs as their proposal to focus on standardized testing is turned
down.
D. Twenty percent of children complete school without a minimum level of education.
E. There is an alarming trend in turnover among teachers, partly due to their perceived lack of
support.
F. Their GDP has risen nearly 40 times thanks to development goals in education.
G. Intense competition is the main cause of a less effective education system.
H. Universal basic skill targets have not yet been met.
I. The attitudes and beliefs make an important contribution to high levels of academic
achievement.
J. A significant proportion of students in certain ages fail to meet their expected level in literacy.
K. Sex education is going to receive greater attention in the future.

Countries
1. The U.S. and Western Countries
2. The U.S. only
3. Finland
4. East Asian countries
5. Ghana

1. E

Over the past few years, the US and other western countries have seen a trend of teachers
quitting their jobs. Among their chief complaints is an overwhelming focus on standardized
testing, and the feeling that their professional opinions are ignored.
2. J

Similarly in the US, around two-thirds of nine and ten year olds cannot yet read at their grade
level.

3. A

And while that’s partially the result of declining Finish test scores

4. I

And they’re all wealthy nations who share similar views on education. Succeeding in school is
culturally stressed as a priority, and has an enormous impact on future prospects. This leads to
intense competition and subsequently higher overall scores.

5. H

The OECD has predicted that if Ghana could meet universal basic skill goals, they could see
their GDP rise 38 times higher over the lifetime of a child born today.

Exercise 11.
https://www.youtube.com/watch?v=HiH0YJeDly4&t=113s
For questions 1-5, listen to a talk about the richest places in the world. What does the speaker
say about these places? Choose five answers from the box and write the correct letter, A-K, in
the corresponding numbered boxes provided.
A. A crash in an industry is closely linked to foreign affairs.
B. Social gaps still persist in the projected wealthiest place worldwide.
C. According to the IMF, this country is following unsustainable development pathways.
D. Authorities need to carefully study the market data and assess the external conditions and
risks.
E. The boom in a lucrative industry originated from a discovery made around 50 years ago.
F. With GDP per capita at nearly $80,000, this country is the richest place in Europe.
G. There is a dark side to the obsession with economic growth targets.
H. Nearly 40 per cent of the total proven natural gas reserves globally are located in this place.
I. This is a success story of a formula implemented to overthrow an autonomous regime.
J. Foreign companies that are seeking asylum from large corporate taxation move to this tax
haven.
K. Holding the largest natural gas reserves, this country claims the first spot of the richest
nation's ranking.

Countries
1. Ireland
2. Singapore
3. Luxembourg
4. Macau
5. Qatar

1. G

According to the WEF, these high levels of inequality are a result of the country’s prioritization
of economic growth, rather than social equity over the past decades.

2. D
but others say we should keep a close eye on the future of Singapore’s economy, due to slight
dips in its domestic property market as well as external factors

3. J

Its central location in Europe as well as its corporate tax breaks, which has caused big companies
to place their corporate headquarters there.

4. B

According to IMF projections, the autonomous region is slated to become richest place on earth
by 2020. Despite its wealth, many of the region’s citizens struggle to make ends meet with basic
necessities, like groceries becoming increasingly unaffordable.

5. E

The country discovered a massive natural gas field off its northeast coast in the 1970s.

A. A crash in an industry is closely linked to foreign affairs. (no link)

C. According to the IMF, this country is following unsustainable development pathways. (the
WEF not the IMF)

D. With GDP per capita at nearly $80,000, this country is the richest place in Europe. (Ireland
has GDP per capita nearly $80,000 but Luxembourg is the richest place in Europe)

H. Nearly 40 per cent of the total proven natural gas reserves globally are located in this place.
(14% not 40%)

I. This is a success story of a formula implemented to overthrow an autonomous regime. ( to


support rather than overthrow)

K. Holding the largest natural gas reserves, this country claims the first spot of the richest
nation's ranking (the third largest)

Exercise 12.
https://www.youtube.com/watch?v=DngZEKg4CKI
For questions 1-5, listen to a report about the reaction of a country after the announcement of
the Nobel Peace Prize and do the following tasks.
Questions 1-2:
Which TWO facts are mentioned about Ethiopia? Choose TWO letters A-E and write your
answers in the corresponding numbered boxes provided.
A. The victory celebration was in full swing in the capital Addis Ababa of Ethiopia.
B. National pride has long been known as an intrinsic part of this country.
C. Ethiopia established the long-standing democracy, which earned recognition from Western
countries.
D. A severe famine in the past was a source of Ethiopia’s prejudice towards Westerners.
E. Ethnic tension is still a crippling burden in this country.
Questions 3-5:
Which THREE facts are mentioned as achievements of Ethiopian Prime Minister Abiy
Ahmed? Choose THREE letters A-F and write your answers in the corresponding numbered
boxes provided.
A. resolved a long-running border conflict with a neighbouring country
B. welcomed home opposition groups and acknowledged past mistakes
C. granted amnesty to political leaders
D. set up a female-dominated cabinet
E. named a woman as head of the Supreme Court
F. got a firm grip on Ethiopia’s internal displacement problem

1-2. B, E (in any order)

A. not in full swing (starting to filter through)

C. not long-standing (young and fledging democracy)

D. actually Western countries’ prejudice towards Ethiopia rather than the opposite.
3-5. A, B, E (in any order)

C. released journalists, not political leaders

D. gender-balanced (50%), not female-dominated

F. internal displacement is still prevalent

Exercise 13.
https://www.youtube.com/watch?v=EyG9noa9AFs
For questions 1-5, listen to a talk about the best recent phone trends. What does the speaker
say about these trends? Choose five answers from the box and write the correct letter, A-K, in
the corresponding numbered boxes provided.
A. have been consistently diverse in style since their first launch
B. are of higher quality than flagship phones
C. witness a shortcoming being rectified properly
D. are more fad than forever devices
E. are a worthy successor to previous models in terms of camera quality
F. were initially monotonous in design
G. need improvements in both charging speed and compatibility with other devices
H. are supported by serious apps designed for average users
I. include cameras attached to phones
J. should include sophisticated applications particularly suited for advanced uses
K. become more available with higher quality

Phone trends
1. Foldables
2. Notch Fight Phones
3. Wireless Charging Phones
4. Low-light Phones
5. Cheaper Phones

1. J

Folding phones should allow for even more serious apps for power users

2. F
Lots and lots of Android phones ended up copying the look. Over time though things started
changing ...

3. C

A downside is that some of these faster changing technologies are not compatible with each
other. Then we’ve got reverse wireless changing. Both Huawei and Samsung have phones that
can wire to charge other devices.

4. E

Lately we’ve seen low-light pictures be a focus of phone makers. Google’s night sight looked
amazing when it was introduced. Huawei’s night mode on the p30 is ridiculous.

5. K

It used to be if you wanted a good camera, fast performance and nice design, you were kind of
limited to just a few phones and they were pricey. Now you can get a pretty awesome phone for
under $500.

Exercise 14.

https://www.youtube.com/watch?v=7rH_W5PN8ns&t=111s

- Google has been hit with a 1.______________ $5 billion fine by EU regulators for breaking
antitrust laws.
- Standard Oil and 2.______________ are given as examples of well-known monopolies. Google
could be this century's Standard Oil, which was required to be 3.______________ by The U.S.
Supreme Court.

- The European Commission says Google has abused its Android market 4.______________.
Google tied together different apps that smartphone manufacturers would have to
5.______________ if they wanted to license its app store, making it difficult for competing apps
to 6.______________.

- In 2017, European Union regulators slapped Google with a $2.7 billion antitrust fine for giving
priority placement in 7.______________ to its own shopping service. In this case, Google is
8.______________ innovation and competition, which is risky.

- The EU has 9.______________ when it comes to regulating big tech. While the EU fines are
still small compared to Google’s revenue, the bills could add up if more regulators start
10.______________.

1. record-breaking 6. gain traction

2. Microsoft 7. search results

3. broken up 8. stifling
4. dominance 9. taken a lead

5. pre-install 10. trust-busting

Exercise 15.
For questions 1-9, listen to a report on a scandal in education and supply the blanks with the
missing information. Write NO MORE THAN THREE WORDS taken from the recording for
each answer in the space provided.
- At least 50 people, including wealthy parents, college sports coaches and administrators have
been charged with participating in 1)_________________ to influence undergraduate admissions
decisions at several top American universities. The scheme involved either cheating on
standardized tests or 2)_________________ college coaches to accept students as college
athletes.

- U.S. Attorney Andrew Lelling called this the largest 3)_________________ ever prosecuted by
the Justice Department. The offences are also described as 4)_________________, selfish and
shameful.

- Parents are reported to have paid more than $25 million to Rick Singer, who orchestrated the
whole scheme. Singer disguised bribe payments as charitable contributions to a purported
nonprofit that was, in fact, a 5)_________________ he used to launder the illegal money.

- Fabrication of sports credentials was carried out in different ways. Firstly, bribed coaches
labelled students as 6)_________________, which gave their applications an advantage.
Secondly, Singer helped parents take 7)_________________ photographs of their children
engaged in particular sports. Finally, Photoshop was used to insert a photograph of a student's
face onto a photograph of another person participating in the sport to document purported
athletic activity.

- College admissions counselors believe 8)_________________ are the main sufferers of the
fraud.
The institutions involved in the scandal took steps to punish the offenders, with Standford’s
9)_________________ being dismissed.

1. alleged conspiracy
2. bribing
3. college admissions scam
4. insidious
5. front
6. recruited competitive athletes
7. staged
8. hardworking students
9. sailing coach

MARY LOUISE KELLY, HOST:

To the news now that dozens of people have been indicted in a college admissions cheating
scam. Among them wealthy parents, including celebrities such as actress Felicity Huffman, also
college sports coaches and people who administer the standardized college admissions tests. Kirk
Carapezza has details from WGBH in Boston.

KIRK CARAPEZZA, BYLINE: At least 50 people have been charged with participating in
alleged conspiracy that involve cheating on college entrance exams, like the SAT and ACT.
Some of their children were admitted to elite colleges, including Yale, Stanford, UCLA and the
University of Texas, by bribing coaches.

(SOUNDBITE OF ARCHIVED RECORDING)

ANDREW LELLING: We're not talking about donating a building so that a school's more likely
to take your son or daughter.

CARAPEZZA: At the federal courthouse in Boston, U.S. Attorney Andrew Lelling announced
the charges in what he called the largest college admissions scam ever prosecuted by the Justice
Department.

(SOUNDBITE OF ARCHIVED RECORDING)

LELLING: We're talking about deception and fraud - fake test scores, fake athletic credentials,
fake photographs, bribed college officials.

CARAPEZZA: Here's how Lelling says it worked. Between 2011 and 2018, wealthy parents
paid Rick Singer, the head of a foundation and a for-profit admissions consulting service, more
than $25 million. Singer would then use that money to pay a ringer to take the SAT or ACT for
children or correct their answers. He'd also bribe Division 1 coaches.
LELLING: Singer's foundation purported to be a charitable organization but was actually a front
Singer used to launder the money that parents paid him.

CARAPEZZA: In return for bribes ranging from 200- to $400,000, coaches agreed to pretend
that certain applicants were recruited competitive athletes.

LELLING: In many instances, Singer helped parents take staged photographs of their children
engaged in particular sports.

CARAPEZZA: In other cases, he helped them use stock photos pulled from the Internet,
Photoshopping the faces of applicants onto the bodies of athletes.

JOSEPH BONAVOLONTA: Make no mistake. This is not a case where parents were acting in
the best interests of their children.

CARAPEZZA: That's Joseph Bonavolonta, special agent in charge of the Boston FBI. He says
more than 30 parents flaunted their wealth to cheat the system and set their children up with the
best education their money could buy.

BONAVOLONTA: Some spent anywhere from 200,000 to $6.5 million for guaranteed
admission. Their actions were, without a doubt, insidious, selfish and shameful.

CARAPEZZA: None of the schools named in the court filings are under investigation for fraud.
The U.S. attorney's office says college admissions officers were tricked. College admissions
counselors say the real victims in this case are hardworking students who did everything they
could to set themselves up for success in this country's crazy college admissions process.

ELIZABETH HEATON: The scale of it is utterly shocking.

CARAPEZZA: Elizabeth Heaton is with the company College Coach in Watertown, Mass. She
works with students and families trying to navigate college admissions.

HEATON: There aren't enough slots for everybody. And it's hard to take - to stomach the idea
that a couple of those slots were taken by people who bought their way in.

CARAPEZZA: In federal court, Rick Singer pleaded guilty to racketeering, money laundering,
conspiracy to defraud the United States and obstruction of justice. Stanford has fired its sailing
coach, who pleaded guilty. In a statement, Yale says the university has been the victim of a crime
perpetrated by its former women's soccer coach, and it's cooperating in the investigation. UCLA
says it has placed its men's soccer coach on leave for allegedly taking bribes and, in a statement,
says the charges against him are deeply disturbing. For NPR News, I'm Kirk Carapezza in
Boston.

Exercise 16.
For questions 1-7, listen to a report on a medical breakthrough and supply the blanks
with the missing information. Write NO MORE THAN TWO WORDS taken from the
recording for each answer in the space provided.
- According to a research paper published in the 1.________________, for only the
second time in recorded medical history, a man’s HIV infection has gone into
2._________________.
- The London patient, who was positive for the virus that causes AIDS, received a stem
cell transplant as treatment for a type of intractable 3._________________.
- The blood stem cells came from a donor with a mutation that makes cells
4._________________ to an HIV infection.
- This was an attempt to replicate the outcomes achieved in the case of the
5._________________ twelve years ago.
- While this type of treatment is clearly not practical to treat all people around the world
living with HIV, it helps researchers find out 6._________________ which may bring
about the ultimate development of a cure for HIV.
- This breakthrough allows HIV patients to stop taking 7._________________ which is
quite burdensome from both economic and health perspective.

1. journal Nature
2. remission
3. cancer
4. highly resistant
5. Berlin patient
6. critical components
7. antiretroviral therapy

MARY LOUISE KELLY, HOST:

Today a big announcement about HIV/AIDS - a second man's HIV infection is in remission.
This is being hailed as a milestone in the search for a cure, which prompts a question. Decades
into the epidemic with drugs available that prevent HIV infection and can treat it, how relevant is
the search for a cure? We're going to put that question to Rowena Johnston. She is in Seattle for
the conference where this news was announced today. She's the vice president and director of
research for the Foundation for AIDS Research - amfAR. And she joined us from member
station KUOW.

Rowena Johnston, welcome.

ROWENA JOHNSTON: Thank you very much.

KELLY: So I want to mention that your group amfAR funded the research, which is published
today in the journal "Nature." Talk to me about this specific case and why it's a breakthrough.
This has to do with a man with HIV and cancer who got a stem cell transplant.

JOHNSTON: That's right. We're referring to this man as the London patient. He was living in
London and was living with HIV and developed cancer. And his cancer was not responding to
normal treatments. And so he became a candidate for a stem cell transplant. And his physicians
were really quite smart. And they decided to look for a donor who also had a CCR5-delta 32
mutation.

KELLY: OK.

JOHNSTON: And this mutation is quite rare, but people who have this mutation are highly
resistant to HIV infection. And so by using cells from this donor, they were replacing the London
patient's immune system with the immune system of a person who's highly resistant to HIV in a
situation that was very closely similar to the Berlin patient, who we now do believe was cured.

KELLY: Between these two patients was - I believe it's a dozen years. And doctors had, of
course, tried to replicate the results in those intervening years. And the virus kept coming back.
Do we know why it was successful with this new patient, with this London patient?

JOHNSTON: You're right that there had been attempts to recapitulate what had happened in the
Berlin patient. In some cases, the transplant recipients were getting donor cells from a person
who did not have that CCR5-delta 32 genetic mutation. So it's beginning to look like having
donors that have that mutation is a key element to this successful outcome. So it's really having
the similarities and the differences between these cases and being able to compare them is where
we're going to learn the valuable lessons to move us forward.

KELLY: Just to be clear, the London patient was dealing with a very specific health situation. In
other words, the breakthrough that is being reported today does not mean that a widespread,
universal cure is within immediate reach. Is that right?

JOHNSTON: That's right. Stem cell transplant is only appropriate for people who are living with
a cancer of the immune system. So this intervention itself is not the way in which we are going to
cure people living with HIV across the world. What this intervention is going to help us
understand, though, is which are the critical components that we can learn from and put together
so that we can develop some different type of cure that is appropriate everywhere that people are
living with HIV.

KELLY: So let me circle you back to the question I posed at the outset, which is we - now
decades into this grappling with HIV and AIDS, there are drugs which help prevent infection,
which help people who are living with HIV infection manage it and live successful, long lives at
this point. Why is it so important to find a cure?

JOHNSTON: A person living with HIV today needs to take their antiretroviral therapy every
single day of their lives for the rest of their lives. And that becomes very burdensome both from
an economic perspective and also, perhaps, from the perspective of their own health. And when
you're taking antiretroviral therapy every day, you're reminded every day that you have this virus
for which you are stigmatized.

And so having a cure for HIV relieves a lot of these burdens. And if we can cure this infection,
that's going to encourage people to get tested for HIV because there's going to be that sense of
optimism that they don't have to live with this virus for the rest of their lives.

KELLY: Rowena Johnston - she is research director for amfAR. That's the Foundation for AIDS
Research. Thanks for your time.

JOHNSTON: Thank you very much

Exercise 17.
For questions 1-9, listen to a report about ways to get into a British university and supply the
blanks with the missing information. Write NO MORE THAN THREE WORDS taken from
the recording for each answer in the space provided.
University applicants can choose from ten of thousands of (1)___________________________.
Students may take A-levels or (2)___________________________ or a mixture of both.
Others may study full-time or two years to obtain a(n) (3)___________________________.
Students with poor A-level results may do (4)___________________________.
The new (5)___________________________ combines academic study with work-based
training.
Mature students should contact (6)___________________________ directly.
(7)___________________________ courses are available at further education colleges.
Sponsorship is a way of obtaining (8)___________________________ and work experience.
To sum up, studying to degree level is a(n) (9)___________________________.

1. course combinations
2. Advanced Vocational Qualifications
3. Higher National Diploma/HND
4. one-year foundation courses
5. Hospitality degree
6. admissions tutors
7. Return-to-study
8. financial assistance
9. life-enhancing experience

Getting the right A-levels is no longer the only way to get to university. New avenues are
opening up all the time. Alice Mills has the report.
Reporter (F): Just a glance at the number of university vacancies in clearing over the past month
will show there are thousands of opportunities within higher education. In fact, around 50,000
course combinations (more than in any other European country) are available at some 500
universities and higher education colleges. Fortunately for the prospective student and his or her
adviser, there are plenty of possible entry routes, ranging from the familiar A-levels and Scottish
Highers through to less well-known diplomas and access certificates. It's perhaps most
straightforward for the 18- to 19-year-old school or college leaver to take A-levels. But since the
advent of Curriculum 2000, many students have followed a slightly different route. Advanced
vocational qualifications exist in a number of subjects and students now have the chance to mix
these with the more traditional A-level subjects. And some will opt for a Higher National
Diploma or HND, a qualification in its own right, which can be topped up to degree level after
completion of the two-year, full-time course. Meanwhile, students who have not done as well as
expected at A-level should be aware that universities are increasingly offering one-year
foundation courses which, if successfully completed, will lead to the first year of a degree. For
some, a more vocational orientation will be better. New employment-related foundation degrees
may provide the answer. They have been developed so that businesses can work with higher
education colleges to address skills shortages. One example here is the Hospitality degree
developed between Radisson Edwardian Hotels and Thames Valley University last year. It has a
strong emphasis on work-based training, while retaining the required academic grounding for the
industry. Recent figures show that two-thirds of those doing foundation degrees are over 25 and
that 40% are part-time. In 2001 almost 25% of those applying to university were mature
students. Many universities and colleges do not insist on formal qualifications which this
category of applicants may lack, for one reason or another, so it's a good tactic to talk directly
with admissions tutors. Mature students are often highly motivated and focused. The mature
student (usually someone over 21 at the start of their course) has several options when
considering university entry. Of course, full-time advanced study is one route, but specially
designed access and return-to-study courses may be better. These are run in many further
education colleges and adult education centres and are designed to offer a quick route into higher
education and a way of returning to study after a break. Those wishing to combine work and
study should be encouraged to look at sponsorship possibilities that provide financial assistance
while you study and, often, work experience in vacation periods. A whole range of organisations
offer sponsorship, including industry, government, armed services, charities, local authorities
and retailers. Studying to degree level is a life-enhancing experience, and one that advisers
should encourage students to take. And it's not all about being 18 with A-levels and taking an
academic degree —there are many other options.

Exercise 18.
For questions 1-5, listen to part of a radio program about online dating and decide whether
the statements are True (T) or False (F).
1. There are nearly 395 million online dating service users worldwide.
2. The dating service app named Momo has the greatest number of subscribers in China.
3. 70% of interracial marriages in the U.S. result from online dating services.
4. The app Bumble is unconventional in that women can take a proactive role there.
5. Britain has witnessed a great increase in the number of sexual predators.

1. False
Today around 295 million people use online dating services all around the world.
2. True
In China, one app alone, Momo, has 180 million registered users.
3. False
70% of gay couples in America now meet online
4. True
Some argue the apps are breaking down barriers and changing social norms. Whitney Wolf
launched Bumble, because she wanted an app where women make the first move.
5. False
In Britain, the increase in online dating has gone hand in hand with a rise in dating-related crime,
although the numbers are small.

Exercise 19.
For questions 1-5, listen to part of a news report about a natural disaster in Venice and
answer the questions. Write NO MORE THAN TWO WORDS taken from the recording
for each answer.
1. In addition to people's residences, what were damaged by the three record-high tides?
2. Except for the sidewalks, what parts of St. Mark's Basilica were also soaked in saltwater?
3. What are put between the wet book pages to absorb the water?
4. What does the Moses project construct to hold back the tide?
5. What factor besides human incapability has accounted for the delays in the Moses project?

1. CHURCHES, BUSINESSES
2. FRESCOES
3. PAPER TOWELS
4. MOVEABLE FLOODGATES
5. FOUL PLAY / CRIMINAL ACTIVITY

AUDIE CORNISH, HOST:


A disaster is unfolding in Venice. The centuries-old city built atop small islands and laced with
canals is flooded. Three record-high tides coming in short order have submerged St. Mark's
Square and damaged churches, homes and businesses. NPR's Sylvia Poggioli reports now Italy
is rallying to save Venice, and volunteers are arriving to help.

(SOUNDBITE OF WATER SPLASHING)

SYLVIA POGGIOLI, BYLINE: Campiello del Tintor full of water as we approach the bookstore
Acqua Alta, which is appropriately named. It means high water. This eccentric bookstore is a
Venice landmark. Because of the constant danger of floods, its books have always been
displayed inside bathtubs, plastic bins and even a full-sized gondola. But last Tuesday, the books
were not high enough for the worst tide in more than 50 years, reaching 6 feet, 1 inch. The shop's
fire escape opens onto a canal, where a gondola now floats above the height of the store's
pavement, which is still under several inches of water. Co-owner Diana Zanda has been
assessing the damage and trying to salvage what she can.

DIANA ZANDA: Nobody was ready for that. But at the end of the situation, I think we're all
feeling pretty lucky because a lot of young people came here in Venice, took care of us and help
us. They helped them a lot.

POGGIOLI: The Italian culture ministry has sent experts to assess damage in the flooded crypt
of St. Mark's Basilica, where mosaic pavements and frescoes were submerged by saltwater. But
in many of the city's less-known cultural institutes, it's volunteers who are doing the salvage
work.

The Querini Stampalia Foundation is located in an 18th century Venetian palazzo. An elegant
room with Murano glass chandeliers is now a rescue center for precious books from the
foundation's seriously damaged library. Anna Dumont is an American Ph.D. student doing
research here on 19th and 20th century textiles. Today she's one of several volunteers.

ANNA DUMONT: We're taking books that are wet with saltwater. And we are, page by page,
putting paper towels in between the pages to soak up the water and hopefully save the books.

POGGIOLI: Working at the next table is Venetian Gianmarco Bondi.

GIANMARCO BONDI: I'm actually a criminal lawyer. I should be at work right now, but I have
a debt towards this place. Given I came here to study for a long time, I felt like I had to give
back.

POGGIOLI: Given his profession, I asked Bondi about Moses, the huge engineering project of
moveable floodgates to hold back the tides from flooding Venice. It's still unfinished after 16
years and $5.5 billion in public funds. Bondi echoes public opinion that believes that
incompetence, foul play and/or criminal activity are behind the delays.

BONDI: Now it's time for people to actually invest in the city and save what's left and finish this
Moses project, hopefully. And eventually, we'll collect what else is needed to save the most
beautiful city we have.

POGGIOLI: Venice is used to high water. A century ago, tides occurred seven times a year. But
today it's closer to 100, with sea levels rising. The latest word from the Moses engineers - they
hope the project can be completed by the end of 2021.

Sylvia Poggioli, NPR News, Venice.

Exercise 20.
There is an 1)___________ in the cosmos where time and space 2)___________.
Feryal Ozel is an 3)___________ at the University of Arizona.
Ozel says a black hole is like a 4)___________with such strong gravity that it 5)___________
everything.
It just distorts the space-time around it, and in the interior, it distorts it so much that even light is
6)___________.
In 2017, they used eight radio telescopes around the world to 7)___________ at the center of a
galaxy around 50 million light years away, a place where a black hole is thought to be
8)___________.
Around the black hole, there's this sort of 9)___________.
Scientists plan to use more telescopes to get better 10)___________ of this black hole.

There is an inexorable force in the cosmos where time and space converge.
She's an astrophysicist at the University of Arizona.
Ozel says a black hole is like a cosmic vacuum with such strong gravity that it sucks up
everything.
It just distorts the space-time around it, and in the interior, it distorts it so much that even light is
trapped.
In 2017, they used eight radio telescopes around the world to peer at the center of a galaxy
around 50 million light years away, a place where a black hole is thought to be lurking.
And then around the black hole, there's this sort of orangey halo.
Scientists plan to use more telescopes to get better close-ups of this black hole.
inexorable: that cannot be stopped or changed
astrophysicist: a scientist who studies the physical and chemical structure of the stars,
planets, etc
lurk: when something unpleasant or dangerous lurks, it is present but not in an obvious
way.
orangey halo: vầng hào quang có màu cam nhạt

ARI SHAPIRO, HOST:

Scientists have taken the first photo of something invisible - a black hole. For decades, black
holes have captured people's imaginations.

(SOUNDBITE OF ARCHIVED RECORDING)

UNIDENTIFIED PERSON #1: There is an inexorable force in the cosmos where time and space
converge.

SHAPIRO: That's the trailer for the 1979 movie "The Black Hole." Well, today researchers
unveiled what a black hole really looks like. NPR's Nell Greenfieldboyce was there.

(CROSSTALK)

NELL GREENFIELDBOYCE, BYLINE: The press briefing in Washington, D.C., was packed.

UNIDENTIFIED PERSON #2: Ladies and gentlemen, in the interest of making sure the fire
marshal does not have any issues, I need a clear lane for...

GREENFIELDBOYCE: Two kinds of people were milling around - folks who were dying to see
the black hole photo and scientists who weren't allowed to show them yet. Feryal Ozel was one
of the latter. She's an astrophysicist at the University of Arizona.

FERYAL OZEL: I have seen these images. I've worked a lot on these images and the
interpretations.

GREENFIELDBOYCE: Ozel says a black hole is like a cosmic vacuum with such strong gravity
that it sucks up everything.

OZEL: We have no other object quite like a black hole. It just distorts the space-time around it,
and in the interior, it distorts it so much that even light is trapped. So it is the absence of light
that we're looking for.
GREENFIELDBOYCE: People sat down, and speakers took the stage, including Sheperd
Doeleman of Harvard, who led this effort. It involved about 200 people in 20 countries. In 2017,
they used eight radio telescopes around the world to peer at the center of a galaxy around 50
million light years away, a place where a black hole is thought to be lurking.

SHEPERD DOELEMAN: We have seen what we thought was unseeable. We have seen and
taken a picture of a black hole.

GREENFIELDBOYCE: And then what looks like a blurry ring of fire appeared on a screen
above him.

DOELEMAN: Here it is.

(APPLAUSE)

GREENFIELDBOYCE: Some high school students were in the audience. I asked Ana
Humphrey (ph) how she'd describe this photo.

ANA HUMPHREY: There's definitely, like, a dark shadow - a circular shadow in the center of
the image. And then around the black hole, there's this sort of orangey halo.

GREENFIELDBOYCE: The halo being bright gasses swirling towards oblivion. Her schoolmate
Gregory Durkin (ph) was glad to see something that matched the predictions of Albert Einstein's
general theory of relativity.

GREGORY DURKIN: So I was not surprised, but it was a great, great pleasure and privilege to
see a black hole and live in the generation that can see it first.

GREENFIELDBOYCE: Scientists plan to use more telescopes to get better close-ups of this
black hole and others like it. Sera Markoff is an astrophysicist at the University of Amsterdam.

SERA MARKOFF: It's just the beginning, you know? You can think of it as a picture, but it's
also one of the most profound things humanity has ever seen.

GREENFIELDBOYCE: She says to her, it's like looking into the pit of nothingness, one of the
most fundamental mysteries of the universe. Nell Greenfieldboyce, NPR News.
Exercise 21.
https://www.youtube.com/watch?v=2DG3pMcNNlw
5G could replace the need for 1)___________ by largely operating on the cloud.
Downloading a two-hour film on 5G would take around 2)___________.
For a world that is increasingly dependent on the internet just to function, it is vital to lower
3)___________.
The efficiency of 4)___________, like self-driving cars, depends on the speed of data transfer.
5)___________ is a technique employed to provide a personalised web experience.
6)___________ is exemplified as an event that requires 5G to operate effectively.
One of the main barriers to the development of 5G is 7)___________.
5G could cause network operators to 8)___________ their current business models.
In order to work properly, 5G needs a frequency with much bigger 9)___________. This is less
of an obstacle in China, who are taking a more 10)___________.
Big 5G 11)___________ are launched to put China at the forefront of equipment production for
the new technology.
5G could replace the need for 1)_____cables______ by largely operating on the cloud.
Downloading a two-hour film on 5G would take around 2)______three and a half seconds_____.
For a world that is increasingly dependent on the internet just to function, it is vital to lower
3)______time delay_____.
The efficiency of 4)_____autonomous vehicles______, like self-driving cars, depends on the
speed of data transfer.
5)______Network slicing_____ is a technique employed to provide a personalised web
experience.
6)_____Mobile World Congress______ is exemplified as an event that requires 5G to operate
effectively.
One of the main barriers to the development of 5G is 7)______cost_____.
5G could cause network operators to 8)_____tear up______ their current business models.
In order to work properly, 5G needs a frequency with much bigger 9)_____bandwith______.
This is less of an obstacle in China, who are taking a more 10)____coherent approach_______.
Big 5G 11)_____trials______ are launched to put China at the forefront of equipment production
for the new technology.
Exercise 22.

https://www.youtube.com/watch?v=Vx0Z6LplaMU&t=23s

The 3D printing process builds an object by successively adding material layer by layer, which is
why it is also called 1)__________. This method is more economical and time-saving compared
to 2)__________.

The first step in 3D printing involves creating a 3)__________ of the 3D modelled object. This
can be done either by using 4)__________ to create your own designs or by finding objects on
websites.

Once the data has been transmitted to the printer, the material is pulled, melted and deposited to
the 5)__________ where the cooling process takes place.

The development of new materials has flourished 3D printing food, in which beautiful
6)__________ are created.

3D printing also has practical values in the medical sector where bio-materials are tested to
invent 7)__________.
The 3D printing process builds an object by successively adding material layer by layer, which is
why it is also called 1)____additive manufacturing______. This method is more economical and
time-saving compared to 2)____standard means______.

The first step in 3D printing involves creating a 3)____blueprint______ of the 3D modelled


object. This can be done either by using 4)____modeling software______ to create your own
designs or by finding objects on websites.

Once the data has been transmitted to the printer, the material is pulled, melted and deposited to
the 5)_____plate_____ where the cooling process takes place.

The development of new materials has flourished 3D printing food, in which beautiful
6)_____intricate treats_____ are created.

3D printing also has practical values in the medical sector where bio-materials are tested to
invent 7)____regenerative medicine______.
Exercise 23.

https://www.youtube.com/watch?v=dDOn_n7tNyo

Quantum computing has great potential to solve some problems that might be too hard for 1)
_____________________ to solve in reasonable amounts of time.

As quantum computing can crack other countries’ 2) _____________________, it attracts the


attention of governments.

Several countries are joining the race for quantum technology research, with China intending to
open national quantum 3) _____________________ by 2020.

Quantum computing promises to make rapid and immediate 4) _____________________


forecasts or compute the formular of a new substance.

With the involvement of technology giants, buiding powerful quantum computers is no longer
the stuff of 5) _____________________.

Quantum computers are often associated with 6) _____________________ machines.

Quantum computers can be used on a kind of 7)_____________________.


1. current supercomputers

2. encrypted networks

3. laboratory

4. stock

5. university physics departments

6. all-singing, all-powerful

7. time-share basis
Exercise 24.

Elie Honig is a former assistant U.S. 1) _____________________ for the Southern District of
New York.

He has been reading the complaints that the Trump campaign and their 2)
_____________________ are filing, and he finds two recurrent faults in these complaints.

Number one, the numbers of ballots that they're talking about are 3) _____________________.
Number two, they just don't have the proof.

When the Trump campaign is pulling back its own lawsuits, it is a 4)


_____________________.that they are in trouble.

Federal prosecutor were advised to examine the so-called 5) _____________________.

Elie thinks there is both good news and bad news regarding opening investigations. The bad
news is Barr has now changed the long-standing rule that you don't do anything public, anything
6)____________________ on an election-related case until that election is certified, finalized,
done and over. It's a continuation of Bill Barr's 7)____________________ of DOJ's power. The
good news is it's not going to make any difference because the 8)____________________ are
too big.
1. attorney
2. surrogates
3. minuscule
4. flashing red light
5. voting irregularities
6. overt: done in an open way and not secretly
7. politicization and abuse
8. margins

Well, joining me now to discuss a CNN legal analyst Elie Honig. He's a former assistant U.S.
attorney for the Southern District of New York.

Good morning, Elie. Always good to have you on.

ELIE HONIG, CNN LEGAL ANALYST: Hey, Jim. Thanks.

SCIUTTO: All right, so let's beginning with the remaining legal challenges. Do they, in your
view, have any legal legitimacy?

HONIG: Jim, remember just this past weekend, three or four days ago, when the president issued
an official statement that starting Monday he would be prosecuting, his word, misuse of the word

SCIUTTO: Yes.
HONIG: But prosecuting his case to show that this election was a fraud.

Here we are. It's Wednesday. They've got nothing.

And, Jim, I've been reading these complaints that the Trump campaign and their surrogates are
filing. There are two recurrent faults in these complaints.

One, the numbers of ballots that they're talking about, if they're even talking about specific
ballots at all, are minuscule, 53 votes here, a couple hundred votes there.

Number two, they just don't have the proof. You just laid it out perfectly. They're claiming voter
fraud. They haven't won a single case.

Look, the record speaks for itself. They've filed cases all across this country. They haven't won a
single piece of litigation yet. They are winless. They're like the New York Jets of constitutional
litigation. The record speaks for itself. They're not turning around anytime soon.

SCIUTTO: Don't insult my New York Jets, but point taken.

HONIG: Sorry.

SCIUTTO: So how quickly -- because, frankly, this is -- this is partly our largely a PR charade
by the president to raise doubts here. How quickly do you expect courts to dismiss these cases
then?

HONIG: Courts are dismissing these cases with alarming speed if you're the Trump campaign.
Some of these cases are getting dismissed within 48, 72 hours of being filed. And, by the way,
that is really rare to see.

And, beyond that, Jim, we're starting to see the Trump campaign and its surrogates dismiss their
own lawsuits. We've now seen one lawsuit withdrawn. We saw another appeal withdrawn just
the other day. That, I think, is a warning sign. You don't see that very often. When the Trump
campaign is pulling back its own lawsuits, I think that's a flashing red light that they're in trouble.

SCIUTTO: OK, I want to talk about the attorney general's role because he told federal
prosecutors that they should examine what he called voting irregularities. And this is before --
and this is the change, before states moved to certify the results, violating or overturning long-
standing DOJ policy.
The quote from his memo is as follows, I authorize you to pursue substantial allegations of
voting and vote tabulation irregularities prior to the certification of elections in your jurisdictions
in certain cases, as I've already done in specific instances.

Tell me what the standard is now for opening investigations. You're a former prosecutor. What
guidance do they take from this?

HONIG: Yes, Jim, so there's bad news and there's good news.

[09:45:00]

The bad news is, William Barr has now changed the rules at this very late date. The rule used to
be you don't do anything public, anything overt on an election-related case until that election is
certified, finalized, done and over. Bill Barr has now, after the election's over, after Donald
Trump has lost, as Donald Trump casts about for any reed to cling to, now Bill Barr says,
actually, we're going to change that long-standing rule and we're going to let you do public
things now if necessary. It's a continuation of Bill Barr's politicization and abuse of DOJ's power.

The good news is, it's not going to make any difference. The facts just are not there. Bill Barr can
change the rules all he wants, he can't generate facts out of nothing. The margins are too big.
They would have to flip more than one state. It's just not going to happen.

So, look, Bill Barr's trying to let DOJ and to use DOJ to sort of support this narrative, but there's
just no substance there.

SCIUTTO: Elie Honig, thanks for walking us all through it.

HONIG: Yes. Thanks, Jim.


Exercise 25.
Matthew Chance is CNN 1) _____________________.
Russians are claiming that their registration of the world's first COVID-19 vaccine would be akin
to a so-called "Sputnik moment," referencing the shocking 2) _____________________ by the
Soviet Union in the 1950s.
Although the vaccine has been 3) _____________________ for a while, it is the first time the
date for approval of vaccine has been clearly indicated.
The vaccine has been 4) _____________________ to develop because it is a modified version
of one already created to fight against other diseases. The conventions are also set aside as the
crucial third phase of 5)____________________ is only conducted when the vaccine is made
publicly available.
6)____________________will be the first vaccinated, once the new drug has been approved for
public use.
There's a great deal of 7)____________________ worldwide about the claims of the
effectiveness and safety of this virus.
1. senior international correspondent
2. satellite launch
3. in the pipeline
4. extraordinarily quick
5. human trials
6. frontline health workers
7. skepticism

BLITZER: Russian officials tell CNN they intend to approve the world's first coronavirus
vaccine in less than two weeks.
Our senior international correspondent Matthew Chance is in Moscow for us with this exclusive
report.
What are you learning, Matthew?
MATTHEW CHANCE, CNN SENIOR INTERNATIONAL CORRESPONDENT: Well, Wolf,
the Russians are calling it a Sputnik moment, a reference to that successful unexpected satellite
launch by the Soviet Union in the 1950s that surprised everyone. This time though, it's a
coronavirus vaccine that Russian scientists are launching into the global pandemic.
It's been in the pipeline for quite a while, but tonight, we've got the first and clearest indication of
when that vaccine will be set for approval for the Russian public. We've been given a date by
Russian officials on the 10th of August or even earlier which is, of course, extraordinarily quick.
Now, there are a couple of reasons for that speed. First of all, Russian officials say that the
technology they've used has been used in the past. They're simply adjusting an old virus to be
relevant to the coronavirus and they're doing that with data they've already got, so they've
managed to do it much quicker.
But they've also set aside the conventions, the human testing, and this is a vaccine that the
scientists injected into themselves. And we've also learned tonight that the phase three crucial
human trials will take place as the vaccine is made publicly available.
BLITZER: Who would be the first, Matthew, to receive this vaccine?
CHANCE: Well, the Russian health ministry saying that's going to be frontline health workers,
followed shortly after by other vulnerable categories. But I've just been given other figures as
well, saying by the end of the year, they expect to manufacture 200 million doses of this new
Russian vaccine, 30 million in Russian alone, the other 170 million in other countries.
And so, you know, there's a great deal of skepticism around the world about the claims of the
effectiveness and safety of this virus, it looks like, you know, within the couple of weeks, the
next couple of months, hundreds of millions of people are going to be taking it.

Exercise 26.
Listen and decide whether these statements are True, False or Not Given?
1. According to the Economist magazine, the pandemic’s real death toll worldwide may be as
many as 1 million.
2. According to the Economist magazine, America's official figures could underestimate the
death toll by around 100,000
3. India now records over 500,000 new infections every week.
4. Coronavirus cases have been going up sharply in parts of Latin America
5. In America, the top 25 percent of income earners and the bottom 25 lose jobs at the same rate
in the wake of the pandemic.
6. According to the Pew Survey, more than forty percent of Americans claim to be unfairly
dismissed.
7. According to the Census Bureau, around 10 percent of American adults say their households
struggle to put enough food on the table
1. FALSE
2. TRUE
3. TRUE
4. TRUE
5. FALSE
6. NOT GIVEN
7. TRUE

ZAKARIA: But first, here is my take. Let me begin by saying I wish President Trump and the
first lady a safe and speedy recovery. I also hope that others who have been infected along the
way who either work with the president or attended an event with him have a mild case of the
disease and a return to normalcy as soon as possible.

The news out of the White House is sad, but perhaps it can remind us of all of the sad news
around. We might have COVID fatigue, but the virus does not. The number of people known to
have died from COVID-19 has crossed one million worldwide, and that is probably a low figure.
An analysis by the "Economist" magazine indicates there may be as many as a million
more uncounted deaths.

It finds that the true count in America is probably not 200,000 dead, but more like 300,000,
which means that the country has lost as many people to COVID in the last eight months as it did
in the four years of combat during World War II.

And we are not done. Egged on by advisers and conservative pundits, the president keeps saying
things like this.
(BEGIN VIDEO CLIP)

DONALD TRUMP, PRESIDENT OF THE UNITED STATES: We're rounding the corner with
or without a vaccine. They hate it when I say that but that's the way it is. We're rounding the
corner.

(END VIDEO CLIP)

ZAKARIA: The reality is the virus is still coursing through country after country. India now has
more than half a million new cases every week and is on track to overtake the U.S. for most
cumulative cases on the planet. Latin America has seen sharp spikes in some places. New
waves have hit parts of Europe and the United States.

Tragic as the deaths are, the broader and more long lasting effects are likely to be economic. The
world economy is in the worst condition it has been since the Great Depression of the 1930s.
And the effects of this downturn have been particularly tough on the most vulnerable sectors of
society.

Take a look at these charts showing the last three recessions in America. The green lines show
the top 25 percent of income earners and the purple show the bottom 25. You will notice that in
most recessions they lose a similar share of jobs. In the current recession, a chasm has opened
up. The bottom 25 percent, the poorest among us, have seen their jobs evaporate.

The jobs report released this week shows 36 percent of unemployed workers are now classified
as permanently unemployed. A Pew survey found that 42 percent of Americans have had
someone in their household lose a job or wages because of COVID-19. Around 10 percent
of adults, 23 million, say their households sometimes or often does not have enough food to
eat according to the Census Bureau.

And that's in America. Imagine the situation in countries like India. But the point is simple. We
are in the midst of the worst global pandemic in 100 years. We are still not handling it well. We
do not have good mass testing and tracing. We are not providing nearly enough economic relief
to the tens of millions whose lives have been devastated not because they ran their businesses
poorly, not because they acted irresponsibly, but because of a pandemic.

Donald Trump's unfortunate diagnosis could serve a useful purpose to remind us that we need to
get our act together because we are all in this together.
Exercise 27.
Volunteers get either 1) _____________________ or the real-deal experimental vaccine from the
George Washington University trial.
Patient 232, Yang, suffers from 2) _____________________ and high blood pressure.
Yang thinks he is recruited because of his 3) _____________________ and his underlying
conditions.
He completed an online application and had to list his 4) _____________________.
After the first shot, he had several side effects as every muscle and joint in his body 5)
_____________________.
Volunteers are instructed not to stay at home and 6) _____________________ all the day, which
surprises the host.
1. placebo shots
2. asthma
3. ethnicity/colour
4. medical history
5. ached
6. self-isolate

MARY LOUISE KELLY, HOST:

It has been a great week for vaccine headlines, with both Pfizer and Moderna reporting
promising results from their coronavirus trials. Behind those headlines are all the people
volunteering to serve as human lab mice in those trials. One of them is Patient 232, the 232nd
person to get either placebo shots or the real-deal experimental vaccine from the George
Washington University trial. Well, Patient 232 joins me now. He is otherwise known as John
Yang. And he's a journalist, a correspondent for PBS' "NewsHour." He wrote about his
experience for STAT. John Yang, welcome.

JOHN YANG: Thank you, Mary Louise.

KELLY: As you wrote, you are at high risk for contracting, if you were to get it, a severe case of
COVID-19.

YANG: I am. I'm older. I am of a certain age, shall we say. I'm over 60. I've got asthma. I have
high blood pressure. And I'm Asian. I had actually read a study earlier that Asians are more
likely to have a bad outcome if they are hospitalized with COVID, which certainly got my
attention.

KELLY: And none of that ruled you out? They didn't care?
YANG: In a way, I think that they wanted it. I think my ethnicity was a big plus because they
really do want to test - widen these tests to have participants of color. And also, they wanted to
find out if it was safe for people with asthma and people with high blood pressure.

KELLY: Right. So walk us through the steps. You completed an online application in July. You
had to list your medical history and all of that. And then what happened? Did you get a call?

YANG: I got a phone call. And I - they tell me they're calling from the George Washington
COVID vaccine trial. And I actually thought that there would be more screening steps. I thought
I'd have to answer more questions and maybe even come in for a physical exam before they said,
OK, you're in. Then later that - in the same visit, they gave me my first shot.

KELLY: Wow. Did you have side effects?

YANG: Well, there were two shots. The first shot, it really wasn't that bad. It was sort of like a
mild case of the flu. I - every muscle and joint in my body ached. I had a fever. I went to bed; I
slept about 10 hours. But as that came on faster, it also resolved faster. By the next day, by
Wednesday - I got the shot on Tuesday. By Thursday, I was fine. The second shot really laid me
low and very quickly.

KELLY: And I gather, in a way, you were actually quite happy to be feeling lousy because...

(LAUGHTER)

KELLY: ...In your mind this suggested that maybe you'd gotten the real vaccine and not a
placebo?

YANG: Exactly. The first day, I was a little disappointed when I went to bed that I was still
feeling OK and woke up the next morning happy to be feeling bad. Like a kid hoping to get out
of school, I was happy to feel that onset of the side effects.

KELLY: One thing that they told you that struck me was they instructed you, you have to carry
on with your usual schedule. You can't just stay home and totally self-isolate. You got to walk
the dog. You got to go for your run. Why?

YANG: It's funny. It was something that made complete sense once he said it, but I had never
thought of it before. The doctor who was the head of the study said that if everyone stays at
home and never goes out and is - and doesn't get sick, if no one gets sick, then the trial will have
failed. They need some people to get sick so they can compare between the placebo group and
the group that has the real - got the real deal. It's something - it's funny because it never occurred
to me that that was the goal.
KELLY: That is John Yang, national correspondent for PBS' "NewsHour" and Patient 232 in one
of the big vaccine trials. John Yang, thank you so much.

YANG: You're welcome. Thanks for having me

Exercise 28.
Yang said he participated in the clinical trials because of the opportunity to get the real vaccine,
not because of 1) _____________________ motives.
He is in a 2) _____________________ because he suffers from several underlying conditions.
After the first day without any side effects, the vaccine 3) _____________________ him a little
bit the next morning. He had soreness in his muscle and 4) _____________________.
After the second shot, he was achy, 5) _____________________, and fatigue. But as the onset
was faster that second time, it 6) _____________________ faster, too.
He is careful not to change his patterns or the 7) _____________________ that he is taking.
Their 8) _____________________ were announced this morning.
1. altruistic
2. high-risk group
3. buoyed
4. joint
5. feverish
6. resolved
7. precautions
8. preliminary results

 Judy Woodruff:
Now to a personal take on the vaccine hunt.
Our own John Yang has been a part of these clinical trials. And he joins me now.
So, John, not everybody would want to take part in one of these. What made you want to do it?
And why do you think they wanted you, other than the fact that you're an amazing, wonderful
human being?
(LAUGHTER)
 John Yang:
Well, Judy, I wish I could say it was something altruistic like wanting to contribute to the
solution, wanting to help the cause of science.
But, quite frankly, it was the opportunity, the chance that I could get the real vaccine. In these
trials, half get the real vaccine, half get the placebo. So, that 50/50 chance is what attracted me.
I am in a high-risk group. My age, I have asthma, I have high blood pressure, things that put me
at a high-risk group. And according to the doctors in the study, that's what also made me
attractive is that they wanted to find out if the vaccine was safe for people in those groups. I'm
also a person of color, which is something they wanted to test.
And my desire to get the real vaccine was so great that, after the first day after I got the vaccine,
I was actually a little disappointed that I hadn't had a reaction to it, to the shot, that maybe I got
the placebo.
(LAUGHTER)
And, paradoxically, the next morning, when I started to feel some of the side effects, it actually
— it buoyed me a little bit. I felt good that. I felt good about feeling bad.
 Judy Woodruff:
So, you thought maybe you did get the real thing.
So, John, we have heard the side effects not too debilitating. And assuming you did get the real
thing, tell us, what did you go through? How did it affect you?
 John Yang:
Well, there were two shots.
The first one, as I say, the first day, I was fine, the second day, started to get a little achy, a little
muscle pain, muscle soreness, joint soreness. I got a fever, not too high. About 99.9 was the
highest it went. I got it on a Tuesday, got the shot on a Tuesday, and those symptoms really did
persist, until about Saturday was the first day that I really felt fine.
The second shot, the onset was much faster. By that night, I was in bed. I was in bed by 7:00,
achy, feverish, fatigue. But as the onset was faster that second time, it resolved faster, too. Again,
I got it on a Tuesday. By Wednesday, I was fine.
 Judy Woodruff:
And so, John, in a situation like this, where you don't know going in whether it is the real thing
or a placebo, what kinds of questions — as this has gone on, what kinds of questions has this
raised for you about the vaccine?
 John Yang:
Well, they asked me to continue my usual routine.
I stayed, obviously, working from home, wearing a mask when I go out, going to go shopping.
And I think I'm — what I'm fighting against is, we still don't know how long the immunity lasts,
or even if I personally have immunity.
So, I am being very careful not to change my patterns, not to change any of the precautions that I
am taking, still being very careful, wearing that mask.
 Judy Woodruff:
And, at some point, you will find out for sure whether you had the real vaccine or not?
 John Yang:
Well, this morning, when they announced their preliminary results, they said that they were so
happy with what they were finding, they were going to offer the real vaccine to the people in the
placebo group.
I'm going for my two-month check-in tomorrow, so I'm going to ask them if they're going to tell
me whether I am — which group I'm in, and if I am in the placebo, whether they will offer me
the real vaccine.
 Judy Woodruff:
Well, it's so helpful to hear your story.
And I think everybody has to say, when you offer to do something like this, it's a — you're
making a sacrifice for everybody else. And we thank you for that, John Yang.
 John Yang:
Thank you.
Exercise 29.
Encouraging data from trials about one type of coronavirus vaccine have been announced, but
they still need to be verified by the 1) _____________________, which has not been available
yet.
The makers claimed that their 2) _____________________ rate was 90 percent.
Researchers adopted a novel technique to make vaccine as they used a piece of 3)
_____________________ called mRNA.
Neil King is a biochemist at the University of Washington's Institute for
4)_____________________.
After providing more data on the safety of this vaccine, they will submit a process and approval
for 5)_____________________for the Food and Drug Administration.
This particular vaccine needs to be transported at 6)_____________________, which creates a
problem called 7)_____________________.
Pfizer has invented its own shipment scheme, with some 8)_____________________ put in a
small box.
9)_____________________ should be given the vaccine first.
Pfizer and many of the others, except for Johnson & Johnson's vaccine, require a
10)_____________________.
1. independent review/peer review/science work
2. efficacy
3. genetic code
4. Protein Design
5. emergency use authorization
6. minus 90 degrees Fahrenheit
7. cold chain logistics
8. dry ice
9. front-line health care workers
10. booster

 William Brangham:
For more perspective on this news, I'm joined now by Miles O'Brien, our science correspondent.
He has been covering the hunt for a vaccine throughout this pandemic.
Miles, great to have you back on the "NewsHour."
Can you help us understand the substance of today's announcement? This was still initially — we
haven't seen the data, correct?
 Miles O’Brien:
Yes. We should do a little bit of trust, but verifying in this one. This is a press release.
We're waiting for the independent review. The peer review, science work is still unknown to us.
But, assuming all of this bears out, this is an extraordinary number. This is — this is a vaccine
that will easily get into the Hall of Fame batting .900 percent, 90 percent.
It's stunning, actually. When you think about it, the typical flu vaccine, its efficacy is somewhere
between 40 and 60 percent. But there's still a lot of work to do here, obviously. We're talking, in
all, of 94 cases among tens of thousands, 94 cases of COVID.
And almost all of those individuals received the placebo, a saline solution, not the vaccine, and
that's where they get that number. It is a vaccine, however, that is really an unproven thing in
humans.
It is — uses a piece of genetic code called mRNA. It's a novel approach to making vaccines. And
there's never been a vaccine brought to the human market that uses this particular technique.
Here's an encouraging part of this, though, William. If all this bears out, all of the vaccines that
are out there right now are going after the spiky surface of the coronavirus, the spike protein,
including this one.
So, if this was so incredibly effective, that bodes well for the other trials that we're looking at
right now.
Back in March, which seems like an eternity ago, before we had a lot of mask requirements, I
spoke with Neil King. He's a biochemist at the University of Washington's Institute for Protein
Design. And he walked me through this process of identifying a vaccine in this manner.
 Neil King:
Figure out how to present this molecule to the immune system in the right way, so that you get
the right response.
And, really, for the spike protein, what that means is, which part of this do you want to hit with
an antibody to shut the virus down? And then how can we shine a light on that to the immune
system?
 Miles O’Brien:
So, the spike protein appears to be the key. And that really does bode well for all the other trials,
William.
 William Brangham:
So, going forward, you heard Dr. Bourla say that, this year, they think they might be able to get
50 million doses out the door, next year, over a billion.
What do we need to do between here and there getting shots into people's arms?
 Miles O’Brien:
So, William, the devil is in the logistics as much as anything. But let's walk you through it.
Over the next few weeks, they are going to try to get a little more data on the safety of this
vaccine. And then they will submit a process and approval for emergency use authorization for
the Food and Drug Administration. Obviously, the FDA knows this is coming, so that should
happen fairly quickly.
But then, when you start thinking about distributing millions, up to a billion doses, this particular
vaccine likes it cold, really cold. It needs to be transported at minus 90 degrees Fahrenheit. That
is colder than some of its competitors on the vaccine front.
And so that's creating a problem with what is called cold chain logistics. UPS is one of the big
ones. They showed us a raft of freezers that they have geared up in order to try to deliver this
vaccine without it getting spoiled.
Pfizer, meanwhile, has come up with its own shipment scheme, a little box that has some dry ice
in it. And, hopefully, they will be able to get these vaccines to people without them being
spoiled. Hopefully, they will manage. But that hasn't been tried.
 William Brangham:
Let's say they do get authorization to distribute these.
Pfizer is not the one who decides who gets those precious first doses, right? Who decides that?
 Miles O’Brien:
The federal government will make the decision, ultimately. And there are all kinds of review
panels that are trying to make specific decisions about the order of business, as it were.
But everyone agrees that front-line health care workers should be right at the head of the queue,
and then, of course, the elderly and other people with preexisting conditions which might be
adversely affected by the coronavirus more than others.
The government has already signed on with Pfizer for $2 billion to buy 100 million doses. But
you have to remember Pfizer and many of the others, except for Johnson & Johnson's vaccine,
require a booster.
So, it's actually two shots, one, and then, a couple of weeks later, a second shot. So, getting
people in for the first shot in a country where we have a lot of people who are suspicious of
vaccines, that's one thing, and then making sure they come back for the second shot, all those
things have to happen in order for this to work.
 William Brangham:
All right, Miles O'Brien, thank you so much for helping us wade through all of this.
 Miles O’Brien:
You're welcome, William.
Exercise 30.
https://www.youtube.com/watch?v=MYtynjvVX5o
The 1) _____________________ of black hole is so powerful that even light can be trapped in it.
Black holes are described as the 2) _____________________ of the universe.
The Nobel Prize in physics has been awarded to three scientists for discovering 3)
_____________________ constituting black holes.
Penrose stated that at the center time and space 4) _____________________.
Genze and Ghez discovered that an 5) _____________________ and extremely heavy object
governs the orbits of stars at the center of our galaxy.
The first direct visual evidence of a black hole was revealed last year, 6)
_____________________ but visible.
Ghez achieved another 7) _____________________ as she was only the fourth woman to win a
Nobel physics prize.
Following 8) _____________________ such as Einstein and Hawking, the winning trio of
scientist keep 9) _____________________ to better understand former mysteries.
1. gravitational pull
2. darkest secrets
3. elements
4. ceased to exist
5. invisible
6. blurry
7. milestone
8. legends
9. paving the way
Exercise 31.
https://www.youtube.com/watch?v=OXR_4NkYZbU
Standford
1. Most people associate Standford with _____________________ Silicon Valley, which is
where the school is located
2. Standford is now the global capital of _____________________
3. A Standford degree is in extremely high demand, and the school maintains the
_____________________ in the country
4. Much of Standford’s _____________________ comes from its impressive list of
_____________________.
Caltech
5. California Institute of Technology is predominantly focused on engineering, technology and
_____________________.
6. In the 1930’s the university founded NASA’s Jet Propulsion Laboratory, which builds space
robots and _____________________.
7. More Caltech graduates go on to earn PhDs than any other _____________________.
Oxford
8. Oxford is _____________________ in comparision with Caltech and Standford
9. It is one of the oldest universities, with historians _____________________ as far back as the
11th century
10. Oxford’s _____________________ to the top comes in part from its exceptionally high
research income.
11. Between 2014 and 2015, charities, individuals and businesses _____________________
nearly $680 million dollars into the university for research.

1. tech-capital
2. entrepreneurship and technology
3. lowest acceptance rate
4. credibility and desirability – alumni
5. applied sciences
6. conducts astronomy missions
7. alma mater
8. enormous
9. tracing instruction
10. ascension
11. funnelled

Exercise 32.
1. The way that the current maths curriculum is organized looks like a pyramid, with
_____________________ forming the basis and _____________________ forming the summit.
2. Statistics should be given priority as people use it every day in various aspects such as
_____________________, reward, randomness, and data comprehension
3. The _____________________ could be avoided if all people knew about probability and
statistics.
4. It's time to change from the more classical, continuous mathematics, to the more
_____________________ mathematics.

1. arithmetic and algebra - calculus


2. risk
3. economic mess
4. modern, discrete

Now, if President Obama invited me to be the next Czar of Mathematics, then I would have a


suggestion for him that I think would vastly improve the mathematics education in this
country. And it would be easy to implement and inexpensive. 
00:22
The mathematics curriculum that we have is based on a foundation of arithmetic and
algebra. And everything we learn after that is building up towards one subject. And at top of that
pyramid, it's calculus. And I'm here to say that I think that that is the wrong summit of the
pyramid ... that the correct summit -- that all of our students, every high school graduate should
know -- should be statistics: probability and statistics. (Applause) 
00:54
I mean, don't get me wrong. Calculus is an important subject. It's one of the great products of the
human mind. The laws of nature are written in the language of calculus. And every student who
studies math, science, engineering, economics, they should definitely learn calculus by the end of
their freshman year of college. But I'm here to say, as a professor of mathematics, that very few
people actually use calculus in a conscious, meaningful way, in their day-to-day lives. On the
other hand, statistics -- that's a subject that you could, and should, use on daily basis. Right? It's
risk. It's reward. It's randomness. It's understanding data. 
01:35
I think if our students, if our high school students -- if all of the American citizens -- knew about
probability and statistics, we wouldn't be in the economic mess that we're in today. (Laughter)
(Applause) Not only -- thank you -- not only that ... but if it's taught properly, it can be a lot of
fun. I mean, probability and statistics, it's the mathematics of games and gambling. It's analyzing
trends. It's predicting the future. Look, the world has changed from analog to digital. And it's
time for our mathematics curriculum to change from analog to digital, from the more classical,
continuous mathematics, to the more modern, discrete mathematics -- the mathematics of
uncertainty, of randomness, of data -- that being probability and statistics. 
02:25
In summary, instead of our students learning about the techniques of calculus, I think it would be
far more significant if all of them knew what two standard deviations from the mean means. And
I mean it. Thank you very much. (Applause) 

Exercise 33.
1. Leonardo described himself as _____________________ among billions of people around the
world who want to find solutions for climate crisis.
2. Every week sees new and _____________________, evidence that accelerates climate change
all around the world.
3. None of the environmental problem is _____________________ and
_____________________. It’s fact.
4. According to the chief of the US Navy’s Pacific Command, climate change is our single
_____________________
5. The UN organization now faces a _____________________ task
6. It’s time to put a price tag on carbon emissions and _____________________ for all oil coal
and gas companies.
7. Solving this crisis is not a question of politics but a question of _____________________.

1) a concerned citizen
2) undeniable climate events
3) rhetoric - hysteria
4) greatest security threat
5) difficult but achievable
6) eliminate government subsidies
7) our own survival

Introducing speaker: Please welcome newly-appointed United Nations Messenger of Peace Mr.


Leonardo DiCaprio.
Leonardo DiCaprio – Actor
Thank you, Mr Secretary General, your Excellencies ladies and gentlemen and distinguished
guests. I’m honored to be here today.
I stand before you not as an expert but as a concerned citizen – one of the 400,000 people who
marched in the streets of New York on Sunday and the billions of others around the world who
want to solve our climate crisis.
As an actor, I pretend for a living. I play fictitious characters, often solving fictitious problems. I
believe that mankind has looked at climate change in that same way, as if it were fiction, as if
pretending the climate change wasn’t real would somehow make it go away.
But I think we all know better than that now.
Every week we’re seeing new and undeniable climate events, evidence that accelerated climate
change is here right now. Droughts are intensifying. Our oceans are acidifying with methane
plumes rising up from the ocean floor.
We are seeing extreme weather events and the West Antarctic and Greenland ice sheets melting
at unprecedented rates, decades ahead of scientific projections.
None of this is rhetoric and none of it is hysteria. It is fact.
The scientific community knows it. Industry knows it. Governments know it. Even the United
States military knows it. The Chief of the US Navy’s Pacific Command Admiral Samuel
Locklear recently said that climate change is our single greatest security threat.
My friends, this body, perhaps more than any other gathering in human history now faces this
difficult but achievable task. You can make history or you will be vilified by it.
To be clear this is not about just telling people to change their light bulbs or to buy a hybrid car.
This disaster has grown beyond the choices that individuals make. This is now about our
industries and our governments around the world taking decisive large-scale action.
Now must be our moment for action.
We need to put a price tag on carbon emissions and eliminate government subsidies for all oil
coal and gas companies. We need to end the free ride that industrial polluters have been given in
the name of a free market economy. They do not deserve our tax dollars. They deserve our
scrutiny for the economy itself will die if our ecosystems collapse.
The good news is that renewable energy is not only achievable but good economic policy.
This is not a partisan debate. It is a human one. Clean air and a livable climate are inalienable
human rights. And solving this crisis is not a question of politics. It is a question of our own
survival.
This is the most urgent of times and the most urgent of messages. Honored delegates, leaders of
the world, I pretend for a living but you do not.
The people made their voices heard on Sunday around the world and the momentum will not
stop. But now it is your turn. The time to answer humankind’s greatest challenge is now.
We beg of you to face it with courage and honesty.
Thank you.

Exercise 34.
https://www.youtube.com/watch?v=YSew_OnDEFE
1. In December 2015, China issued a red alert warning for their capital, Beijing due to its
excessive ____________________.
2. Many ____________________ occupied top spots in terms of cleanliness according to the
Green City Index report.
3. While Denmark and Switzerland impose heavy restrictions on high polluting vehicles, there is
also a ____________________ to use public transit, bicycling and walking.
4. Curitiba in Brazil was the first city to introduce ____________________ in 1974.
5. Singapore keep the city clean through ____________________ and infrastructure investment.
6. ____________________ recycling and water treatment plants in Singapore are designed
around supplying the uniquely compact city.
7. Recycling a significant amount of waste, San Francisco is described as a
____________________ city.
8. A way to maintain low use of energy in San Fransico is to require ____________________ to
submit energy usage reports on a regular basis.
9. Accra in Ghana get high scores for establishing a ____________________with the
government.
10. In general, it is ____________________ to be involved that fundamentally contributes to the
protection of the environment.

1. smog and air particle levels


2. Nordic countries
3. societal push
4. bus rapid transit
5. high-density planning
6. state – of – the – art
7. health-conscious
8. commercial building owners
9. direct bureaucratic link
10. society’s willingness

Exercise 35.
https://www.youtube.com/watch?v=Xc0bR9tiDyU

1. Everyone can theoretically win by ____________________ in an honest manner.

2. While poor countries receive an enormous ____________________, international companies


can take advantage of the ____________________.
3. Consumers in rich countries benefit from ____________________ and better prices thanks to
imports

4. The world would move towards a more ____________________, with countries focusing on
the products they are better at making and importing those they do not have competitive
advantage.

5. However, most countries are engaged in unfair practices , for example, rich countries
encourage poor countries to sell them resources but discourage them from selling
____________________ products

6. Countries want to curb the system by ____________________ to boost exports, subsidizing


industries and so on

7. The speaker wonders what we have today truly globalization or just one big ______________.

1. eliminating trade barriers

2. influx of capital - lower wages

3. greater product variety

4. efficient allocation of capital


5. high value-added

6. artificially weakening their currency

7. masquerade

Exercise 36.
https://www.youtube.com/watch?v=IMmnSZ7U1qM
The advancement of Industry 4.0 will be driven by a smart interconnected 1)_____________.
The opportunities for disruption are huge and those left behind will feel it 2)_____________.
KPMG has conducted research on what the 3)_____________ were doing in their factories and
offices and it revealed some thought – provoking findings:
- The major players are moving away from isolated 4)_____________ to large-scale and
5)_____________ across their enterprise and among customers
- It’s important for the players to:
+ think big, 6)_____________ and nurture innovation
+ develop disruptive thought processes aimed at devastating the 7)_____________
+ look for opportunities in every element of their 8)_____________ during the production
process

1. pervasive environment
2. acutely
3. trailblazers
4. silo driven development
5. proactive integration
6. act bold
7. status quo
8. value chain

Exercise 37.
The Nobel Peace Prize was awarded today to the U.N.'s World Food Programme for its efforts in
1) _____________________.
David Beasley is the 2) _____________________ of the World Food Programme.
Starvation is a critical issue in Niger because of 3) _____________________, as well as war and
conflict.
Beasley thinks that most of hunger around the world is 4) _____________________.
With the billionaires making hundreds of billions of dollars with COVID, we're facing the worst
5) _____________________ since World War II
The starvation rate is 6) _____________________ because of COVID and economic
deterioration.
It people do not receive support, three things are going to happen. One, you are going to have
famine literally of 7)_____________________. Number two, you're going to have 8)
_____________________. And, number three, you're going to have 9)
_____________________.
If you don' thave a cure against starvation, you are going to pay for it 10)
_____________________ more with the problems that result from food insecurity.

1. combating global hunger


2. executive director
3. climate extremes
4. manmade-driven
5. humanitarian crises
6. spiking
7. biblical proportions
8. destabilization
9. mass migration
10. 1,000-fold

 Judy Woodruff:
As we reported, the Nobel Peace Prize was awarded today to the U.N.'s World Food Programme
for its work combating global hunger, especially during the pandemic.
Amna Nawaz spoke early this morning with the WFP's leader, who was in Niger as part of a
mission.
 Amna Nawaz:
And joining me now is David Beasley. He's the executive director of the World Food
Programme.
David Beasley, welcome to you. And congratulations.
So, tell us, how does it feel?
 David Beasley:
You know, it's absolutely unbelievable.
I'm here in Niger, which is probably the most appropriate place to be as we receive this award,
because I want the world to understand that people are struggling all over the world.
And so I was in a meeting, as we were talking about issues in Niger, starvation because of
climate extremes, as well as war and conflict from extremist groups. And so somebody walked in
the meeting and said, a Nobel Peace Prize. I'm like, yes, wow. Who got it? Who got it? And they
said, we did, the World Food Programme. And I was like, oh, my gosh. Wow.
I mean, the first time I — I think, in my life, I was speechless.
(LAUGHTER)
 David Beasley:
Thank you, all. Thank you.
(APPLAUSE)
 Amna Nawaz:
Anyone who knows you knows that it is very rare for David Beasley to be speechless.
(LAUGHTER)
But let me ask you, because they said your agency was able to intensify the work that you're
doing to meet this dramatic rise in global hunger around the world during the pandemic. They
said you met it with impressive ability.
But tell us, what has that rise been like? How much worse has global hunger gotten, and how
have you been able to meet that need?
 David Beasley:
Well, this is what's really, I think, the great news that we have gotten this award, so we can really
have a call to action.
The bad news is the fact that we should be getting this award because of all the hunger around
the world. And, quite frankly, it's — most of it is manmade-driven. And if you compound that
with climate extremes, when you look at the fact of, just in the last three years, the number of
people on the brink of starvation had risen before COVID 80 million to 135 million.
And now, with COVID, the number of people — and I'm not talking about people going to bed
hungry — on the brink of starvation is now to 270 starvation 270 million people.
And, quite frankly, with the billionaires making hundreds of billions of dollars with COVID,
we're facing the worst humanitarian crises since World War II. They need to step up. We need an
extra $5 billion to save millions of lives around the world.
This is a call to action. With all the wealth in the world today, no one should be dying from
hunger, not a single person.
 Amna Nawaz:
David, you have mentioned $5 billion in need just to keep people off the brink of starvation.
And you reference those billionaires. There's more than 2,000 billionaires in the world. You have
made the point before. When talk to them, do you think that they will step up in this moment?
What's been the response from the world's wealthiest people?
 David Beasley:
Well, the $5 billion that we're talking about is additional money, because we feed 100 million
people.
It literally is — the starvation rate is spiking because of COVID and economic deterioration. The
billionaires have got to step in. We're just asking them to step in this one time to help humanity.
The world needs them.
And I will be very disappointed if they don't. But what is — I mean, I know $5 billion is a lot of
money, but for the billionaires that are making literally hundreds of billions during COVID,
come on. Come on. Please, be with us. Join our hands. Show the world you care. Let's do it
together, because no one should go to bed hungry. No one should starve to death today because
of hunger, with the wealth we have today.
 Amna Nawaz:
David, where are the hot spots? You and I have spoken before about Yemen, where two-thirds of
the population is on — is food-insecure at the moment. Where else is the greatest need right
now?
 David Beasley:
Well, economic deterioration is really causing disruption for a lot of people's lives all over the
world, but especially in places like Yemen, Sudan, South Sudan, Syria, Lebanon, Iraq.
And where I am right now is in Niger, in the Sahel, where millions of people have been impacted
because of climate extremes and compounded by extremist groups that are coming in, exploiting
the situation, and now COVID on top of that.
And there are literally about a dozen or two dozen places around the world that, if we don't get
the support that they need, three things are going to happen. One, you are going to have famine, I
mean, literally of biblical proportions. Number two, you're going to have destabilization. And,
number three, you're going to have mass migration.
And we can solve all that. We have a cure against starvation, and it is called food. And we need
money to get it to the people that need the help. If you don't, you're going to pay for it 1,000-fold
more with the problems that result from the lack of security, because, when you have food
insecurity, you have destabilization, war and conflict, and migration.
 Amna Nawaz:
David Beasley, it is an incredible honor for a wonderful organization, led by you, the executive
director of the World Food Programme, today awarded the Nobel Peace Prize.
Congratulations to you and your team. And thank you for being with us.
 David Beasley:
Thank you so much. Great to be with you.
And I hope to see you again with better news, with more money.

Exercise 38.
The storm struck Honduras earlier this week as a Category 4 hurricane with 1)
_____________________ of 155 miles per hour.
Hurricane Eta caused more than 130 deaths and triggered 2) _____________________ as it
made its way across the region.
When storms make landfall, they cause hazards which come through things like 3)
_____________________, high wind speeds and rainfall.
One of the feasible solutions is to use 4) _____________________ designed specifically for
forecasting storms.
The sea surface has increased in temperature over the last 150-plus years due to 5)
_____________________ climate change.
We basically have two sets of forecasts: actual and 6) _____________________
For individual basins like the North Atlantic, it is a little bit harder to understand, in part because
of 7) _____________________from year to year.
We expect about a 5 to 7 percent increase in rainfall within 8) _____________________ and
hurricanes for every degree Celsius of warming that we have.
We're getting quicker turnaround in terms of 9) _____________________.
A glance at the storm under one or two or three additional degrees of warming would help both
10) _____________________and allow us to communicate that the impacts of climate change
are not one hundred years off.

1. sustained winds
2. mudslides
3. storm surge
4. state-of-the-art models
5. human-induced
6. counterfactual
7. natural variability
8. tropical cyclones
9. running simulations
10. inform decision making

 Hari Sreenivasan:
Tens of thousands of Hondurans have been left homeless after flooding and damage caused by
Hurricane Iota.
The storm struck Honduras earlier this week as a Category 4 hurricane with sustained winds of
155 miles per hour. Rain from the storm flooded neighborhoods and swelled rivers.
Iota was the second Category 4 hurricane to hit this part of Central America in two weeks.
Hurricane Eta caused more than 130 deaths and triggered mudslides as it made its way across the
region.
Iota is the 30th named storm of the Atlantic Hurricane season, topping off a record year that
resulted in the national hurricane center resorting to the Greek alphabet for letters.
As this record hurricane season officially comes to a close at the end of this month, we wanted to
understand more about what's been discovered about the connection between climate change and
extreme weather events like hurricanes.
I recently spoke with Kevin Reed, an associate professor at Stony Brook University who leads
the school's Climate Extremes Modeling Group. I began by asking him how scientists are teasing
out the effect of climate change on individual storms.
 Kevin Reed:
When storms make landfall, they make an impact. They have hazards. And those hazards come
through things like storm surge, high wind speeds and rainfall, right? Because extreme rainfall
can cause flooding.
And so, one of the things we can do is we can use state-of-the-art models that are used for
forecasting storms, and we can run these under different conditions that have climate change to-
date in the signal, or removed. And so we basically come up with two sets of reality.
 Hari Sreenivasan:
So you're taking a forecast like we would see on the Weather Channel and you're saying, what
are the impacts that climate change could have on this storm. And then we take a look at kind of
two outcomes with the rain and without the rain that's attributable to climate change?
 Kevin Reed:
Yes. And so another way to put it is we run a forecast just like we typically would, the difference
is then we also run a forecast in which we've removed the climate signal to-date, right?
So in the North Atlantic, that's approaching over one to two degrees Fahrenheit. The sea surface
has increased in temperature over the last 150-plus years due to human-induced climate change.
And we can remove that signal and we can rerun the forecasts. And so, we basically have two
sets of forecasts. One we call the actual forecast, right? The kind of the real forecast, as well as
the one in which we have this counterfactual, which we've removed warming.
 Hari Sreenivasan:
When you look at a storm, say, for example, like Hurricane Laura now, what does this type of
modeling tell us?
 Kevin Reed:
Yes, so this type of modeling tells us that what we call the maximum accumulated rainfall
amount, right, which is just the fancy way of saying how much rain fell during the lifetime of the
storm. We're seeing increases of 5 to 10 percent. Meaning that if an example of Hurricane Laura,
we had about 12 inches of rainfall in some regions. And so, that's an increase of about an inch in
some cases of rainfall. So we're attributing how much rainfall in an individual hurricane is due to
climate change.
 Hari Sreenivasan:
When you are looking out into the future. Are we likely to see more storms or more intense
storms or both?
 Kevin Reed:
Yeah, so that's to some extent an open question still. And the consensus is that there will be a
decrease or the number of hurricanes, for example, globally will remain about the same or
decrease.
What that means for individual basins like the North Atlantic is a little bit harder to understand,
in part because there are things like natural variability from year to year, right? But we do know
our models do tell us that the storms are becoming more intense, both in terms of the the
maximum wind speed, but also in the amount of rainfall.
We expect about a 5 to 7 percent increase in rainfall within tropical cyclones, within hurricanes
for every degree Celsius of warming that we have. And so if you see in the North Atlantic, right,
if we were to flash forward 50 years in the North Atlantic is you know two or three degrees
warmer than it is now, then you could start to expect upwards of over 10, maybe approaching 20
percent increase in tropical cyclone rainfall.
 Hari Sreenivasan:
What are the data sets that you're looking at now that you hope will help refine the way that you
model things, the way that people can prepare going forward? I mean, is there a way that we
could look at hurricane forecasts when we're watching TV before the storm sets in and realize
that this is going to be worse each time because of all of these other factors of sea level rise and
ocean temperatures warming?
 Kevin Reed:
Yeah, these type of analysis in which we're able to kind of quantify the impact of climate change
on things like hurricanes as well as other extreme weather events has definitely increased in
sophistication, meaning each time we do these type of things, like most things in life, we're
getting better at analyzing the data, we're getting quicker turnaround in terms of running
simulations. And I think that in the future we could have a system in which we're doing that real
time. Not only are we exploring the impact of climate change on the storm that occurred to date,
but also providing some future, a peek into the future.
What would the storm look like under one or two or three additional degrees of warming? And I
think that that would help both inform decision making, right? To see, OK, this storm was was
really impactful, how much worse would this storm be in the future? But also, it allows us to
communicate that the impacts of climate change are not one hundred years off.
The impacts of climate change are here now. They are changing the weather around us and they
are having a real impact on society through that.
 Hari Sreenivasan:
All right. Kevin Reed, associate professor at Stony Brook University, thanks so much for joining
us.
 Kevin Reed:
Great. Thanks for having me.

Exercise 39.
https://www.youtube.com/watch?v=D2Ipdxx7Ri0
Israeli scientists claim they have managed to not only successfully stop the biological ageing
process but to 1) _____________________ it.
They use 2) _____________________ to target specific cells and DNA and discovered the secret
of forever youth.
The oxygen therapy involves breathing pure oxygen in a 3) _____________________.
Reserachers studied the effect of the therapy on 4) _____________________, which are
associated with tissue and organ deterioration.
Telomere is a molecule linked to 5) _____________________.
Telomere shortening is considered the 6) _____________________ of the biological ageing
process.
7) _____________________ is believed to be the result of brief oxygen shortages triggered by
conditions in the chamber.
Dr. Amir is proud that his team has achieved significant telomere 8) _____________________
within a short period.

1. reverse
2. hyperbaric oxygen chambers: phòng oxy có áp suất cao
3. pressurized environment
4. senescent cells: tế bào lão hóa
5. premature cellular ageing
6. Holy Grail: something that is extremely difficult to find or get
7. cell regeneration
8. elongation
Exercise 40.
https://www.youtube.com/watch?v=u_C-o_9_E9I
Besides two current pandemics (Covid-19 and HIV/AIDS), four other pandemics are mentioned,
including 1) _____________________.
Swnie flu vaccination led to a 2) _____________________.
Swine flu infected a large percentage of the world’s population before it 3)
_____________________ in 2010.
“The Hot Zone” by Richard Preston detailed the orgins of two horrible 4)
_____________________ including Ebola.
While Hollywood often deals in fast-moving fiction, the world has witnessed slower, more 5)
_____________________.
The 6)_____________________ was the cause of the Black Death that swept through Europe in
the 14th century and claimed the lives of an estimated 200 million people.
7)_____________________ was a major problem in areas without access to clean water.
8)_____________________ is endemic in tropical regions.
No pandemic can spread like 9)_____________________.
1. Spanish flu, smallpox, typhus and tuberculosis
2. chaotic scramble
3. tapered off
4. Central African diseases
5. sustained spreads
6. bubonic plague
7. cholera
8. malaria
9. fear

You might also like